You are on page 1of 320

www.jeebooks.

in
Advanced Problems in
Inorganic Chemistry
ZA
IIT-JEE
&
Other Competitive Examinations

Ramashish Paul
B. Tech. Calcutta University

� G K Publications (P) Ltd

www.jeebooks.in
CL MEDIA (P) LTD.
Ed i t i on : 2017

© PU B L I SH ER Ad m i n i st r at i v e an d Pr od u ct i on Of f i ces

No part of this book may be reproduced in a retrieval system Pu bl i sh ed by : CL Media (P) Ltd.
or transmit ted, in any form or by any means, electr onics, KH No. 1027, Ramnagar, Salempur,
mechani cal, phot ocopyi ng, r ecor di ng, scanni ng and or
Rajputana, Roorkee (U.K.)-247667
without the wr itten permission of the publisher.
M ar k et ed by : G.K. Publications (P) Ltd.
A-41, Lower Ground Floor,
Espire Building,
I SB N : 978-81-8355-864-8 Mohan Cooperative Industrial Area,
Main Mathura Road,
New Delhi - 110044

For product information :


Visit w ww .gk pu bl i cat i on s.com or email to gk p@gk pu bl i cat i on s.com

www.jeebooks.in
Preface
As we know st udent s ar e t he backbone of our count r y and t hey ar e having t he power t o move
t he count r y ahead of t ime. The whole wor ld would salut e t hem in fut ur e for t heir gr eat r esear ch
wor k for r evolut ionar y invent ion of M edical dr ug. I nor der t o help t hem t o achieve t he t ar get we
tried to publish the book based on “ I NORGANI C CH EM I ST RY” which will serve as an impor tant
t ool for t hose who want t o shine in var ious compet it ive examinat ions (Engineer ing & M edical).
I t helps t o get an edge over ot her t o become an “ I I T” ian ; a “ NI I T” ian or an AI I M S scholar. M y
aim in t his book is t o pr ovide a compr ehensive and cont empor ar y int r oduct ion about t he subject .
The pr oblems given in t his book ar e t hor oughly innovat ive.

M ost of t he st udent s find I nor ganic Chemist r y as bor ing subject . I n my opinion it ’s not a bor ing
subject , it ’s a scor ing subject . But t o scor e it one needs t o develop int er est wit hin t he subject .

Along wi t h so many object i ve t ype quest ions which ar e desi gned scient i fi call y for var i ous
engineer ing examinat ions, a lot of ot her t ype quest ions e.g., mult iple choice quest ions, int eger
t ypes quest ions ar e given in t he book which pr ove t o be ext r emely useful t o cr ack t he I I T-JEE
Advanced examinat ions. Also emphasis is given t o pr ovide “ Compr ehension t ype” & “ M at r ix
M at ch t ype” quest ions which make t he book unique and differ ent fr om all ot her books available
in t he mar ket . So by solving all t ypes of pr oblems an aver age st udent can develop confidence in
I I T-JEE (M ain & Advanced) examinat ion. By t his way t his book can set a visionar y example t o
connect best chunk of st udent s wit h I I T-JEE examinat ion.

I have t he pr ivilege t o t each t he st udent s “ I N ORGAN I C CH EM I ST RY” ever y year. But a


lar ge number of st udent s who ar e not t aught by me can be immensely benefit t ed by t his book.
They can connect myself wit h my t eaching met hodology t hr ough t his book. M y pur pose will be
accomplished when someone can feel “ Pau l Si r ’s” met hods t o t each I nor ganic chemist r y in
absence of myself.

Ramashi sh Paul
B.Tech. Cal cutta Uni ver si ty

www.jeebooks.in
Acknowledgement
Fir st ly, I would l ik e t o t hank t he almight y God for hi s bl essi ngs t o complet e t his book
successfully wit hout my par ent s uncondit ional suppor t , I could have not been mot ivat ed t o
t ake such a big st ep in my life. The cont inuous suppor t of my wife & family was ver y much
needed dur ing t his pr ocess and I am r eally t hankful t o t hem for it .

I would also like t o t hank M r. Jit endr a Gaur of GK Publicat ions for all his editor ial suppor t t o
mould t he cont ent .

I am also t hankful to Mr. Ashish Shar ma for all his dat a suppor t. Finally I would like to thank
my st udent s as my car eer would be incomplet e wit hout t hem.

Wi th Best Regar ds
Ramashi sh Paul
B.Tech. Cal cutta Uni ver si ty

www.jeebooks.in
Contents

 QU AN T I TAT I VE AN A LYSI S OF QU EST I ON S I N EACH CH APT ER

1. Chemical Bonding 1.1 - 1.54

2. Coordination Compounds 2.1 - 2.45

3. Metallurgy 3.1 - 3.26

4. Qualitative Salt Analysis 4.1 - 4.42

5. The s-block Elements and Their Compounds 5.1 - 5.21

6. The p-block Elements and Their Compounds 6.1 - 6.46

7. The d-block Elements and Some of Their Compounds 7.1 - 7.20

8. Hydrogen and The Hydride 8.1 - 8.11

9. Periodic Table 9.1 - 9.36

10. Answers 10.1 - 10.18

www.jeebooks.in
Quantitative Analysis of Questions in Each Chapter
NUMBER OF QUESTIONS ON DIFFERENT TOPICS

No. NAME OF CHAPTER Single Multiple Comprehension Matrix Match Integer

Objective Types Choice Questions Type Type Type

1. Chemical Bonding 200 150 65 35 80

2. Coordination Compounds 150 100 75 31 50

3. Metallurgy 100 60 20 15 40

4. Qualitative Salt Analysis 120 65 100 25 50

5. The s-block Elements and Their 75 65 28 17 21


Compounds

6. The p-block Elements and 150 80 60 30 50


Their Compounds

7. The d-block Elements and 65 45 38 17 30


Some of Their Compounds

8. Hydrogen and The Hydride 46 20 15 11 15

9. Periodic Table 150 100 100 40 50

TOTAL QUESTIONS : 1056 685 456 221 386

Note : Along with this, More than 500, IIT-JEE (Main & Advanced) Questions are given with solutions. So near about total 3300
questions are given in this book which are more than sufficient for IIT-JEE preparation.

www.jeebooks.in
CHEMICAL BONDING
PROB L EM S B ASED ON GI VEN T OPI CS  Rules for linear combination of atomic orbitals
 Attainment of s stable configuration  H2+ molecule ion
 Types of Bonds  H2 molecule
 Transitions between the main types of bonding  He2+ molecule ion
 Ionic bonds  He2 molecule
 Covalent bonds  Li2 molecule
 Oxidation numbers  Be2 molecule
 Coordinate bonds  B2 molecule
 Double and triple bonds  C2 molecule
 Metallic bonds and metallic structures  N2 molecule
 The Covalent bond  O2 molecule
 The Lewis theory  O2– ion
 Sidgwick-Powell theory  F2 molecule
 Valence bond theory  Examples of molecular orbital treatment for
 What is the essence of hybridization? heteronuclear diatomic molecules
 Features of hybrid orbitals  MO molecule
 Calculation of steric number  CO molecule
 Valence shell electron pair repulsion (VSEPR)  The ionic bond
theory  Radius ratio rules
 Effect of lone pair  Calculation of some limiting radius ratio values
 Effect of double bond  Dipole moment
 Effect of electronegativity  Application of dipole moment
 Back bonding  Some special cases
 The extent od d orbital participation in molecular  Close packing
bonding  Ionic compound of the type AX (ZnS, NaCl, CsCl)
 Types of covalent bonds (sigma () and (pi ()) bonds)  Structures of zinc sulphide
 Bridge bonding  Sodium chloride structure
 Molecular orbital method
 Caesium chloride structure
 LCAO method
 Ionic compounds of the type AX2 (CaF2 , TiO2 ,
 s-s combination of orbitals SiO2)
 s-p combination of orbitals  Calcium fluoride (fluorite) structure
 p-p combination of orbitals
 Rutile structure
 d-d combination of orbitals
 -cristobalite (silica) structure
 Non-bonding combination of orbitals

www.jeebooks.in
 Layer structures (CdI2, CdCl2, [NiAs]))  Electrical conductivity and colour
 Cadmium iodide structure  Acidic nature of oxides
 Cadmium chloride structure  Thermal stability of ionic compounds
 Nickel arsenide structure  Weak forces

 Lattice energy  Attractive intermolecular forces


 Stoichiometric defectes  Repulsive intermolecular forces
 Schottky defects  Lennard- Jones potential
 Frenkel defects  Interactions between ions and covalent molecules
 Nonstoichiometric defects  The metallic bond
 Metal excess  Conductivity
 Metal deficiency  Lustre
 Born - Haber cycle  Malleability and cohesive force
 Polarizing power and polarizability - Fajans’ Rules  Bond lengths
 Properties of ionic compounds affected by  Theories of bonding in metals
polarization  Free electron theory
 Melting point of ionic compounds  Valence bond theory
 Solubility of ionic compounds  Molecular orbital or band theory
 Prediction of solubility order in ionic compounds  Conductors, Insulators and semiconductors

www.jeebooks.in
CHAPTER
1
Chemical Bonding

EXERCISE # I
 Only one correct answer : 7. In which of the following compound, central atom
1. Compound having ionic bond as well as covalent has highest oxidation state -
bond - (a) N 2O 5 (b) IF7
(a) KI3 (b) NaN 3 (c) OsO4 (d) HMnO4
(c) LiAlH4 (d) All 8. Number of planes containing maximum number of
2. Which is planar ion ? atoms in SeF6 is x and number of planes containing
 
(a) C H3 (b) C Me3 at least three chlorine atoms in PCl5 is y. Then the
 value of x – y is -
(c) BF (d) C(CN)3–
4
(a) 1 (b) 2
3. Which of the molecule is non polar as well as
(c) 3 (d) 4
nonplanar ?
(a) CS2 (b) BF3 9. In trigonal bipyramidal geometry -
(c) CCl4 (d) B3N3H6 (a) There are 3 arrangement where 3 atoms are at
an angle 120° with respect to each other.
4. Which of the molecule is polar as well as planar ?
(a) H3C – C CH (b) SF4 (b) There are 6 arrangement where 2 atoms are at
Cl an angle 90° with respect to each other
(c) (d) CHCl3 (c) There is 1 arrangement where 2 atoms are at
5. What is the correct order of dipole moment of the an angle 180° with respect to each other
molecule ? (d) All
CH3 CH3 CH3 10. The shape of cationic part and anionic part when
NO2 molten I2Cl6(liquid) undergo ionisation -
(a) > > (a) trigonal planar and trigonal bipyramidal
NO2 respectively
NO2
CH3 CH3 CH3 (b) linear and octahedral respectively
CH3 (c) angular and square planar respectively
(b) > > (d) trigonal planar and tetrahedral respectively
CH3 11. Which of the molecule does not exist ?
CH3
(c) HF > HCl > HBr > HI (a) ClF3 (b) ICl3
(d) All (c) BrF3 (d) ClI3
6. Which of the following molecule has highest dipole 12. Which compound has N–O–N linkage?
moment among methyl halide ? (a) N 2O 3 (b) N 2O 4
(a) CH3F (b) CH3Cl (c) N 2O 5 (d) All
(c) CH3Br (d) CH3I

www.jeebooks.in
1.4 CHEMICAL BONDING
13. The maximum number of 90° angles between bond 21. Two compounds X1 & X2 have general molecular
pair - bond pair electrons observed in which formula AB4.
hybridisation - X1 is nonplanar all A – B bond lengths are identical
(a) d sp2 (b) sp3d X2 is nonplanar but all A – B bond lengths are not
(c) sp3d2 (d) sp3d3 identical
14. Which of the following case C – C bond length X1 and X2 can be -
will be highest ? (a) Square planar structure and tetrahedral
(a) CF3 – CF3 (b) F2CH – CHF2 structure
(c) FCH2 – CH2F (d) F2CH – CF3 (b) Square planar structure and sea saw strucuture
15. The hybridisation state of central atom of anionic (c) Sea saw structure and tetrahedral structure
part of product when XeF2 and PF5 combined (d) tetrahedral structure and Sea saw structure
is - 22. The minimum number of 90° angles between bond
(a) sp3 (b) sp3d pair - bond pair of electrons is observed in -
(c) sp3d2 (d) sp3d3 (a) sp3 hybridisation
16. Which of the following pair has same shape and (b) sp3d hybridisation
total number of lone pair and  bond pair on central (c) sp3d2 hybridisation
atom - (d) sp3d3 hybridisation
(a) XeF6 , XeF5– (b) XeOF4 , XeF5+ 23. Incorrect option regarding bond angle -
(c) XeO2F2 , XeOF4 (d) XeF4 , XeO3 (a) PH3 > PF3
17. Which of the following is bent in shape with sp2 (b) H2O < F2O < Cl2O
hybridisation in central atom ?
(c) NH3 > NF3
(a) SnCl2 (b) Triplet carbene ( CH2 )  in H O < HOC  in CH OH
(d) HOH 2 3
(c) Both (a) & (b) (d) None
24. F – M – F bond angle is maximum for -
18. Silicate in which discrete tetrahedral unit is present
(a) BeF2 (b) MgF2
are also called naso silicate. Identify example of
nasosilicate :- (c) CaF2 (d) SrF2
3
(a) BaTi(Si3O9) 25. Number of sp hybridised carbon and sp
hybridised carbon present in C(CN)4 molecule is-
(b) Sc2Si2O7
(a) 0, 5 (b) 0, 5
(c) Ca2Mg5(Si4O11)2(OH)2
(c) 1, 4 (d) 4, 1
(d) Be2SiO4
 bond angle in XeO4 and XeO3F2 are
26. OXeO
19. Bond order of SO2 and O3 are respectively :-
(a) 2 & 1.5 (b) 1.5 & 1.5 respectively :-
(c) 1.5 & 1.33 (d) 2.4 & 1.5 (a) 120° & 109°28’
20. Sorosilicates contain units made of two tetrahedral (b) 109°28’ & 120°
that share an oxygen. The example of (c) > 109°28’ & < 120°
sorosilicate - (d) 110° & 110°
(a) BaTi(Si3O9)  (X = halogen atom) in NOF (A) ; NOCl
27. ONX
(b) Sc2Si2O7 (B) ; NOBr (C) -
(c) Ca2Mg5(Si4O11)2(OH)2 (a) A = B = C (b) A > B > C
(d) Be2SiO4 (c) C > B > A (d) A > C > B

www.jeebooks.in
CHEMICAL BONDING 1.5
28. Which of the following option regarding bond angle 33. F–P–F bond angle in PF3Cl2 can be approximately:-
is correct ? (a) only 120° (b) only 180°
O P O (c) 90° & 180° (d) 90° & 120°
P P P 34. Which one is correct order regarding property of
F  F Cl  Cl  Br Br molecule ?
F Cl Br (a) % p character in hybrid orbital sp > sp2 > sp3
(a)  <  <  (b)  =  =  (b) bond angle NO2– > NO3– > NO2+
(c)  >  >  (d)  >  <  (c) C–H bond length CH3 – F > CH2F2 > CHF3
29. Which of the following option regarding bond angle  CH > H O > NH
(d) HMH 4 2 3
is correct ?
35. What is the difference in between bond angles in
O O O cationic species of PCl5 and PBr5 in solid state ?
H  H HC 
H H3C

CH3
3 (a) 60° (b) 109°28’
(a)  <  <  (b)  =  =  (c) 0° (d) 90°
(c)  >  >  (d)  >  <  36. Select the incorrect statement for molecule
30. Which of the following option regarding bond angle Cl3P = CH2
is correct ? (a) P is sp3 hybridised and Sea saw strcuture
Se Te (b) molecule has p-d bond
O S
H  H H  H H  H H  H (c) C atom is sp2 hybridised
(a)  <  < <  (b)  =  = =  (d) Planar molecule
(c)  >  > >  (d)  >  > >  37. Number of exactly 90° angle in SF4 is -
31. Which of the following option regarding bond angle (a) 4 (b) 3
is correct ? (c) 1 (d) zero
38. The maximum % of s character on central atom
H Cl H H
 C=O  C=O  C=C present in -
H Cl H H (a) N2H2 (b) NH3

(a)  <  <  (b)  =  =  (c) NH 4 (d) NH2OH
(c)  >  >  (d) None 39. Product of which reaction can form dimer ?
32. Which of the following statements is correct ? (a) Cl2 + F2 Product
(a) In trigonal bipyramidal electron geometry, if two (excess)
lone pair is present then shape of the molecular (b) I2 + Cl2 Product
is linear (equimolar)
(b) In pentagonal bipyramidal electron geometry, (c) I2 + Cl2 Product
if two lone pair is present then shape of the (excess)
molecule is distorted octahedron. (d) Br2 + F2 Product
(c) In octahedral electron geometry, if two lone (excess)
pair is present then shape of the molecule is 40. XeF4 when combines with SbF5 the product formed
square planar. is ionic. The hybridisation of cationic part and
anionic part is respectively.
(d) In tetrahedral electron geometry, if two lone pair
is present the shape of the molecule is trigonal (a) sp3d2 , sp3d2 (b) sp3d , sp3d
planar . (c) sp3d2 , sp3d (d) sp3d , sp3d2

www.jeebooks.in
1.6 CHEMICAL BONDING
41. Which of the following geometry is not formed from 50. Molecule AHn does not exist because d orbital
sp3d hybridisation of the central atom ? contraction is not possible. Select the correct
(a) Linear (b) Tetrahedral statement -
(c) T-shaped (d) Sea-saw (a) A belongs to 2nd period
42. The orbital not participated in sp3d2 hybridisation- (b) Hybridisation of A may be sp2 or sp3
(a) px (b) dz2 (c) A must have more than 4 valence electrons
(c) pz (d) dxz (d) Minimum value of n must be 5
43. SnCl2 ; SnCl3 ; I3 ; I3– ; H3O+
+ + 51. BF3 + O(SiH3)3  No reaction.
Which of the following shape does not describe Which following factor has no significance ?
any of the above species ? (a) lone pair of oxygen are involved in back
(a) Angular (b) Linear bonding with Si
(c) Pyramidal (d) Tetrahedral (b) F can easily donate its lone pair to boron in
44. All possible bond angles in octahedral geometry comparison of oxygen
like PCl6– is - (c) O(SiH3)2 is very stable due to back bonding
(a) 109°28’ (b) 90°, 180° (d) O(SiH3)2 can’t act as Lewis base
(c) 90°, 120° (d) 109°28’, 180° 52. Choose the correct order of bond angles for given
45. % s character in hybrid orbital - compound -
(a) sp3d < sp3 < sp2 < sp (a) NO2 > O3 (b) O3 > NO2

(b) sp < sp2 < sp3 < sp3d (c) NO2 = O3 (d) Can’t be preicted
(c) sp3d < sp < sp2 < sp3 53. The molecule having only one lone pair is / are -
(d) sp2 < sp3 < sp < sp3d (a) ClF3 (b) NH3
46. Which of the following pair of species have identical (c) H2O (d) PCl3
shape ? 54. Which of the following species is non polar ?
(a) I3+ ; I3– (b) NO2+ ; NO2– (a) ClF3 (b) ICl4–
(c) TeF5– ; XeF5+ (d) SF4 , XeO4 (c) I3+ (d) OCN –
47. In which case maximum number of atoms are 55. The molecule which does not consist of any
present in same plane - F–X–F bond angle which is less than 90° -
(a) B3N3H6 (b) C(CN)4 (X = central atom) -
(c) I2Cl6 (d) IF7 (a) ClF3 (b) IF7
48. Cl – O bond order - (c) PF5 (d) SF4
(a) ClO4– > ClO3– > ClO2– > ClO– 56. Highest boiling point is observed in -
(b) ClO– > ClO2– > ClO3– > ClO4– (a) CH4 (b) BF3
(c) ClO3– > ClO3– > ClO– > ClO4– (c) Silica (d) CO 2
(d) ClO2– > ClO– > ClO3– > ClO4– 57. In a compound AB3 , A is a central atom and B is
49. Find the overlapping which do not result in  bond surrounding atom. Then which of the following
combination ofA and B gives minimum BAB  bond
formation, if z axis is considered to be
intermolecular axis - angle :-
(a) s + px (b) pz + dz2 (a) A = N ; B = Cl (b) A = P ; B = Cl
(c) s + pz (d) dx2 – y2 + dz2 (c) A = N ; B = H (d) A = P ; B = H

www.jeebooks.in
CHEMICAL BONDING 1.7
58. % of s character in the orbital occupied by lone 68. Which of the following set of overlap can not
pair in H2O. provide  bond formation ?
(Given : cos104.5° = –0.25 , HOH  = 104.5°) (a) 3d – 2p (b) 2p – 3p
(a) 25% (b) 20% (c) 2p – 2p (d) 3p – 1s
(c) 80% (d) 30% 69. The number of  bond and  bond ratio in N2 and
59. % of s character in the orbital occupied by lone P2 molecule are -
pair in NH3 molecule. 1 1
(Given : cos107° = –0.292) (a) , (b) 1, 1
2 2
(a) 68% (b) 32% 1 1
(c) 38% (d) 25% (c) ,1 (d) 1,
2 2
60. Which of the following orbital is commonly involved 70. Which orbitals of two atoms can produce 
in hybridisation of only in two of following species bond?
PCl5 ; PCl4+ ; PCl6– ; PCl3 ?
(a) dz2 overlap on z axis dz2
(a) dz2 (b) dx2 – y2
(b) dxy overlap on x axis dxy
(c) s (d) px, py, pz
(c) dx2 – y2 overlap on y axis dx2 – y2
61. Which of the following order is correct for
(d) dxz overlap on y axis dxz
increasing p-character in orbitals used for bonding
by central atom ? 71. Consider the following molecules,
(a) SiH4 > CH4 (b) PH4+ > PH3 I (CH3)3N ; II CH3 – N = C = O ; III (CH3)3 P
(c) H2S > H2O (d) NH3 > PH3 In which molecule, geometry around underlined
atom is not changed when all CH3 groups are
62. Which of the following molecule does exist as oxy
replaced by SiH3 group.
acid of fluorine ?
(a) II, III
(a) HFO4 (b) HFO3
(b) III only
(c) HFO2 (d) HOF
(c) II only
63. Which of the following fluoride does exist ?
(d) I & II
(a) OF 6 (b) OF 4
72. p – dbonding is most effective in -
(c) IF7 (d) ClF7
(a) PF3 (b) PCl5
64. Which of the following is thermally most stable?
(c) PBr3 (d) PI3
(a) H2O (b) H2S
73. In which case back bonding takes place -
(c) H2Se (d) H2Te
CCl3 C O
65. Shape of the molecule is decided by - (A) ; CCl3 (B) ;
(a)  bond (b)  bond O
CCl2 (C) ; Cl3C – O– (D)
(c) both (d) None
(a) only B
66. Which of the following bond has highest bond
dissociation energy ? (b) B,C & D
(a)  bond (b)  bond (c) B & C
(c) H-bond (d) None (d) A,B,C,D
67. Which of the following is example of strongest  74. p-flurophenol is less acidic than p-chlorophenol
bond if the molecular axis is x axis ? because -
(a) 2px – 4px (b) 2py + 2py (a) –I effect –F > –Cl
(c) 2py + 3dxy (d) 2pz + 4pz (b) –I effect of –Cl > –F

www.jeebooks.in
1.8 CHEMICAL BONDING
(c) Congugate base, of p-fluoro phenol is more 80. If back bonding does not take place in N(SiH3)3
stable than conjugate base of p-chloro phenol then what will be the hybridisation of N atom and
due to vacant d-orbital present in F. Si atom -
O– O– (a) sp2, sp3 (b) sp2, sp2
(c) sp3, sp2 (d) sp3, sp3
(d) Stability > because the charge on
81. In which of the following, correct shape and
Cl F hybridisation of central atom is given ?
oxygen in more dispersed in vacant d orbital of Ion Shape Hybridisation of
–Cl atom.
central atom
75. The correct order of viscosity of ethanol , ethylene
glycol and glycerol is :- (a) I. MnO4–1 Tetrahedral sp3
(a) ethanol > ethylene glycol > glycerol (b) II. CrO4–2 Square planar d3 s
(c) III. XeF4 Tetrahedral sp3d2
(b) ethanol > glycerol > ethylene glycol
(d) IV. CoCl4–2 Tetrahedral sp3
(c) ethylene glycol> ethanol > glycerol
82. The hybridisation of N in N(SiH3)3 ; N in H3Si – N
(d) glycerol > ethylene glycol > ethanol
= C = S ; O in O(SiH3)3 are respectively :-
76. Which of the following pair of molecular orbital is
(a) sp3 , sp2, sp3 (b) sp2 , sp2, sp2
the pair of degenerate orbital ?
(c) sp2 , sp, sp2 (d) sp3 , sp, sp2
(a) 2py , 2py (b) 2px , 2py
83. Back bonding does not take place for which
 
(c)  2py ,  2pyz (d) 1s , 2s molecule :-
77. The O – O bond length in O2 ; O2[AsF4] ; KO2 (a) H2N – BH2 (b) SCl2
is :- 
(c) H2N – PH2 (d) CF3
(a) O2[AsF4] < O2 < KO2
(b) O2 < O2[AsF4] < KO2 84. In the dimer of BeH2 , the kind of overlap present
in bridge bond :-
(c) KO2 < O2[AsF4] < O2
(a) sp3–s–sp3 (b) sp3–s
(d) O2[AsF4] < KO2 < O2
(c) sp2–s–sp2 (d) sp2–sp2–sp2
78. In B(OCH3)3 (methyl borate), due to back bonding
85. In pentagonal bipyramidal geometry -
 angle.
COB (a) Groups / atoms which are occupied at
(a) becomes equal to 109°28’ equatorial position will be at 72°
(b) less than 109°28’ (b) Groups / atoms which are occupied at axial
(c) more than 109°28’ position will be at 180°
(d) can’t be predicted (c) Equatorial groups & axial groups are at 90°
w.r.t. each other
79. For succinic acid which of the following option is
(d) All
correct ?
86. In which of the following molecule 3C–2e bonding
(K1 & K2 are Ist dissociation constant and IInd
is present ?
dissociation constant respectively)
(a) Monomer of diborane
K1 K1
(a) K  1 (b) K  1 (b) Monomer of (BeH2)n
2 2
(c) (AlH3)n
K1
(c) K  1 (d) All (d) All of these
2

www.jeebooks.in
CHEMICAL BONDING 1.9
87. Find the number of molecule having 3c–2e bond:- 95. X (1mole) + H2O H3PO3(1 mole) + H3PO4
(i) O2Cl2 (i) Al2Cl6 (1 mole) ;
(iii) Al(CH3)3 (iv) [Be(CH3)2] X is :-
(a) 0 (b) 1 (a) H4P2O8 (b) H4P2O6
(c) 2 (d) 3 (c) H4P2O7 (d) H4P2O5
88. Which of the following is correct regarding I2Cl6 ? 96. AsCl3 + H2O X1 + HCl
(a) It is planar like Al2Cl6 SbCl3 + H2O X2 + HCl
(b) It is non-planar like Al2Cl6 BiCl3 + H2O X3 + HCl
(c) During ionisation of I2Cl6 , cationic part and X1 ; X2 ; X3 are respectively :-
anionic part formed are bent and square planar (a) AsOCl , SbOCl , BiOCl
respectively. (b) As(OH)3 , Sb(OH)3 , Bi(OH)3
(d) It has 2c–2e as well as 3c–2e bond (c) As(OH)3 , SbOCl , BiOCl
89. Find the molecule in which different bond angle is (d) AsOCl , Sb(OH)3 , Bi(OH)3
observed but dipole moment of the molecule is 97. Which of the following compound when undergoes
zero :- hydrolysis oxyacid of halogen is formed as one of
(a) PCl4+ (b) BF3 the product ?
(c) PCl3F2 (d) PCl2F3 (a) SF4 (b) XeF4
90. Which of the following molecule is planar as well (c) NCl3 (d) SeF6
as polar ?
98. SiCl4 undergoes hydrolysis by :-
(a) B3N3H6 (b) ClO2
(a) SN1 mechanism (b) SN2 mechanism
(c) NH3 (d) SOF4
(c) SNi mechanism (d) E2 mechanism
91. Which of the following molecule contains atleast
99. Which of the molecule do not undergo hydrolysis
one lone pair of electron on central atom and has
at room temperature :-
non-zero dipole moment ?
(a) XeF2 (b) CClF3 (I) NF 3 (II) CCl4
(c) XeF4 (d) NH3 (III) SF4 (IV) SF6
92. During the hydrolysis of SiCl4 transition state is (a) II, III (b) I, II, III
formed in Ist step. What is the hybridisation of (c) only II (d) I, II, IV
central atom in that transition state ? 100. PCl3 and PCl5 when undergo hydrolysis, products
(a) sp3d (b) sp3d2 formed are respectively :-
(c) sp3 (d) sp2 (a) Tribasic acid (H3PO3) & Tribasic acid (H3PO4)
93. Which of the following oxide when reacts with (b) Dibasic acid (H3PO3) & Dibasic acid (H3PO4)
water, mixture of acids is formed ? (c) Dibasic acid H3PO3 & Tribasic acid H3PO4
(a) CO 2 (b) N 2O 5 (d) Tribasic acid H3PO3 & Dibasic acid H3PO4
(c) Mn2O7 (d) NO 2
101. TeF6 + H2O Oxyacid P1
94. Which of the following compounds gives only
SeF6 + H2O Oxyacid P2
oxyacid as a final product by the hydrolysis in cold
condition ? P1 & P2 are respectively :-
(a) NF 3 (b) SbCl3 (a) H2TeO4 , H2SeO4 (b) H6TeO6 , H6SeO6
(c) H2S2O7 (d) SO2Cl2 (c) H2TeO4 , H6SeO6 (d) H6TeO6 , H2SeO4

www.jeebooks.in
1.10 CHEMICAL BONDING
102. NF3 at drastic condition undergoes hydrolysis 109. Which of the following statement is incorrect ?
through :- 
(a) In ClO 3 , odd electron resides in one of the
3
(a) SN1 mechanism sp hybridised orbital
(b) SN2 mechanism 
(b) CHF is pyramidal in nature
2

(c) E2 mechanism   CH
(c) CH    H 3C  CH 3 . G° of this
3 3

(d) E1 mechanism reaction in +ve.


103. CCl4 when undergoes reaction with superheated (d) NO2 is called mixed anhydride
steam the product formed is :- 110. Correct order of boiling point of noble gases :-
(a) CO 2 (b) CHCl3 (a) He > Ne > Ar > Kr > Xe
(c) COCl2 (d) CCl3(OH) (b) He > Xe > Kr > Ar > Ne
104. Which xenon fluoride when undergoes hydrolysis, (c) Xe > Kr > Ar > Ne > He
redox reaction does not take place :- (d) Kr > Ar > He > Xe > Ne
(a) XeF2 (b) XeF4 111. Correct order of boiling point :-
(c) XeF6 (d) All (a) HF > HI > HBr > HCl
105. In which of the molecule, hydrolysis can not (b) H2O > H2Te > H2Se > H2S
generate oxyacid of central atom :- (c) PbH4 > SnH4 > SiH4 > CH4
(a) NCl3 (b) PCl3 (d) All
(c) SF4 (d) P 4O10 112. Which diagram correctly represents Wallastonite
Ca3Si3O9.
106. Which of the following compounds are obtained – –
as common product during hydrolysis of XeF4 and O O
XeF6 ? O– O–
(a) O
(a) Xe
(b) HF –O O–

O
(c) XeO3

(d) both (b) and (c) O
O O
107. In methyl radical, % of p character of C–H bond (b) – –
is :- O O
– –
O O O
(a) greater than % p character of C–F bond in –
O
trifluoro methyl radical

(b) less than % p character of C–F bond in trifluoro (c) O
– –
methyl radical O O
(d) None
(c) is same as % p character of C–F bond is
trifluoro methyl radical 113. In amphibole which are essentially double chain
silicate, which of the following statement is correct :-
(d) none
(a) No. of oxygen shared in every tetrahedral unit
108. NeF2 , KrCl2 ; ArH2 do not exist because :- is 2
(a) Nonavailability of d orbital for Ne, Kr, Ar (b) No. of oxygen shared in every tetrahedral unit
(b) d orbital contraction is not possible for NeF2 , is 3
KFCl2 , ArH2 (c) 2 & 3 oxygen atoms shared in tetrahedral unit
(c) Inertness of noble gases to form compounds alternatively
(d) Ne2+ , Kr2+ , Ar2+ have noble gas configuration (d) None

www.jeebooks.in
CHEMICAL BONDING 1.11
114. In diopsite CaMg(SiO3)2 , the type of silicate and 120. Dipole-dipole interaction exist between which
number of oxygens shared in tetrahedral unit are pair :-
respectively :-
(a) Double chain silicate ; 3 (a) KCl, H2O (b) EtOH
(b) Single chain silicate ; 2
(c) Single chain silicate ; 3 (c) , CCl4 (d) Acetone and acetonitrile
(d) Double chain silicate ; 2 –
I a

115. Which of the compound has odd e– molecule and 121. I – I I–I In I5 :-
b
central atom is sp3 hybridised :-
(a) a = b (b) a > b
(a) NO 2 (b) ClO2
(c) b > a (d) a = b = 2 pm
(c) ClO3 (d) NO
122. The internuclear interaction that is independent on
116. Which can form dimer ? the inverse cube of distance between the molecule
(a) BCl3 (b) B(OCH3 )3 is :-
(a) London force
(c) Al(CH 3 ) 3 (d) BBr3
(b) Ion-Ion interaction
117. In boric acid hybridisation state of B and O are
(c) Ion dipole interaction
respectively :-
(d) H-bonding
(a) sp2, sp2
123. Which of the compound does not have H-bonding?
(b) sp2, sp3
(a) K2HPO4 (b) KHCO3
(c) sp3, sp2
(c) K2HPO3 (d) KHF2
(d) sp3, sp3
124. Select incorrect statement :-
118. Correct statement :-
(a) Ammonium is more soluble than Na+
(a) 8-hydroxy quinoline can be separated from 4-
hydroxy quinoline by fractional distillation (b) He/Ne can not form clathrate
(b) As branching of isomeric alkane increases (c) ICl is more reactive than I2
boiling point decreases (d) H3O4+ , H-bonding is absent
(c) Salicylic acid has less boiling point that p 125. Which of the following has highest pka in aqueous
hydroxy benzoic acid because former solution ?
compound can form intramolecular H bonding (a) CH3NH2 (b) (CH3)2NH
but latter compound form intermolecular H (c) (CH3)3N (d) NH3
bonding.
126. Which of the following compound of heating gives
(d) All
N2 gas ?
119. Which of the following statement is correct ?
(a) NH4NO3 (b) NH4NO2
(a) 12 crown-4-ether dissolves Li+ most effectively
(c) NaNO 3 (d) Li3N
because cavity size is 75 pm
127. KCl and KI will be soluble (highly) in :-
(b) 15 crown-5-ether dissolves Na + most
effectively because cavity size is 110 pm (a) H2O, H2O respectively
(c) 18 crown-6-ether dissolves Rb + most (b) H2O & acetone respectively
effectively because cavity size is 160 pm (c) Acetone, H2O respectively
(d) All (d) Acetone, Acetone respectively

www.jeebooks.in
1.12 CHEMICAL BONDING
128. Highest polarising power is observed in :- 138. In PTFE (polytetrafluoro ethene) the hybridisation
(a) Na+ (b) K+ of carbon atom is :-
(c) Cu+ (d) Zn2+ (a) sp (b) sp2
129. Highest polarisability is observed in :- (c) sp3 (d) sp3d
139. In vinyl acetylene the hybridisation of carbon atom :-
(a) F – (b) Cl–
(a) only sp2 (b) only sp3
(c) Br– (d) I–
(c) both sp2 , sp3 (d) both sp, sp2
130. Most covalent halide :-
140. The correct order of B–F bond lengths :-
(a) CaCl2 (b) PbCl2
(a) BF3 < BF2 – NH2 < BF2 – OH < BF4–
(c) AlCl3 (d) FeCl2
(b) BF2NH2 < BF2 – OH < BF3 < BF4–
131. Which of the following is thermally most stable ?
(c) BF3 < BF4– < BF2 – OH < BF2 – NH2
(a) Na2CO3 (b) BeCO3 (d) BF3 < BF2 – OH < BF2 – NH2 < BF4–
(c) MgCO3 (d) CaCO3 141. Total 2c–2e and 3c–2e bonds in Be(BH4)2 are
132. Which of the following compound has highest lattice respectively :-
energy ? H H
Be
(a) NaCl (b) MgO
(c) ScN (d) TiC H H
133. Which compound has highest covalent character?
H
(a) NaCl (b) MgCl2 B
B
(c) AlCl3 (d) SiCl4 H H
H
134. Which of the following order is correct regarding (a) 6,4 (b) 4,6
solubility of group IIA halide ? (c) 4,4 (d) 4,8
(a) BeX2 < MgX2 < CaX2 < SrX2 < BaX2 142. What will be the oxidation state of P in H4P2O8 &
(b) LiX > NaX > KX > RbX < CsX HPO3 ?
(c) BeX2 > MgX2 > CaX2 > SrX2 > BaX2 (a) +5, +5 (b) +5, +3
(d) LiX < NaX < KX < RbX < CsX (c) +5, 0 (d) +5, +7
135. Which compound is least thermally stable ? 143. The mineral Na 2 Fe 3 II Fe 2 III [Si 8 O 22 ](OH) 2
(chrocidolite) is a :-
(a) LiNO3 (b) NaNO 3
(a) pyroxene chain silicate
(c) KNO 3 (d) RbNO3
(b) Sheet silicate
136. Out of BeCl2 , CaCl2 , CsCl , KCl ; BeCl2 has :-
(c) amphiboles chain silicate
(a) highest ionic chracter
(d) 3D-silicate
(b) highest solubility in H2O
144. The silicate anion in the mineral kinoite is a chain
(c) highest lattice energy of three SiO4 tetrahedra that share corners with
(d) lowest charge : size ratio in cation adjacent tetrahedra. The mineral also conain Ca2+
137. % s-character of the hybrid orbitals used for C–H ions, Cu2+ ions, and water molecules in a 1 : 1 : 1
bond formation in CH2F2 molecule :- ratio. Mineral is represented as :-
 CH F  111.9 , cos 111.9° = –0.3729
 HCH (a) CaCuSi3O10. H2O
2 2
(b) CaCuSi3O10. 2H2O
(a) 27.1% (b) 37.3%
(c) Ca2Cu2Si3O10. 2H2O
(c) 29% (d) 30%
(d) None of these

www.jeebooks.in
CHEMICAL BONDING 1.13
145. An atom A has atomic number less than 21. What 152. The solubility of anhydrous AlCl3 and hydrated
will be the hybridisation of ACl3 , if ACl3 has zero AlCl3 in diethyl ether are S1 and S2 respectively.
dipole moment ? Then :-
(a) pure p (b) sp (a) S1 = S2 (b) S1 > S2
(c) sp2 (d) sp3 (c) S1 < S2 (d) none
146. Which of the following halide of silver is water 153. The critical temperature of water is higher than that
soluble ? of O2 because the H2O molecule has :-
(a) AgF (b) AgCl (a) fewer electrons than O2
(c) AgBr (d) AgI (b) two covalents bonds
147. Acetylene gets dissolved in acetone, it is because (c) V-shape
of :- (d) more dipole moment
(a) intramolecular hydrogen bonding 154. Which of the following boiling point order is
(b) intermolecular hydrogen bonding correct ?
(c) london forces (a) He > T2 > D2
(d) ionic interaction (b) He < T2 < D2
148. Ca3(PO4)2 + SiO2  Calcium silicate + P1 (c) T2 > He > D2
P1 + C  P2 + CO (d) He < D2 < T2
P1, P2 are respectively :- 155. Two ice cubes are pressed over each other and
(a) P4O6, P4 (b) P4O10, P4 unite to form one cube. Which force is responsible
(c) P4O6, PH3 (d) P4O10, PH3 for holding them together :-
149. In methylene sulphur tetrafluoride (CH2 = SF4) ; (a) Vander wall’s forces
the hydrogen atoms are in the same plane with (b) Covalent attraction
the :- (c) H-bond formation
(a) equatorial fluorine atoms (d) Dipole-dipole attraction
(b) axial fluorine atoms– 156. Which has maximum dipole moment ?
(c) axial as well as equatorial fluorine atoms Cl
(d) None Cl Cl
Cl Cl
150. In which of the following compounds phosphorous
(a) (b)
atoms are at the corner of tetrahedral
unit :- Cl
(a) P 4 (b) P 2O72– Cl
(c) PO43– (d) KH2PO3 Cl Cl
151. Which of the following equilibria would have the
least value of KP at a common temperature? (c) (d)
Cl Cl
 MgO + CO2
(a) MgCO3 
157. NMe3 + H2O2  product
 CaO + CO2
(b) CaCO3 
What is the formal charge of N in product ?
 SrO + CO2
(c) SrCO3 
(a) +1 (b) –1
 BaO + CO2
(d) BaCO3 
(c) +2 (d) 0

www.jeebooks.in
1.14 CHEMICAL BONDING
158. When container containing calcium carbide and 164. Which of the following option is correct :-
calcium phosphide are pierced and thrown in the CH3 C CH2 C CH3 CH3 C CH C CH3
sea, then the gases evolved burn and serve as a
signal called as Holme’s signal. The gases evolved O O OH O
are :- Keto form Enol form
(a) C2H4 , P2H4 (b) CH4 , P2H4 (a) enol form is less stable than keto form due to
(c) C2H4 , PH3 (d) C2H2 , PH3 hydrogen bonding
159. Elements of which groups are polymorphic (exist (b) keto and enol form both are of same stability
in more than one allotropic form) :- (c) enol form is more stable than keto form due to
(a) group 13 (b) group 14 hydrogen bonding
(c) group 15 (d) group 16 (d) none
160. Boric acid and fluoro boric acid are respectively :- x CH2–CH3 y CH=CH2
(a) Monobasic acid and monobasic acid 165.
(b) Monobasic acid and tribasic acid
z CCH w CCH
(c) Tribasic acid and tribasic acid
(d) Tribasic acid and monobasic acid
161. In CO2, SO2, SiO2 central atom is covalently x, y, z, w are bond length between C–C bond.
bonded with m1, m2, m3 number of oxygen atoms
The correct order is :-
respectivley then :-
(a) x > y > z > w (b) x > z > w > y
(a) m1 = 2, m2 = 2, m3 = 2
(c) x > y > w > z (d) none
(b) m1 = 4, m2 = 4, m3 = 4
166. In energy level diagram or O22+ , 13th electron is
(c) m1 = 2, m2 = 4, m3 = 4
present in :-
(d) m1 = 2, m2 = 2, m3 = 4
(a) 2p orbital
162. Which of the following reactions is spontaneous ? x

(a) Pb2+ Pb4+ + 2e (b) Sn2+ Sn4+ + 2e (b)  2p y or 2pz orbital


(c) Tl1+ Tl3+ + e (d) Bi3+ Bi+5 + 2e *
(c)  2 p y or *2 pz orbital
163. The correct structure of Cl2O7 :-
O O (d) *2px orbital
O O O
Cl Cl 167. There is largest energy difference between M.O.
(a)
of dioxygen :-
O O
O (a) 2p , 2p (b) 2p , 2p
O O O x y y y
(c) 2p , 2s (d) 2p , 2s
(b) Cl Cl O x x
168. Which of the energy order is same for both M.O.
O O diagram of N2 and O2?
O O
(a) *2s < 2p (b) *2s < 2s
x
Cl O Cl O (c) 2p < 2p (d) *2p < 2p
(c) x y y y
O 169. Assuming 2s-2p mixing is not operative the
O O paramagnetic species among the following :-
O O O
(a) Be2 (b) B2
Cl Cl (c) C 2 (d) N 2
(d)
O O O
O
www.jeebooks.in
CHEMICAL BONDING 1.15
170. If the filling of electrons in M.O. of O2 does not 178. If we consider no mixing of 2s and 2p orbitals,
obey Hund’s rule , then which of the following then the bond order and magnetic moment of the
property in O2 gets changed :- diatomic molecule C2 is :-
(a) bond order (a) 3 & dimagnetic (b) 2 & dimagnetic
(b) magnetic behaviour (c) 2 & paramagnetic (d) 2.5 & dimagnetic
(c) number of electrons in bonding M.O. 179. If C–C bond in C2H6 undergoes heterolytic fission, the
(d) number of electrons in antibonding M.O. hybridisation of two resulting carbon atoms is / are:-
171. Which allotrope of phosphorous is (a) sp2 both (b) sp3 both
thermodynamically most stable ?
(c) sp2 , sp3 (d) sp3 , sp3
(a) white phosphorous
180. Which of the following halide when undergo
(b) black phosphorous hydrolysis the intermediate form during the reaction
(c) purple phosphorous have sp3 , sp3d , sp3d2 hybridisation ?
(d) red phosphorous (a) SF6 (b) SF4
172. In which case , both of the following species are (c) PCl5 (d) PCl3
paramagnetic ?
181. Which halogen does not react with water ?
(a) O2 , Na2O2 (b) BaO2 , KO3
(a) F 2 (b) Cl2
(c) O2 , KO3 (d) N2 , KN3
(c) Br2 (d) I2
173. Which of the following species is expected to be
coloured ? 182. Which halogen when reacts with water, it oxidises
water to form O2 gas at a fastest rate ?
(a) KO 3 (b) Li2O
(a) F 2 (b) Cl2
(c) Na2O (d) K 2O 2
(c) Br2 (d) I2
174. If inter nuclear axis is z axis then HOMO orbital(s)
of CO is :- 183. BCl3 + H2O P1 + P2
(a)   2p (b) N.B.M.O Correct statement regarding P1 and P2 :-
z
(c) 2p = 2p (d) None of these (a) P1, P2 both are proton donor acid
y z
175. Find the number of nodal plane in * (ABMO) :- (b) P1, P2 no one is proton donor acid
(a) 1 (b) 2 (c) In between P1, P2 one is proton donor acid &
another is not a proton donor acid
(c) 3 (d) 4
(d) P1, P2 both are base
176. If internuclear axis is assumed to be z axis then
which of the following pair(s) of orbitals are 184. BF3 + H2O B(OH)3 + X1 X1 + BF3 X3
HOMO of O2 molecule :- SiF4 + H2O Si(OH)4 + X2 X2 + SiF4 X4
(a) 2p , 2p (b) 2p , 2p X3 & X4 are respectively :-
x y y z
(c) 2p ,  
(d) 2p , 2p (a) HF, HF (b) HBF4 , HSiF5
x 2p y z z
177. Which of the following molecule or molecular ion (c) H2BF5 , H2SiF6 (d) HBF4 , H2SiF6
has highest number of total electron in antibonding 185. In the cyclo-S8 molecule of rhombic sulphur, all
molecular orbital ? the S–S bond length and all the S–S–S bond
angles are respectively
(a) O 2 (b) N 2
(a) 204 pm and 105° (b)102 pm and 120°
(c) N2+ (d) O 2 2–
(c) 204 pm and 180° (d) 102 pm and 60°

www.jeebooks.in
1.16 CHEMICAL BONDING
186. Which is correct physical state of boron halide ? 192. Which type of  bonds present in solid Cl2O6 ?
(a) BF3 is gas ; BCl3 , BBr3 both are liquids ; BI3 is (a) 2p-3d
solid (b) 2p-3p
(b) BF3 is solid ; BCl3 , BBr3 both are liquids ; BI3 (c) 3p-3d
is gas
(d) both (a) & (b)
(c) BF3 , BCl3 both are gases ; BBr3 is liquid ; BI3
193. Which of the following molecule is diamagnetic and
is solid
has last electron in sigma () B.M.O. :-
(d) None
(a) O 2 (b) N 2
187. Which of the following is correct statement ?
(c) B2 (d) C 2
(a) HF forms more effective hydrogen bonding
than H2O but Hvap of HF is less than Hvap 194. Which of the following has dipole-dipole
of water interaction between the species possessing
permanent dipole ?
(b) HF forms less effective hydrogen bonding than
H2O but Hvap of HF is more than Hvap of (a) liquid-NH3 (b) liquid-He
water (c) solid-I2 (d) liquid-Br2
(c) HF forms less effective hydrogen bonding than 195. Select the most ionic and most covalent compound
H2O and Hvap of HF is less than Hvap respectively from the following :-
of water CrO5, Mn2O7, PbO, P4O10, SnO2
(d) HF forms more effective hydrogen bonding than (a) CrO5, Mn2O7 (b) PbO, Mn2O7
H2O and Hvap of HF is more than (H)vap of
(c) CrO5, SnO2 (d) CrO5, P4O10
water
188. SnCl2 , HgCl2 can not coexist together because :- 196. Which of the following statements is true for IO2F2–
according to VSEPR theory ?
(a) Sn2+ oxidises Hg2+
(a) the lone pair and two I–O double bonds occupy
(b) Sn2+ reduces Hg2+ to Hg+1 & finally Hg+1
the equatorial positions of trigonal bipyramid
converted into Hg metal
(b) it has sp3d hybridisation and T-shaped
(c) Sn2+ oxidises Hg to Hg2+
(d) Sn2+ reduces Hg4+ to Hg2+ (c) its structure is analogous of SF4
189. In aqueous solution, the hydronium ion is further (d) (a) and (c) both
hydrated to give speices like :- 197. What is the hybridisation of boron atoms in
(a) H5O2+ (b) H7O3+ compound Mg[B2O(OH)6] ?
(c) H9O4+ (d) All (a) both sp3
190. In aqeuous solution, the hydroxyl ion (OH–) is (b) one sp2 and other sp3
hydrated to give speices like :- (c) one sp3 and other sp3d
(a) H3O2– (b) H5O3– (d) both sp2
(c) H7O4– (d) All 198. A diatomic molecule has a dipole moment of 1.2
191. The number of P–O–P linkage in D. If its bond length is equal to 10–10 m. Then the
cyclictetrametaphosphoric acid (H4P4O12) which fractions of an electronic charge on each atom will
is formed during stepwise hydrolysis of P4O10 ? be :-
(a) zero (b) two (a) 42% (b) 52%
(c) three (d) Four (c) 37% (d) 25%

www.jeebooks.in
CHEMICAL BONDING 1.17
199. Which of the following is correct structure of 7. Which of the following molecule have sp3d2
S2Cl2 ? hybridised atom with square pyramidal
Cl shape ?
(a) S = S (b) Cl–S–S–Cl (a) BrF5 (b) XeF5+
Cl (c) XeOF4 (d) TeF5–
Cl
8. Choose the correct option :-
S–S
(c) (d) Cl=S=S=Cl (a) In d3s hybridisation, the d-orbital involved is
Cl dx2–y2 , dz2 , dxy
200. Which of the following atomic orbital does not
(b) In sp3d hybridisation, the d-orbital involved is
participate in trigonal bipyramidal electron
dz 2
geometry i.e., sp3d hybridisation :-
(c) In sp3d2 hybridisation, the d-orbital involved is
(a) dz2 (b) dx2–y2
dx2–y2 , dz2 respectively
(c) p y (d) px
(d) In sp3d2 hybridisation, the d-orbital involved is
EXERCISE # II dxy , dzx respectively
 One or More Than One Correct Answer : 9. Which of the following are polar and planar ?
1. In which of the following compounds, central atom (a) CO
has one lone pair of e– ? (b) SF6
(a) BrF5 (b) XeOF4 (c) CH2Cl2
(c) SF2Cl2 (d) SOCl2 HO2C
2. For which of the following compounds, underlined (d) CH=CH (fumaric acid)
atom has +1 oxidation state ? CO2H
H 3C 10. Which of the following are nonpolar and
S=O nonplanar ?
(a) TI3 (b)
H 3C (a) PCl5
(c) Cl – F (d) KI3 (b) SF6
3. In which of the following compounds, total six lone (c) H3C–CC–CH3
pair is present ? (d) XeF4
(a) Melamine [C3N3(NH2)3] 11. Which of the following molecule do exist ?
(b) HClO2 
(a) NeF2 (b) N F4
(c) Cl2O7
(c) IF32– (d) GeCl62–
(d) SO3
12. Choose the correct statements -
4. Which of the following are linear in shape ?
O–
(a) N 3 – (b) I3+
+
(c) I3– (d) XeF2 (a) the correct structure of NOF3 is N
5. How many ions are non planar ions ? F F
F
(a) XeF5– (b) SO32– (b) maximum number of atoms lying is one plane
(c) SO42– (d) ClO4– of C(CN)4 molecule is 5
6. Which of the following molecule have sp3d2 (c) among halogens Cl2 only exist as solid state
hybridised atom with square planar shape ? (d) only oxyacid of fluorine which can be isolated
(a) XeF4 (b) ICl4– is HOF where fluorine has +1 oxidation state
(c) BrF4– (d) I2Cl6

www.jeebooks.in
1.18 CHEMICAL BONDING
13. In which of the following compound, dz2 orbital is 18. For which of the following compounds, dz2 orbital
involved in hybridisation of their central atom- is involved in hybridisation ?
(a) BrF3 (b) H3O+ (a) BrF3 (b) SF6
(c) CH3+ (d) SF6 (c) H3O +
(d) SF4
14. Which of the following are pyramidal in shape ? 19. For the molecule, OF2 (I) ; HOF (II) ; H2O (III),
(a) Amide ion (NH2–) which statements are correct -
(b) XeO3 (a) dO–F bond length I > II
(c) ClO3– (Chlorate ion) (b) dO–F bond length II > I
(d) Sulphite ion (c) dO–H bond length II > III
15. Which of the following are angular in shape with (d) dO–H bond length III > II
sp3 hybridisation on central atom ? 20. Which of the following statements is correct ?

(a) I3+ (a) CH 3 Hybridisation of carbon is sp2
(b) ClO2– (chlorite ion) 
(b) CF3 Hybridisation of carbon is sp2
(c) HOCl (Hypochlorous acid)
(d) Water (c) C2F4 Hybridisation of carbon is sp3
16. Which process are non spontaneous ? +
(d)  Hybridisation of carbon having
O O positive charge is sp2
+ +
(a) NO2 + NO2  N–N 21. Incorrect order -
O

O– (a) C – C bond length FCH2CH2F > CHF2CHF2
(b) ClO2 + ClO2 O = Cl – O – O – Cl = O > F3C–CF3
CMe3 CMe3 (b) Oxidation state of P H3PO4 > PCl3 > NaH2PO2
. . > P4
(c) CMe3 O +O CMe3 (c) Number of 90° repulsion between bond pair
CMe3 XeF4 > PF5 > PF6–
CMe3
CMe CMe3 CMe3 (d) Number of lone pair present on central atom
XeOF2 > Cl+OF4– > ClOF2 > BH4–
CMe Me3C O O CMe3 22. Which molecular geometry are most likely to result
CMe3 from an octahedral electron geometry ?
CMe CMe3
 
(a) square planar (b) square pyramidal
(d) C H 3  C H 3 
 CH 3 – CH 3 (c) linear (d) v-shaped
17. Colourless gas (X) + O2 Brown colour gas 23. Bond energy order -
NO2 (Y) (a) sp3 C–H > sp2 C – H > sp C – H
Correct statements are : (b) sp C–H > sp2 C – H > sp3 C – H
(a) X, O2 , Y all are example of odd electron (c) F – F > Cl – Cl > Br – Br > I – I
molecule
(d) C – F > C – Cl > C – Br
(b) Y maintains planarity on dimerisation
24. Which of the following statements are correct ?
(c) X, Y are examples of odd electron molecule
(a) ClF2– is linear but ClF2+ is bent
but O2 is not
(b) In pentagonal bipyramidal geometry, axial
(d) Y during dimerisation forms compound which
orbital length is greater than equitorial bnond
is diamagnetic in nature
length

www.jeebooks.in
CHEMICAL BONDING 1.19
(c) In trigonal bipyramidal geometry, axial orbital 31. Which of the following representation of molecules
length is greater than equitorial bond length are not correct -
(d) Dimer of ICl3 i.e., I2Cl6 has planar structure with O
sp3d2 hybridisation S
(a) N  N  N (b)
O O– O

25. Which of the following molecule itself exist but has
no definite hybridisation ? O
– F F F F
(a) XeF3 (b) PH3 P
(c) F Xe (d) Cl F
(c) H2S (d) None

..
F Cl
26. Which of the following pair of species have different 32. Which of the following molecule have only
hybridisation but same shape ? p – d type of  bonds ?
(a) ICl2– and BeCl2 (gas phase) (a) SO3 (b) XeO4
(b) PCl5 (liq.) and XeOF4 (c) XeO2F2 (d) SO2
(c) ICl4– , Pt(NH3)2Cl2 33. The correct order of single bond energy ?
(d) XeF2 and HC  CH (a) P–P > N–N (b) S–S > O–O
27. Which of the following options are correct (c) C–C > Si–Si (d) C–H > C–D
regarding XeO3F2 and XeOF4 molecule ? 34. Which of the following compound are planar ?
(a) dXe–O(XeO3F2) < dXe–O(XeOF4) (a) ClF3 (b) ICl4–
(b) dXe–F(XeO3F2) < dXe–F(XeOF4) (c) I3+ (d) OCN –
(c) shapes are trigonal bipyramidal and square 35. Choose the correct statements regarding SF4
pyramidal respectively molecule -
(d) the nature of  bond is 5d–2ptype (a) Hybridisation of sulphur occurs in Ist excited
state.
28. Considering internuclear axis is x axis, then which
of the following overlapping are possible - (b) Total number of l.p – b.p. repulsion at 90° is 2
(a) s + px  (c) There are six possible F–S–F bond angles
which are less than 180°
(b) px + px 
(d) It is hypervalent molecule
(c) px + dxy 
36. In which of the following compounds, sigma
(d) dxy + py  coordinate bond is absent -
29. Which of the following molecule has dative (a) NO 3 – (b) CO
bond ? +
(c) NH4 (d) HNC
(a) NH4+ (b) NOF3 37. Which of the following molecule is aromatic ?
(c) PCl3 (d) SF6 (a) B3N3H6 (b) B3H3O3
30. Which of the following process involves absorption (c) C3N3Cl3 (d) C 12 O 9
of energy ?
38. Correct statements about C12O9 is are :-
(a) S(g) + e– S–(g)
(a) It is example of cyclic trianhydride
(b) Xe(g) + e– Xe–(g)
(b) It is formed when melittic acid undergoes heating
(c) O–(g) + e– O2–(g) (c) It is example of cyclic ether
(d) Cl–(g) Cl(g) + e– (d) It is non aromatic

www.jeebooks.in
1.20 CHEMICAL BONDING
39. Correct order regarding dipole moment - 42. Example of a three - dimensional silicate is :
(a) HF < HCl < HBr < HI (a) Feldspars
(b) CH3F < CH3Cl < CH3Br < CH3I (b) Ultramarines
(c) CH3Cl > CH3F > CH3Br > CH3I (c) Beryls
(d) CH3Cl > CH2Cl2 > CHCl3 > CCl4 (d) Zeolites
40. What is the correct order of dipole moment ?
43. Which of the following statements are correct ?
(a) NH3 > NF3 (b) H2O > F2O
(a) d orbital used in hybridisation of P in PBr5(s) is
(c) XeO4 > XeO3 (d) XeO3F2 > XeO2F2
dz2
41. Correct statement regarding P4O6 & P4O10 -
(b) all possible angles in BF2Cl is 120°
O
(c) PI5(s) does not exist due to steric crowding
P P
O (d) PH5 does not exist because of no possibility of
O O O
O d orbital contraction
O P=O
P O P O P 44. Select the correct statements -
O
O O O O  – N is less than
P P (a) In HN3 , the bond angle H – N
120°
O (b) In CH2Cl2 , at least 2 bond angles are identical
(a) Both have same number of P–O–P linkage
 is exactly equal to 180°
(c) In ClF3 , FaxClF
(b) Both have same number of P=O linkage ax

(c) Both have same valancey of phosphorus (d) In Na2S4O6, the number of S-S linkage is
(d) Both have same number of P–P linkage zero

45. In XeF4 , 2 possible structures are given as follows -


F
F F F
Xe Xe
F F F
F
I II
l.p.l.p repulsion at 90° l.p.-b.p repulsion at 90° b.p-b.p repulsion at 90°
(a) Str.-I 0 7 5
(b) Str.-I 0 8 4
(c) Str.-II 0 7 4
(d) Str.-II 1 6 4

46. F F F F Correct option regarding I & II :-


(a)  F  , 
ax S Fax  I  180  Fax S Fax  II  180
S S 
(b)  F  , 
ax S Fax  I  180  Fax S Fax  II  180
O F F
F F (c)  F  
eq S Feq  I  120 ,  Feq S Feq  II  120

I II (d)  F  
eq S Feq  I  120 ,  Feq S Feq  II  120

www.jeebooks.in
CHEMICAL BONDING 1.21
47. Correct order of bond angle - 55. As per VBT, which of the following overlapping
(a) PF3 < PCl3 < PBr3 < PI3 are possible ?
(b) F2O< H2O < Cl2O (a) dxy + dxy 
x axis

(c) PH3 < PF3 (b) py + px x axis

(d) NH3 < NF3 (c) dxy + pzy  
48. Correct order of bond angle - (d) sp2 – sp2  
(a) H2O > F2O 56. Find the number of molecule in which direction of
(b) H2O > H2S > H2Se > H2Te back bonding is from central atom to surrounding
(c) SbH3 > AsH3 > PH3 > NH3 atom -

(d) N O2 > NO2 > NO2– > NO3– (a) BBr3 (b) OF 2

49. The correct statements are - (c) OCl2 (d) CCl3
(a) the reduction of bond angle in H2O is greater
57. Find out the correct statements -
than reduction of bond angle in NH3 from

(a) Si NSi bond angle in N(SiH3)3 is greater than
109°28’.
(b) in both compounds hybrid orbitals containing 
Si PSi bond angle in P(SiH ) 3 3
lone pair have p character more than 75%. (b) B–F bond dissociation energy in BF(CH3)2 is
(c) % s character of lone pair at nitrogen atom in greater than B–F bond dissociation energy in
NH3 is less than % s character of each lone BF3
pair at oxygen atom in H2O. (c) In BF3 back bonding is possible due to this B–F
(d) in NH3 & BF3 , all bonds in each molecule is bond has partial double bond character whereas
identical. in BF4– , back bonding is not possible so B–F
50. Which of the following molecules have cation and bond has 100% single bond character.
anion pair ? (d) All hybrid orbitals of O-atom in H2O lie in the
(a) PBr5(g) (b) PBr5(s) same plane.
(c) N2O5(s) (d) dry ice 58. Which of the following statements are correct ?
51. Which of the following molecules do not exist ? (a) B–F bond length in BF4– is more w.r.t. B–F
(a) CF62– (b) NeF2 bond length in BF3
(c) XeCl82– (d) XeH6 (b) Correct order of lewis acidic strength SiF4 >
52. Which of the following molecules do not exist ? SiCl4 > SiBr4 > SiI4
(a) PH5 (b) PH3 (c) Correct order of lewis acidic strength BF3 >
(c) NH3 (d) SH6 BCl3 > BBr3 > BI3
53. If y axis is the approaching axis then which set of (d) AlF3 does not show back bonding
orbitals can not form the  bond between two atom 59. Which of the following statements are correct ?
in general - (a) there is no scope of axial or equatorial position
(a) pz – pz (b) px – px in octahedral and tetrahedral geometry
(c) py – py (d) py – px (b) CF2 = C = CF2 molecule has all atoms in one
54. Which of the following overlapping is involved in plane
formation of only  bond ? (c) the bond order in NO is 2.5 and while that in
(a) s-p overlapping (b) s-s overlapping NO+ is 3
(c) p-d overlapping (d) p-p overlapping (d) X+ ion is smaller than X– ion

www.jeebooks.in
1.22 CHEMICAL BONDING
60. Correct acidic strength order : 65. Which structure shown below are correct shape ?
S O H3C
(a) < (a) N=C=S(Bent shape)
S O Methylisothiocyanate
(b) H3C–N=C=S (Linear shape)
>
(b) Methylisothiocyanate
PH3 NH3 (c) H3Si–N=C=S (Linear shape)
+ +
(c) SiH3OH (Silanol) > CH3OH (methanol) Silylisothiocyanate

OH SH (d) H3Si (Bent shape)


N=C=S
(d) Silylisothiocyante
>
66. In which of the following molcules during monomer
61. Silyl isocyanate is a compound having molecular formation, central atom has incomplete octet (less
formula H3SiNCO. Which of the following bond than 8 electrons) :-
has significant  bond character -
(a) Al2Cl6 (b) N 2O 4
(a) C–O linkage (b) N–C linkage
(c) (BeCl2)n (d) I2Cl6
(c) Si–N linkage (d) Si–H linkage
67. Which molecule under following condition does
62. Which of the following statements are correct ? not have bridging bond :-
(a) In back bonding of CCl3 , e– movement is from (a) SO3 during trimer formation
surrounding atom (Cl) to central atom carbon. (b) P2O5 during dimer formation
(b) In back bonding of :CCl2, e– movement is from (c) BeCl2 during polymer formation
surrounding atom (Cl) to central atom carbon.
(d) Cl2C = CCl2 during polymer formation
(c) When boric acid is dissolved in water, back
bonding is going to be lost in product side. 68. Which molecule has 3c–4e bond ?

(d) CH 2 – CH 3 (ethyl carbocation) is more stable (a) Al2Br6 (b) (BeH2)n
 (c) I2Cl6 (d) Fe2Cl6
than CH 2 – OCH 3 (methoxy methyl 69. Reaction of diborane with NMe3 in product
carbocation) formation :-
63. I. O(CH3)2 + BF3  (a) Hybridisation of B does not change
II. O(SiH3)2+BF3  (b) Hybridisation of N does not change
III. NMe3 + HCl  (c) Hybridisation of B changes
IV. N(SiH3)3 + HCl  (d) Hybridisation of N changes
Correct statement :- 70. Which molecules are electron defficient ?
(a) Reaction I is faster than reaction II (a) SO3 (b) SOCl2
(b) Reaction II is faster than reaction I (c) B2H6 (d) NH3
(c) Reaction III is faster than reaction IV 71. Which of the following reactions products formed
(d) Reaction IV is faster than reaction III are correctly written :-
64. In which of the following compound, bond angle (a) BCl3 + H2O B3+(aq) + 3Cl–(aq)
increases due to back bonding ? (b) POCl3 + 3H2O H3PO4 + 3HCl
(a) BI3 (b) N(CH3)3 (c) SF4 + H2O H2SO3 + HF
(c) N(SiH3)3 (d) H3Si–N=C=S (d) NaCl + H2O Na+(aq) + Cl–(aq)

www.jeebooks.in
CHEMICAL BONDING 1.23
72. Which is the correct order of hydrolysis rate :- 77. Correct number of –OH groups present in product
(a) NCl3 > NF3 when following compound reacts with water?
(b) SF6 > SeF6 > TeF6 (a) PCl3  Product
3OHgroup
(c) CCl4 > SiCl4 > GeCl4
(b) NCl3  Product
1OHgroup
(d) PCl3 > AsCl3 > SbCl3 > BiCl3
r.d.s r.d.s (c) TeF6  Product
73. A – X   A+ + X– A+ + Y–   6OHgroups
A – Y (Y = OH) .........(I):
(d) PCl5  Product
5OHgroups
Y– + A – X Y – A + X– .......(II) Transition
  78. Which molecules can form dimer and dimer is
state Y ...f..... A .....b...... X .
planar in nature ?
(a) I is called dissociative process
(a) BBr3 (b) AlCl3
(b) II is called associative process
(c) BeH2 (d) BeCl2
(c) I is SN1 mechanism
79. Which molecules undergo restricted rotation ?
(d) II is SN2 mechanism
(a) B2H6 (b) C2H6
74. Which xenon fluorides when undergo hydrolysis ,
(c) C2H4 (d) C4H10
O2 gas is evolved :-
80. Which of the following are correct :-
(a) XeF2 (b) XeF4
(a) NO2 at –11°C can form dimer and dimer form
(c) XeF6 (d) All
is diamagnetic
75. Which of the following oxyacids are formed during
(b) Cl2O6(s) ionises to give Cl2– and ClO4–
stepwise hydrolysis of P4O10 :-
(c) ClO2 can not form dimer because odd electron
(a) Tetrameta phosphoric acid
is delocalised
(b) Tetrapoly phosphoric acid
(d) (P–O) bond length in P4O6 > (P–O) bond
(c) Pyro phosphoric acid length in P4O10
(d) Phosphoric acid 81. Correct statements :-
76. SOCl2 when undergoes hydrolysis which of the (a) NO2 is paramagnetic and has tendency to form
following statements are correct :- dimer
(a) The oxidation state of central atom in oxyacid (b) ClO2 is paramagnetic and has no tendency to
formed is +4 form dimer
(b) The oxyacid formed is example of dibasic acid (c) ClO2 is diamagnetic and has no tendency to
(c) The hydrolysis mechanism is example of SNAE form dimer
mechanism (d) NO2 is brown colour gas but its dimer N2O4 is
(d) During hydrolysis, H2O acts as electrophile and solid in nature
SOCl2 acts as nucleophile
– –
82. HO2C C O 2H K 1 O2 C CO2H K2 O2C CO2–
C=C C=C C=C
H H H H H H
K1 and K2 are Ist and IInd dissociation constant of maleic acid :-

H CO2H K1 H C O2 H K 2 H CO 2
C=C – C=C – C=C
HO2C H O 2C H O2 C H
’ ’
K1 and K are Ist & IInd dissociation constant of fumaric acid :-
2
(a) (k1)maleic acid > (k1)fumaric acid (b) (k1)maleic acid < (k1)fumaric acid
(c) (k2)maleic acid > (k2)fumaric acid (d) (k2)maleic acid < (k2)fumaric acid

www.jeebooks.in
1.24 CHEMICAL BONDING
83. Select correct statement :- CO2H P4O10
(a) If molecule has any polar bond then it is always 89. H2C  X
polar
CO2H
Correct statements regarding X :-
(b) Solubility of noble gas increases in water down
the group (a) X is linear in shape
(b) X has 3 and 3 bond
(c) London dispersion forces also contribute in net
interaction between nonpolar molecule (c) all carbons in X have same hybridisation state
(d) Molecular interaction between CF4 molecule (d) X is CH3CO2H
is higher as compared to molecular interaction 90. Which of the compounds do have H-bonding ?
between NF3 molecule (a) NH3 (b) CH3OCH3
84. Which compounds undergo hydrolysis at very high (c) CH3CH2OH (d) CH3NHCH3
temperature through SN1 ? 91. For which compounds, intramolecular hydrogen
(a) SF6 (b) NF 3 bonding is possible :-
(c) CCl4 (d) SF4 (a) o-nitrophenol
85. Which reactions product are correctly written ? (b) p-nitrophenol
O
O
(a) B2H6 + H3BO O
OH
C
O (c) C

(b) B2H6 + R3N  H 3B  NH 3 C OH

(c) B2H6 + NH3  H 3B  NH 3 O
(d) All (d) HNO3
86. Which of the following compound on heating give 92. H-bonding is responsible for which of the following
NO2 gas ? phenomenon :-
(a) AgNO3 (b) LiNO3 (a) Acetylene is dissolved in acetone
(c) Be(NO3)2 (d) KNO 3 (b) DDT is soluble in water
87. Which of the following statements are correct ? (c) Formic acid is more acidic than acetic acid
(a) OF during dimer formation planarity is lost (d) H3PO4 is more viscous w.r.t. Me3PO4
(b) Pentasulphide S52– is bent in shape 93. Which of the following are correct statements ?
(c) Chloral CCl3CH = O forms stable hydrate (a) the direction of dipole moment in CO is from
because of H bonding. The structure of chloral O to C
H (b) white phosphorus is less stable than red
Cl O
phosphorus
hydrate is given as follows Cl – C – CH . (c) H2 molecule is more stable than He–H molecule
Cl O (d) b & * orbitals obtained from 2p orbital are
H lying in the same plane
(d) In S2F10, all S–F bond lengths are identical. 94. Which of the following order are correct regarding
88. Which are correct representation of boranes ? electronegativity of elements ?
(a) BnHn+4 (b) BnHn+6 (a) P > S (b) P > N
(c) BnH2n+2 (d) None (c) Cl > S (d) Cl > Br

www.jeebooks.in
CHEMICAL BONDING 1.25
95. Correct order of stability :- (c) s-p mixing is operative and Hund’s rule is
(a) PbCl2 > PbCl4 (b) CCl2 > PbCl2 followed
(c) SnCl2 > SnCl4 (d) PbCl4 > PbI4 (d) None
96. All the given molecule have triangular faces. Find 101. Which of the following molecule / ions do not
molecules in which all the triangular faces are contain unpaired electron ?
equilateral triangular faces :- (a) N2+ (b) O 2
2–
(a) CHCl3 (b) PCl5 (c) O 2 (d) Br2
(c) CH4 (d) SF6 102. Which of the following overlapping(s) result
97. During linear combination of atomic orbitals to form antibonding molecular orbital with two nodal plane?
molecular orbitals, which of the following
conditions must be satisfied :- +
(a)
(a) the combining atomic orbitals must have the
same or nearly the same energy
(b) the combining atomic orbitals must have the
same symmetry about the molecular axis (b) +
(c) the combining atomic orbitals must overlap to
the maximum extent
(d) the combining atomic orbitals must overlap to (c) +
the minimum extent
98. For which of the following ion , correct resonating
structures are given :- (d) +
..


(a) S
.. – C  S
.. = C = 
..

..

..

.. 103. Which of the following process is not correct


 
(b) N =C= NC–N 2– against the mentioned property ?
O O

O
+
O
– (a) N2+  N2 (magnetic moment increases)
(c) S S (b) O2  O2+ (bond order decreases)

O O –
O O– (c) Li2+  Li2– (magnetic property changes)
+ +
O O (d) C2  C2+ (electron is removed from bonding
(d) – – orbital)
O O O O
99. Select incorrect statement :- 104. Which of the following options are incorrect ?
(a) In general no S–H bond is present in oxy acid (a) dimer of NO2 contains N–O–N linkage
of sulphur (b) all diatomic halogens are coloured due to
(b) Oxyacids of P containing P–H bonds are HOMO-LUMO electron transition
reducing in nature (c) all diatomic halogens are coloured due to
(c) Both O2 and S2 are diamagnetic in nature LUMO-HOMOelectron transition
(d) General reaction of O2 with many metals and (d) dimer of NO2 does not contain N–O–N linkage
non metals are endothermic in nature because 105. The correct statements are :-
energy is required to break double bond (a) B2 solid does not exist
present in O2. (b) B2 have basic building B12 icosahedral
100. B2 molecule will be diamagnetic when :- (c) B12 icosahedral is made up of polyhedron
(a) s-p mixing is not operative having 20 faces and 12 corners
(b) s-p mixing is operative and Hund’s rule is (d) B12 icosahedral is made up of polyhedron
violated having 10 faces and 5 corners

www.jeebooks.in
1.26 CHEMICAL BONDING
106. Which of the following compounds when undergo 114. Which of the following order are correct ?
reaction with water give methane gas ? (a) CHF > CHCl > CHBr (stability)
(a) Be2C (b) CaC2 (b) CHF > CHCl > CHBr (extent of back bonding)
(c) Mg2C3 (d) Al4C3 (c) CF2 > CCl2 > CBr2 (extent of back bonding)
107. Which of the following compounds when undergo (d) CF2 > CCl2 > CBr2 (lewis acidity)
reaction with water give terminal alkyne ? 115. O O
(a) Li2C2 (b) CaC2  
(c) Al4C3 (d) Mg2C3 H3Si SiH3 H3C CH3
A B
108. Which compounds when react with water , it gives Correct option is :-
ammonia as one of the product ? (a)  > 
(a) CaCN2 (b) AlN (b) A is more lewis basic than B
(c) Ca3N2 (d) Li3N (c)  > 
109. Which compounds consume six moles of water (d) B is more lewis basic than A
per one mole of compound ?
116. Which of the following compounds on heating give
(a) Ca3N2 (b) P 4O 6 O2 gas ?
(c) BrF5 (d) SiF4 (a) BeC2O4 (b) MgCO3
110. Which of the following statements are incorrect ? (c) NaNO 3 (d) H2O2
(a) maximum covalency of nitrogen is 3 117. Out of BeCl2 , CaCl2 , CsCl, KCl :-
(b) in glyoxal, carbon atom is sp2 hybridised
(a) highest covalent character is observed in BeCl2
(c) size of d orbital decreases : Si > P > S > Cl
(b) highest covalent character is observed in CaCl2
(d) maximum covalency of Sulphur is 4
(c) Least covalent character is observed in KCl
111. How many statements are correct :-
(d) Least covalent character is observed in CsCl
(a) In PCl5 ; axial P–Cl bond length > equatorial
118. In Cr2O72– (dichromate ion) :-
P–Cl bond length
(a) 8Cr–O bond lengths are identical
(b) In PF4Cl ; axial P–F bond length < equatorial
P–Cl bond length (b) 6Cr–O bond lengths are identical
(c) PCl4F has more dipole moment than PCl3F2 (c) Cr is in +6 oxidation state
(d) PF3Cl2 has more dipole moment than PCl5 (d) Cr is sp3 hybridised
112. Correct statement :- 119. In which all hydrogens are not of same acidic
character :-
(a) Solid state PCl5 exists as PCl4+ and PCl6–
(a) H3PO5 (b) H2S2O5
(b) Solid state PBr5 exists as PBr4+ and Br–
(c) H2S2O8 (d) H3PO4
(c) Solid state N2O4 exists as NO+ and NO3–
120. The correct order of boiling point :-
(d) Solid state N2O5 exists as NO2+ and NO3–
(a) C2H6 > C2F6 (b) NH3 > NF3
113. Which of the following statement are incorrect for
PO43– ? (c) C10H22 > C10F22 (d) CCl4 > SiCl4
(a) Numberof identical resonating structures are 3 121. Which of the following order are correct regarding
(b) Bond order of P–O bond is 1.25 solubility in water for following compounds ?
(c) No. of identical P – O bonds are 3 (a) KCl > AgCl (b) CaCl2 > ZnCl2
(d) Maximum no. of atoms are in one plane is 4 (c) BeSO4 > BaSO4 (d) LiI > RbCl

www.jeebooks.in
CHEMICAL BONDING 1.27
122. Which of the following order are correct regarding (c) during loss of 1 electron , bond order of the
size of cation or anion ? product formed becomes 3
(a) S2– > O2– (b) N3– > P3– (d) its bond order is 2.5
(c) K+ > Rb+ (d) Ba2+ > Be2+ 129. Choose the incorrect statement from the following:-
123. Which of the following order are incorrect (a) HOMO of N2 molecule is bM.O.
regarding tendency towards hydrolysis ? (b) LUMO of N2 molecule is M.O.
(a) CCl4 > SiCl4 (c) HOMO of N2 molecule is pM.O.
(b) SF6 > SeF6 (d) Halogens are colourless
(c) BiCl3 > NCl3 130. Choose correct statement regarding N2 N2+ + e :-
(d) (CH3)3CCl > (CH3)2CHCl (a) process is endothermic
124. Which of the following order are correct regarding
(b) bond order decreases
stability of compounds of different metal / element
in same oxidation state ? (c) electron removed from BMO
(a) B in univalent state > Tl in univalent state (d) electron removed from gerade MO
(b) Carbon in divalent state > Pb in divalent state 131. Which of the following pair of species are
isostructural but not isoelectronic ?
(c) Carbon in tetravalent state > Pb in tetravalent
state (a) O3 & NO2– (b) NF3 & NH3
(d) B in trivalent state > Tl in trivalent state (c) CH4 & CCl4 (d) N2O & NO2+
125. In peroxy disulphuric acid H2S2O8, which of the 132. Molecule in which any hybrid orbital(s) of central
following options are correct :- atom which form bond contains s-character less
(a) central atom is sp3 hybridised than 25% :-
(b) –O–O– linkage is present (a) PCl5 (b) H2O
(c) S has +7 oxidation state (c) NH3 (d) NO 3 –
(d) this reagent is used in ELBS oxidation reaction 133. Which of the following statement(s) are correct
about Al2(CH3)6 ?
126. Choose the correct statement regarding ionisation
energy :- (a) number of 2c–2e bonds are 22
(a) molecular O2 < atomic O (b) molecule is non polar
(b) atomic N < molecular N2 (c) maximum number of atoms may lie in one plane
(c) N2+ < N2 is 10
(d) None (d) total number of 2 vacant orbitals are involved
in hybridisation of central atom
127. Regarding colour which of the following options
are correct :- 134. Which of the following statement are correct for
(a) O2 - colourless (b) Se2 - red F – Xe – O F
S2 - pale yellow Te2 - black XeFOSO2F ? S
(c) I2(g)- violet (d) NO - colourless O O
Br2(g)- reddish brown NO2 - reddish brown (a) the number of sp3 hybridised central atom is 3
128. Regarding NO which of following options are (b) the number of identical S–O bond is 2
correct :-
(c) maximum number of atoms may lie in one plane
(a) it is diamagnetic in gaseous state is 5
(b) it combines with O2 to give paramagnetic NO2 (d) all atoms are lying in the same plane
gas

www.jeebooks.in
1.28 CHEMICAL BONDING
135. Which of the following are incorrect order against 139. If polarising power is in the order of Ma+ > Mb+ >
indicated properties ? M c + and polarisability is in the order of
(a) AgF > AgCl > AgI : covalent character X– > Y– > Z– then select which have more covalent
(b) NaHCO3 > KHCO3 > RbHCO3 : solubility in character as compared to Mb+Y– :-
water (a) Ma+Y– (b) Mb+X–
(c) NaF < MgF2 < AlF3 : melting point (c) Ma+X– (d) Mb+X–
(d) MgC2O4 > CaC2O4 > BaC2O4 : solubility in 140. Which of the following statements are incorrect ?
water
(a) P–F bond length is longer than that of P–Cl
136. Which of the following statements are correct ? bond length in PF2Cl3
(a) compounds of Hg2+ ions having an ionic radius
(b) F atom ocupy in equatorial position of T.b.p
of 116 pm are more covalent in character than
structure of PF2Cl3
those of Ca2+ ion with almost identical size (114
pm) and same charge (c) In PF3Cl2 all axial and all equatorial bond length
(b) ethers behave as bases in the presence of are identical
mineral acids (d) Fluorine atoms prefers to attach at the axial
(c) carbon has unique ability to form p-p position than that of equatorial position of
multiple bonds with itself and with other atoms PF2Cl3
of small size and high electronegativity. 141. Which of the following options are incorrect ?
F (a) Salicyaldehyde has a higher melting point as
 compared to its positional isomer because of
(d) Cl F bond angle = bond angle but intra molecular hydrogen bonding

(b) Ionic forces are directional in nature
F
not precisely 90°. (c) In a crystals of I2 , the forces of attraction will
137. From the following options which are correct for be covalent
double chain silicate :- (d) crystalline boric acid forms two dimensional
(a) average charge on each tetrahedral = –1.5 sheet with almost hexagonal symmetry
(b) all the tetrahedron have one unshared oxygen 142. Which of the following statement is incorrect ?
/ corner (a) the number of bonding electrons in N2 and N2–
(c) all the silicon atoms have +4 oxidation state is same
(d) average shared oxygen / corner per (b) the bond order and bond length of N2+ and
tetrahedron = 2.5 N2– is same
138. Which of the following options are correct ? (c) among O2– , O22– , O22+ ; O22+ has longest bond
(a) in alkaline medium, the hydrolysis of BeCl2 length
produces clear solution consisting of (d) bond order (O–O) O22+ > O2+1 > O2 > O2–1 >
[Be(OH)4]2– and HCl. O 2 2–
(b) in the clear solution of BiCl3 when large quantity 143. Which of the following statement are correct :-
of water is added, the white turbidity of BiOCl
(a) the C–Cl distance in CH3Cl and CF3Cl are
is obtained
1.78Å and 1.75Å respectively
(c) SiF4 undergoes partial hydrolysis
(b) the C–C single bond distance in methyl
(d) the final hydrolysis products of PCl3 and POCl3 acetylene H3C – C  CH is only 1.46Å
are identical
compared to 1.54 Å is CH3–CH3

www.jeebooks.in
CHEMICAL BONDING 1.29
O 147. Which of the following molecules are nonplanar ?
(a) NH3
(c) H3C P OH is less acidic than (b) N(CH3)3 [trimethyl amine]
CH3 (c) N(SiH3)3 (trisilylamine)
O (d) P(SiH3)3
148. Select the correct statement(s) :-
P OH (a) in ClO4– , all Cl – O bonds are identical and
there is strong p-d bonding between
(d) SF6 sublimes at –64°C where graphite sublimes chlorine and oxygen atom
at 3700°C (b) in P4S3 molecules there are a six P – S bonds ,
144. Incorrect statement - three P–P bonds and ten lone pairs of electrons
(a) POF3 and NOF3 both have coordinate bond (c) N2H4 is pyramidal about each N-atom
(b) POF3 and NOF3 does not have coordinate (d) Br2 and ICl have same reactivity and same
bond boiling point
(c) POF3 has coordinate bond but NOF3 does not 149. Which of the following are correct ?
have coordinate bond (a) thio-ether have less boiling point than ether
(d) POF3 does not have coordinate bond but NOF3 (b) thiol have more boiling point than alcohol
has coordinate bond (c) the maximum possible number of hydrogen
145. Select the correct statements for sulphuric acid :- bonds a water molecule can form is 4
(a) it has high boiling point and viscocity (d) Al2S3 when undergoes reaction with water the
(b) there are two types of bond lengths in its products formed are Al(OH)3 and H2S
bivalent anion 150. Which are correct statements ?
(c) p-dbonding between sulphur and oxygen (a) Borazine has less intermolecular force of
is observed attraction as compared to benzene
(d) sulphur has the same hybridisation as that of (b) D2O has less boiling point than H2O
boron in diborane (c) CuI2 is unstable even at ordinary temperature
146. Which of the following statement are correct ? (d) NaClO4 is about 1000 times as soluble as
(a) the percentage of s character in the orbital KClO4 in water
forming S–S bond and P–P bonds in S8 and P4 EXERCISE # III
molecules respectively are same
 Linked Comprehension Type :
(b) aqueous H3PO4 is syrupy
Passage Type:
(c) SiO2 crystal may be considered as giant Passage for Q.1 to Q.3
molecule in which eight - membered rings are XeF6 undergo hydrolysis in 3 stages giving different
formed with alternate silicon and oxygen products in each stage. Here H2O acts as nucleophile
atoms. and XeF6 acts as electrophile. The reaction is as
(d) In SF4 the bond angles instead of being 90° follows :-
and 180° are 89° and 177° respectively due XeF6 + H2O P1 + 2HF
to the repulsion between lone pair and bond P1 + H2O P2 + 2HF
pairs of electrons P2 + H2O P3 + 2HF

www.jeebooks.in
1.30 CHEMICAL BONDING
1. The hybridisation of Xe atom in P1 :- 5. Correct geometry and polar character of AB7 :-
(a) sp3 (b) sp3d (a) AB7 : octahedral ; polar
3 2 3 3
(c) sp d (d) sp d (b) AB7 : octahedral ; nonpolar
2. The hybridisation of Xe atom in P2:- (c) AB7 : pentagonal bipyramidal ; polar
2 3
(a) sp (b) sp (d) AB7 : pentagonal bipyramidal ; non polar
3 3 2
(c) sp d (d) sp d 6. Correct shape geometry and polar character of AB3-
3. The hybridisation of Xe atom in P3 :- (a) AB3 : bent ; nonpolar
2 3
(a) sp (b) sp (b) AB3 : Trigonal planar ; nonpolar
3 3 2
(c) sp d (d) sp d (c) AB3 : T shaped ; nonpolar
Passage for Q.4 to Q.6 (d) AB3 : T shaped ; polar
Interhalogen compound of the type AmBn are possible. Passage for Q.7 to Q.8
It can have formula AB ; AB3 ; AB5 ; AB7 where A is BeF2  F
A  F
B.
(1 mole) (1 mole)
halogen of larger size and B is halogen of smaller size 1 mole

and B is more electronegative than A. As the ratio 7. In above reaction, hybridisation of central atom in
between radii of A,B increases, the number of atoms species A is -
per molecule increases- (a) sp (b) sp2
4. Correct geometry and polar character of AB :- (c) sp3 (d) sp3d
(a) AB : linear ; polar 8. In above reaction, hybridisation of central atom in
(b) AB : linear ; nonpolar species B is -
(c) AB : bent ; polar (a) sp (b) sp2
(d) AB : bent ; nonpolar (c) sp3 (d) sp3d
Passage for Q.9 to Q.11
When the central atom bears both the bond pairs & lone pairs the structure are devided from regular geometries
produced from rule-1 of VSEPR. (Valence shell electron pair repulsion) theory -
9. In ClF3 , 3-possible structures given as follows -
F F F

Cl Cl Cl
F F F F
F F
I II III
l.p.l.p repulsion at 90° l.p.-b.p repulsion at 90° b.p.-b.p. repulsion at 90°
(a) Str.-I 0 4 2
(b) Str.-II 1 3 2
(c) Str.-III 0 6 0
(d) All are correct
10. In ICl2– , 3 possible structures given as follows -
–1 –1 –1
Cl Cl Cl

I I I
Cl Cl
Cl
I II III

www.jeebooks.in
CHEMICAL BONDING 1.31
l.p.-l.p repulsion at 90° l.p.-b.p repulsion at 90° b.p.-b.p.repulsion at 90°
(a) Str.-I 0 6 0
(b) Str.-II 2 4 0
(c) Str.-III 2 3 1
(d) All are correct
11. In SF4 , 2 possible structures given as follows -
F F F

S S
F F
F
F F
I II
l.p.-l.p repulsion at 90° l.p.-b.p repulsion at 90° b.p.-b.p.repulsion at 90°
(a) Str.-I 0 3 3
(b) Str.-I 0 2 4
(c) Str.-II 0 2 4
(d) Str.-II 0 3 3
Passage for Q.12 to Q.14 15. Hemimorphite having formula Zn4(OH)2 . Si2O7 .
2– H2O. It is example of :-
Cl Cl Cl Cl Cl (a) Orthosilicate
2Cs
+ Be Be Be Be
Cl Cl Cl Cl Cl (b) Pyrosilicate
(c) single chain silicate
A segment of polymeric BeCl2 can be given as follows. (d) double chain silicate
12. What is maximum number of atoms present in 16. In Beryl Be2Al2 (Si6O18) , the number of oxygen
same plane ?
shared in every tetrahedral :-
(a) 6 (b) 8
(a) 2 (b) 3
(c) 10 (d) 12
(c) 6 (d) 4
13. What is the number of 2c–2e bonds present ?
17. Kaolin, an important clay mineral having molecular
(a) 12 (b) 4 formula Al2(OH)4(Si2O5) is example of :-
(c) 8 (d) 6
(a) Single chain silicate
14. What is the number of 3c–2e bonds present ?
(b) 3-D-silicate
(a) 12 (b) 4
(c) 2D/Sheet silicate
(c) 8 (d) 6
(d) Double chain silicate
Passage for Q.15 to Q.17
Passage for Q.18 to Q.20
Silicate may be regarded as the metal derivative of
100 C 200 C
silicic acid H4SiO4. Silicate can be prepared by fuming 2H5IO6 
4H 2O
 2P1   P2  H 2O  O 2
paraperiodic acid
metal oxide or metal carbonate with sand.
SiO2 + Na2CO3  fused
 Na2SiO4 , (Na2SiO3)n etc. 18. The structure of P1 is :-
Silicate are classified according to the nature of linking (a) trigonal (b) tetrahedral
between the tetrahedral SiO44– anions. (c) octahedral (d) bent

www.jeebooks.in
1.32 CHEMICAL BONDING
19. Hybridisation of iodine atom in P2 :- 26. Which of the following compound can form
(a) sp2 (b) sp3 hydrogen bonding with each other :-
(c) sp (d) sp3d (a) NH4Cl + Water
20. The shape of paraperiodic acid is :- (b) KF + HF
(a) tetrahedral (b) pentagonal bipyramidal (c) CH3OCH3 + Water
(c) octahedral (d) angular (d) All
Passage for Q.21 to Q.23 Passage for Q.27 to Q.29
Powdered borax Na2B4O7 . 10H2O is used in borax – N=N + N=N  product (A)
bead test to identify different metal ions. For this , it is H H H H
taken in a Pt wire, heated, the salt swells up and shrinks N2 + product (B) (N2H4)
upon the loop forming a colourless transparent glass - 27. What kind of hybridisation change is observed for
like bead. N atom from reactant to product (A):-
21. Correct statement about borax :- (a) sp2  sp3 (b) sp3  sp
(a) each boron atom has one –OH group (c) sp  sp2 (d) sp2  sp
(b) each boron atom has two –OH group 28. What kind of hybridisation change is observed for
(c) each boron atom has three –OH group N atom from reactant to product (B) :-
(d) each boron atom has four –OH group (a) sp2  sp3 (b) sp3  sp
22. Borax is actually made of :- (c) sp  sp2 (d) sp2  sp
(a) two tetrahedra and one triangular unit 29. This reaction is example of :-
(b) one tetrahedra and one triangular unit (a) non-redox reaction
(c) two tetrahedra and two triangular unit (b) disproportionation reaction
(d) None (c) comproportionation reaction
23. Correct statement regarding borax :- (d) acid base reaction
(a) each boron atom has four B–O bonds Passage for Q.30 to Q.31
(b) each boron atom has three B–O bonds Polystyrene is a polymer which is used as insulator ,
(c) two boron atoms has four B–O bonds while manufacture of toys , radio, television cabinet. Its
other two have three B–O bonds monomeric unit is styrene (CH 2 = CHPh).
Polyacrylonitrile is another polymer which is ued as
(d) none
substitute for wool in making commercial fibres. Its
Passage for Q.24 to Q.26
momomeric unit is acrylonitrile. (CH2= CH–CN)
The hydrogen bond is generally represented as A–
30. The hybridisation of carbon in polystyrene :-
H.......B. Where A is sufficiently electronegative element
to cause acidic nature on the hydrogen and B has (a) sp2 and sp
sufficient electron density to act as a base to this (b) sp and sp3
hydrogen. (c) sp3 and sp2
24. Which hydrogen bonding is strongest :- (d) only sp2
(a) F–H......N (b) N–H......O 31. The hybridisation of carbon in Polyacrylonitrile :-
(c) F–H......O (d) F–H......F (a) sp2 and sp
25. Hydrogen bonding takes place in :- (b) sp and sp3
(a) KHF2 (b) KH2PO4 (c) sp3 and sp2
(c) KH2PO2 (d) both (a) and (b) (d) only sp2

www.jeebooks.in
CHEMICAL BONDING 1.33
Passage for Q.32 to Q.33 the amplitude of the electron waves. These are obtained
Ammonia molecule undergoes a type of motion known from the solution of Schrodinger wave equation.
as inversion ; The N atom oscillate throgh the plane of Molecular orbitals which are one electron wave
the three H atoms much as an umbrella turns inside functions for molecules are difficult to obtain directly
out. This is called umbrella inversion. from the solution of Schrodinger wave equation. To
.. overcome this problem, an approximate method known
N H as linear combination of atomic orbitals (LCAO) has
H H H H been adopted.
H N
.. 36. If the atomic orbitals of these atoms are
represented by the wave functions A and A ,
32. During umbrella inversion the hybridisation of N in then which of the following options is correct
intermediate stage :- regarding bonding molecular orbital :-
(a) sp (b) sp2
(a) bonding molecular orbitals are formed as
(c) sp3 (d) sp3d = A + B
33. The potential energy barrier for inversion is :- (b) antibonding molecular orbitals are formed as
(a) 6 KJ mole–1 = A – B
(b) 12 KJ mole–1 (c) both (a) and (b)
(c) 18 KJ mole–1 (d) None
(d) 24 KJ mole–1 37. Which of the following option is correct regarding
Passage for Q.34 to Q.35 bonding molecular orbital ?
Because of very high electronegativity of F(4.0) , it (a) In a bonding molecular orbital, electron density
has the tendency to form hydrogen bonds. The liquid is located between the nuclei of the bonded
acid and its vapour at low temperature consists of atoms
polymeric chain (HF)x. The association of HF molecule (b) A bonding molecular orbital always possesses
is attributed to hydrogen bonding. lower energy than either of the atomic orbital
F  F (c) In the formation of the bonding molecular
H H F H H F H
F orbital, the two electron waves of bonding

atoms reinforce each other due to constructive
34.  and are respectively :-
interference.
(a) 1.22°A ; 120°
(d) All
(b) 0.77°A ; 70°
38. Which of the following option is correct regarding
(c) 2.55°A ; 140° anitbonding molecular orbital ?
(d) 3.66°A ; 210° (a) In a antibonding molecular orbital, most of the
35. Which of the following option is correct regarding electron density is located away from the space
HF ? between the nuclei.
(a) HF has highest boiling point among all HX (b) An antibonding molecular orbital always
(b) HF is most polar among all HX possesses higher energy than either of the atomic
(c) HF is least acidic among all HX orbital
(d) All (c) In the formation of the antibonding molecular
orbital, the two electron waves cancel each
Passage for Q.36 to Q.38
other due to destructive interference.
According to wave mechanics, the atomic orbitals can
(d) All
be expressed by wave functions (’s) which represent

www.jeebooks.in
1.34 CHEMICAL BONDING
Passage for Q.39 to Q.41 44. HF is :-
Type of molecule Example Dipole moment (a) paramagnetic
µ(D) (b) diamagnetic
AB2 H2O x1
(c) can not be predicted
H2S y1
(d) base
CO 2 z1
AB3 NH3 x2 Passage for Q.45 to Q.47
NF 3 y2 
369K
-sulphur   -sulphur
369K
BF3 z2
45. -sulphur is :-
AB4 CH4 x3
(a) rhombic sulphur (b) monoclinic sulphur
CHCl3 y3
CCl4 z3 (c) plastic sulphur (d) colloidal sulphur
39. Correct option is :- 46. -sulphur is :-
(a) x1 > y1 > z1 (b) x1 < y1 < z1 (a) monoclinic sulphur
(c) x1 = y1 = z1 (d) x1 > z1 > y1 (b) rhombic sulphur
40. Correct option is :- (c) plastic sulphur
(a) x2 > y2 > z2 (b) y3 > x3 = z3 (d) colloidal sulphur
(c) x2 = y2 = z2 (d) both (a) & (b) 47. Correct statements are :-
41. Which one has zero dipole moment ? (a) both and variety are soluble in CS2
(a) CO 2 (b) BF3 (b) both and variety are insoluble in water
(c) CCl4 , CH4 (d) All (c) variety is lemon yellow and variety is
Passage for Q.42 to Q.44 needle like transparent crystal
M.O. diagram of HF molecule is given below : (d) all
*
 Passage for Q.48 to Q.50
H 1s
AO
Interhalogen compounds are represented as XX’,
XX’3 , XX’5 , XX’7
when X’ is more electronegative element
X is less electronegative element
nb
F when they undergo hydrolysis a mixture of hydraacid
2p and oxyacid is formed.
nb XX’ + n1H2O  P1+P2
b 2s
F XX’3 + n2H2O  P1+P3
AO
XX’5 + n3H2O  P1+P4
42. HOMO of HF is :- (HOMO = highest occupied
molecular orbital) XX’7 + n4H2O  P1+P5
48. n1 , n2 , n3 , n4 are respectively
(a)  * (b)  b
(a) 2,3,4,5 (b) 0,1,2,3
(c)  nb (d)  nb
(c) 1,2,3,4 (d) 0,2,4,6
43. LUMO of HF is :- (LUMO = lowest unoccupied
molecular orbital) 49. The common product P1 is :-
(a)  * (b)  b (a) HOX’ (b) HX’

(c)  nb (d)  nb (c) HOX (d) HX

www.jeebooks.in
CHEMICAL BONDING 1.35
50. P2 , P3 , P4 , P5 are respectively :- 56. Correct order regarding property between
(a) XO0(OH) ; XO1(OH) ; XO2(OH) ; XO3(OH) diamond and graphite :-
(b) X’O 0 (OH) ; X’O 1 (OH) ; X’O 2 (OH) ; (a) thermal conductivity : diamond > graphite
X’O3(OH) (b) electrical conductivity : diamond < graphite
(c) HOX ; HOX2 ; HOX3 ; HOX4 (c) C–C bond length : diamond > graphite
(d) HOX’ ; HOX2’ ; HOX3’ ; HOX4’ (d) All
Passage for Q.51 to Q.53 Passage for Q.57 to Q.59
Marshal’s acid + H2O X (acid) + Y (acid) Nitrogen forms p-p multiple bonds, nitrogen exists
Y (acid) + H2O X (acid) + Z (unstable compound) as triply bonded diatomic gaseous molecule. Bond
51. The product X, Y are respectively :- strength is very high which is responsible for inertness
(a) H2SO3 ; H2SO4 (b) H2SO4 ; H2SO5 at ordinary conditions, while other members of nitrogen
family form d-p bonding :-
(c) H2SO3 ; H2SO5 (d) H2SO5 ; H2SO3
57. Thermal and electrical conductivity is highest
52. Unstable compound Z is :-
(a) N (b) P
(a) H2O (b) H2
(c) As (d) Bi
(c) O 2 (d) H2O2
58. Which is most acidic in nature ?
53. Which compound has peroxide linkage ?
(a) Bi2O3 (b) Bi2O4
(a) X,Y,Z all (b) X, Z both
(c) Sb2O3 (d) Bi2O5
(c) X, Y both (d) Y, Z both
59. Maximum covalency of Sb will be :-
Passage for Q.54 to Q.56
Carbon exists in two allotropic forms : (i) crystalline (a) 0 (b) 2
form, (ii) amorphous form . The crystalline form of the (c) 4 (d) 6
allotropes of carbon includes (a) Diamond, (b) Passage for Q.60 to Q.62
Graphite. The amorphous form of allotropes of carbon Chemical bonding between two atoms is necessarily
include (a) Charcoal (b) Lampblack (c) Coke (d) Gas associated with an electrical moment arising out of the
carbon. difference in electronegativity of two atoms. This means
54. Which of the following options are correct that every bon carries with it an electrical moment called
regarding Diamond ? to bond moment’. To compute the dipole moment it is
(a) Diamond is chemically inert, it is not attacked necessary to find out the values of various bond
by acids and alkalies. moment. in the following table dipole moment of
(b) It is the hardest substance, specific gravity is 3.5 different bonds are as given.
  
(c) It has carbon with sp3 hybridisation Bond H—C C — Cl CO
(d) All
Bond moments 0.4D 1.5D 2.5D
55. Which of the following options are correct
The group moments of few group as given :
regarding Graphite ?
(a) Graphite is used as dry lubricant in machine in Group NO2 OH CN CH3
place of oil Direction toward toward toward away
(b) It is a good conductor of heat and electricity of dipole N O N from
CH3
(c) It is soft and slippery, specific gravity is 2.2
Dipole 4D 1.6D 3.8D 0.4D
(d) All moment

www.jeebooks.in
1.36 CHEMICAL BONDING
60. The bond angle in H2S is 97° and its dipole moment 63. Number of carbon atoms with sp3 hybridisation
is 1.5D. The S–H bond distance is (a) 2 (b) 4
0.15 nm. Therefore approximate percentage ionic (c) 6 (d) 8
character of S–H bond is (neglect the effect of
64. Number of carbon atoms with sp2 hybridisation
dipole moment of lone pair on sulphur atom in
H 2 S). (Given cos 97° = – 0.12 and (a) 2 (b) 4
(c) 6 (d) 8
0.88  0.94 )
65. Number of nitrogen atoms with sp hybridisation
(a) 32% (b) 16%
(a) 2 (b) 4
(c) 84% (d) 10%
(c) 6 (d) 1
61. In CH3CCl3 (I), CHCl3 (II) and CH3Cl (III) the
normal tetrahedral bond angle is maintained. Also MATRIX MATCH TYPE
1 66. Match list-I with list-II and select the correct
given cos70.5° = . Therefore dipole moments answer :-
3
of the given compounds are. :- Species O–N–O angle
(a) I = 1.9 D, II = 1.9 D, III = 1.7 D (A) NO2 +
(P) 180°
(b) I = 1.9 D, II = 1.7 D, III = 1.9 D (B) NO 2 (Q) 134°
(c) I = 1.9 D, II = 1.7 D, III = 1.0 D (C) NO 2 – (R) 120°

(d) I = 1.9 D, II = 1.1 D, III = 1.9 D (D) NO 3 (S) 115°
62. In the acetone molecule considering the normal 67. Match list-I with list-II and select the correct
planer structure , the observed dipole moment of answer :-
acetone molecule is :- Species HMH/FMF bond angle
(a) 2.9D (b) 2.75D (A) NF 3 (P) 102°
(c) 3D (d) none of these (B) PF3 (Q) 97.8°
(C) NH3 (R) 107.3°
Passage for Q.63 to Q.65
(D) PH3 (S) 93.8°
The structure of AZT drug (azidothymidine) also called
68. Match list-I with list-II and select the correct
zidovudone, is given below.
answer :-
O
Molecule/Species Unpaired electron
H C CH3 resides in
N.. C (A) NO 2 (P) vacant d-orbital
C (B) ClO2 (Q) sp2-orbital
H
O .. C
O N H (C) ClO3 (R) sp3-orbital
H O C
(D) CH3 (S) vacant p-orbital
H CH CH
69. Match the column :-
CH CH2
Column - I Column - II
N Compound Characteristic feature
+
N (A) POCl3 (P) Covalent bond present
N (B) XeO2F2 (Q) Molecule violets octet rule
It is used to delay development of AIDS in patients (C) CH3–NC (R) One lone pair on any atom
infected with HIV (Human immuno defficiency virus) (D) CO (S) -bond present

www.jeebooks.in
CHEMICAL BONDING 1.37
70. Match the column :- 74. Match the column :-
Column - I Column - II Column - I Column - II
Compound Characteristic feature Compound Features
(A) XeO4 (P) p–p bond present (A) X eO 4 (P) p-p bond present
(B) CH2 = SF4 (Q) p–d bond present (B) SO3 (Q) p-d bond present
Cl (C) COCl2 (R) Number of total lone pair
(C) O=C (R) Number of total lone is even
Cl pair present is even (D) H2S2O7 (S) sp2 hybridisation
(D) XeO3F2 (S) Non polar molecule (T) sp3 hybridisation
71. Match the column :- 75. Match the column :-
Column - I Column - II Column - I Column - II
Compound Hybridisation Compound Features
(A) XeO64– (P) sp (A) Be2Cl4 (P) 3c–4 e bond is present
(B) IF7 (Q) sp2 (B) C2Cl6 (Q) 3c–2 e bond is present
(C) CO 32– (R) sp3d3 (C) B2H6 (R) 2c–2e bond is present
(D) Ag(CN)2– (S) sp3d2 (D) Al2(C6H5)6 (S) Molecule is planar
72. Match the column :- 76. Match the column :-
Column - I Column - I Column - II
Atom in a compound Compound Features
(A) Carbon atom in cyclopentadienyl anion (A) BF3 (P) electron difficient molecule
either in monomer or in
(B) Oxygen atom in diethyl ether
dimer form
(C) Carbon atom in singlet carbene
(B) SiCl4 (Q) non-polar in dimeric form
(D) Nitrogen atom in triethyl amine
(C) ICl3 (R) bridging atoms are present
Column - II in molecular plane in its
Hybridisation dimeric form
(P) sp2 hybridisation (D) BeCl2 (S) dimer form does not exist
(Q) sp3 hybridisation 77. Match the column :-
(R) one lone pair of e– present (localised) Column - I Column - II
(S) two lone pair of e– present (localised) Species Features
73. Match the column :- (A) XeF5 +
(P) Axial bond length is longer
Column - I Column - II than equatorial bond
Molecule/ Shape/Hybridisation length

Intermediate of central atom (B) IF4 (Q) sp 3 d hybridisation of
central atom
(A) NO 2 (P) sp2
(C) PCl5 (R) Non-planar species
(B) Trifluoro methyl (Q) sp3
(D) ClF3 (S) Central atom has 2 lone
radical
pair of electrons on it
(C) Methyl radical (R) Trigonal planar
(T) dz2 orbital is involved in
(D) ClO3 (S) Pyramidal hybridisation

www.jeebooks.in
1.38 CHEMICAL BONDING
78. Match the column :- 81. Match the column :-
Column - I Column - I Column - II
Process Compound Features
(A) Solubility of noble gas is in water (A) Dimer of acetic acid (P) 2c–2e bond present
(B) The interaction between chlorobenzene molecule (B) Dimer of borane (Q) 3c–2e bond present
(C) Interaction between alkane (C) Dimer of AlCl3 (R) 3c–4e bond present
(D) Dissolution of ionic compound in water (D) Dimer of NO2 (S) H bond present
Column - II (T) Molecule existing
Operating interaction involved 82. Match the column :-
(P) Debye forces Column - I
(Q) Keesom forces
Compound
(R) London forces
(A) CN22– , CO2
(S) Weak forces
(B) Inorganic benzene, borazene
79. Match the column :-
(C) PCl3F2 , PF3Cl2
Column - I
(D) BrF5 , XeF5+
Process
Column - II
(A) Clathrate compound of Xe in ice
Features
(B) Liquation of Xe gas
(P) same number of total electrons
(C) Liquation of HCl gas
(D) Hydration of Na+ (Q) pair of isostructural species
Column - II (R) same number of valence electrons
Operating interaction involved (S) both structures are planar
(P) Ion-dipole (T) both cases, central atom has same hybridisation
(Q) Dipole-dipole 83. Match the column :-
(R) Dipole-Induced dipole Column - I Column - II
(S) London forces Species Features
80. Match the column :- (A) C 12 O 9 (P) M–O–M linkage is
Column - I present
Compound (B) Si2O76– (Q) open chain silicate
(A) P(SiH3)3 (C)  – SO3 (R) cyclic structure
(B) N(CH3)3 (trimeric form)
(C) N(SiH3)3 (D) Cl2O7 (S) anhydride of HClO4
(D) SiF4 84. Match the column :-
Column - II Column - I Column - II
Features Species Hybridisation of
(P) pyramidal in shape central atom
2–
(Q) no back bonding possible (A) PtCl4 (P) sp3
(R) trigonal planar in shape (B) CrO42– (Q) sp3d2
(S) 2p– 3d back bonding possible (C) XeF4 (R) d3s
(T) tetrahedral in shape (D) BF4– (S) dsp2

www.jeebooks.in
CHEMICAL BONDING 1.39
85. Match the column :- 88. Match the column :-
Column - I Column - II Column - I Column - II
Species Features Type of  bond Molecules
(A) AlF3 (P) dipole moment is (A) 2p–2p (P) O=C=O
zero (B) 2p–3p (Q) S=C=S

(B) I(CN)2 (Q) overall molecule is (C) 2p–3d (R) POCl3
linear (D) 3p–3d (S) PSCl3
(C) Ba2XeO6 (R) ionic compound (T) SOCl2
exist as lattice
89. Match the column :-
(D) H3Ge – N = C = S (S) the central atom
Column - I Column - II
has dz2 orbital for
hybridisation Oxyacid of chlorine Features
(T) at least one of the (A) HClO4 (P) weakest acid
atom is sp (B) HClO3 (Q)strongest acid
hybridised (C) HClO2 (R) C.B. has 3 identical R.S.
86. Match the column :- (D) HOCl (S) sp3 hybridisation of
Column - I central atom
Species (T) Formula of anhydride
(A) H3BO3 ; H3PO4 is Cl2O7
(B) CH2=C=O ; CH2=C=CH2 90. Match the column :-
(C) LiI, LiF Column - I
(D) H2O, H2S Miscellaneous oxy acid
Column - II (A) HMnO4
Features (B) H2CrO4
(P) Ist cpd is planar ; IInd cpd is non planar (C) H2CO3
(Q) Ist cpd is not proton donor but IInd cpd is (D) CH3CO2H
proton donor Column - II
(R) Ist cpd is covalent but IInd cpd is ionic in nature Features
(S) For Ist cpd hybridisation of central atom is (P) Monobasic acid
sp3 whereas for IInd cpd, hybridisation of (Q) Pink / purple colour
central atom in molecule is not defined (R) Yellow colour
87. Match the column :- (S) C–O bond lengths are not identical
Column - I Column - II (T) Thermally unstable and gives CO2 gas on heating
Oxyacids of N Features 91. Match the column :-
(A) HNO4 (P) Monobasic acid Column - I Column - II
(B) HNO3 (Q) C.B. has 2 identical R.S. Compound/molecule Structures
(C) HNO2 (R) Compound showing G.I. (A) S 8 (P) Zig-zag structure
(D) H2N2O2 (S) Highest oxidation state (B) H2O2 (Q) open book like structure
of central atom (C) (HF)n (R) crown shape
(T) Peroxy linkage present (D) CrO5 (S) Butterfly structure

www.jeebooks.in
1.40 CHEMICAL BONDING
92. Match the column :- 95. Match the column :-
Column - I Column - I
Compound
Oxyacids of S
(A) NH4ClO4
(i) H2SO2 (sulphoxylic acid)
(B) MgC2O4
(C) HCOONa (400°C) (ii) H2SO3
(D) (NH4)2CO3 (iii) H2SO4
Column - II (iv) H2SO5 (Caro’s acid)
Products on heating (v) H2S2O7 (Oleum)
(P) CO2 gas is evolved
(vi) H2S2O8 (Marshall acid)
(Q) H2 gas is evolved
(R) N2 gas is evolved (vii) H2S2O3 (Thiosulfuric acid)
(S) Same gas is evolved which is obtained by (viii) H2S2O2 (Thiosulfurous acid)
heating (NH4)2SO4 (ix) H2SnO6 (Polythionic acid)
(T) Intramolecular redox reaction (x) H2SnO4 (Polythionous acid)
93. Match the column :-
Column - II
Column - I
Compounds Features
(A) AlF3, SF4 , PF5 (P) S–O–S linkage present
(B) O(CH3)2, S(CH3)2 , Se(CH3)2 (Q) sp3 hybridisation of S
(C) HCl, HF, HBr (R) Peroxy linkage present
(D) CH4 , CH3Cl, CH3OH, CH3CO2H
(S) S=S linkage present
Column - II
(T) S–S linkage present
Boiling point
(P) Ist compound has highest boiling point (U) At least one sulphur atom having oxidation
(Q) IIIrd compound has highest boiling point state of (+6)
(R) IInd compound has highest boiling point (V) Oxidation state of S is zero in S–S linkage
(S) IVth compound has highest boiling point (W) Dibasic acid
94. Match the column :- (X) Sulphur can have –ve oxidation state
Column - I Column - II
(Y) Oxyanion undergoes resonance
Oxyacids of P Features
(i) H3PO4 (P) All Hs’ are not ionisable 96. Match the column :-
(ii) H3PO3 (Q) Dibasic reducing agent Column - I Column - II
(iii) H3PO2 (R) Monobasic reducing agent Compounds Type of bond/
(iv) H4P2O7 (S) P–H linkage present interaction
(v) H5P3O10 (T) P–O–P linkage present
(A) Blue vitriol (P) Ionic bond
(vi) H3P3O9 (U) P–P linkage present
(vii) H4P2O6 (V) Cyclic structure (B) Gypsum (Q) Covalent bond
(viii) H3PO5 (W) Peroxy linkage present (C) Pure orthophosphoric (R) Hydrogen bond
(X) Highest oxidation state of P acid
(Y) Tetrabasic acid (D) Chloral hydrate (S) Resonance
(Z) Tribasic acid stabilisation

www.jeebooks.in
CHEMICAL BONDING 1.41
97. Match the column :- 100. Match the column :-
Column - I Column - I
Different system Observed order
(A) Liquid bromine (A) H2 < CO2 < H2O
(B) Noble gas clathrate (B) PH3 < AsH3 < NH3 < SbH3
(C) Solution of sodium fluoride in water (C) D2O > H2O
(D) (CH3)3N < CH3NH2 < (CH3)2NH
(D) Liquid methylamine
Column - II
Column - II
Property
Type of bond/interaction
(P) order of boiling point
(P) hydrogen bond
(Q) order of density
(Q) ion-dipole force
(R) order of intermolecular forces of attraction
(R) london dispersion force
(S) order of basic strength in aqueous medium
(S) dipole induced dipole interaction
98. Match the column :-
EXERCISE # IV
Column - I  Integer Type :
Molecules 1. Molecules having total eight lone pair present :-
(A) F2 , N2H4 , H2O2 , NF3 XeO4 ; HClO4 ; H2SO4 ; N2O3 ; C3N3Cl3 ; COCl2 ;
(B) XeF4 , SO2 , PCl3F2 , PCl5(g) H2SO3
(C) H3BO3 , Graphite, NH4Cl , Na2CO3 , SO3 2. How many correct bond energy order is given :-
(D) Inorganic benzene , Benzene, (a) H–H > C–H > C–C
Column - II (b) C–F > C–Cl > C–Br > C–I
Features (c) Cl–Cl > Br–Br > F–F > I–I
(P) all except one are examples of non polar (d) C–C > Si–Si > Ge–Ge
compounds (e) N–N > P–P > As–As > Sb–Sb > Bi–Bi
(Q) all except one are planar molecules (f) CN > CN > C–N
(R) all except one have at least one atom which is 3. How many molecules /ions are result of dative -
sp2 hybridised bond ?
  
(S) all except one have less bond energy because (a) NH 4 (b) H 3 N– BF3
ICl4– , SO42–, NO3– of lone pair - lone pair
repulsion (c) I3 (d) BH 4
99. Match the column :- (e) SbF6– (f) H3O+
Column - I Column - II 4. How many molecule having negative fractional
Compounds Feature/Use oxidation state for nitrogen -
(A) H2O (P) polar solvent N2O3 ; N2O5 ; Hydrazine ; Hydrazoic acid ; NF3 ;
(B) H2O2 (Q) non linear molecule N2O ; NH3
(C) D2O (R) more acidic than C2H5OH 5. How many molecules do not exist ?
(D) CH3OH (S) used as moderator in CCl62– ; CBr62– ; SiF62– ; AsF5 ; AsH5 ; XeF2 ;
nuclear reactions KrCl2

www.jeebooks.in
1.42 CHEMICAL BONDING
6. How many of the following has M–O–M linkage? 15. Find the number of planes containing maximum
(a) trimer of SO3 which is represented as S3O9 number of fluorine atoms (only) in SbF5.
(b) H2S2O8 16. How manmy molecule / species central atom is
(c) Cl2O7 sp3 hybridised

(d) P 4O10 BeF2 ; BF4– ; SO3 ; AlCl4– ; NH4+ ; ClO4– ; N3– ;


XeO4
(e) P 4O 6
17. Find the number of molecule in which p – p
(f) H2SO5
sideways overlapping is present -
(g) H2S2O7
N2 ; HCl ; Cl2 ; HF ; S8 ; F2 ; HCP
7. For how many molecule where all following points
18. Total number of bond angle of 90° in SiF62– is
are satisfied :
19. Find the number of atomic orbital which forms only
(i) central atom is sp3 hybridised
 bond.
(ii) tetrahedral in shape
s, px, py, pz, , dxy , dx2–y2
(iii)  bond is present along with  bond
20. The number of molecule having perfectly tetrahedral
CH4 , SiCl4 , NH4+ , BF4– , XeO4 , ClO4– , SO42– , shape among the following are -
PO43–
NH3, CCl4, PO43–, CHCl3, BF4–
8. How many molecules / species are example of 16
21. Find the ratio of number of lone pairs to number of
e– system with linear inshape -
 bonds in a particular resonating structure of
(a) CO 2 (b) Nitronium ion SO42– -
(c) Azide ion (d) Nitrous oxide 22. In which of the following compounds all atoms are
(e) Cyanate ion (f) Isocyanate ion present in same plane -
(g) Cyanamide (CN22–) ion (a) I3– (b) ICl4–
9. Total number of odd electron system : (c) XeF4 (d) XeF5+
NO2 ; NO : ClO2 ; OF ; O3– ; O2 ; ClO3 (e) XeF5– (f) C(CN)4
10. Find the number of sp3 hybrid orbital in SO3 :- 23. In which of the following compound octet of central
11. How many molecules having molecular formula atom is incomplete or molecule is hypovalent-
PClnFm will be polar - (a) BF3 (b) ICl
(a) m = 0 ; n = 5 (b) n = 0 ; m = 5 (c) ClF3 (d) BeH2
(c) n = 3 ; m = 2 (d) m = 3 ; n = 2 (e) PCl3 (f) AlCl3
(e) n = 4 ; m = 1 24. Identify the total number of molecules (amongest
12. Identify number of molecules in which only two the following) which have bond angle lesser
types of bond angles are present than -
CH4 ; CH3Cl ; CH2Cl2 ; CHCl3 (Chloroform) ; O– +
 N=O
CCl4 (Carbon tetrachloride) ; COCl2
O–
13. If number of planes present in CH4 molecule which
CO2 ; CH4 ; BF3 ; CH3–O–CH3 ; SOCl2 ; NH3 ;
contain maximum number of atoms of
H2O
corresponding molecule is x then find out x – 1.
25. Find the number of chemical species in which bond
14. If number of planes present in IF7 molecule which
angle is less than the bond angle in CCl4-
contain maximum number of atoms of
PH3 ; H2S ; OCl2 ; O(CH3)2 ; ClO2 ; NCl3
corresponding molecule is y then find out y.

www.jeebooks.in
CHEMICAL BONDING 1.43
+ –
26. Find the molecule in which bond angle is equal to 37. XeF4 + PF5 XeF3 + PF6
the expected bond angle according to hybridisation If total numbers of 90° angle in reactant = m then
of underlined atom - find out and if total numbers of 9° angles in
(a) PF3 (b) BF3 product = n.
(c) PCl3 (d) COCl2 38. In how many of the following species, % s character
of all bonds is not identical.
(e) NH4+ (f) NMe3 S
27. Find out no. of total lone pair present in trimer of IF7 ; CH4 ; SF4 ; XeF3+ ; OCl2 ; ICl2– ; BF3 ;
O O
isocyanic acid (HNCO)3. 39. Cos92° = –0.0.3, What is the % of s-character in
28. If number of  bonds present in melamine = x and hybrid orbital containing lone pair of e–s :-
if no. of lone pair present in melamine = y then find 40. Find out number of molecule which do not exist -
out y/x. XeF3– ;XeI3– ; IF32– ; FeI3 ; AI3 ; ClI3– ; PI5 in T.b.p
29. If number of  bond present in C 3N3(N 3)3 state.
cyanauric triazide = x and if number of lone pair 41. Find the number of molecules which has 3p – 2p
present in C3N3(N3)3 is = y & if no. of sp hybridised back bonding :
yz OCl2 ; :CCl2 ; BCl3 ; CCl3
N present = z then is :-
x 42. Find the number of molecules in which 2p – 3d
30. Drago rule is applicable for which of the following back bonding is present.
molecule - OCl2 ; BF3 ; N(SiH3)3 ; N(CH3)3 ;

SiH4 ; PH3 ; PF3 ; H2S ; H2Te ; AsH3 ; PH4+ ; C Cl3 ; O(SiH3)2 ; :CCl2
SbH3 43. Correct statement regarding B2H6 :-
31. How many structures are cyclic in nature ? (a) Terminal B–H bond length 119 pm and it is
(a) S 3O 9 (b) P 3O93– 2c–2e bond
(c) P 4O124– (d) (PNCl2)3 (b) Bridging B–H bond length 113 pm and it is
3c–2e bond
(e) H2S2O7 (f) H2S2O8
(c) It is non-polar molecule
32. Number of S–O–S linkage present in cyclic trimer
(d) There are only twelve bonding electrons available
of SO3.
(e) B atom in B2H6 is approximately sp3 hybridised
33. Number of bond angle which are at 90° w.r.t. each
other present in SF4. (f) It has momentarily complete octet
(g) The plane containing the two boron atoms and
34. Number of bond angle which are at 90° w.r.t. each
four terminal hydrogen atom is perpendicular to
other present in BrF5.
that containing the two bridging hydrogen atoms.
35. In which case compound has bond angle which is (h) The kind of overlap present in bridge bond in
not exactly equal according to hybridisation? B2H6 sp3–s–sp3
36. F 44. In this following molecule the number of bridging
F
..

bond :-
M H H
F  F
F Be

 bond angle in MF is  = 87°.


If FMF H H
5

 bond angle in MF is 
& if FMF H
4 2
Be
Then find out 2– 1
H H
H

www.jeebooks.in
1.44 CHEMICAL BONDING
45. Find the maximum number of atoms may lie in a 57. In IF7 ; what is the total number of bonds which
single plane for the following molecule are at 72° ?
H3 C CH3 58. For C70 fullerene find the number of hexagonal
C=C=C :- face :-
H3 C CH3 59. For which compound all M–O bonds have bond
46. Maximum number of 2c–2e bonds which are order 1.5 :-
formed by sp3–sp3 overlapping in Al2(CH3)6 :- SO2 ; CH3CO2H ; CH3CO2– ; NO2– ; NO3–
47. Maximum number of atoms present in one plane 60. For [ABSi3Ox] , where A & B have +2 & +4
in Al2(CH3)6 :- oxidation state respectively. What is the value of
48. How many compounds on reaction with H2O give X?
product with pH < 7 in aqueous solution ? 61. Number of S–S linkage in H2S6O6 :-
MgO ; Na2O ; SO3 ; Cl2O7 ; P4O10 ; BaO ; N2O3 ; 62. Find the number of S atom which are at zero
SO2 oxidation state in H2S6O6 :-
49. Find out number of molecules which on complete 63. Find the number of species having X–O–X
hydrolysis produce sulphuric acid as one of the linkage :-
product :- Pyroxene / chain silicate, Perdisulphate ion, Anion
SOF4 ; SF4 ; H2S2O8 ; H2S2O7 ; SO2Cl2 ; S3O9 of Caro’s acid, Pyrosulphate ion, Tetrathionate ion,
Thiosulphate ion.
50. Find the number of water molecules required for
64. How many orbitals are having higher energy
complete hydrolysis of one molecule of XeF6 :-
compared to 2s for Li2 molecule from M.O. given
51. Find the number of water molecules which are used below :-
for complete hydrolysis of one molecular
1s* ; 2s* ; 1s ; 2py ; 2py* ; 2pz* ; 2pz* ;
P4O10 :-
 pz
52. How many water molecules are associated with
65. How many of the following are paramagnetic :-
hydrogen bond but not coordinated with metal ion
in CuSO4.5H2O :- He2+ ; H2+ ; H2– ; NO2 ; NO ; OF
66. How many molecules / ion have bond order less
53. Among the following find out the total number of
than or equal to two
molecules have more % of p-character in
E–H bond than that of PH3 :- B2 ; C2 ; O22– ; O2+1 ; H2+1
AsH3 ; SbH3 ; NH3 ; H2O ; B2H6 67. Considering MOT find out number of diatomic
species which do not have  bond :-
54. Find out the number of molecules which are non-
planar :- F2 ; C2 ; N2 ; H2 ; C22–
C2(CN)4 ; P4 ; S8 ; H2O2 ; F2O2 ; CF4 ; S2F10 68. Find the ratio of electrons in the C2 molecule
with that of B2 molecule according to MOT :-
55. For how many molecule C–C bond length is
shorter than C–C bond length in graphite :- 69. Find the number of chemical species which are
paramagnetic in nature :-
(i) acetylene (ii) ethene (iii) benzene (iv) ethane
O22– ; NO ; ClO2 ; OF ; B2
56. How many molecules have two peroxy linkage :-
70. Find out the number of species having
CO3H paramagnetic in nature as well as fractional bond
order :-
, B4O72– , [B2(O2)2(OH)4]2– , CrO5
B2 ; NO ; OF ; N2 ; KO2 ; K2O2
Cl

www.jeebooks.in
CHEMICAL BONDING 1.45
71. If sp mixing is not operative then total number of 80. The difference in the number of  and  bonds in
electron in B.M.O. of N2+ will be :- trimer of SO3 is (considering only covalent bond is
72. Find out total number of species containing N–N/ present) :-
P–P linkage :- EXERCISE # V(A) JEE-MAIN
N2O ; N2O3 (symmetrical) ; N2O5 ; H2N2O2 ; 1. Which of the following statements is true?
H4P2O6 ; P4O6 ; P4O10 [AIEEE–2002]
73. Find the number of chemical species which are (1) HFis less polar than HBr
isoelectronic and have same bond order as of CO:- (2) Water does not contain any ions
CN– ; NO+ ; N2 ; C22– ; O22+ (3) Chemical bond formation takes place when
74. Find the number of compounds which can form forces of attraction overcome the forces of re-
pulsion
intermolecular hydrogen bonding but cannot form
intramolecular hydrogen bonding :- (4) In covalent bond, transfer of electrons takes place
Dimer of acetic acid ; Quinol ; Catechol ; 2. The reason for double helical structure of DNA is
operation of: [JEE2003]
(KHCO3)2 ; (NaHCO3)n
(1) dipole–dipole interaction
75. Number of bond pairs at 90° angle in XeF5– is x
(2) hydrogen bonding
Number of bond pairs at 90° angle in XeF4 is y
(3) elcctrostatic attractions
The value of y – x :-
(4) vander Walls forces
76. Select the no. of orbitals which can produce 
3. Which of the following pair of molecules will have
bond while overlapping with an s-orbital - permanent dipole moments for both members
(a) px (b) p y [AIEEE–2003]
(c) pz (d) dxy (1) NO2 and CO2 (2) NO2 and O3
(e) dyz (f) s (3) SiF4 and CO2 (4) SiF4 and NO2
77. Find out total number of bond in P6O186– :- 4. The pair of species having identical shapes for
molecules of both species is [AIEEE–2003]
78. How many statements are correct :-
(1) XeF2,CO2 (2) BF3,PC13
(a) Li2O has highest melting point among all alkali
(3) PF5, IF5 (4) CF4, SF4
metal oxide
5. The correct order of bond angles (smallest first) in
(b) BeO has highest melting point among all alkaline H2S, NH3, BF3 and SiH4 is :– [AIEEE–2004]
earth metal fluoride
(1) H2S < NH3 < SiH4 < BF3
(c) LiF1 has highest melting point among all alkali (2) NH3 < H2S < SiH4 < BF3
metal fluoride
(3) H2S < SiH4 < NH3 < BF3
(d) BeF2 has highest melting point among all (4) H2S < NH3 < BF3 < SiH4
alkaline earth metal fluoride
6. The bond order in NO is 2.5 while that in NO+ is
(e) Li3N has highest melting point among all alkali 3. Which of the following statements is true for
metal nitride these two species ? [AIEEE–2004]
(f) Be3N2 has highest melting point among all (1) Bond length in NO+ is equal to that NO
alkaline earth metal nitride (2) Bond length in NO is greater than NO+
79. The bond order of the underlined species (3) Bond length in NO+ is greater than NO
NOHSO4 is :- (4) Bond length is unpredictable

www.jeebooks.in
1.46 CHEMICAL BONDING
7. The stales of hybridization of boron and oxygen 14. Of the following sets which one does not contain
atoms in boric acid (H3BO3) are respectively isoelectronic species. [AIEEE 2005]
[AIEEE–2004] (1) PO43–, SO42–, ClO4–
(1) sp3 and sp2 (2) sp2 and sp3
(2) CN–, N2, C22–
(3) sp2 and sp2 (4) sp3 and sp3
(3) SO32–, CO32–, N3–
8. Which one of the following has the regular tetra-
(4) BO33–, CO32–, NO3–
hedral structure ? [AIEEE–2004]
– 15. The number and type of bond between two earbon
(1) BF4 (2) SF4
atom in calcium carbide are: [AIEEE–2005]
(3) XeF4 (4) [Ni(CN)4]2–
(1) one sigma, one pi (2) one sigma, two pi
(Atomic nos.: B = 5, S = 16, Ni = 28, Xe = 54)
(3) two sigma, one pi (4) two sigma, two pi
9. The maximum number of 90° angles between bond
pair–bond pair of electrons is observed – 16. Which of the following molecules / ions does not
[AIEEE–2004] contain unpaired electron ? [AIEEE–2005]
(1) dp2 hybridization (2) sp3d hybridization (1) N2+ (2) O 2
2–
(3) dsp3 hybridization (4) sp3d2 hybridization (3) O 2 (4) B2
10. Beryllium and aluminium exhibit many proper- 17. Among the following mixtures , dipole – dipole as
ties which are similar. But , the two elements the major interaction , is present in :
differ in :– [AIEEE 2004] [AIEEE–2006]
(1) Forming covalent halides (1) KCl and water
(2) Forming polymeric halides (2) benzene and carbon tetrachloride
(3) Forming maximum covalency in compounds (3) benzene and ethanol
(4) Exhibiting amphoteric nature in their oxides (4) acetonitrile and acetone
11. Which one of the following species is diamagnetic 18. A metal , M forms chlorides in its +2 and +4 oxi-
in nature ? [AIEEE2005] dation states. Which of the following statement
(1) He2 +
(2) H2 about these chlorides is correct ? [AIEEE–2006]
(3) H2+ (4) H2– (1) MCl2 is more ionic than MCl4
12. Lattice energy of an ionic compound depends upon (2) MCl2 is more easily hydrolysed than MCl4
[AIEEE2005] (3) MCl2 is more volatile than MCl4
(1) charge on the ion only (4) MCl2 is more soluble in anhydrous ethanol than
(2) size of the ion only MCl4
(3) packing of the ion only 19. In which of the following molecules / ions ae all the
(4) charge and size of the ion bonds not equal : [AIEEE 2006]
13. The molecular shapes of SF4, CF4 and XeF4 are– (1) XeF4 (2) BF4–
[AIEEE2005] (3) SF4 (4) SiF4
(1) the same with 2, 0 and 1 lone pair of electrons
20. The decreasing values of bond angles from NH3
on the central atom, respectively
(106°) to SbH3 (91°) down group–15 of the peri-
(2) the same with 1, 1 and 1 lone pair of electrons odic table is due to – [AIEEE 2006]
on the central atom, respectively
(1) decreasing lp – bp repulsion
(3) the same with 1, 1 and 2 lone pair of electrons
on the central atom, respectively (2) increasing electronegativity
(4) the same with 1, 0 and 2 lone pair of electrons (3) increasing bp – bp repulsion
on the central atom, respectively (4) increasing p–orbital character in sp3

www.jeebooks.in
CHEMICAL BONDING 1.47
21. In which of the following ionization process, the 28. The structure of IF7 is :– [AIEEE–2011]
bond order has increased and the magnetic (1) octahedral (2) pentagonal bipyramid
behaviour has changed [AIEEE 2007] (3) square pyramid (4) trigonal bipyramid
+
(1) NO  NO (2) O2  O2+ 29. Among the following the maximum covalent
(3) N2  N2+ (4) C2  C2+ character is shown by the compound:
22. Which of the following hydrogen bond is the stron- [AIEEE–2011]
gest ? [AIEEE 2007] (1) A1C13 (2) MgCl2
(1) F–H.......F (2) O–H.......O (3) FeCl2 (4) SnCl2
(3) O–H.......F (4) O–H.......N 30. Which of the following has maximum number of
23. Which of the following species exhibits the dia- lone pairs associated with Xe
magnetic behaviour ? [AIEEE 2007] (1) XeO3 (2) XeF4
(1) O2+ (2) O 2 (3) XeF6 (4) XeF2
(3) NO (4) O 2 2– 31. The number of types of bonds between two carbon
24. Which one of the following pairs of species have atoms in calcium carbide is :– [AIEEE–2012]
the same bond order [AIEEE 2008] (1) One sigma, two pi
(1) CN–and NO+ (2) CN–and CN+ (2) One sigma, one pi
(3) O2– and CN– (4) NO+ and CN+ (3) Two sigma, one pi
25. The bond dissociation energy of B–F in BF3 is (4) Two sigma, two pi
646 kj mol–1 whereas that of C–F in CF4 is
32. The molecule having smallest bond angle is –
515 kj mol–1. The correct reason for higher B – F
[AIEEE–2012]
bond dissociation energy as compared to that of
C–F is :– [AIEEE–2009] (1) PCl3 (2) NC13
(1) Significant p – p interaction between B and (3) AsCl3 (4) SbCl3
F in BF3 whereas there is not possibility of such 33. In which of the following pairs the two species are
interaction between C and F in CF4. not isostructural ? [AIEEE–2012]
3–
(2) Lower degree of p – p interaction between (1) A1F6 and SF6
B and F in BF3 than that between C and F in (2) CO32– and NO3–
CF4 (3) PCl4+ and SiCl4
(3) Smaller size of B–atom as compared to that of (4) PF5 and BrF5
C–atom
34. The number of S–S bonds in SO3, S2O32–, S2O62–
(4) Stronger  bond between B and F in BF3 as and S2O82– respectively are :–
compared to that between C and F in CF4
[JEE–MAINS–2012] (On–line)
26. Using MO theory predict which of the following
(1) 1,0,1,0 (2) 0,1,1,0
species has the shortest bond length ?
[AIEEE–2009] (3) 1,0,0,1 (4) 0,1,0,1
(1) O 2 –
(2) O 2 2– 35. Dipole moment is shown by :–
(3) O22+ (4) O2+ [JEE–MAINS–2012] (ON-line)
27. The hybridisation of orbitals of N atom in NO3–, (1) trans–2. 3–dichloro–2–butene
NO2+ and NH4+ are respectively:– [ArEEE–2011] (2) 1, 2–dichlorobenzene
(1) sp, sp3, sp2 (2) sp2, sp3, sp (3) 1,4–diclorobenzene
(3) sp, sp2, sp3 (4) sp2 , sp, sp3 (4) trans–1,2–dinitroethene

www.jeebooks.in
1.48 CHEMICAL BONDING
36. Among the following species which two have 44. Which of the following is the wrong statement?
trigonal bipyramidal shape :– [JEE–M–2013]
[JEE–MAINS–2012] (On–line) (1) ONCl and ONO are not isoelectronic
(I) NI3 (II) I3– (2) O3 molecule is bent
(III) SO32– (IV) NO 3 – (3) Ozone is violet–black in solid state
(1) II and III (2) III and IV (4) Ozone is diamagnetic gas
(3) I and IV (4) I and III 45. In which of the following pairs of molecules/ions,
37. Among the following, the species having the both the species are not likely to exist ?
smallest bond is - [JEE–M–2013]
[JEE–MAINS–2012] (On–line) (1) H2+,He22– (2) H2–,He22–
(1) NO (2) NO+ (3) H22+,He2 (4) H2–,He22+
(3) O 2 (4) NO – 46. Which of the following exists as covalent crystals
in the solid state ? [JEE–M–2013]
38. Based on lattice energy and other considerations ,
which one of the following alkali metal chloride is (1) Iodine (2) Silicon
expected to have the highest melting point :– (3) Sulphur (4) Phosphorus
[JEE–MAINS–2012] (On–line) 47. Stability of the species Li2, Li2– and Li2+ increases
(1) RbCl (2) LiCl in the order of [JEE–M–2013]
(3) KCl (4) NaCl (1) Li2 < Li2 < Li2 (2) Li2 < Li2+ < Li2
+ – –

39. Which of the following has the square planar (3) Li2 < Li2– < Li2+ (4) Li2–< Li2 < Li2+
structure :– [JEE–MAINS–2012] (On–line) 48. Trigonal bipyramidal geometry is shown by.
(1) NH4 +
(2) CCl4 [JEE–Mains–2013, Online]
(3) XeF4 (4) BF4– (1) XeO3F2 (2) XeOF2
40. The compound of Xenon with zero dipole moment (3) XeO3 (4) FXeOSO3F
is :– [JEE–MAINS–2012] (On–line) 49. In which of the following ionization processes the
(1) XeO3 (2) XeO2 bond energy has increased and also the magnetic
behaviour has changed from paramagnetic to
(3) XeF4 (4) XeOF4
diamagnetic ? [JEE-Main 2013]
41. Among the following molecule with the lowest +
(1) NO NO (2) O2 O2+
dipole moment is: [JEE–MAINS–2012] (On–line)
(3) N2 N2+ (4) C2 C2+
(1) CHCl3 (2) CH2Cl2
50. Which one of the following molecule is polar ?
(3) CCl4 (4) CH3Cl
[JEE.MAINS.2013]
42. The formation of molecular complex BF3 – NH3
results in a change in hybridisation of (1) CF4 (2) SbF5
boron :– [JEE–MAINS–2012] (3) IF5 (4) XeF4
(1) from sp3 to sp3d (2) from sp2 to dsp2 51. Oxidation state of sulphur in anions SO32–, S2O42–
(3) from sp3 to sp2 (4) from sp2 to sp3 and S2O62– increases in the order–
(on–line) [JEE.MAINS.2013, Online]
43. Which one of the following molecules is expected (1) S2O62–, S2O42– < SO32–
to exhibit diamagnetic behaviour ? [AIEEE–2013] (2) SO32–< S2O42–< S2O62–
(1) C 2 (2) N 2 (3) S2O42–< SO32– < S2O62–
(3) O 2 (4) S 2 (4) S2O42– < S2O62– < SO32–

www.jeebooks.in
CHEMICAL BONDING 1.49
52. Bond order normally gives idea of stability of a (1) 348, 260, 297 (2) 348, 297, 260
molecular species. All the molecules viz. H2 , Li2 (3) 297, 348, 260 (4) 260, 297, 348
and B2 have the same bond order yet they are not 59. In which of the following sets, all the given species
equally stable. Their stability order is - are isostructural ? [JEE–MAINS–2013]
[JEE–MAINS–2013] (On–line) (1) BF3 , NF3 , PF3 , AlF3
(1) Li2 > H2 > B2 (2) H2 > B2 > Li2 (2) PC13, A1C13, BC13, SbCl3 (On–line)
(3) B2 > H2 > Li2 (4) Li2 > B2 > H2 (3) BF4–, CCl4 , NH4+ , PCl4+
53. The solubility order for alkali metal fluoride in water (4) CO2, NO2, ClO2, SiO2
is– [JEE–MAINS–2013] (On–line)
60. The inter molecular distances in O –O bonds for
(1) LiF < NaF < KF < RbF O2+, O2, O2– and O22– respectively are :
(2) LiF > NaF > KF > RbF [ JEE–MA INS– 2013], (On–line)
(3) RbF < KF < NaF < LiF (1) 1.49 Å. 1.21 Å, 1.12 Å, 130 Å
(4) LiF < RbF < KF < NaF (2) 1.30 Å, 1.49 Å, 1.12 Å, 1.21 Å
54. XeO4 molecule is tetrahedral having : (3) 1.12 Å. 1.21 Å, 1.30 Å, 1.49 Å
[JEE–MAINS–2013] (On–line) (4) 1.21 Å, 1.12 Å, 1.49 Å, 1.30 Å
(1) Two p–d bonds 61. Which one of the following properties is not shown
(2) Four p–d bonds by NO ? [JEE–Main–2014]
(3) One p–d bond (1) It combines with oxygen to form nitrogen dioxide
(4) Three p–d bonds (2) It's bond order is 2.5
55. Bond distance in HF is 9.17 × 10–11 m. Dipole (3) It is diamagnetic in gaseous state
moment of HF is 6.104 × 10–30 Cm. The percent (4) It is a neutral oxide
ionic character in HF will be : 62. For which of the following molecule significant
(electron charge = 1.60 × 10–19 C) µ  0 :- [JEE–Main–2014]
[JEE–MAINS–2013] Cl CN
(1) 61.0% (2) 38.0%
(3) 35.5% (4) 41.5%
(1) (2)
56. The shape of IF6– is : [JEE–MAINS–2013]
Cl CN
(1) Trigonally distorted octahedron
OH SH
(2) Pyramidal
(3) Octahedral
(3) (4)
(4) Square antiprism
57. Which has trigonal bipyramidal shape ? OH SH
[JEE–MAINS–2013], Online (1) Only (3) (2) (3)and (4)
(1) XeOF4 (2) XeO3 (3) Only (1) (4) (1) Only (2)
(3) XeO3F2 (4) XeOF2 63. Among the following oxoacid, thh correct
decreasing order of acid strength is– [JEE–M014]
58. The catenation tendency of C, Si and Ge is in the
order Ge < Si < C. The bond energies (1) HClO4 > HClO3 > HC1O2 > HOC1
(in kJ mol–1) of C – C, Si –Si and Ge–Ge bonds (2) HC1O2 > HCIO4 > HC1O3 > HOCl
are respectively : (3) HOCl > HClO2 > HClO3 > HC1O4
[JEE–MAINS–2013]–(On–Iine) (4) HClO4 > HOCl > HClO2 > HClO3

www.jeebooks.in
1.50 CHEMICAL BONDING
64. The number and type of bonds in C22– ion in CaC2 71. Which of the following xenon–oxo compounds
are– [JEE–MAINS–2014–(On–line) may not be obtained by hydrolysis of xenon
(1) Two  bonds and two  – bonds fluoride XeF6 :– [JEE–MAINS–2014]
(2) Two  bonds and two  – bonds (1) XeO2F2 (2) XeO3
(3) One  bonds and two  – bonds (3) XeO4 (4) XeOF4
72. Which one of the following molecules is
(4) One  bond and one  – bond
paramagnetic - [JEE–MAINS–2014]
65. For the compounds
(1) NO (2) O 3
CH3Cl, CH3Br, CH3l and CH3F,
(3) N 2 (4) CO
the correct order of increasing C–halogen bond
73. Amongst LiCl, RbCl, BeCl 2 and MgCl2 the
length is: compounds with the greatest and the least ionic
(1) CH3F < CH3Br < CH3C1 < CH3I character, respectively are: [JEE–MAINS–2014]
(2) CH3F < CH3C1 < CH3Br < CH3I (1) RbCl and MgCl2 (2) LiCl and RbCl
(3) CH3C1 < CH3Br < CH3F < CH3I (3) MgCl2 and BeCl2 (4) RbCl and BeCl2
(4) CH3F < CH3I < CH3Br < CH3C1 74. Example of a three - dimensional silicate is :
66. Which of the following has unpaired electron(s) ? [JEE–MAINS–2014]
[JEE–MAINS–2014] (1) Beryls (2) Zeolites
(1) O 2 – (2) N22+ (3) Feldspars (4) Ultramarines
(3) O 2 2– (4) N 2 75. Which one of the following alkaline earth metal
67. In allene (C3H4), the type(s) of hybridization of the sulphates has its hydration enthalpy greater than
carbon atoms is (are): [JEE–MAINS–2014] its lattice entahalpy - [JEE–MAINS–2014]
2 (1) BeSO4 (2) SrSO4
(1) only sp (2) sp2 and sp
(3) CaSO4 (4) BeSO4
(3) sp and sp3 (4) sp2 and sp3
76. The intermolecular interaction that is dependent on
68. Shapes of certain interhalogen compounds are
the inverse cube of distance between the
stated below. Which one of them is not correctly
molecules is " [JEE–MAINS–2015]
stated? [JEE–MAINS–2014]
(1) London force
(1) IF7 : Pentagonal bipyramid
(2) Hydrogen bond
(2) BrF5: Trigonal bipyramid
(3) ion–ion interaction
(3) IC13: Planar dimeric
(4) ion–dipole interaction
(4) BrF3: Planar T–shaped 77. Which one has the highest boiling point ?
69. The correct order of bond dissociation energy [JEE–MAINS–2015]
among N2 , O2 , O2– is shown in which of the (1) Kr (2) Xe
following arrangements - (3) He (4) Ne
[JEE–MAINS–2014 (On–line)]
78. Which one of the following statements about water
(l) N2 > O2 > O2– (2) O2 , O2– , N2 is FALSE ? [JEE–MAINS–2016]
(3) N2 , O2– , O2 (4) O2– , O2 , N2 (1) Ice formed by heavy water sinks in normal water.
70. Which of the following molecules has two sigma (2) Water is oxidized to oxygen during
() and two pi () bonds :– photosynthesis.
[JEE–MAINS–2014, (On–line)] (3) Water can act both as an acid and as a base.
(1) HCN (2) C2H2Cl2 (4) There is extensive intramolecular hydrogen
(3) N2F 2 (4) C2H4 bonding in the condensed phase.

www.jeebooks.in
CHEMICAL BONDING 1.51
79. The species in which the N atom is in a state of sp 3. The common features among the species CN–, CO
hybridization is :– [JEE–MAINS–2016] and NO+ are - [IIT–2001]
(1) NO 2 (2) NO2+ (a) bond order three and isoelectronic
(3) NO 2 – (4) NO 3 – (b) bond order three and weak field ligands
80. The pair in which phosphorous atoms have a formal (c) bond order two and -acceptors
oxidation state of + 3 is: [JEE–MAINS–2016]
(1) Pyrophosphorous and pyrophosphoric acids (d) isoelectronic and weak field ligands
(2) Orthophosphorous and pyrophosphorous acids 4. Specify the coordination geometry aronnd and
hybridisation of N and B atoms in a 1:1 complex
(3) Pyrophosphorous and hypophosphoric acids
of BF3 and NH3 - [IIT 2001]
(4) Orthophosphorous and hypophosphoric acids
(a) N: tetrahedral sp3; B : tetrahedral sp3
81. Which intermolecular force is most responsible in
allowing xenon gas to liquefy? (b) N: pyramidal sp3; B : pyramidal sp3
[JEE–MAINS–2016 (On–line)] (c) N: pyramidal sp3; B : planar sp2
(1) ionic (d) N: pyramidal sp3; B : tetrahedral sp3d
(2) Instantaneous dipole– induced dipole 5. Specify hybridization of N and B atoms in a 1 : 1
(3) Dipole – dipole complex of BF3 and NH3 - [JEE 2002]
(4) Ion – dipole (a) N: tetrahedral sp3; B : tetrahedral sp3
83. The group of molecules having identical shape is: (b) N: pyramidal sp3; B : pyramidal sp3
[JEE–MAINS–2016]
(c) N: pyramidal sp3; B : planar sp2
(1) SF4, XeF4, CCl4
(d) N: pyramidal sp3; B : tetrahedral sp3d
(2) ClF3, XeOF2, XeF3+
(3) PC15, IF5 , XeO2F2 6. The nodal plane in the -bond of ethene is located
in [JEE 2002]
(4) BF3, PC13, XeO3
(a) the molecular plane
84. The bond angle H–X–H is the greatest in the
compound : [JEE–MAINS–2016 (On–line)] (b) a plane parallel to the molecular plane
(1) NH3 (2) H2O (c) a plane perpendicular to the molecular plane
(3) PH3 (4) CH4 which bisects, the carbon-carbon bond at right
angle
EXERCISE # V(B) (JEE-ADVANCED)
(d) a plane perpendicular to the molecular plane
1. The hybridisation of atomic orbitals of nitrogen in
which contains the carbon-carbon bond.
NO2+ , NO3– and NH4+ are : [IIT–2000]
2 3 2 7. Which of the following molecular species has
(a) sp , sp and sp respectively
unpaired electron(s) ? [JEE 2003]
(b) sp, sp2 and sp3 respectively
(a) N 2 (b) F 2
(c) sp2, sp and sp3 respectively
(c) O 2 – (d) O 2 2–
(d) sp2, sp3 and sp respectively
8. Which of the following are isoeIectronic and
2. The correct order of hybridization of the central atom
in the following species NH3 , [PtCl4]2–, PCl5 and isostructural ?
BCl3 is - [IIT–2001] NO3– . CO32–, ClO3–, SO3
(a) dsp2, sp3d, sp2 and sp3 (a) NO3– , CO32–
(b) sp3, dsp2, sp3d, sp2 (b) SO3, NO3–
(c) dsp2, sp2, sp3, sp3d (c) ClO3–, CO32-
(d) dsp2, sp3, sp2, sp3d (d) CO32–, SO3

www.jeebooks.in
1.52 CHEMICAL BONDING
9. According to molecular orbital theory which of the 16. Statement-1 : In water, orthoboric acid behaves
following statement about the magnetic as a weak monobasic acid. [JEE 2007]
character and bond order is correct regarding Statement-2 : In water, orthoboric acid acts as a
O2+ : [JEE 2004] proton donor.
(a) Paramagnetic and Bond order < O2 (a) Statement-1 is True, Statement-2 is True;
(b) Paramagnetic and Bond order > O2 Statement-2 is a correct explanation for
(c) Diamagnetic and Bond order < O2 Statement-1.
(d) Diamagnetic and Bond order > O2 (b) Statement-1 is True, Statement-2 is True;
10. Which species has the maximum number of lone Statement-2 is NOT a correct explanation for
pair of electrons on the central atom? Statement-1.
(a) ClO3– (b) XeF4 (c) Statement-1 is True, Statement-2 is False.
(c) SF4 (d) I3– [JEE 2005] (d) Statement-1 is False, Statement-2 is True.
11. The percentage of p-character in the orbitals 17. Statement-1 : Pb +4 compounds are stronger
forming P-P bonds in P4 is [JEE 2006] oxidizing agents than Sn4+ compounds
(a) 25 (b) 33 Statement-2 : The higher oxidation states for the
(c) 50 (d) 75 group 14 elements are more stable for the heavier
members of the group due to 'inert pair effect'.
12. Among the following, the paramagnetic compound
is [JEE 2007] [JEE 2008]
(a) Na2O2 (b) O 3 (a) Statement-1 is True, Statement-2 is True;
Statement-2 is a correct explanation for
(c) N 2 O (d) KO 2
Statement-1.
13. The species having bond order different from that
(b) Statement-1 is True, Statement-2 is True;
in CO is [JEE 2007]
Statement-2 is NOT a correct explanation for
(a) NO – (b) NO+ Statement-1.
(c) CN – (d) N 2
(c) Statement-1 is True, Statement-2 is False.
14. The structure of XeO3 is [JEE 2007]
(d) Statement-1 is False, Statement-2 is True.
(a) linear (b) planar
18. Match each of the diatomic molecules in coloumn I
(c) pyramidal (d) T-shaped with its property / properties in column - II
15. Statement-1 : p-Hydroxybenzoic acid has a lower Column-I Column-II
boiling point than o-hydroxybenzoic acid.
(a) B2 (P) paramagnetic
Statement-2 : o-Hydroxybenzoic acid has
(b) N 2 (Q) undergoes oxidation
intramolecular hydrogen bonding. [JEE 2007J
(c) O 2 – (R) undergoes reduction
(a) Statement-1 is True, Statement-2 is True;
Statement-2 is a correct explanation for (d) O 2 (S) Bond order 2
Statement-1. (T) Mixing of ‘s’ and ‘p’
(b) Statement-1 is True, Statement-2 is True; orbitals
Statement-2 is NOT a correct explanation for 19. The nitrogen oxides that contains N–N bonds is
Statement-1. are - [JEE 2009]
(c) Statement-1 is True, Statement-2 is False. (a) N 2 O (b) N 2O 3
(d) Statement-1 is False, Statement-2 is True. (c) N 2O 4 (d) N 2O 5

www.jeebooks.in
CHEMICAL BONDING 1.53
20. In the reaction [JEE 2009] 29. The total number of diprotic acids among the
2X + B2H6 [BX2(X2)]+ [BH4]– following is ?
the amines X is/are - H3PO4 H2SO4 H3PO3 H2CO3 H2S 2O 7
(a) NH3 (b) CH3NH2 H3BO3 H3PO2 H2CrO4 H2SO3
(c) (CH3)2NH (d) (CH3)3N 30. Among the following , the number of elements
21. The species having pyramidal shape is - showing only one non-zero oxidation state -
[JEE 2010] O, Cl, F, N, P, Sn, Tl, Na, Ti [JEE 2010]
(a) SO3 (b) BrF3 31. The difference in the oxidation numbers of the two
(c) SiO32– (d) OSF2 types of sulphur atoms in Na2S4O6 - [JEE 2011]
22. Assuming that Hunds rule is violated, the bond 32. The total number of lone-pairs of electrons in
order and magnetic nature of the diatomic melanin is. [JEE Adv. 2013]
molecule B2 is [JEE 2009]
33. Assuming 2s-2p mixing is NOT operative, the
(a) 1 and diamagnetic paramagnetic species among the following:-
(b) 0 and diamagnetic (a) Be2 (b) B2
(c) 1 and paramagnetic (c) C 2 (d) N 2
(d) 0 and paramagnetic
34. Match the orbital overlap figures shwon in List -
23. In allene (C3H4) the type(s) of hybridisation of the I with the description given in List - II and select
carbon atom(s) is/are - [JEE 2012] the correct answer using the code given below
3 2
(a) sp and sp (b) sp and sp the lists: [JEE-Adv.2014]
(c) only sp2 (d) sp2 and sp3 List - I List-II
24. Which ordering of compounds is according to the
decreasing order of the oxidation state
of nitrogen :- [JEE 2012] (P) (1) p - d  antibonding
(a) HNO3 , NO, NH4Cl, N2
(b) HNO3 , NO, N2, NH4C1
(c) HNO3 , NH4Cl, NO, N2 (Q) (2) d – d  bonding
(d) NO, HNO3, NH4C1, N2
25. The shape of XeO2F2 molecule is : [JEE 2012]
(a) Trigonal bipyramidal
(R) (3) p – d  bonding
(b) Square planar
(c) tetrahedral
(d) see-saw
Subjective : (S) (4) d – d  antibonding
26. The number of water molecules) directly bonded
to the metal centre in CuSO4.5H2O is :- P Q R S
[JEE 2012] (a) 2 1 3 4
27. Based on VSEPR theory, the number of 90 degree
(b) 4 3 1 2
F-Br-F angles in BrF5 is.
(c) 2 3 1 4
28. The value of n in the molecular formula
BenAl2Si6O18 is ? [JEE 2010] (d) 4 1 3 2

www.jeebooks.in
1.54 CHEMICAL BONDING
35. Among the triatomic molecules / ions, BeCl2, N3– , (a) The number of Cl=O bonds in (ii) and (iii)
N2O, NO2+ , O3, SCl2 , ICl2–, I3– and XeF2 the together is two
total number of linear molecules(s) / ion(s) where (b) The number of lone pairs of electrons on Cl in
the hybridization of the central atoms does not have (ii) and (iii) together is three
contribution from the d-orbital(s) is : (c) The hybridization of Cl in (iv) is sp3
[JEE Adv. 2015] (d) Amongst (i) to (iv), the strongest acid is (i)
(Atomic number : S = 16, CI = 17, I = 53 and 38. When O, is adsorbed on a metallic surface,
Xe = 54) electron transfer occurs from the metal to O2. The
36. The total number of lone pairs of electrons in N2O3 TRUE, statement (s) regarding this adsorption is
is : (are) [JEE Adv. 2015]
37. The correct statement(s) regarding, (a) O2 is physisorbed
(i) HClO, (ii) HC1O2, (b) heat is released
(iii) HC1O3 and (iv) HC1O4, is (are) (c) occupancy of 2p* of O2 is increased
(d) bond length of O2 is increased



www.jeebooks.in
COORDINATION COMPOUNDS
PROB L EM S B ASED ON GI VEN T OPI CS  Tetragonal distortion of octahedral complexes
 Double salt and coordination compounds (Jahn-Teller Distortion)
 Werner’s work  Square planar arrangements
 More recent methods of studying complexes  Tetrahedral complex
 Classification of ligands  Magnetism
 Effecive atomic number (EAN)  Extension of the crystal field theory to allow for
 Sidgwick EAN rule some covalency
 Shape of d orbitals  Nomenclature of coordination compounds
 Bonding in transition metal complexes  Isomerism
 Valence bodn theory  Polymerisation isomerism
 Crystal field theory  Ionisation isomerism
 Molecular orbitals theory  Hydrate isomerism
 Valence bond theory  Linkage isomerism
 Crystal field theory  Coordination isomerism
 Octahedral complexes  Coordination position isomerism
 Effects of crystal field splitting  Geometrical isomerism and optical isomerism

www.jeebooks.in
CHAPTER
2
Coordination Compounds

EXERCISE # I
 Only one correct answer : 5. On adding AgNO3 solution to a solution of
1. The formula of sodium nitropruside is Pt(NH3)3Cl4 , the percentage of total chloride ion
Na2[Fe(CN)5NO]. In sodium nitropruside, the precipitated is :-
oxidation state, coordination number and EAN of (a) 100 (b) 75
iron are respectively :- (c) 50 (d) 25
(a) +3, 6, 36 (b) +2, 6, 36 6. The effective atomic number of iron in complex
(c) +3, 3, 36 (d) 6, +3, 35 [(n5C5H5)Fe(CO)2Cl] :-
2. What is the oxidation state of Th in organometallic (a) 34 (b) 36
compound [ThHOR(C5Me5)2] ? (c) 38 (d) 32
(a) +1 7. If EAN of metal is 36 in [M(CO) 2
(b) +2 (C5H5)(C5H5)], then what is the atomic number
(c) +4 of metal M ?
(d) +3 (a) 23 (b) 24
3. CoCl3.4H2O is an anhydrous binary soluble (c) 25 (d) 26
compound. Hence its Werner’s representation is:- 8. What is the coordination number of central atom
Cl Cl of the complex [M(NH3)4SO4]Br which has one
Cl OH2 Cl OH2 ring ?
(a) 4 (b) 5
(a) Co OH2 (b) Cl Co OH2
(c) 6 (d) None
H2O Cl OH2 H2O Cl OH2 9. Oxidation state of Co in Ag[Co(CO)4] and Mo in
oxo complex species [Mo2O4(C2H4)(H2O)2]2– are
Cl OH2 respectively :-
(a) +1, +3 (b) –1, –3
(c) H2O Co OH2 (d) None
(c) +1, –3 (d) –1, +3
Cl Cl 10. In Tollen’s reagent, the oxidation number,
OH2 coordination number and effective atomic number
4. Concentrated H2SO4 will not dehydrate which of of central metal ion are respectively. (At. no. of
the following complex Ag = 47) :-
(a) [Cr(H2O)5Cl] Cl2.H2O (a) +1, +2, 50
(b) [Cr(H2O)6]Cl3 (b) +1, +2, 51
(c) [Cr(H2O)4Cl2] Cl.2H2O (c) +2, +1, 50
(d) All of these (d) +1, +1, 50

www.jeebooks.in
2.4 COORDINATION COMPOUNDS
11. Which will not give test of all the anions present in 17. Which of the following are examples of tetradentate
it ? ligand where all donor atoms are same ?
(a) K2Fe2(SO4)4.24H2O (a) N(CH2CH2NH2)3
(b) (NH4)2Fe(SO4)2.6H2O (b) N(CH2CO2–)3
(c) NH2(CH2)2NH(CH2)2NH(CH2)2NH2
(c) K3[Fe(CN)6]
(d) Both (a) & (c)
(d) All
18. In which case Pt has +2 oxidation state :-
12. In the compound CoCl3.4NH3 :-
(a) [Pt(NH3)4] [PtCl4] (b) [Pt(NH3)2Cl2]
(a) All the Cl shows primary valency only (c) [Pt(C2H4)Cl3]–1 (d) All
(b) Two Cl show primary valency and two Cl 19. KCN(excess) + CuSO4 Product (Complex).
shows secondary valency Complex is :-
(c) Two Cl show primary valency and one Cl (a) K2[Cu(CN)4] (b) K[Cu(CN)2]
shows primary valency as well as secondary (c) K3[Cu(CN)4] (d) None
valency 20. Mohr salt in aqueous solution gives :-
(d) 3 Cl shows primary valency and two Cl shows (a) 3 different ions (b) 2 different ions
primary valency as well as secondary valency (c) 4 different ions (d) 5 different ions
13. For which of the following complex, metal carbon 21. Which of the following statement is correct ?
linkage is present :- (a) NH2– will be named as amido
(a) Cr(acac)3 (b) Ni(DMG)2 (b) SO42– will be named as sulphito
(c) Fe(CO)5 (d) [Co(C2H5O)6]3– (c) CH3– will be named as methylo
14. Which of the following ligand can act as chelating (d) SnCl3– will be named as trichloro stannito
ligand with more than one chiral centre ? 22. Which of the following ligand forms only two ring
(a) 1, 2 propanediamine (pn) towards a central atom ?
(a) en (b) bn
(b) 2,3 butane diamine (bn)
(c) dien (d) trien
(c) Nitrilotriacetato(nta–3)
23. For which of the following complex, chelate is not
(d) glycinato formed ?
15. Which one of the following is example of 1 : 3 (a) Bis(dimethyl glyoximato)nickel(II)
electrolyte :- (b) Potassium ethylene diamine tetrathiocyanato
(a) PtCl4 . 6NH3 (b) CrCl3 . 6NH3 chromate(III)
(c) CoBr3 . 5NH3 (d) PtCl4 . 3NH3 (c) Pentaammine carbonato cobalt(III) nitrate
16. Which of the following are examples of hexadentate (d) Trans-diglycinatoplatinum(II)
chelating ligand where four oxygen atoms and 24. Which of the following pair of molecule have
two nitrogen atoms act as donor atoms ? identical shape ?
(a) [NiCl4]2– & XeF4 (b) [Ag(NH3)2]+1 , H2S
(a) [EDTA]4–
(c) [Zn(H2O)4]2+, SiCl4 (d) Fe(CO)5 , XeOF4
N(CH2CO2–)2
(b) 25. A complex of Pt, ammonia & chlorine produces
N(CH2CO2–)2 four ions per molecule in the solution. The solution
consistent with the observation is :-
(c) both (a) and (b)
(a) [Pt(NH3)5Cl]Cl3 (b) [Pt(NH3)4Cl2]Cl2
(d) None
(c) [Pt(NH3)3Cl3]Cl (d) All of these

www.jeebooks.in
COORDINATION COMPOUNDS 2.5
2+
26. [M(H2O)6] complex typically absorbs light at 33. The ratio of trans and cis isomers of the complex
around 600 nm. It is allowed to react with ammonia [Ma2bcde] ?
to form a new complex [M(NH3)6]2+. It will absorb
(a) 2 : 3 (b) 2 : 1
light at around :-
(c) 1 : 2 (d) 3 : 5
(a) 800 nm (b) 620 nm
(c) 580 nm (d) 320 nm 34. Which of the following can show geometrical
27. Arrange the following compound according to isomerism -
decreased C–C bond length :- (a) [Co(en)2(NH3)Cl]
(I) C2F4 (b) (Ph3P)ClPdCl2PdCl(PPh3)
(II) C2H4 (c) Both (a) and (b)
(III) [PtCl3(C2H4)]–1 (d) K4[Fe(CN)6]
(a) I > II > III (b) III > II > I
35. Which will give a pair enantiomers (which are
(c) II > I > III (d) II > III > I optically active) ?
28. 1 mole of which complex produce minimum number
(a) [Co(NH3)4Cl2]NO2
of K+ ions on dissociation :-
(a) Potassium tris(oxalato)coblaltate(III) (b) [Cr(NH3)6][Co(CN)6]
(b) Potassium tetrahydroxido zincate(II) (c) [Co(en)2Cl2]Cl
(c) Potassium hexacyanidoferrate(II) (d) [Pt(NH3)4]PtCl6]
(d) Potassium ammine penta chlorido platinate(IV) 36. For the complex, K[CoBr2(CN) (NO2) (NH3)
29. Facial and Merdional isomers are associated with (H2O)]
which of the general formula of complex - X = Number of possible stereoisomer where both
A,B,C,D are example of monodentate ligand. Br are at cis position
& AA is example of bidentate ligand. Y = Number of possible stereoisomer where both
(a) [M(AA)2] (b) [M(AA)3] Br are at trans position
(c) [MABCD] (d) [MA3B3]
Find out X & Y :-
30. How many geometrical isomers possible for

(a) 6, 3 (b) 3, 6
[Pd(NH2 – CH – CO2 )2]
(c) 12, 3 (d) 3, 12
Cl 37. A complex with the composition [MA3B]x± is
(a) 2 (b) 3 found to have no geometrical isomers. The possible
(c) 4 (d) 6 structures of the complex is (where A and B are
–1 monodentate ligands) :-
31. In [Fe(EDTA)] , which of the following statements
are correct ? (a) Tetrahedral (b) Square planar
(a) It shows optical isomerism (c) Octahedral (d) (a) & (b) both
(b) C.N. of Fe+3 is 6
38. Select incorrect option code about complex
(c) 5 chelates are formed in the complex
[Cr(NO2)(NH3)5] [ZnCl4] :-
(d) All
(a) IUPAC name of the compound is pentaammine
32. A coordination complex of type Mx2y2 (M metal
nitrito-N-chromium(III) tetrachlorido
ion ; x, y monodentate ligands). can have either a
zincate(II)
tetrahedral or a square planar geometry. The number
of structures in these two cases are respectively:- (b) It shows geometrical isomers
(a) 1 & 2 (b) 2 & 1 (c) It shows linkage isomerism
(c) 1 & 3 (d) 3 & 2 (d) It shows coordination isomerism

www.jeebooks.in
2.6 COORDINATION COMPOUNDS
39. Select the correct code regarding total number of 44. Total possible geometrical isomers and
stereoisomers for the following compounds :- stereoisomers are existing respectively for
[Ma3b2c] (I) ; [M(AB)3]x± (II) [Ma2b2c2]x± [M(gly)2(en)] type octahedral complex :-
I II III (a) 3, 6 (b) 3, 4
(a) 4 4 6 (c) 4, 8 (d) 3, 4
(b) 4 3 5 45. How many coordination isomers of
(c) 3 3 5 [Pt(NH3)4][PtCl4] showing geometrical isomers :-
(d) 3 4 6 (a) Four (b) One
40. For the complex dichlorido bis(ethylenediamine) (c) Two (d) Zero
cobalt(III) ion, 46. Which of the following ions are optically active
Select the correct statement :- (exists as pair of enantiomers) ?
en en
(a) It has three isomers, two of them are optically
active and one is optically inactive. Cl +1 Cl +1
(b) It has three isomers, all of them are optically (a) Co (b) Co
active
Cl Cl
(c) It has three isomers, all of them are optically en
inactive en
X Y
(d) It has one optically active isomer & two en
geometrical isomer. 3+
Cl en +1
41. When [Co(CO 3 )(NH 3 ) 4 ] + is reacted with en Co Co
concentrated HCl, the violet coloured (c) (d)
[CoCl2(NH3)4]Cl compound is formed. Predict the Cl
geometry of the product formed. en en
(a) Only cis isomer Z W
(b) Only trans isomer (a) X only (b) Y only
(c) Cis and trans isomer both (c) Y & Z both (d) W only
(d) Can’t be predicted 47. The complex has two optical isomers which are
related as enantiomers. Their correct structures
42. Which of the following complex only show linkage
are-
isomerism ?
(a) [Co(NH3)4(H2O)Cl]CN Br Br
I I
(b) [Co(NH3)5(SCN)]2+
(c) [Co(H2O)6]Cl3 (a) en M ; M en
(d) [Co(NH3)5SO4]Br I I
43. Which of the following complex only show Br Br
ionisation isomerism ? (among all type of structural I I
isomerism) Br Br
(a) [Co(NH3)4(H2O)Cl]Br M ; M
(b) en en
(b) [Co(NH3)5(SCN)]2+
(c) [Co(H2O)6]Cl3 Br Br
I I
(d) [Co(NH3)5Br]Br2

www.jeebooks.in
COORDINATION COMPOUNDS 2.7
en
Br en Br
(b) Linkage isomer and ionisation isomer respectively
(c) Ionisation and coordination isomer respectively
(c) M ; M (d) Coordination isomer and coordination position
isomer respectively
I Br Br I
I I 54. Which of the following isomers of [M(NH3)2Cl2]
Br Br would react with silver oxalate Ag2C2O4 ?
Br Br (a) H3N Cl
(d) en M ; M en M
I I H3N Cl
I I (b) H3N Cl
48. Find complex which has maximum number of
geometrical isomers ? M
(a) Ma3b3 (b) Ma3b2c
(c) Ma2b2c2 (d) M(AA)a2b2 Cl NH3
49. The formula for the compound tris (ethane-1,2- (c) Both
diamine)cobalt(III) sulphate is - (d) None
(a) [Co(en)3]SO4 (b) [Co(SO4)4(en)3] 55. In Zieses salt C–C bond length is -
(c) [Co(en)3 (SO4)2] (d) [Co(en)3]2(SO4)3 (a) 1.37°A (b) 1.19°A
50. For which of the following dn configuration , (c) 1.87°A (d) 1.34°A
octahedral complexes can not exist in both high 56. Identify incorrect statement for the behaviour of
and low spin forms - ethane 1,2-diamine as a ligand.
I. d3 II. d5 III. d6 IV. d8 (a) It is a neutral ligand
(b) It is a didentate ligand
(a) I, II (b) I, IV
(c) It is a chelating ligand
(c) III, IV (d) II, III
(d) It is a unidentate ligand
51. Correct option regarding complex [Co(en)3]2
(SO4)3 :- 57. The stabilisation of coordination compounds due
to chelation is called the chelate effect. Which of
(a) low spin inner orbital octahedral complex with the following is most stable complex species ?
d2sp3 hybridisation
(a) Fe(CO)5 (b) [Fe(CN)6]3–
(b) optically active (c) [Fe(C2O4)3]3– (d) [Fe(H2O)6]3+
(c) C2 axis of symetry is present 58. Which of the following complex formed by Cu2+
(d) All ion is most stable ?
52. In which ligand, only two nitrogen acts as donor (a) Cu2+ + 4NH3 [Cu(NH3)4]2+log k = 11.6
site ? (b) Cu2+ + 4CN– [Cu(CN)4]3– log k = 27.3
(a) nta3– (b) NH3 (c) Cu2+ + 2en [Cu(en)2]2+ log k = 15.4
(c) en (d) H2N–CH2–CO2– (d) Cu2+ + 4H2O [Cu(H2O)4]2+ log k = 8.9
53. [Ru(NH3)5(NO2)]Cl vs [Ru(NH3)5Cl]NO2 & 59. The correct IUPAC name of [Pt(NH3)2Cl2] is -
[Ru(NH3)5(NO2)]Cl vs [Ru(NH3)5(ONO)]Cl are (a) Diamminedichloridoplatinum(II)
related as - (b) Diamminedichloridoplatinum(IV)
(a) Ionisation isomer and linkage isomer (c) Dichloridoammineplatinum(II)
respectively (d) Diamminedichloridoplatinum(0)

www.jeebooks.in
2.8 COORDINATION COMPOUNDS
60. The colour of the coordination compound depend 67. What kind of isomerism exist between violet colour
on the crystal field splitting. What will be the correct [Cr(H2O)6]Cl3 & green colour [Cr(H2O)5Cl]Cl2 .
order of absorption of wavelength of light in the H2O.
visible region for complex ? (a) Linkage isomerism
[Co(NH3)6]3+ ; [Co(CN)6]3– ; [Co(H2O)6]3+ (b) Solvent isomerism
(a) [Co(CN)6]3– > [Co(NH3)6]3+ > [Co(H2O)6]3+
(c) Ionisation isomerism
(b) [Co(NH3)6]3+ > [Co(H2O)6]3+ > [Co(CN)6]3–
(d) Coordination isomerism
(c) [Co(H2O)6]3+ > [Co(NH3)6]3+ > [Co(CN)6]3–
68. The CFSE for octahedral [CoCl6]4– is 18000 cm–1 .
(d) None The CFSE for tetrahedral [CoCl4]2– will be -
61. The type of isomerism arises due to presence of (a) 18000 cm–1 (b) 16000 cm–1
ambidentate ligands attached to central metal ion
is (c) 8000 cm–1 (d) 20000 cm–1
(a) Linkage isomerism (b) Ligand isomerism 69. [Co(NH3)5(CO3)]ClO4 , for this complex the
coordination number, oxidation number, number
(c) Hydrate isomerism (d) Ionisation isomerism
of d-electrons and number of unpaired d electrons
62. The type of isomerism arises due to presence of
are respectively -
isomeric ligands is
(a) 4, 2, 6, 0 (b) 6, 3, 6, 0
(a) Ionisation isomerism
(c) 4, 2, 6, 3 (d) 6, 3, 6, 3
(b) Coordination position isomerism
70. Select correct statement for [Cu(CN) 4] 3– ,
(c) Ligand isomerism
[Cd(CN)4]2– , [Cu(NH3)4]2+ :-
(d) Linkage isomerism
(a) Both [Cd(CN)] 42– & [Cu(NH 3)4] 2+ have
63. [Fe(CN 6 )] 3– ion shows magnetic moment
square planar geometry
coressponding to -
(b) [Cu(CN)4] 3– & [Cu(NH3)4] 2+ have equal
(a) 2 unpaired electrons
number of unpaired electron
(b) 3 unpaired electrons
(c) [Cu(CN)] 4 3– & [Cd(CN) 4 ] 2– can be
(c) 1 unpaired electron
distinguished by passing H2S gas.
(d) None
(d) All complexes have magnetic moment equal to
64. Toxic metal ions are removed by chelating ligand. zero.
The chelating ligand used can be -
71. Which of the following cation is colourless in
(a) en (b) pn aqueous solution ?

(c) H2N–CH2–CO2 (d) EDTA
(a) Ni2+ (b) Co3+
65. Complex [Fe(H2O)6]3+ ion has magnetic moment-
(c) Cr3+ (d) Ti4+
(a) 2.83 B.M. (b) 3.87 B.M.
72. What is the magnetic moment of high spin
(c) 4.92 B.M. (d) 5.92 B.M. octahedral complex that has 6 electron in 3d orbitals?
66. A chelating ligand has two or more than two donor
(a) 0 B.M. (b) 24 B.M.
atoms to bind to a single metal ion. Which of the
following is not a chelating ligand ? (c) 8 B.M. (d) 15 B.M.
73. For the complex, [Fe (en)3]n [CoIII(NH3)2(NO2)4]2
II

(a) N (b) Oxalato the value of n is -


(a) 1 (b) 2
(c) Glycinato (d) Ethan-1, 2 diamine
(c) 3 (d) 4

www.jeebooks.in
COORDINATION COMPOUNDS 2.9
74. In which of the following pair , hybridisation , 81. The spin magnetic moment of K3[Fe(C2O4)3] is :-
magnetic moment and geometry are same :- (a) 15 B.M. (b) 35 B.M.
(a) [IrF6]3– , [Co(NH3)6]3+
(c) 24 B.M. (d) 2 B.M.
(b) [CrF6]3– , [Co(H2O)6]2+
82. What is the magnetic nature of Co(II) in tetrahedral
(c) [Cu(NH3)4]2+ , [Ni(NH3)4]2+ and octahedral complexes respectively ?
(d) [Co(H2O)6]3+ , [Co(H2O)6]4– (a) Paramagnetic, Paramagnetic
75. What is the oxidation state, magnetic moment of (b) Paramagnetic, Diamagnetic
central metal cation in the following oxo complex (c) Diamagnetic, Diamagnetic
[Os(ONO)(O)2(O2)(SCN)(H2O)]OH ? (d) Diamagnetic, Paramagnetic
(a) +8, 15 B.M. (b) +6 ; 0 83. For octahedral complex the value of the spin only
(c) +6, (d) +8, 0 BM magnetic moment for one of the following
35 B.M.
configuration is 2.84 B.M. The correct one is :-
76. If 10Dq0 < P then crystal field splitting energy for
(a) d4 (strong field ligand)
d7 ion in octahedral field will be :-
(b) d4 (weak field ligand)
(a) 20 Dq – 2P (b) 4 Dq
(c) d3 (in weak as well as strong field ligand)
(c) 8 Dq (d) 24 Dq – 2p (d) d5 (strong field ligand)
77. Which of the following complex is low spin as well CH2–CH2–NH2
as diamagnetic ? 84. CrCl . 3 CH2 shows which
(a) K4[Fe(CN)6] (b) [Co(C2O4)3]3– NH2
type of isomerism :-
(c) [Ni(H2O)6]2+ (d) both (a) & (b)
(a) Ligand isomerism
78. Correct statements about complex consisting Ni, (b) Polymerisation isomerism
NH3, H2O and NO3– ion -
(c) Linkage isomerism
(a) If magnetic moment is zero, then the formula of (d) Ionisation isomerism
the complex will be [Ni(NH3)4](NO3)2 . 2H2O
85. Which of the following property of [Ni(H2O)6]SO4
(b) If magnetic moment is 2 2 & conducts can not be explained by valence bond theory ?
electricity, then the formula of the complex is (a) It has 6 M-L coordinate bond
[Ni(NH3)4(H2O)2](NO3)2 BM
(b) Geometry of Ni2+ is octahedral
(c) The higher and lower value of magnetic moment (c) Total six Ni–O linkage is present
of the given complex corresponds to
(d) It is green solution
octahedral and square planar complex.
86. The correct order of magnetic moment (spin only
(d) All magnetic moment in BM) among is :-
79. Which of the following complex is paramagnetic (a) [Fe(CN)6]4– > MnCl42– > CoCl42–
as well as square planar ? (b) MnCl42– > CoCl42– > [Fe(CN)6]4–
(a) NiCl42– (b) [Cu(CN)4]3– (c) MnCl42– > [Fe(CN)6]4– > CoCl42–
(c) [Cu(DMG)2] (d) [PtBr4]2– (d) [Fe(CN)6]4– > CoCl42– > MnCl42–
80. Which of the following complex has lowest number 87. Which of the following is paramagnetic ?
of unpaired electron ? (a) K2[NiF6]
(a) [NiF6]2– (b) [Cu(NH3)4]2+ (b) [Co(H2O)6]3+
(c) [CoF6]3– (d) [Fe(EDTA)]–1 (c) K4[Fe(CN)5O2] [O22– = Peroxido]
(d) None of these

www.jeebooks.in
2.10 COORDINATION COMPOUNDS
88. For which complex compound, optical activity 92. In NiCl42– , the magnetic moment is :-
does not depend upon the orientation of the ligand (a) 2.82 B.M. (b) 1.41 B.M.
arround metal cation :- (c) 1.82 B.M. (d) 5.46 B.M.
(a) [CoCl3(NH3)3]
93. Geometrical isomerism is found in coordination
(b) [Co(en)3]Cl3 compounds having coordination number :-
(c) [Co(C2O4)2(NH3)2]– (a) 2 (b) 3
(d) [CrCl2(NH3)2en] (c) 4 (tetrahedral) (d) 6
89. Which of the following complex is paramagnetic
94. Cis trans isomerism is found in square planar
as well as low spin complex ?
complexes of molecular formula (a & b are
(a) [Cr(CN)6]4– (b) [Ni(NH3)6]2+ monodentate ligand) :-
(c) [Fe(CN)6]4– (d) [Co(H2O)6]3+ (a) Ma4 (b) Ma3b
Br
+1 (c) Ma2b2 (d) Mab3
Br
95. The complexes [Pt(NH 3 ) 4 ] [PtCl 6 ] &
90. Co en ;
[Pt(NH3)4Cl2] [PtCl4] are :-
H3 N (a) Linkage isomers
NH3 (b) Optical isomers
I
+1
(c) Coordination isomers
Br
(d) Ionisation isomers
H3 N
Co en ; 96. [Sc(H2O)6]3+ ion is :-
(a) Coloured and paramagnetic
H3N
(b) Colourless and paramagnetic
Br
II
(c) Colourless and diamagnetic
Br +1 (d) Coloured and octahedral
Br 97. Maximum paramagnetism in 3d series is shown
en Co by:-
(a) Mn (b) Co
NH3
NH3 (c) Ni (d) Fe
III 98. Which of the following complexes has geometry
Incorrect statement are :- different from others :-
(a) I, II are G.I. (a) [NiCl4]2– (b) [Ni(CO)4]
2–
(b) II, III are optical isomers (c) [Ni(CN)4] (d) [Zn(NH3)4]2+
(c) I, III are optical isomers 99. Which of the following is a high spin complex ?
(d) II & III are Geometrical isomers (a) [Co(NH3)6]3+ (b) [Fe(CN)6]4–
91. Which of the following option has maximum number (c) [Ni(CN)4]2– (d) [FeF6]3–
of unpaired electrons?
100. In the complex Co(NH3)3Cl3 chloride ion satisfy :-
(a) A tetrahedral d6 ion
(a) only primary valency
(b) [Co(H2O)6]3+
(b) only secondary valency
(c) A square planar complex with d7 ion
(c) both primary and secondary valencies
(d) A coordination compound with magnetic
(d) None
moment of 5.92 B.M.

www.jeebooks.in
COORDINATION COMPOUNDS 2.11
+4
101. A complex cation is formed by Pt ion with ligands. 106. A complex Kn[MnF6] has magnetic moment 4.9
Which of the following can be its correct IUPAC B.M. What is the value of n -
name :-
(a) 2 (b) 3
(a) Diammineethylenediaminedithiocyanato-s
(c) 4 (d) 5
platinum(II)ion
(b) Diamineethylenediamminedithiocyanato-s 107. WhiCh of the following is copper (I) compound :-
platinum(IV)ion (a) [Cu(H2O)4]2+ (b) [Cu(CN)4]3–
(c) Diamminebis(ethylenediamine)dithiocyanato-s (c) [Cu(NH3)4]2+ (d) All of these
platinum(IV)ion 2+
108. Cu (aq) + KCN(aq) (excess)  Complex X1
(d) Diammineethylenediaminedithiocyanato-s Co2+(aq) + KNO2(s) + H+(aq)  Complex X2
platinum(IV)ion
Zn2+(aq) + NaOH(aq) (excess)  Complex X3
102. The number of geometrical isomers possible for
complex [M(gly)2Br2] is - X1 , X2 , X3 are respectively :-
(a) 3 (b) 5 (a) Tetrahedral , Octahedral and Square planar
(c) 7 (d) 9 (b) Tetrahedral , Octahedral and Tetrahedral
103. If NO reacts with [Cr(CO)6] , how many CO (c) Square planar, Octahedral and Tetrahedral
groups can be replaced by NO ? (d) Octahedral, Tetrahedral and Square planar
(a) All 6 CO groups are replaced by 6 NO groups. 109. Which of the following is not correctly matched ?
(b) 4 CO groups are replaced by 6 NO groups. (a) Sodium (ethylenediaminetetraacetato)
(c) 2 CO groups are replaced by 3 NO groups. chromate(II) : Na2[Cr(CH3COO)4(en)]
(d) All the 6 CO groups are replaced by 4 NO groups. (b) Dichloridobis(ethane-1,2-diamine)cobalt(III)
104. During developing of a photographic plates , one ion : [Co(en)2Cl2]+
of the step involves washing the plate with hypo
(c) Tris(bipyridine) iron(II)ion : [Fe(NH4C5 –
solution. The purpose of this is to :-
C5H4N)3]2+
(a) Reduce reacted AgBr to Ag
(d) Ammineaquadibromidocopper(II) :
(b) Convert unreacted AgBr to silver thiosulphate
[Cu(H2O)(NH3)Br2]
complex [Ag(S2O3)2]3–
(c) Convert unreacted AgBr to Ag2SO4 110. Which of the following has total 18 valence
electron:-
(d) Convert reacted AgBr to Ag2S2O3
4 S2 O82–
(a) [Fe(CO)3(4–C8H8)]
105.  NH 3 5 Co  O  O  Co  NH 3 5  

Green colour complex
Oxidised
(b) [Cr(6–C6H6)2]
5 (c) [Mo(CO)3(6–C7H7)]+1
 NH 3 5 Co  O  O  Co  NH 3 5 
Brown colour complex
(d) All
The magnetic moment of green complex is 1.73 111. Jahn-Teller distortion in octahedral field will occur
B.M. and for brown complex magnetic moment is for :-
zero. (O–O) is same in all replaced in both the (a) d1 ion (b) d4 ion
complex. The oxidation state of Co in brown
(c) d9 ion (d) All
complex and green complex respectively are :-
(a) III, II ; IV, III (b) III, II ; III, III 112. Jahn-Teller distortion in octahedral field will not
occur for :-
Brown Green Brown Green
(c) III, III ; III, II (d) III, IV ; III, III (a) d3 ion (b) d8 ion
Brown Green Brown Green (c) d5 ion (d) (a) & (b) both

www.jeebooks.in
2.12 COORDINATION COMPOUNDS
113. Which of the following is incorrect about :- n1 and n2 are :-
Tetraamminedithiocyanato-Scobalt(III) (a) 4 & 6 (b) 7 & 3
tris(oxalato)cobaltate(III) (c) 3 & 7 (d) 6 & 4
(a) Formula of the complex is [Co(SCN) 2 119. The order of 0 :-
(NH3)4][Co(Ox)3]
(a) Co3+ > Rh3+ > Ir3+ (for same ligand)
(b) It is a chelating complex and show linkage
isomerism (b) F– > Br– > Cl– (for same metal ion)
(c) It shows optical isomerism (c) CN– > NH3 > S2– (for same metal ion)
(d) It shows geometrical isomerism (d) Fe2+ > Fe3+ (for same ligand)
114. Irrespective of the nature of ligand, inner orbital 120. If the splitting energy of octahedral field is 0 and
octahedral complex is formed by :- if the splitting energy of tetrahedral field is t then :-
(a) d1 ion (b) d2 ion (a) 0 = t (b) 0 < t
(c) d3 ion (d) All 2 4
2– (c) 0 = t (d) t = 0
115. CoCl4 is observed to be tetrahedral whereas 3 9
CoCl64– is observed to be high spin octahedral 121. In which of the following reactions, correct product
complex. The configuration in tetrahedral field and is given ?
octahedral field are respectively :- 2+
2 1
H3 N Cl +en H3 N NH2 CH2
5 6
(a) e4t23 ; t e
2g g (b) e4t23 ; t e2g g (a)
Pt Pt
H3 N Cl H3 N NH2 CH2
1 2
(c) t24e3 ; t 62g eg (d) t24e3 ; t 52g eg
H3N Cl +H2C2O4
H3 N O CO
116. Incorrect statement regarding dichromate ion :- (b) Pt
–2HCl
Pt
H3N Cl H3 N O CO
(a) Six Cr–O bond lengths are identical
en en
(b) Cr in dichromate ion is sp3 hybridisation Cl O CO
(c) the colour is due to d–d transition +H2C2O4
(c) Co
–2HCl
Co
(d) the colour is due to LM charge transfer Cl O CO
(where M - Metal ion, L - Ligand) en en
(d) All
117. In crystal field theory, each of p orbitals in
octahedral field :- 122. In which of the following reactions, correct product
(a) equally face two ligands along the coordinate is given :-
O
axis
Cl O–C–CO2H
(b) unequally face two ligands along the coordinate
axis (a) en Cr en
+2H2C2O4
en Cr en
–2HCl
(c) equally face two ligands in between the
coordinate axis Cl O–C–CO2H
(d) unequally face two ligands in between the O
O
coordinate axis
H3N Cl H3N O–C–CO2H
eg +2H2C2O4
(b) Pt
–2HCl
Pt
n1Dq CO2H–C–O
Cl NH3 NH3
118. Degenerate
O
d-orbitals H 3N Cl H3N NH2–CH2CO2H
n2Dq (c) 2H2NCH2CO2H
Pt Pt
–2HCl
t2g Cl NH3 HO2C–H2C–H2N NH3
Splitting in Octahedral field (d) All

www.jeebooks.in
COORDINATION COMPOUNDS 2.13
123. For Xe[PtF6] ; oxidation state of Xe and Pt are 129. Which of the following complex can have optical
respectively :- isomer ?
(a) +4, +2 (b) +2, +4 (a) [Pt(bn)2]2+ (b) [PtCl4]2–
(c) +1, +5 (d) +6, 0 (c) [Pt(en)2]2+ (d) [Pt(NH3)3Cl]1+
124. Name the following coordination species 130. Geometrical isomerism can be shown by :-
Cl NH3 (a) [Ag(CN)(NH3)] (b) Na2[Cd(NO3)4]
:- 2–
Zn (c) [PtCl4I2] (d) [PtCl(NH3)3]
Cl NH3 131. Which of the following options is correct :-
(a) Cis-diamminedichloridozinc(II) (a) Pi-acid ligands favour spin pairing
(b) Trans-diamminedichloridozinc(II) (b) In low spin complex, the number of unpaired
(c) Diamminedichloridozinc(II) electrons is equal to that present in the free ion
(d) Dichloridodiamminezinc(II) (c) In high spin complex, the number of unpaired
NH3 electron is minimum
Cl (d) Ti(H2O)63+ ; V(H2O)63+ ; Cr(H2O)63+ all are
H3N
sp3d2 hybridised
125. Pt Cl IUPAC name of given 132. [Cr(H2O)6]3+ ; [Fe(H2O)6]3+ ; [Co(H2O)6]3+ are:-
Cl Cl (a) sp3d2 ; sp3d2 ; sp3d2 hybridised
NH3 (b) d2sp3 ; d2sp3 ; d2sp3 hybridised
complex :- (c) d2sp3 ; sp3d2 ; d2sp3 hybridised
(a) Cis-triamminetrichloridoplatinum(IV)chloride (d) sp3d2 ; d2sp3 ; sp3d2 hybridised
(b) Trans-triamminetrichloridoplatinum(IV)chloride 133. In [Cu(NH3)4]2+ , dsp2 hybridisation takes place
(c) Fac-triamminetrichloridoplatinum(IV)chloride for Cu2+ ion . The d orbital involved is :-
(d) Mer-triamminetrichloridoplatinum(IV)chloride (a) (n–1)dz2 (b) (n–1)dx2 – y2
126. IUPAC name of Xe[PtF6] is :- (c) (n–1)dxy (d) (n–1)dxz
(a) Xenoniumhexafluoridoplatinum(V) 134. [BeCl4] ; [MnCl4] ; [CoCl4]2– all are sp3
2– 2–

(b) Xenonhexafluoridoplatinate(V) hybridised with :-


(c) Xenonhexafluoridoplatinate(VI) (a) 2,5,3 unpaired electron respectively
(b) 0,5,2 unpaired electron respectively
(d) Hexafluoridoxenoniumplatinum(VI)
(c) 1,5,1 unpaired electron respectively
127. Type of isomerism which may be possible for six
coordinated complex of metal M having formula (d) 0,3,3 unpaired electron respectively
Cr(NO2)2 . 6H2O :- CH3CHNH2 CH2CH2NH2
135. If pn = and tn = then
(a) Hydrate isomerism CH2NH2 CH2NH2
(b) Linkage isomerism CoCl2(pn)2 and CoCl2(tn)2 are example of :-
(c) Geometrical isomerism (a) ligand isomerism
(d) All of them (b) linkage isomerism
128. [NiCl2(PMe3)2] is a paramagnetic complex of (c) coordinate position isomerism
Ni(II). How many isomers are possible of (d) ionisation isomerism
analogous diamagnetic complex of Pd2+ :-
136. Which of the following is optically active :-
(a) Zero (b) Five
(a) [Zn(gly)2]–1 (b) [Ca(EDTA)]–2
(c) Two (d) Three
(c) [Co(Ox)3]–3 (d) All of these

www.jeebooks.in
2.14 COORDINATION COMPOUNDS
137. For the complex ion, dichloridobis (ethylenediamine) 146. [Fe(H2O)6]2+ has crystal field splitting energy value
cobalt(III), which of the following is incorrect:- 10,400 cm –1 and pairing energy value
(a) it has two optically active isomers 17,600 cm–1 then it is :-
(b) it has two optically inactive isomers (a) low spin complex
(c) it is example of cationic complex (b) paramagnetic in nature
(d) it is example of octahedral complex (c) diamagnetic in nature
138. In [Cu(NH3)4]2+ outermost electron is present in :- (d) none of these
(a) 3d orbital (b) 4s orbital 147. An ion M2+ forms the complexs [M(H2O)6]2+,
(c) 4p orbital (d) 4d orbital [M(en)3]2+ and [MBr6]4–, match the complex with
the appropriate colour :-
139. The type of hybridisation of Fe(CO)5 is :-
(a) Green , blue and red
(a) dx2–y2 sp3 (b) sp3 dx2–y2
(b) Blue , red and green
(c) dz2 sp3 (d) sp3 dz2
(c) Green , red and blue
140. The hybridisation of metal ion and shape of
[CuCl5]3– are respectively :- (d) Red , blue and green
(a) dx2–y2 sp3 , square pyramidal 148. Which one of the following species does not
(b) sp3 dx2–y2 , trigonal bipyramidal represent cationic species of vanadium formed in
aqueous solution :-
(c) dz2 sp3 , trigonal bipyramidal
(a) VO2+ (b) VO2+
(d) sp3 dz2 , tetrahedral
(c) [V(H2O)6]3+ (d) VO22+
141. VO43– :-
149. In which of the following C–O bond length is
(a) square planar geometry with dsp2 hybridisation
shorter as compared to CO.
(b) tetrahedral geometry with sp3 hybridisation
(a) CO+ (b) H2CO
(c) tetrahedral geometry with d3s hybridisation
(c) [Ni(CO)4] (d) [V(CO)6]–1
(d) None
150. PtCl4 . 3NH3 is example of :-
142. In crystal field splitting diagram for square planar
(a) 1 : 1 electrolyte (b) 1 : 2 electrolyte
complex the d-orbital having highest energy is :-
(c) 1 : 3 electrolyte (d) 1 : 4 electrolyte
(a) dx2–y2 (b) dxy
(c) dz2 (d) dxz & dyz EXERCISE # II
143. In crystal field splitting diagram for square planar  One or More Than One Correct Answer :
complex the d-orbital having least energy is :-
1. To fom a coordination bond, one needs a ligand.
(a) dx2–y2 (b) dxy
Which of the following species can not be a ligand?
(c) dz2 (d) dxz & dyz
(a) NH4+ (b) PH4+
144. Among the following the compound that is both
(c) H3O+ (d) NO +
paramagnetic and coloured is :-
2. Which of the following are examples of bidentate
(a) K2Cr2O7 (b) (NH4)2[TiCl6]
ligand ?
(c) VOSO4 (d) K3[Cu(CN)4]
(a) C2O42– (b) en
145. Which of the following is correct electronic
(c) Glycinato (d) NH2–NH2
configuration of 3d orbital in excited state of central
metal ion, when [Ti(H2O)6]3+ absorbed yellow - 3. Among the properties of CN– (cyanid ion), which
green light :- of the following properties are correct :-
(a) 3d1 (b) t2g1eg1 (a) oxidising (b) reducing
(c) t2g1eg0 (d) t2g0eg1 (c) acidic (d) complexing

www.jeebooks.in
COORDINATION COMPOUNDS 2.15
4. Which of the following are anionic didentate ligands 12. Which ligand are examples of  donor ligand ?
where oxygen is donor atom ? (a) cyclobutadiene (b) benzene
(a) H2O (b) Acetato (c) SO4 2–
(d) CO32–
(c) Carboxylato (d) Oxalato 13. Which can act as +ve charged ligand ?
5. Which of the following is example of ambidentate  

monodentate ligand ? (a) NO 2 (b) NH2– NH 3



(a) SCN (b) NO 2– (c) CH 3 (d) CN–
(c) S2O32– (d) CN– 14. Which can act as reducing agent :-
6. Which of the following are colourless ? (a) Fe(CO)4 (b) [Co(CO)4]
(a) [Cu(CN)4]3– (b) [Ti(NO3)4] (c) [V(CO)6] (d) [Mb(CO)6]
3+
(c) [Co(NH3)6] (d) [Cu(NH3)4]2+ 15. Which of the following statements are correct :-
7. The complex ion which has no d electrons in the (a) In first transition series Cr can show highest
central metal atom is :- value of magnetic moment in the ground state.
(a) MnO4–1 (b) [Co(M3)6]3+ (b) [Cr(NH3)6] [Cr(NO2)6] & [Cr(NH3)4(NO2)2]
(c) [Cr(H2O)6]3+ (d) CrO42– [Cr(NH3)2(NO2)4] can be distinguished from
8. Select the correct statement about metal carbonyl conductivity measurment in aqous solution.
complex compound :- (c) [Ti(H2O)6]3+ is coloured white [Sc(H2O)6]3+ is
(a) metal carbon bond in metal carbonyls possess colourless
both &  character (d) [Al(H2O)6]3+ can act as proton donor Bronsted
acid.
(b) due to synergic bonding metal carbon bond
becomes weak 16. Which of the following statements are incorrect ?
(c) due to synergic bonding carbon oxygen bond (a) Among TiCl3, MnCl2, FeSO4 , CuSO4 ; MnCl2
strength increases contains highest number of unpaired electron
(d) in metal carbonyles extent of synergic bonding (b) In forming metal complex, metal ion functions
will increase with increases in negative change as lewis acid and ligand functions as lewis base
on central metal ion. (c) In aq. solution, Mn3+ is stronger oxidising
reagent as compared to Co3+
9. Paramagnetic species are :-
(d) All
(a) H2[NiF6] (b) K3[CoF6]
17. For which of the following complex, correct
(c) O 2 (d) B2
IUPAC name is given :-
10. Which ligands are examples  donor ligand ?
(a) K2[SiF6] : Potassiumhexafluorido Silicate(IV)
(a) Propylene
(b) [Cr(NH3)6]ClSO4 : Hexaamminechromium
(b) Propylene diamine (III) chloridesulphate
(c) NH2– (c) [Ru(NH3)5(N2)]Cl3 : Pentaammine(dinitrogen)
(d) 1, 2,3-triaminopropane ruthenium(III)chloride
11. Which of the following are example of bidentate (d) [Pt(PPh3)4] : Tetrakis(triphenylphosphane)
chelating ligand ? platinum(0)
18. When metal is present in anionic part of the
(a) H N NH2 (b) N N complex, then correct naming of metal will be :-
2
(a) Ag : Silverate (b) Co : Cobaltate
(c) N N (d)
(c) Fe : Ferrate (d) Zn : Zincate
N N

www.jeebooks.in
2.16 COORDINATION COMPOUNDS
19. For which ligand in IUPAC nomenclature the term 24. The reactivity of cis and trans isomeric compounds
bis / tris / tetrakis is used up , when more than one may differ widely. It may help to distinguish the
such type of ligands is present :- cis-trans isomers :-
(a) (CH3)2NH (b) NH2CH2CH2NH2 +1
[PtCl2(NH3)2] 
aquation
  [PtCl(NH3)2(OH2)]
(c) 5C5H5 (d) NH3 aquation
   [Pt(NH3)2(OH2)2]
+2
..........(i)
20. What are the correct statements ?
[CoCl2(en)2]+1  aquation
  [CoCl(en)2(OH2)]
+2

(a) [Co(en) 3 ]Cl 3 has four optically active ..........(ii)


stereoisomers
Correct statements :
(b) Ma3bcd has total five stereoisomers out of
(a) Reactivity order in (I) : cis >>> trans
which two are optically active and three are
(b) Reactivity order in (I) : trans >>> cis
optically inactive.
(c) Reactivity order in (II) : cis >>> trans
(c) Resolution of cis[CoCl2(en)2]Cl is possible
(d) Reactivity order in (II) : trans >>> cis
(d) Resolution of trans[CoCl2(en)2]Cl is not
25. Which of the following  bonded organometallic
possible
compound follow Sidgwick EAN rule ?
21. Which of the following statements are incorrect ?
(a) Ferrocene (b) Zeises salt
(a) Charge on the complex of ferric ion with EDTA (c) Cisplatin (d) Cr(C6H6)2
is –1
26. Aqueous solution of potash alum gives :-
(b) [Ni(en) 3 ]Cl 2 has lower stability than (a) white precipitate with BaCl2
[Ni(NH3)6]Cl3
(b) violet colour flame
(c) NF3 is a weaker ligand than NMe3
(c) green colour flame
(d) The complex [Cr(H2O)6]Cl3 looses six water (d) white gelatinous ppt. with NaOH
moleculer to concentrated H2SO4 and does not
27. Which of the following are the correct naming
give any precipitate with AgNO3.
regarding ligand ?
22. Which of the following statement are correct ? (a) NH3Ammine
(a) Identification of cis-trans isomers of compound (b) CH3–NH2Methylamine
Ma2b2can be done by measuring dipole (c) (CH3)2NHDimethylamine
moment.
(d) (CH3)3NTrimethylamine
(b) Aqueous solution of copper sulphate solution
28. Which of the following statements are correct :-
gives green precipitate with aquous potassium
(P) [Cr(H2O)4Cl2]Br.2H2O
fluoride.
(Q)[Cr(H2O)4Br2]Cl.2H2O
(c) The magnetic moment changed when CN–
(R)[Cr(H2O)5Cl]Br2.H2O
ligands in octahedral complex are replaced by
Cl– ligand for d6 ion. (S) [Cr(H2O)6]Cl3
(d) The magnetic moment remains same when (a) P & Q are example of ionisation isomers.
CN– ligands in octahedral complex are replaced (b) Q & R are example of hydrated isomers.
by Cl– for d3 ion. (c) S is most conducting.
23. For which complex , facial and merdional isomers (d) P & R are example of ionisation isomers.
are possible ? :- 29. Which of the following have two stereoisomeric
(a) [Cr(gly)3] (b) [CrCl3(NH3)3] forms ?
(a) [Cr(NO3)3(NH3)3] (b) K3[Fe(C2O4)3]
(c) [RuCl3(OH)3] (d) [PtCl2(NH3)2(py)2]
(c) [Fe(en)Br3Cl]–1 (d) [CoBrCl(Ox)2]3–

www.jeebooks.in
COORDINATION COMPOUNDS 2.17
30. Which of the following statements are correct for 35. Which of the following relationship between
2+
[Pt(H2N–CH–CH–NH2)2 ] :- complex are correct ?
CH3 CH3 OH
(a)
(a) It has total 6 geometrical isomers [(NH3)4Co C o Cl2(NH3)2]SO4 ;
OH
(b) It has total 5 geometrical isomers
OH
(c) One geometrical isomers has COS and 4
[Cl(NH3)3Co Co Cl(NH3)3]SO4
geometrical isomers have POS
OH
(d) Two geometrical isomers has POS and 4 : Coordination position isomers
geometrical isomers have COS
(b) [Pt(NH3)4] [PtCl6] ; [PtCl2(NH3)4] [PtCl4] :
31. Which of the following statements are correct ?
Coordination isomers
Br Br Br
Br H3N Br (c) [Co(NH 3 ) 3 (NO 3 ) 3 ] ; [Co(NO 2 )(NH 3 ) 5 ]
Co en Co en en Co [Co(NH3)2(NO2)4]2 : Polymerisation isomers
H3N H3N NH3 (d) [Co(NH 3)5SCN] 2+ ; [Co(NH 3)5NCS] 2+ :
NH3 Br NH3
I II III
Linkage isomers
(a) I, II are diastereoisomers 36. Which of the following compounds show optical
(b) II, III are enantiomers isomerism ?
(c) I, III are enantiomers (a) [Pt(bn)2]2+ (b) [CrCl2(en)2]+1
(d) II, III are diastereoisomers (c) [Co(en)3] [CoF6] (d) [Zn(gly)2]
32. Which of the following statements are incorrect ? 37. Which of the following statements are correct about
(a) In Mabcd, square planar complexes show both the complex ion [CrCl(NO 2 )(en) 2 ] +1
optical as well as geometrical isomerism (en = ethylenediamine) ?
(b) In Mabcd, tetrahedral complex, optical isomerism (a) It has two geometrical isomers ; cis and trans
& geometrical isomerism both are observed isomer
(c) [Fe(EDTA)]–1 shows geometrical isomerism (b) Cis and trans isomers are diastereomers w.r.t.
(d) tris(ethan-1,2-diamine)nickel(II)chloride is to each other
homoleptic outer orbital complex (c) Only cis isomer is optically active, exists as d
33. Which of the following statements are correct ? and  pair
(a) In [PtCl2(NH3)2]2+ , the cis form is optically (d) It has three stereomers ; two of which are
inactive while trans form is optically active optically active and one is optically inactive
(b) In [Cr(C2O4)3]3+ , geometrically isomerism which is meso form.
does not exist while optical isomerism exists 38. Which of the following statements are correct
(c) [Co(en)2Br2]+1 , cis form is optically active while regarding the following compound [Pt(NH3)4]
trans form is optically inactive [PtCl4]:-
(d) [Co(Ox)2Cl(NO2)] , cis form is optically active (a) It is the polymerisation isomer of [Pt(NH3)3Cl3]
while trans form is optically inactive (Ox - oxalato)
(b) E.A.N. of metal in cationic part and anionic
34. (PPh3) Cl Pd Cl2Pd Cl (PPh3) shows - part both are same
(a) geometrical isomerism (c) It is the coordination isomer of [Pt(NH3)3Cl]
(b) optical (space) isomerism [Pt(NH3)Cl3]
(c) co-ordination position isomerism (d) Synergic bonding is not involved in the complex.
(d) ionisation isomerism

www.jeebooks.in
2.18 COORDINATION COMPOUNDS
39. Correct option regarding Tetrakis (pyridine) 46. Which of the following are inner orbital octahedral
dithiocyanato-N cobalt(III)nitrate. complex ions and diamagnetic ?
(a) It is chelating complex (a) [Co(NH3)6]3+ (b) [Mn(CN)6]3–
(b) It exist in linkage isomeric form (c) [Fe(CN)6]4– (d) [Fe(CN)6]3–
(c) It exist in geometrical isomeric form 47. Which of the following are outer orbital octahedral
(d) Number of ions formed in solution of the complex with same number of unpaired
complex > 2 electrons ?
40. Wilkinson catalyst [RhCl(PPh3)3] is - (a) [MnCl6]3– (b) [FeF6]3–
(a) square planar complex with dsp2 hybridisation (c) [CoF6]3– (d) [Ni(NH3)6]2+
(b) optically inactive 48. Correct statement regarding [MnCl42–] :-
(c) diamagnetic (a) It has magnetic moment 5.92 B.M.
(d) example of heterogeneous catalyst (b) It has five unpaired electrons
2
41. Which of the following are dsp hybridised ? (c) It is tetrahedral complex
(a) [AuCl4]–1 (b) [Pt(NH3)4]2+ (d) The hybridisation of Mn in the complex is sp3
(c) [AuBr2Cl2]–1 (d) Ni(CO)4 49. Correct order of ligand strength -
42. Which of following are sp3 hybridised for Ni- (a) CN– > CO (b) F– > Cl–
complexes ? (c) NO2– > NO3– (d) H2O > NH3
(a) [NiCl4]2– (b) [Ni(CN)4]2– 50. Which of the following complexes are homoleptic?
(c) [Ni(CN)4]4– (d) [Ni(NH3)6]2+ (a) [Co(NH3)6]3+ (b) [Co(NH3)4]+1
43. [Zn(gly)2] is - (c) [Ni(CN)4]2– (d) [Ni(NH3)4Cl2]
(a) sp3 hybridised (b) optically inactive 51. Which of the following complexes are heteroleptic?
2
(c) dsp hybridised (d) optically active (a) [Cr(NH3)6]3+ (b) [Fe(NH3)4Cl2]+1
(c) [Mn(CN)6]4– (d) [Co(NH3)4Cl2]
44. [Pt(NH3)4(NO2)2] exists in 2 geometrical isomers.
Which of the following options are correct ? 52. Which of the following complexes are optically
active?
(a) Cis & trans isomer both have plane of symmetry
(a) [Co(en)3]3+ (b) trans [Co(en)2Cl2]+1
(b) Cis & trans isomer both have C2 axis of symmetry
(c) Cis[Co(en)2Cl2]+1 (d) [Cr(NH3)5Cl]2+
(c) both are optically active
53. An aqueous pink solution of cobalt(III)chloride
(d) both have centre of symmetry changes to deep blue on addition of excess HCl.
45. Brown ring - having molecular formula This is because -
[Fe(H2O)5NO]SO4. Which of the following (a) [Co(H2O)6]3+ is transformed into [CoCl6]4–
statemens are correct regarding brown ring - (b) [Co(H2O)6]3+ is transformed into [CoCl4]2–
(a) Outer orbital octahedral complex with sp3d2 (c) Tetrahedral complexes have smaller crystal field
hybridisation splitting than octahedral complex
(b) Fe has (+2) configuration and NO is example (d) Tetrahedral complexes have larger crystal field
of neutral ligand splitting than octahedral complex
(c) It has 2 unpaired electron and magnetic moment 54. Which of the following complex is tetrahedral but
2.73 B.M. diamagnetic ?
(d) It has 3 unpaired electron and magnetic moment (a) K3[Cu(CN)4] (b) Ni(CO)4
3.82 B.M. (c) K4[Ni(CN)4] (d) [Zn(H2O)4]2+

www.jeebooks.in
COORDINATION COMPOUNDS 2.19
55. Which of the following are correct statements ? 61. Which of the following are examples of bidentate
(a) Pd(II), Pt(II) forms mostly 4 coordination, chelating ligand ?
square planar diamagnetic coordination (a) [NH3–CH2–CH2–NH3]2+
compounds.
CH2CO2–
(b) In Mn2(CO)10 there are total 70 electrons (b) NH
present CH2CO2–
(c) [Ni(CN4)]4– & [Ni(CN4)]2– have different (c) H2N–CH2– C O–
dipole moment
O
(d) The difference of the number of unpaired O –

electron in Zn2+ and [Zn(NH3)4]2+ is zero


(d)
56. FeCl3 + K4[Fe(CN)6]  Complex product.
O–
Correct statement regarding complex product :-
62. In [Fe2(CO)9], which of following statements are
(a) IUPAC name of complex product is iron(II) correct :-
hexacyanidoferrate(III)
(a) Fe has zero oxidation state
(b) Total number of Fe ions in the solution is 4
(c) Average oxidation state of Fe is 2.57 (b) EAN of each Fe atom is 36
(d) EAN of central metal atom of complex is 36 (c) EAN of each Fe atom is 35
6 0
57. Which complex compound having t 2 ge g (d) None
configuration ? 63. For which of the following compound secondary
(a) K3[Co(NO2)6] (b) K4[Fe(CN)6] valency of metal is six :-
(c) K3[Fe(CN)6] (d) Na2[Fe(CN)5NO] (a) PdCl2.4NH3 (b) NiCl2.6H2O
3
58. Which complex compound having t 2g eg2 (c) PtCl2.2NH3 (d) CoCl3.4NH3
configuration ?
64. Which of the following are example of symmetrical
(a) [Mn(H2O)6]2+ (b) [Mn(NH3)6]2+
electronic configuration :-
(c) [Mn(CN)6]4– (d) MnO2
59. Co-ordination entity Wave length of light (a) d5 in weak field ligand
absorbed (nm) (b) d7 in strong field ligand
[CoCl(NH3)5] 2+
P (c) d8 in weak field ligand
3+
[Co(NH3)5(H2O)] Q (d) d10 in strong field ligand
3+
[Co(NH3)6] R 65. Which of the following are example of
3–
[Co(CN)6] S asymmetrical electronic configuration :-
2+
[Cu(H2O)4] T (a) d3 in strong or weak field ligand
Select the correct orders :- (b) d7 in strong field ligand
(a) P > R > Q (b) T > P > Q > R > S
(c) d4 in weak field ligand
(c) P > Q > R (d) T > Q > P > R > S
(d) d9 in strong or weak field ligand
60. What are the basic features of crystal field theory?
(a) ligands are considered as point like charge 66. Which of the complex give same number of
particles as that of LaCl3 :-
(b) the metal ligands interaction are purely
electrostatic in nature (a) K3[CO(NO2)6]
(c) metal electrons are considered as non bonding (b) Co(NO2)3.KNO2.2NH3
electrons (c) CoCl3.3NH3
(d) covalent interaction takes place between ligand (d) CoCl3.6NH3
and metal ion

www.jeebooks.in
2.20 COORDINATION COMPOUNDS
67. Correct option regarding Pentaamminethiocyanato- 74. In spherical octahedral crystal field, average energy
N cobalt(III) trioxalatochromate(III) - of the 3d orbital is :-
(a) It is chelating complex (a) higher than eg orbital
(b) It exist in linkage isomeric form (b) lower than eg orbital
(c) It exist in geometrical isomeric form (c) higher than t2g orbital
(d) Number of ions formed in solution of the (d) lower than t2g orbital
complex > 2 75. Complex Absorption peak (cm–1)
68. Which of the following IUPAC names are correct:- (A) [Cr(III)Cl6]3– x
(III) 3+
(a) [(CO)3Fe(CO)3Fe(CO)3] : Tri-µ-carbonyl- (B) [Cr (H2O)6] y
(III) 3+
bis(tricarbonyliron(0)) (C) [Cr (NH3)6] z
(III) 3–
(b) [(NH3)5CoNH2Co(NH3)5] (NO3)5 : µ-amido- (D) [Cr (CN)6] w
bis[pentaamminecobalt(III)]nitrate] Correct order between x,y,z,w are :-
(c) Na3[Ag(S2O3)2] : Sodiumbis(thiosulphato) (a) x > y (b) y > z
argentate(I) (c) w > z (d) w > z > y > x
(d) K2[CrVI(CN)2O2(O2)NH3] : Potassiumammine 76. Complex 0 (cm–1)
dicyanidodioxoperoxochromate(VI) (A) [Co(III)(NH3)6]3+ x
(III) 3+
69. Which of the following statements are incorrect :- (B) [Rh (NH3)6] y
(a) Maximum coordination number of Ag+1 is 6 (C) [Ir(III)(NH3)6]3+ z
(b) Maximum coordination number of Pt2+ is 6 Correct order between x,y,z are :-
(c) Maximum coordination number of Al3+ is 6 (a) z > y (b) z > x
(d) Maximum coordination number of Ni2+ is 6 (c) y > x (d) x = y = z
70. Which of the following have eg0t2g0 configuration :- 77. Which of the following do not act as an oxidizing
agent :-
(a) FeO42– (b) MnO4–
(a) Mn(CO)5 (b) Fe(CO)5
(c) TiCl4 (d) ZnCl42–
(c) Mn2(CO)10 (d) Fe2(CO)9
71. Which of the following statements are incorrect :-
78. Prussian blue is formed by combining between :-
(a) [HgI3]– is planar and diamagnetic (a) Fe3+ (b) [Fe(CN)6]3–
(b) In Zeise’s salt central metal act as  donor,  (c) Fe2+ (d) [Fe(CN)6]4–
acceptor
79. For d7 ion like Co2+ with very strong ligand like
(c) Zeise’s salt has planar anion NO2– :-
(d) NO3– is example of ambidentate ligand (a) Co2+ ion can be reduced to Co
72. Which of the following complexes/compounds, (b) Co2+ ion can be oxidised to Co3+
atleast one metal ion has +3 oxidation state:- (c) The excited 7th electron resides in 5s orbital
(a) Ferric alum (b) Cryolite which is of higher energy level.
(c) Blue vitriol (d) Potassium fericyanide (d) The complex formed is d2sp3 hybridised
73. A complex which is meso isomer can :- 80. Which of the following complexes are examples
of ternary complexes :-
(a) Have plane of symmetry
(a) [CoCl(NH3)5]2+ (b) [Cu(bpy)en]2+
(b) Have one chiral centre
(c) [Cu(bpy)ox] (d) [Co(NH3)5(OH2)]3+
(c) Have centre of symmetry
(bpy = bipyridine ; en = ethylene diamine ; ox =
(d) Have nonsuperimposable mirror image oxalato)

www.jeebooks.in
COORDINATION COMPOUNDS 2.21
81. Which of the following statements are incorrect ? 89. Cr2O72– II
+ Fe + H2SO4 Final solution (a)
(a) NO+ has lower p accepting tendency than CO Correct option regarding A :-
(b) The C-O bond length in bridging carbonyl (a) A is [Cr(SO4)3]3– (b) A is [Cr(H2O)6]3+
compound is shorter than that of terminal (c) A is violet in colour (d) A is green in colour
carbonyl group
90. In the crystal structure of MnSO4.4H2O, which of
(c) The C-O bond length in bridging carbonyl
the following statement are correct ?
compound is longer than that of terminal
carbonyl group (a) SO42– acts as bidentate ligand where 2 oxygen
atoms of SO42– form chelate with Mn2+
(d) Ni(H2O)62+ is green in colour but [Ni(CN)4]2–
is colourless (b) There are 6 water molecule associated with
each Mn2+ ion
82. Correct order of CO bond length :-
(a) [M(CO)3PF3] < [M(CO)3PCl3] (c) SO42– acts as bridging ligand.
(b) [M(CO)3PF3] > [M(CO)3PCl3] (d) Mn2+ has coordination number six
(c) [M(CO)3PF3] < [M(CO)3PMe3] 91. Incorrect statements are :-
(d) [M(CO)3PCl3] < [M(CO)3PMe3] (a) Ni4+ is more stable than Ni2+
83. The value of x in :- (b) Ni2+ is more stable than Ni4+
(a) [Hg(CN)4]x can be –2 (c) Pt2+ is more stable than Pt4+
(b) [Fe(CN)6]x can be –3 (d) Pt4+ is more stable than Pt2+
(c) [Fe(CN)6]x can be –4 92. X forms pentacarboyl of the type X(CO)5
(d) [Co(NH3)2Cl4]x can be –1 Y forms tetracarboyl of the type Y(CO)4
84. Which of the following ligands can act as Z forms hexacarboyl of the type Z(CO)6
flexidentate ligand ? If in all the complexes metal follows EAN rule then:-
(a) SO42– ion (b) CH3CO2– ion (a) X can be Fe (b) Y can be Ni
(c) CO32– ion (d) HCO2– ion (c) Z can be V (d) Z can be Cr
85. Which of the following are example of perfect 93. When Gold is attacked by aqua regia :-
complex :-
(a) the coordination number of Au in complex
(a) [Fe(CN)6]4– (b) [Ag(CN)2]–1
formed is +2
(c) [Cu(CN)4]3– (d) [Cu(NH3)4]2+
(b) the coordination number of Au in complex
86. Which of the following are example of inperfect formed is +4
complex :-
(c) the complex formed can act as a bronsted acid
(a) [Cd(CN)4]2– (b) [CdI4]2–
(d) NO2 gas is evolved
(c) [Co(NH3)]3+ (d) [Fe(CN)6]4–
94. An aqueous solution of Fe(III) sulphate is
87. Which of the following are example of anionic
paramagnetic but when an excess of KCN is
complex :-
added to the solution which of the following
(a) K2[HgI4] (b) K3[Co(NO2)6] properties are changed :-
(c) Ni(CO)4 (d) H[AuCl4] (a) Shape of the molecule
88. Which of the following are example of cationic
(b) Hybridisation of Fe(III)
complex :-
(c) Number of unpaired electron
(a) [Rh(PPh3)Cl3] (b) [Ag(NH3)2]Cl
(d) magnetic behaviour
(c) [Co(NH3)6]Cl3 (d) [Co(NH3)5 SO4]Br

www.jeebooks.in
2.22 COORDINATION COMPOUNDS
95. Correct statement regarding [Co(en)3]3+ :- 100. Select the correct IUPAC name for the following
(a) It exists as cis and trans isomer complex
(b) It is nonresolvable Me Me
(c) It is resolvable Br S Br
Pt Pt :-
(d) It is optically active Br S Br
96. Correct statement regarding Ferrocene :-
Me Me
(a) It is an orange solid with melting point 174°C (a) D i b r o m i d o p l a t i n u m ( I I ) b i s - µ -
(b) It is prepared as 2C5H5MgBr + FeCl2  (dimethylthioether) dibromidoplatinum(II)
Fe(C5H5)2 + MgCl2 + MgBr2 (b) Bis{µ(dimethylthioether) dibromido-
(c) IUPAC name of ferrocene is bis platinum(II)}
(cyclopentadienyl) iron(II) (c) Bis-µ-dimethylthioethertetrabromid-
(d) It is an example of sandwich compound oplatinum(II)
97. Which of the following statements are correct :- (d) Bis-µ-dimethylthioethertetrabromid-
(a) In an octahedral ligand field the five fold oplatinate(II)
degenerate 3d level will split into doubly EXERCISE # III
degenerate eg set and triply degenerate t2g set
(b) The symbol g denotes the presence of centre  Linked Comprehension Type :
of inversion Paragraph for Q.1 to Q.2
(c) In the square ligand field, the 3d orbitals will An octahedral complex of cobalt has its composition
be split into four levels Co(NH3)3(H2O)3(SO4)(NO3) and it exists in four
isomeric forms A, B, C and D . Simple heating of
(d) In the square ligand field, the 3d orbitals will
either A or B loses 5.6% of its original weight whereas
be split into two levels
similar treatment on C causes 11.2% loss in its original
98. The substance CoBr3 . 4NH3 . 2H2O has a molar weight D did not lose any weight on simple heating.
conductivity of 420 ohm–1 at infinite dilution. Which Either A or C did not result in any precipitation if treated
corresponds to (3+, 1–) electrolyte like AlCl3. with aqueous BaCl2 solution, whereas B and D
Correct option regarding that substance :- resulted a white precipitate when treated with aqueous
(a) Substance is [Co(NH3)4(H2O)2]Br3 BaCl2 solution. Also C does not conduct electricity in
(b) IUPAC name of substance is tetraammine its aqueous solution,
diaquacobalt(III)bromide 1. A is :-
(c) Substance is [Co(NH3)3(H2O)3]Br3 (a) [Co(NH3)3(H2O)3]SO4.NO3
(d) IUPAC name of substance is tetraammine (b) [Co(NH3)3(H2O)2NO3]SO4.H2O
diaquacobalt(II)bromide (c) [Co(NH3)3(H2O)2SO4]NO3.H2O
99. Select the correct IUPAC name for [(NH 3)4 (d) [Co(NH3)3(H2O)SO4NO3]2H2O
Co(OH)(NH2)Co(NH3)4]4+ :- 2. B & C are respectively :-
(a) µ-Amido-µ-hydroxidobis {tetraamminecobalt (a) [Co(NH3)3(H2O)3]SO4.NO3 ;
(4+)} ion [Co(NH3)3(H2O)SO4.NO3]2H2O
(b) µ-Amido-µ-hydroxidobis{tetraamminecobalt (b) [Co(NH3)3(H2O)2NO3]SO4.H2O ;
(III)} ion [Co(NH3)3(H2O)SO4]NO3.H2O
(c) µ-Amido-µ-hydroxidobis{tetraamminecobaltate (c) [Co(NH3)3(H2O)2NO3]SO4.H2O ;
(4+)} ion [Co(NH3)3(H2O)SO4.NO3]2.H2O
(d) µ-Amido-µ-hydroxidobis{tetraamminecobaltate (d) [Co(NH3)3(H2O)2SO4]NO3.H2O ;
(III)} ion [Co(NH3)3(H2O)2.NO3]SO4.H2O

www.jeebooks.in
COORDINATION COMPOUNDS 2.23
3. D is :- 8. The white ppt is :-
(a) [Co(NH3)3(H2O)3]SO4.NO3 (a) BaSO4 (b) BaCl2
(b) [Co(NH3)3(H2O)2NO3]SO4.H2O (c) Ba(NO3)2 (d) BaCO3
(c) [Co(NH3)3(H2O)2SO4]NO3.H2O 9. The brown and blue colour ppt are respectively :-
(d) [Co(NH3)3(H2O)SO4NO3]2H2O (a) I HgO Hg NH2 & ferro-ferri cyanide
Paragraph for Q.4 to Q.6 (b) I HgO Hg NH2 & ferri-ferro cyanide
A, B and C are three complexes of chromium (ii) (c) Fe(OH)3 & Fe(CN)3
with their formula H12O6Cl3Cr. Complex A does not (d) Fe(OH)3 ; [Cn(NH3)4]SO4
react with concentrated H2SO4 solution whereas Paragraph for Q.10 to Q.11
complexes B and C loses 6.75% and 13.5% of their In metal carbonyls, synergic bonding takes place
weight, respectively, on treatment with concentrated between metal and carbonmonooxide. This leades to
H2SO4 solution. increase in strength of metal ligand bond and decrease
4. Correct option regarding A :- in bond order of CO in carbonyl complex as compared
(a) A is violet with 3 chloride ions/unit formula to bond order in carbon monooxide.
(b) B is light green colour with 2 chloride ions / 10. Which among the following metal carbonyls are
unit formula inner orbital complexes with diamagnetic property ?
(c) C is dark green colour with 1 chloride ions / (X) Ni(CO)4 ; (Y) Fe(CO)5 ; (Z) V(CO)6 ; (W)
unit formula Cr(CO)6
(a) (X) and (Y) only (b) (Y), (Z) and (W) only
(d) All
(c) (Y) and (W) only (d) (X), (Y) and (W) only
5. Electrical conductivity order of A,B,C
11. Which of the following metal carbonyls involve the
(a) A > B > C (b) B > A > C
d2sp3 hybridisation for the formation of metal
(c) C > A > B (d) C > B > A carbon  bonds and is paramagnetic ?
6. A,B,C are related as (a) [Cr(CO)6] (b) [V(CO)6]
(a) Ionisation isomers (b) Hydrate isomers (c) [Mo(CO)6] (d) [W(CO)6]
(c) Optical isomers (d) Geometrical isomers Paragraph for Q.12 to Q.14
Paragraph for Q.7 to Q.9 Co (aq.) + SCN–(aq.) Complex (X)
2+

A light green coloured substance A has 27.55% H2O. Ni2+(aq.) + Dimethylglyoxime 
NH 4OH Complex (Y)
A gives the following reaction, :-
The coordination number of cobalt and nickel in
(i) it gives white ppt. with BaCl2 which is soluble in complexes X and Y is four.
conc. HNO3
12. The IUPAC names of the complexes (X) and (Y)
(ii) it gives brown colour ppt when reacts with alkaline are respectively :-
K2[HgI4] solution
(a) tetrathiocyanato-S-cobalt(II)ion and
(iii) it gives dark blue ppt when reacts with bis(dimethylglyoximate) nickel(II)
K3[Fe(CN)6] (b) tetrathiocyanato-S-cobaltate(II)ion and
7. A is :- bis(dimethylglyoximato) nickel(II)
(a) KCl.MgCl2.6H2O (c) tetrathiocyanato-S-cobaltate(II)ion and
(b) Fe2(SO4)3.(NH4)2SO4.6H2O bis(dimethylglyoximato) nickelate(II)
(c) Cr2(SO4)3.K2SO4.6H2O (d) tetrathiocyanato-S-cobaltate(III)ion and
(d) FeSO4.(NH4)2SO4.6H2O bis(dimethylglyoximato) nickel(II)

www.jeebooks.in
2.24 COORDINATION COMPOUNDS
13. The geometry of complexes (X) and (Y) are 17. Identify the incorrect statement :-
respectively :- (a) Mn2+ has the highest paramagnetism amongst
(a) tetrahedral and square planar the bivalent cations of the Ist transition series
(b) both tetrahedral (b) The coloured ions or compounds of transition
(c) square planar and tetrahedral elements are due to d-d transition, polarisation
(d) both square planar of anion and charage transfer spectrum
14. Select the correct statement for the complexes (X) (c) In 3d series the paramagnetic character first
and (Y) :- increase to maximum and then starts decreasing
(a) (X) is paramagnetic with two unpaired electrons (d) None of these
(b) (Y) is diamagnetic and shows intermolecular Paragraph for Q.18 to Q.20
H-bonding In general, the crystal field splitting energy 
(c) (X) is paramagnetic with three unpaired corresponds to wavelengths of light in visible region of
electrons and (Y) is diamagnetic the spectrum, and coloures of the complexes can
(d) (X) and (Y) both are diamagnetic therefore be attributed to electronic transition between
Paragraph for Q.15 to Q.17 the lower- and higher energy sets of d-orbitals.
Paramagnetism is a property due to the presence of In general, the colour that we see is complementry to
unpaired electrons. In case of transition of metals, as the colour absorbed.
they contain unpaired electrons in the (n – 1) d orbitals,
most of the transition metal ions and their compounds 650nm 600nm
If a substance Orange
are paramagnetic. Paramagnetism increases with absorbs here
increase in number of unpaired electrons. Magnetic
moment is calculated from ‘Spin only formula’ vis. Red
800nm Yellow
560nm
µ = n(n  2) B.M. n = number of unpaired electrons 400nm Green
Violet
Similarly the colour of the compounds of transition It appears as
metals may be attributed to the presence of incomplete Blue this colour
(n – 1) d subshell. When an electron from a lower 430nm 490nm
energy of d-orbital is excited to a higher energy Different metal ions have different values of , which
d-orbital, the energy of excitation corresponds to the
explains why their complexes with the same ligand have
frequency of light absorbed. This frequency generally
different colour.
lies in the visible region. The colour observed
corresponds to complementary colour of the light. The Similarly the crystal field splitting also depends on the
frequency of the light absorbed is determined bt the nature of ligands and as the ligand for the same metal
nature of the ligand. varies from H2O to NH3 to ethylenediamine,  for
15. Which of the followig pair of compounds is complexes increases. Accordingly, the electronic
expected to exhibit same colour in aqueous solution? transition shifts to higher energy (shorter wavelength)
as the ligand varies from H2O to NH3 to en, thus
(a) FeCl2 , CuCl2 (b) VOCl2 , CuCl2
accounting for the variation in colour.
(c) VOCl2 , FeCl2 (d) FeCl2 , MnCl2
Crystal field theory accounds for the magnetic
16. Titanium shows magnetic moment of 1.73 B.M. in
properties of complexes in terms of the relative values
its compounds. What is the oxidation state of
of  and the spin pairing energy P. Small  values favour
titanium in the compound ?
high spin complexex, and large  values favour low
(a) +2 (b) +1 spin complexed.
(c) +3 (d) +4

www.jeebooks.in
COORDINATION COMPOUNDS 2.25
3–
18. The [Ti(NCS)6] ion exhibits a single absorptin 23. If 10Dq0 > P then crystal field splitting energy for
bond at 544 nm. What will be the crystal field d6 ion in octahedral field will be :-
splitting energy (in kJ mol–1) of the complex ? (a) 20 Dq – 2P (b) 4 Dq
(h = 6.626 × 10–34 J.s ; C = 3.0 × 108 m/s ; (c) 8 Dq (d) 24 Dq – 2p
NA = 6.02 × 1023 ions.mjole.)
Paragraph for Q.24 to Q.25
(a) 240 (b) 220
The splitting diagram for square planar complexes is
(c) 270 (d) 250
more complex than for octahedral and tetrahedral
19. Whcih of the following statement is incorrect :- complexes and is shown below with the relative energies
(a) The Ni2+ (aq) cation is coloured because Ni2+ of each orbital.
ion can absorb light , which promotes electrons dx2–y2 1.230
from the filled d-orbitals to the higher half filled eg
d - orbitals dxy 0.230
0
(b) The Zn2+ (aq) cation is coloured because the d
orbitals are completely filled and no electrons d z2 –0.430
can be promoted, so no light is absorbed e2g
(c) A complex which has just one absorption band –0.510
dxz dyz
at 455 nm, must be red coloured.
Octahedral field Square planar field
(d) None
20. Which of the following complexes are diamagnetic:- 24. Crystal field stabilisation energy for a diamagnetic
square planar d8 metal complex with the help of
[Pt(NH3)4]2+[Co(SCN)4]2– [Cu(en)2]2+ [HgI4]2–
above diagram neglecting pairing energy (P) :-
square planar tetrahedralsquare planar tetrahedral
(a) – 4.88 0 (b) – 2.44 0
(i) (ii) (iii) (iv)
(c) – 0.68 0 (d) – 3.88 0
(a) (i) and (ii) (b) (ii) and (iii)
(c) (i) and (iv) (d) (iii) and (iv) 25. Which of the following set of d electrons have
same crystal field stabilisation energy value for
Paragraph for Q.21 to Q.23
tetrahedral complex neglecting pairing energy (P):-
In the octahedral systems there are two possible ways
to distribute the d-electron in the t2g and eg level for (I) d1, d6 (II) d2, d7
each dn system. There will be two opposing factors. (III) d3, d8 (IV) d4, d9
The crystal field splitting energy , i.e., 10Dq0 (or 0) (V) d5, d10
will try to place the electrons into the more stable t2g
(a) Only V (b) II, III, V
level as many as possible while the pairing energy (P)
required to pair two electrons in the same orbital will (c) I, III, IV (d) I, II, III, IV, V
try to avoid the pairing as far as possible. Paragraph for Q.26 to Q.27
21. If 10Dq0 > P the correct electronic configuration According to crystal field theory, the electronic
for d4 ion is :- configuration of ions are given below :
(a) t 32g eg1 (b) t 42g eg 0 d1 : t12gd8 : t62geg2
(c) t 22g eg 2 (d) eg3t12g d2 : t22gd9 : t62geg3
22. If 10Dq0 < P then correct electronic configuration d3 : t32gd10 : t62geg4
for d5 ion is :- 26. Irrespective of the nature of ligand outer orbital
octahedral complex is formed by :-
(a) t 52g eg 0 (b) eg3t 22g
3 2 4 1
(a) d8 ion (b) d9 ion
(c) t eg (d) t eg
2g 2g
(c) d10 ion (d) All

www.jeebooks.in
2.26 COORDINATION COMPOUNDS
27. Irrespective of the nature of ligand inner orbital 32. Ionic potential of :-
octahedral complex is formed by :- (a) Zn2+ is in between Fe3+ and Cd2+
(a) Only d1 ion (b) d1 , d2 ion (b) Cu2+ is in between Cr3+ and Cu+
(c) Only d3 ion (d) d1 , d2 , d3 ion (c) Mn2+ is in between Rh3+ and Ba2+
Passage for Q.28 to Q.29 (d) All
AlF3 is not soluble in analysis HF but not soluble in KF. Passage for Q.33 to Q.35
When BrF3 is added to the above solution, AlF3 is
According to Sidgwick’s Effective Atomic Number rule,
precipitated.
the ligands donate the electrons to the central metal ion
28. What will be the product when AlF3 is dissolved in through the covalent coordinate bonds and the total
KF ? number of electrons on the central metal ion including
(a) K[AlF4] (b) K2[AlF5] those gained from ligands is the atomic number of
(c) K3[AlF6] (d) K4[AlF7] nearest noble gas.
29. What will be the product when this dissolved Given :At. No. of Cd & Pt are 48, 78 respectively.
solution combines with BF3 ? 33. In [Cd(NH3)4]2+ , EAN value of metal ion :-
(a) K[BF4] (b) K2[BF5] (a) is same as atomic number of Kr
(c) K3[BF6] (d) K4[BF7] (b) is same as atomic number of Xe
Passage for Q.30 to Q.32 (c) is same as atomic number of Rn
Electrostatic theory of bonding in coordination (d) does not match any noble gas configuration
compound consider the electrostatic attraction between
34. In [Pt(NH3)4Cl2]2+ , EAN value of metal ion :-
the positively charged metal ion and negatively charge
ligads or negative ends of polar molecule acting as the (a) is same as atomic number of Kr
ligands. Thus complexing power of particular metal ion (b) is same as atomic number of Xe
depends on the charge density called ionic potential. (c) is same as atomic number of Rn
In fact ionic potential is a good parameter to measure (d) does not match any noble gas configuration
the complexing power of metal ion.
35. In [Pt(2 C2H4)Cl3]–1 , EAN value of metal ion :-
30. Metal ion like Cs + , Rb + , K + , Na + , Li + ,
(a) is same as atomic number of Kr
complexing power is very low. It is because these
metal ions have :- (b) is same as atomic number of Xe
(a) very high ionic potential (in the range of 3.7 - (c) is same as atomic number of Rn
5.2 ) (d) does not match any noble gas configuration
(b) moderately high ionic potential (in the range of Passage for Q.36 to Q.38
1.5 - 2.5) The following complexes are given below :
(c) low ionic potential (in the range of 0.55 - 1.11) [Ag(NH3)2]+ ; [Ag(CN)2]–1;
(d) moderately high ionic potential (in the range of X Y
2 - 3) [Ag(SCN)2] ; –1
[Pt(CN)4]2–
31. Ionic potential of Co3+ , Al3+ , Sn4+, Be2+, Pt4+are
Z W
very high. So these ions have :-
36. Which of the following complex is linear in shape
(a) very poor complexing ability with ligands
having all 5 atoms including metal ion are in a
(b) moderate complexing ability with ligands straight line :-
(c) very high or most pronounced complexing (a) X (b) Y
ability with ligands
(c) Z (d) W
(d) do not form complex at all

www.jeebooks.in
COORDINATION COMPOUNDS 2.27
37. Which of the following complex is square planar in 43. X can be :-
shape having at least 5 atoms including metal ion (a) [Co(NH3)6]Cl3 (b) K3[Co(C2O4)3]
are in a straight line :- (c) [Co(H2O)6]SO4 (d) [CoCl4]2–
(a) X (b) Y 44. Y can be :-
(c) Z (d) W (a) [CoBr6]4– (b) [Co(CO)4]–1
38. Which of the following complex is linear in shape (c) [Co2(CO)8] (d) [CoF6]3–
having at least 7 atoms including metal ion are in a
Passage for Q.45 to Q.47
straight line :-
The following complexes P and Q are given as follows
(a) X (b) Y
(c) Z (d) W
Br Br
Cl Br Br NH 3
Passage for Q.39 to Q.41 M M
The following complexes are given below : NH 3 Cl
Cl HN
3
[Ni(H2O)6]2+; [Ni(H2O)4en]2+; NH3 Cl
X1 X2
P Q
2+
[Ni(H2O)2(en)2] ; [Ni(en)3]2+ 45. P and Q are related as :-
X3 X4 (a) Structural isomers (b) Geometrical isomers
39. Correct statement regarding X4 :- (c) Optical isomers (d) None
(a) X4 has 2 stereoisomers 46. P :-
(b) X4 has 3 stereoisomers
(a) can exist as enantiomeric pair.
(c) X4 has 4 stereoisomers
(b) can not exist as enantiomeric pair.
(d) X4 has 2 optically inactive stereoisomers
(c) is resolvable
40. Correct statement regarding X3 :-
(d) both (a) and (c)
(a) X3 has 2 stereoisomers
47. Q :-
(b) X3 has 3 stereoisomers
(a) can exist as enantiomeric pair.
(c) X3 has 4 stereoisomers
(b) can not exist as enantiomeric pair.
(d) X3 has 2 optically inactive stereoisomers
(c) is resolvable
41. Correct statement regarding X1 and X2:-
(d) both (a) and (c)
(a) X1 , X2 both are optically inactive
Passage for Q.48 to Q.51
(b) X1 , X2 both are optically active
A qualitative correlation between the various Lewis
(c) X1 is optically inactive but X2 is optically active
acids and bases is done by classifying the acids or bases
(d) X2 is optically inactive but X1 is optically active
into two classes hard and soft. Acid base reactions are
Passage for Q.42 to Q.44 then treated by the general principle that hard acid
A complex of cobalt , X has magnetic moment 4.89 prefer to combine with hard bases and soft acids prefer
BM and EAN value 36. Another complex of cobalt, Y to combine with soft bases. A firmly held electron -
has magnetic moment 3.87 BM and EAN value 37. cloud with low polarisability makes a species “hard”
Third complex of cobalt , Z has magnetic moment 0 while an easily polarisable electron cloud characterises
BM and EAN value 36. the species as “soft”.
42. Z can be :- 48. Which of the following are example of hard acids?
(a) [Co(H2O)6]SO4 (b) [CoCl4]2– (a) Co3+ (b) Fe3+
(c) K3[Co(NO2)6] (d) [CoBr4]2– (c) Mg2+ (d) Cr3+

www.jeebooks.in
2.28 COORDINATION COMPOUNDS
49. Which of the following are example of soft acids ? 54. The donor atoms of cryptands :-
(a) Cu+ (b) Ag+ (a) O, O (b) O, N
(c) Hg2+ (d) Pd2+ (c) N, N (d) None
50. Which of the following are example of hard bases? N

(a) NO3– (b) F– O O


(c) NH3 (d) ROH 55. O is example of :-
51. Which of the following are example of soft base ? O
O
(a) RSH (b) R3P N

(c) I (d) CN–
(a) crypt-[2,2,1] (b) crypt-[2,2,2]
Passage for Q.52 to Q.53
(c) crypt-[2,3,2] (d) crypt-[0,2,0]
Irving and Williams calculated the series of stability
56. Which of the following statements are correct :-
constants of octahedral complexes of the bivalent metal
ions of first transition series with the common ligands (a) Four member chelate ring produced by 2
having the donor sites like oxygen, sulphur, nitrogen didentate ligands like acetato, carbonato,
and halogen. This series is called Irving and Williams sulphato and nitrato are fairly stable and
stability order of octahedral complexes.This occurs due preferred in complexes of high coordination
to the following contributing factors. number 8,10,12
 decrease of ionic radii, i.e., increase of (b) The five member chelate ring formed by amino
electronegativity along the series. acid or amino carboxylate are more stable than
six member chelate ligands formed by similar
 increase of crystal field stabilization energy along
ligands
the series.
(c) Resonance or delocalisation of charge stabilises
52. The stability constant of the octahedral complexes
the six membered chelate rings
for d block ions :-
(d) Generally chelate rings less than five membered
(a) Mn2+ < Fe2+ < Co2+ or more than six membered will be less stable
(b) Mn2+ > Fe2+ > Co2+ Passage for Q.57 to Q.58
(c) Co2+ < Ni2+ < Cu2+ > Zn2+ oxidised
K6 [(CN)5–Co–O–O–Co(CN)5]  K5
(d) Ni2+ < Cu2+ < Zn2+ X
53. The stability constant of the octahedral complexes [(CN)5–Co–O–O–Co(CN)5]
for s-block & d-block ions :-
Y
(a) Ba2+ < Ca2+ < Mg2+
57. In both the complex Co have t2g 6 eg 0
(b) Ba2+ > Ca2+ > Mg2+ congiguration. The bond energy of (O–O) in X &
(c) Ca2+ < Mg2+ < Mn2+ Y is :-
(d) Mg2+ < Ba2+ < Ca2+ (a) B.E. of (O–O) in Y < B.E. of (O–O) in X
Passage for Q.54 to Q.56 (b) B.E. of (O–O) in X < B.E. of (O–O) in Y
The crown ethers are monocyclic while the cryptands (c) B.E. of (O–O) in X = B.E. of (O–O) in Y
are bicyclic or tricyclic. The multi cyclic cryptands are (d) None
basically the amino ethers where the N-sites acts as 58. Both the complex are :-
bridgehed atoms. The cryptands can provide the cage
(a) Tetrahedral and Diamagnetic
structure to accommodate the metal ion and other
suitable guest species. Thus these multicyclic ligands (b) Octahedral and Paramagnetic
provide the heterocyclic rings to encapsulate the metal (c) Diamagnetic
ion in a cage-like structure. (d) Paramagnetic

www.jeebooks.in
COORDINATION COMPOUNDS 2.29
Passage for Q.59 to Q.60
Complex  M (Ohm–1 cm2 mol–1) No. of ions No. of ionic Cl–1
per molecule per molecule for AgCl ppt.
COCl3.6NH3 m1 x p
(Lutecobaltic chloride)
COCl3.5NH3 m2 y q
(Purpurecobaltic chloride)
COCl3.4NH3 m3 z r
(Praseocobaltic chloride)
COCl3.5NH3.H2O m4 w s
(Roseocobaltic chloride)
59. Correct option is :- (b) M–C vibrational frequency : III > II > I
(a) m1 > m2 > m3 > m4 (b) m1 > m4 > m2 > m3 (c) Synergic effect order : I > II > III
(c) m1 > m3 > m4 > m2 (d) m1 > m2 > m4 > m3 (d) C–O vibrational frequency : III > II > I
60. Correct option is :- Passage for Q.63 to Q.65
(a) w = x = 4 ; s = p = 3 The stability of a complex in solution refers to the degree
(b) y = 3 ; q = 2 of association between the two species involved in the
(c) z = 2 ; r = 1 state of equilibrium. The magnitude of the equilibrium
constant expresses the stability. Thus, for the reaction
(d) All  ML if equilibrium constants
of the type : M + 4L  4
Passage for Q.61 to Q.62 K1, K2, K3, K4 for the stepwise formation of the
The metal carbon bond in metal carbonyls possess both successive complexes ML, ML 2 , ML 3 , ML 4
 and  character. The M–C bond is formed by respectively are called the stepwise stability constants.
donation of lone pair of electrons on the metal carbonyls 63. If overall stability constant is 4 for the above
into a vacant orbital of the metal. The M–C  bond is reaction then
formed by the donation of a pair of electrons from a (a) 4 = K1 + K2 + K3 + K4
filled d orbital of metal into the vacant antibonding
*orbital of carbon monooxide. The metal to ligand (b) 4 = K1 – K2 + K3 – K4
bonding creates a synergic effect. (c) 4 = K1 × K2 × K3 × K4
61. For the complexes - (d) None
Ni(CO) (PF3)3 Ni(CO) (PCl3)3 Ni(CO) (PPh3)3 64. Thermodynamic stability of the complex -
(I) (II) (III) (a) Reflects how slowly it reacts with the ligand
Which of the following option is correct ? (b) Reflects how fast it reacts with the ligand
(a) M–C bond order : III < II < I (c) Measures the strength of metal-ligand bond in
(b) C–O bond order : III < II < I the complex
(c) C–O vibrational frequency : I < II < III (d) Measures how easily it dissociates
(d) All 65. A complecx is kinetically stable provided -
62. For the complexes - (a) it reacts very fast with the ligand
M(CO) M2(CO) M3(CO) (b) it reacts very slow with the ligand
(I) (II) (III) (c) complex is extremely stable in nature of
bonding
Which of the following option is correct ?
(d) activation energy for the complex formation is
(a) M–C bond length : III > II > I very low

www.jeebooks.in
2.30 COORDINATION COMPOUNDS
Passage for Q.66 to Q.68 70. Total charge of the complex is unchanged in -
According to Werner’s coordination theory complex (a) Step-I (b) Step-II
has two types of valencies. (c) Step-III (d) Step-IV
(i) Primary valency : It is the oxidation state of the metal  Match the Column Type
ion in the complex
71. Match the column :-
(ii) Secondary valency : It is number of ligands atoms
Column - I Column - II
coordinatated to metal ion
Complex Features
Three complexes are given
(a) Fe(CO)5 (P) can act as reducing agent
Cl NH3 NH3 Cl NH3 (b) Mn(CO)6 (Q) can undergo dimerisation
H3N
Co (c) Mn(CO)5 (R) can act as oxidising reagent
NH3 H3N Co NH3
H3N (d) [Ti(CO)6] (S) follows Sidgwick E.A.N.
Cl Cl rule
NH3 Cl Cl
NH3 (T) does not follow Sidgwick
(X) (Y) E.A.N. rule
Cl NH3 72. Match the column :-
Column - I Column - II
H3N Co NH3 Complex Features
+1
(a) [Co(NH3)5SO4] (P) only monodentate
Cl NH Cl
3 ligand is present
(Z) +1
(b) [Co(NH3)4SO4] (Q) only bidentate ligand is
66. X is represented as - present
+3
(a) [Co(NH3)6]Cl3 (b) [Co(NH3)4Cl2]Cl (c) [Co(en)3] (R) monodentate &
(c) [Co(NH3)5Cl]Cl2 (d) None bidentate ligand both
67. Z is represented as - are present
2+
(a) [Co(NH3)6]Cl3 (b) [Co(NH3)4Cl2]Cl (d) [Co(NH3)6] (S) chelating ligand present
(c) [Co(NH3)5Cl]Cl2 (d) None (T) metal has +3 oxidation
state
68. In Y which of the following option is correct ?
73. Match the column :-
(a) 1 Cl shows primary valency only
Column - I Column - II
(b) 2 Cl shows secondary valency only
(c) 2 Cl shows primary valency as well as Complex Features
secondary valency (a) [Co(H2O)BrClI] (P) show optical
(d) both (a) and (c) isomerism
Passage for Q.69 to Q.70 (b) [Co(gly)3] (Q) show geometrical
isomerism
n
CoF6  
step  I
[Co(H 2 O)3 F3 ]n  
step  II (c) [Co(NH3)2Cl2(en)]NO3 (R) metal is in +3
n n oxidation state
Co(H 2 O) 6  
step  III
  Co(NH 3 ) 6 
n
(d) K3[CoCl2(C2O4)2] (S) aqueous solution

step  IV
  Co(NO 2 )6  acts as non-
69. Magnetic behaviour is changed in - electrolyte
(a) Step-I (b) Step-II (T) homoleptic
(c) Step-III (d) Step-IV complex

www.jeebooks.in
COORDINATION COMPOUNDS 2.31
74. Match the column :- 78. Match the column :-
Column - I Column - II Column - I Column - II
Complex Features Ions Colour in aqueous solution
3+ 2 3 2+
(a) [Co(H2O)6] (P) d sp (a) Cu (P) Purple
(b) [CoF6]3– (Q) sp3d2 (b) Ni2+ (Q) Green
(c) [Co(NO2)6]3– (R) diamagnetic (c) Co2+ (R) Blue
3+
(d) [CoF3(H2O)3] (S) paramagnetic (d) Ti (S) Pink
(T) central metal ion with 79. Match the column :-
+3 oxidation state Column - I Column - II
75. Match the column :- Complex Type of s, p, d orbitals
Column - I Column - II participated in hybridisation
3+
Complex Features (a) [Ti(H2O)6] (P) 4s, 4p, 4dx2–y2 , 4dz2
(a) [AuF4]– (P) dsp2 hybridisation (b) [Ni(NH3)6]2+ (Q) 3dx2–y2 , 3dz2 , 4s, 4p
(b) [Cu(CN)4]3– (Q) sp3 hybridisation (c) [Zn(NH3)4]2+ (R) 3dx2–y2 , 4s, 4px , 4py
2+ 3 2 2+
(c) [Ni(NH3)6] (R) sp d hybridisation (d) [Cu(NH3)4] (S) 4s, 4px , 4py , 4pz
(d) [Fe(CN)5NOS]4– (S) d2sp3 hybridisation 80. Match the column :-
(T) contains two unpaired Column - I (Complex)
electron (a) [PtCl(NO2)(NH3)4]SO4
76. Match the column :- (b) [Mn(NH3)5(C3H5)]
Column - I Column - II (c) [Pt(NH3)4][Cr(NCS)(NO2)2(NH3)2I]
Ligand Name NH2 2+
(a) N 3 – (P) azido (d) Cl(NH )
3 3Co Co(NH )
3 3 Cl
(b) N 3– (Q) nitrido O2
Column - II (Isomerism shown)
(c) ONO (R) nitrito-O
(P) shows linkage isomerism

(d) NO 3 (S) nitrato
(Q) shows coordination position isomerism
77. Match the column :-
(R) shows ionisation isomerism
Column - I Column - II
(S) shows coordination isomerism
Ligand Features
(T) shows ligand isomerism
(a) NH2–NH2 (P) didentate nonbridging
81. Match the column :-
ligand
Column - I Column - II
(b) NH2CH2CO2– (Q) didentate symmetrical
chelating ligand Ions Colour in aqueous solution
(c) CH3NH2 (R) didentate unsymmetrical (a) V4+ (P) Blue
chelating ligand (b) V 3+
(Q) Green
(d) NH2CH2CH2NH2 (S) monodentate ligand (c) V2+ / Cr3+ / Mn3+ (R) Violet
(T) at least one lone pair of (d) Fe3+ (S) Yellow
electron is present

www.jeebooks.in
2.32 COORDINATION COMPOUNDS
82. Match the column :- (c) Ma3bcd (R) Number of Geometrical
Column - I (Complex / Colour of light absorbed) isomers = 4
(a) [CoCl(NH3)5]2+ / Yellow (d) Ma4bc (S) Total 3 stereoisomers are
possible
(b) [Co(NH3)5(H2O)]3+/ Blue Green
(T) Only one ENT pair is possible
(c) [Co(NH3)6]3+/ Blue
86. Match the column :-
(d) [Cu(H2O)4]2+/ Red/Orange
Column-I Column-II
Column - II (Colour of complex)
Complex Type of isomerism exhibited
(P) Violet
(a) Na2[Pt(SCN)2(Ox)2] (P) Ionisation
(Q) Red
(b) [CrCl2(NH3)4]NO3 (Q) Linkage
(R) Yellow Orange (c) [Pt(NO2)(gly)(NH3)] (R) Geometrial
(S) Blue (d) K3[Fe(OH)2(C2O4)2] (S) Optical
83. Match the column :- 87. Match the following :-
Column - I (Ligand) Column-I
(a) O 2CCH 2 NHCH 2CO 2 +4
(b) O 2CCH 2 N  C(O ) CH 2CO 2 NO2
(a) (en)2Co Co(en)2
(c) NH 2 (CH 2 ) 2 NH(CH 2 )2 NH 2
NH2

(d)
N en NO2 en
N N

Column - II (Characteristic feature) (b)


(P) tridentate ligand
(Q) chelating ligand en
NH2
(S) nitrogen and oxygen can act as donor atom en
(R) only nitrogen can act as donor atom en en
NO2
(T) two oxygen atoms and one nitrogen atom act
as donor atom (c)
84. Match the column :-
Column - I Column - II NH2
2–
en en
(a) [Ti(CO)6] (P) Highest C–O bond vibration en
en NO2
streching frequency
(b) [V(CO)6]1– (Q) Longest M–C bond length (d)
(c) [Cr(CO)6] (R) Follows sidgwick rule
(d) [Mn(CO)6]+1 (S) Synergic effect is observed
NH2 en
85. Formula of complex features - en
Column-I Column-II Column-II
(a) Ma3b2c (P) All stereoisomers are optically (P) Meso form
inactive (Q) Optically active form (either d or s form)
(b) Ma3b3 (Q) Number of Geometrical (R) Polynuclear complex
isomers = 2 (S) Plane of symmetry if present

www.jeebooks.in
COORDINATION COMPOUNDS 2.33
88. Match the column :- 92. Match the column :-
Column - I Column - II Column - I Column - II
Complex Features Complex Features
4–
(a) [Cr(CN)3(NO2)3] (P) Outer orbital octahedral (a)  Ni(H 2O)6  Cl2 (P) d2sp3 hybridised
complex
(b) Co(CN)2 (NH3 )4  OEt (Q) I o n i s a t i o n
(b) [Co(C2O4)3]3– (Q) Inner orbital octahedral isomerism
complex 3
(c)  IrCl6  (R) µ = 2.83 B.M.
–1
(c) [Fe(EDTA)] (R) µ = 2 2 B.M.
(d)  PtCl2 (NH3 ) 4  Br2 (S) 2 unpaired
(d) [Ni(en)3] (NO3)2 (S) Shows optical activity
electron is present
89. Match the column :- 93. Match the column :-
Column - I Column - II Column - I Column - II
Complex Features Complex Features
(a) [Co(CN)(NH3)5]SO4 (P) Diamagnetic (a) [NiCl2(PPh3)2] (P) Paramagnetic with 1
(b) K[Co(CO)4] (Q) O.N. of metal 0 unpaired electron
(c) K4[Co(C2O4)3] (R) Metal - Metal (b) V(CO)6 (Q) Paramagnetic with 2
linkage unpaired electron
(d) Co2(CO)8 (S) Paramagnetic (c) [Cr(CN)6]4– (R) C.N. of metal ion = 4
(T) Optically active (d) Ni(CO)4 (S) Diamagnetic
90. Match the column :- (T) C.N. of metal ion = 6
Column - I Column - II 94. Match the column :-
Complex Features Column - I (Compound)
(a) Sodiumnitropruside (P) µ = 0 BM (a) Double salt
(b) Brown ring complex (Q) Octahedral (b) Ethylene diaminetriacetato
(c) Complex of Ag used for (R) µ = 15 B.M. (c) [Cr(NH3)6]3+
(d) t2g
its extraction
Column - II (Features)
(d) Potassium ferrocyanide (S) NO + acts as
ligand (P) FeSO4.(NH4)2SO4.6H2O
91. Match the column :- (Q) Pentadentate ligand
Column - I Column - II (R) Paramagnetic
Types of complex Total possible (S) Triply degenrate orbitals
geometrical isomers 95. Match the column :-
n Column - I Column - II
(a)  M  AB 2 ab   (P) 2
 a / b Geometry Complex
n
(b)  Ma 2 b 2 c 2  
 b / d
 (Q) 6 (a) Trigonal bipyramidal (P)  Cu(NH 3 ) 4 
2

n
 a / c
(c)  M  AA 2 a 2    (R) 3 (b) Tetrahedral (Q)  Cr(H 2O) 6 
3

 a / b
(d)  M  AB  a 3b    (S) 5 (c) Square planar (R)  Ni(CN)5 
3

(T) 0 2–
(d) Octahedral (S)  NiCl 4 

www.jeebooks.in
2.34 COORDINATION COMPOUNDS
96. Match the column :- Column - II
Column - I Column - II (P) In one of the isomeric forms , the metal satisfies
(Ion) (cfse in high-spin octahedral its secondary valences by only neutral
molecules
complex)
6 (Q) The magnetic moment of one of the isomeric
(a) d (P) 4Dq0
7
form is 3.87 B.M.
(b) d (Q) 8Dq0
(R) Show ionisation isomerism
(c) d8 (R) 12Dq0
(S) Show hydrate isomerism
9
(d) d (S) 6Dq0
(T) Show linkage isomerism
97. Match the column :-
101. Match the column :-
Column - I Column - II
Column - I (Ligands)
(Ion) (cfse in high-spin octahedral
(a) CO, CN–, C2H4, NO+
complex)
(b) H–, NH3
4
(a) d (P) 16Dq0 – P
(c) H2O, NH2–, ROH, R2S
5
(b) d (Q) 20Dq0 – 2P (d) PF3, PR3 , AsMe3
(c) d6 (R) 24Dq0 – 2P Column - II (Nature of ligands)
7
(d) d (S) 18Dq0 – P (P)  donor ligand
98. Match the column :- (Q)  donor ligand
Column - I Column - II (R)  acceptor ligand
Formula of complex Number of stereoisomers (S) strong field ligand
(a) M(AA)(BC)de (P) 4 (AA = Bidentate (T) weak field ligand
symmetrical ligand)
(b) M(AB)(AB)cd (Q) 13 (CD/BC/AB = EXERCISE # IV
Bidentate non symmet-  Integer Type :
rical ligand)
1. How many compounds are expected to yield a
(c) Mabcdef (R) 11(abcdef = Monodentate white precipitate with AgNO3 solution ?
ligand)
PtCl4.4NH3 ; CoCl3.3NH3 ; Vinyl chloride ; Allyl
(d) M(AB)3 (S) 10 chloride, Carnalite ; Tertiary butyl chloride
99. Match the column :- 2. What is the coordination number of central atom
Column - I Column - II in [Pt(trien)]2+ ?
Complex Total number of ions 3. Find out number of compleses which follow
(a) PtCl4 . 6NH3 (P) 2 sidgwick’s rule of EAN :-
(b) PtCl4 . 5NH3 (Q) 3 H[AuCl4] ; [Pt(NH3)4Cl2]2+ ; [HgI4]2– ; NiCl42– ;
[Ti(C 5 H 5 ) 2 (C 5 H 5 ) 2 ] ; K 3 [Fe(CN) 6 ] ;
(c) PtCl4 . 4NH3 (R) 4
K4[Fe(CN)6] ; [Ag(S2O3)2]–1 ; [Fe(H2O)5NO]2+
(d) PtCl4 . 3NH3 (S) 5
4. How many statements are correct regarding
100. Match the column :- [Ni(DMG)2] ?
Column - I (i) the complex is diamagnetic.
(a) CoCl3 . 5NH3 . H2O (ii) the complex forming 5 membered chelate with
(b) Co(NH3)5BrSO4 ligand and metal ion.
(c) CrCl3 . 6H2O (iii) the complex forming hydrogen bonding and 6
(d) Cr(NO2)2 . 6H2O membered chelation is possible.

www.jeebooks.in
COORDINATION COMPOUNDS 2.35
(iv) the complex forming 6 membered chelate with 12. Find out number of bidentate ligand where both
ligand and metal ion. nitrogen atom act as donor atom and 5 member
(v) it is rosy red colour. chelate is formed with metal ion :-
(vi) it is tetrahedral complex. CH3
NH2
(vii) it is square planar complex. CH2 – CH2
CH2 – C – CH3
(viii)O, O is the donor atom for ligand. ; ; ;
NH2 NH2 NH
NH2 NH2 2
(ix) N, N is the donor atom for ligand.
5. How many ligands are example of non classical
ligand ? ; NH2CH2CH2CH2NH2
 – –
C2H4 , CO, N O , CN , PF3, PR3 . C2(CN)4 O NH2
6. In Zeise’s salt, how many statements are correct :- 13. What is the coordination number of aluminium in
cryolite ?
(a) oxidation state of Pt is +2 & corrdination
number of Pt is 4 14. Number of optically active isomers of Mabcdef in
(b) first organometallic complex discovered which a and e are on opposite position.
(c) it is example of cationic complex  NH3
15. [M(NH 3 ) 2 (py) 2 (NO 2 ) 2 ]   [M(NH 3 ) 3
 NO 2
(d) the carbon atom exhibit some electrophilic (py)2(NO2)]
character Number of geometrical isomers reduced in product
(e) HCH bond angle is 115° side as compared to reactants -
(f) carbon atom is Zeise’s salt is perfectly sp3 16. Total number of isomers for the complex
hybridised K2[PtCl2BrSCN].
(g) ethylene moiety of Zeise’s salt looses planarity 17. Number of geometrical isomer in Mabcdef where
(h) C–C bond length in C2H4 < C–C bond length a,b,c are at adjacent position with respect to each
in Zeises salt other.
(i) synergic effect takes place (back donation takes 18. Find out number of linkage isomers possible for
place) between filled d orbital of Pt to * K4[Fe(CN)6] :-
ABMO of ethylene molecule
19. Find out number of coordination isomers for the
7. How many isomers are possible for the complex
complex [Cr(NH3)6][Co(NO2)6] :-
[Ir(CO)Cl(PPh3)2] ?
20. Find out number of coordination isomers for the
8. How many type of isomerism is exhibited by the
complex [Co(NH3)4(NO2)2]Cl ? complex [Pt(NH3)4][CuCl4]
9. How many total stereoisomers are possible for the 21. Find out geometrical isomers are possible for
complex [Rh(en)2(NO2)(SCN)]+ ? complex [Mab(AB)2]n±
10. Find the number of ions which have higher splitting 22. How many geometrical isomers are possible for
energy than the Mn2+ in octahedral complex compound [Co(gly)Cl2(pn)].
considering same ligands for all ions :- 23. How many ligands forming d-d back bonding
Cu2+ ; Ni2+ ; Co2+ ; Fe2+ ; Cr3+ (synergic bonding) with metal ion,
11. Find out the number of bidentate ligands where CO ; NO+ ; CN–, PF3 ; PPh3 ; H2O ; en ; AsR3
both oxygen atoms act as donor atom :-
24. In [Co(EDTA)]–1 number of N–Co–O linkage
(a) acetylacetonato (b) biuret present ?
(c) dimethylglyoximato (d) glycinato
25. Find the total number of stereo isomers present in
(e) biguanido (f) oxalato [Pt(bn)2] :-
(g) dimethylglycol

www.jeebooks.in
2.36 COORDINATION COMPOUNDS
26. Find the change in magnetic moment of the complex 41. If crystal field stabilization energy of [ML6]+n is
formed in following process :- –0.80. (0 = 10dq)
2 6H2O
Fe  NH3 6  
2 Find minimum number of electron in t2g orbital of
–6NH3
 Fe  H2O6 
metal ion ?
27. Find the maximum number of atoms lying in one 42. A co-ordination compound have magnetic moment
plane for [Cr(CN)6]3– :- 3.83 B.M.. Find out the number of unpaired
28. How many hexagonal rings are present for C84 electron(s) in the compound:-
fullerene :- 43. Find number of ligands which are stronger ligand
29. How many of the following have square planar as compared to pyridine :-
geometry ? NO2– ; H2O ; NO3– ; F– ; en ; C2O4– ; Cl– ; CN– ;
[PtCl4]2– , PdCl42– , AgF4–1 , AuCl4– , ICl4– , CoCl4–, CO ; o-phenanthroline ; dipyridyl.
FeCl42– , NiCl42– 44. In K2[Ni(EDTA)] total number of 5 member ring
30. What are the number of all possible constitutional is m and total number of G.I. possible is n.
isomers of Rn(NH3)5(NO2)Cl ? :- Find out m × n :-
31. Maximum numebr of atoms which are in same 45. In [Co(gly)3] how many O–Co–N linkage is
plane in Ni(CO)4 :- present -
32. Find the number of ligands which are stronger ligand 46. Find out number of unpaired electron for d5 ion
as compound to ethylenediamine(en) having d2sp3 hybridisation :-
NO2– ; H2O ; NO3– ; F– ; C2O42– ; NH3 ; Cl– ; 47. Find out number of unpaired electron for d7 ion
CN– having sp3d2 hybridisation :-
33. Find the number of optically active isomers for 48. If number of unpaired electron for d6 ion having
sp3d2 hybridisation is n1 and number of unpaired
 Pd(en)2 (NH3 )(H 2O) :- electron for d6 ion having d2sp3 hybridisation is n2
cation

34. What are the number of all possible isomeric then find out n1 – n2 :-
compound for a square planar palladiium(II) 49. What is the number of valence shell electron for
complex that contant two Cl– and two SCN– the following complex : [Fe(CO)2(NO)2]
ligands ? 50. Total isomeric complex salt isolated having formula
35. Find out the number of unpaired electron present CrCl3.6H2O
in FeCl42– :-
36. Find out the number of unpaired electron present
EXERCISE # V(A) JEE–MAIN
in NiCl42– :- 1. In [Cr(C2O4)3]3– , the isomerism shown is :–
37. In MnCl42– , how many unpaired electron present [AIEEE 2002]
in e orbital according to CFT :- (a) Ligand (b) Optical
38. In CoCl42– , how many unpaired electron present (c) Geometrical (d) Ionisation
in t2 orbital according to CFT :- 2. In the complexes [Fe(H2O)6]3+, [Fe(SCN)6]3–,
39. Find out the number of stereoisomer present in [Fe(C2O4)3]3– and [FeCl6]3– , most stability is
M(AB)2a2 :- shown by :– [AIEEE 2002]
3+
Where AB is bidentate nonsymmetrical ligand and (a) [Fe(H2O)6] (b) [Fe(SCN)6]3–
a is monodentate ligand. (c) [Fe(C2O4)3]3– (d) [FeCl6]3–
40. Find out the number of stereoisomer present in 3. One mole of the complex compound Co(NH3)5Cl3,
M(AB)2ab :- gives 3 moles of ions on dissolution in water. One
Where AB is bidentate nonsymmetrical ligand and mole of the same complex reacts with two moles
a & b is monodentate ligand. of AgNO3 solution to yield two moles of AgCl(s).

www.jeebooks.in
COORDINATION COMPOUNDS 2.37
The structure of the complex is :–[AIEEE 2003] 9. Which of the following complex is an outer orbital
(a) [Co(NH3)3Cl3].2NH3 complex :– [AIEEE 2004]
3+
(b) [Co(NH3)4Cl2]Cl.NH3 (a) [Co(NH3)6] (b) [Mn(CN)6]4–
(c) [Co(NH3)4Cl].Cl2.NH3 (c) [Fe(CN)6]4– (d) [Ni(NH3)6]2+
(d) [Co(NH3)5Cl]Cl2 10. Coordination compounds have great importance
4. In the coordination compound K4[Ni(CN)4], the in biological systems. In this context which of the
oxidation state of nickel is :– [AIEEE 2003] following statement is incorrect ? [AIEEE 2004]
(a) 0 (b) +1 (a) Cyanocobalamin is vitamin B12 and contains
cobalt
(c) +2 (d) –1
(b) Haemoglobin is the red pigment of blood and
5. The number of 3d–electrons remained in Fe2+ contains iron
(At. no. of Fe = 26) ion is :– [AIEEE 2003]
(c) Chlorophylls are green pigment in plants and
(a) 4 (b) 5 contains calcium
(c) 6 (d) 3 (d) Carboxypeptidase – A is an enzyme and
6. Ammonia forms the complex ion [Cu(NH3)4]2+ contains zinc
with copper ions in alkaline solutions but not in 11. The correct order of magnetic moments (spin only
acidic solution. What is the reason for it :– values in B.M.) among is :– [AIEEE 2004]
[AIEEE 2003]
(a) [Fe(CN)6] > [MnCl4] > [CoCl4]2–
4– 2–
(a) In acidic solutions hydration protects copper
ions (b) [MnCl4]2– > [Fe(CN)6]4– > [CoCl4]2–
(b) In acidic solutions protons coordinate with (c) [MnCl4]2– > [CoCl4]2– > [Fe(CN)6]4–
ammonia molecules forming NH4+ ions and (d) [Fe(CN)6]4– > [CoCl4]2– > [MnCl4]2–
NH3 molecules are not available. 12. For octahedral complex , the value of the ‘spin
(c) In alkaline solutions insoluble Cu(OH2) is only’ magnetic moment for one of the following
precipitated which is soluble in excess of any configuration is 2.84 B.M. The correct one is :–
alkali [AIEEE 2005]
(d) Copper hydroxide is an amphoteric substance (a) d4 (in strong ligand field)
7. Among the properties (b) d4 (in weak ligand field)
(a) reducing (b) oxidising (c) complexing, (c) d3 (in weak as well as strong field)
the set of properties shown by CN– ion towards (d) d5 (in strong ligand field)
metal species is :– [AIEEE 2004] 13. The IUPAC name for the complex
(a) c,a (b) b,c [Co(NO2)(NH3)5]Cl2 is :– [AIEEE 2006]
(c) a,b (d) a,b,c (a) pentaammine nitrito–N–cobalt(II)chloride
8. The coordination number of a central metal atom (b) pentaammine nitrito–N–cobalt(III)chloride
in a complex is determined by :– [AIEEE 2004] (c) nitrito–N–pentaamminecobalt(III)chloride
(a) The number of ligands around a metal ion (d) nitrito–N–pentaamminecobalt(II)chloride
bonded by sigma and pi–bonds both 14. Nickel (Z = 28) combines with a uninegative
(b) The number of ligands around a metal ion monodentate ligand X– to form a paramagnetic
bonded by pi –bonds complex [NiX4]2–. The number of unpaired
(c) The number of ligands around a metal ion electron in the nickel and geometry of this complex
bonded by sigma bonds ion are, respectively :– [AIEEE 2006]
(d) The number of only anionic ligands bonded to (a) one , square planar (b) two , square planar
the metal (c) one, tetrahedral (d) two , tetrahedral

www.jeebooks.in
2.38 COORDINATION COMPOUNDS
15. In Fe(CO)5 , the Fe–C bond possesses :– 23. Which one of the following has an optical isomer ?
(a) ionic character [AIEEE 2006] (en = ethylene diamine) [AIEEE 2010]
(b) –character only (a) [Zn(en)2]2+ (b) [Zn(en)(NH3)2]2+
(c) –character only (c) [Co(en)3]3+ (d) [Co(H2O)4(en)]3+
(d) both  and character 24. A solution containing 2.675 g of CoCl3 . 6NH3
16. How many EDTA (ethylenediaminetetraacetate) (molar mass = 267.5 g mol–1) is passed through a
molecule are required to make an octahedral cation exchanger. The chloride ions obtained in
complex with a Ca2+ ion ? [AIEEE 2006] solution were treated with excess of AgNO3 to
(a) One (b) Two give 4.78 g of AgCl (molar mass = 143.5 g mol–1).
(c) Six (d) Three The formula of the complex is :– [AIEEE 2010]
17. The “spin–only” magnetic moment [in units of Bohr (At. mass of Ag = 108 u)
magneton, (µB)] of Ni2+ in aqueous solution would (a) [CoCl(NH3)5]Cl2 (b) [Co(NH3)6]Cl3
be (At. no. Ni = 28) :– [AIEEE 2006] (c) [CoCl2(NH3)4]Cl (d) [CoCl3(NH3)3]
(a) 0 (b) 1.73 25. Which of the following facts about the complex
(c) 2.84 (d) 4.90 [Cr(NH3)6]Cl3 is wrong :– [AIEEE 2011]
18. Which one of the following has a square planar (a) The complex is an outer orbital complex
geometry :– [AIEEE 2007] (b) The complex gives white precipitate with silver
(Co = 27, Ni = 28, Fe = 26, Pt = 78) nitrate solution
(a) [CoCl4]2– (b) [FeCl4]2– (c) The complex involves d2sp3 hybridisation and
(c) [NiCl4]2– (d) [PtCl4]2– is octahedral in shape
19. The coordination number and the oxidation state (d) The complex is paramagnetic
of the element ‘E’ in the complex [E(en)2(C2O4)] 26. The magnetic moment (spin only) of [NiCl4]2– is :–
NO 2 (where en is ethylenediamine) are, [AIEEE 2011]
respectively :– [AIEEE 2008]
(a) 2.82 B.M. (b) 1.41 B.M.
(a) 6 and 2 (b) 4 and 2
(c) 1.82 B.M. (d) 5.46 B.M.
(c) 4 and 3 (d) 6 and 3
27. Among the ligands NH3 , en, CN– and CO the
20. In which of the following octahedral complex of
correct order of their increasing field strength, is :–
Co(at. no. 27), will the magnitude of 0 be the
highest :– [AIEEE 2008] (a) CO < NH3 < en < CN– [AIEEE 2011]

(a) [Co(CN)6] 3–
(b) [Co(C2O4)3]3– (b) NH3 < en < CN < CO
(c) [Co(H2O)6]3+ (d) [Co(NH3)6]3+ (c) CN– < NH3 < CO < en
21. Which of the following pairs represents linkage (d) en < CN– < NH3 < CO
isomers :– [AIEEE 2009] 28. Which one of the following complex ions has
(a) [Co(NH3)5NO3]SO4 and [Co(NH3)5SO4]NO3 geometrical isomers :– [AIEEE 2011]
(b) [PtCl2(NH3)4]Br2 and [PtBr2(NH3)4]Cl2 (a) [Co(en)3]3+ (b) [Ni(NH3)5Br]+
(c) [Cu(NH3)4][PtCl4] and [Pt(NH3)4][CuCl4] (c) [Co(NH3)2(en)2]3+ (d) [Cr(NH3)4(en)]3+
(d) [Pd(PPh3)2(NCS)2] and [Pd(PPh3)2(SCN)2] 29. Which among the following will be named as
22. Which of the following has an optical isomer :– dibromidobis (ethylene diamine) chromium (III)
[AIEEE 2009] bromide [AIEEE 2012]
3+
(a) [Co(H2O)4(en)] (b) [Co(en)2(NH3)2]3+ (a) [Cr(en)Br2]Br (b) [Cr(en)3]Br3
(c) [Co(NH3)3Cl]+ (d) [Co(en)(NH3)2]2+ (c) [Cr(en)2Br2]Br (d) [Cr(en)Br4]–

www.jeebooks.in
COORDINATION COMPOUNDS 2.39
30. The complex ion 37. The equation which is balanced and represents
[Pt(NO2)(Py)(NH3)(NH2OH)]+ will give :– the correct product (s) is : [JEE–MAIN–2014|
[JEE–MAIN–2012, Online] (a) [Mg(H2O)6]2+ + (EDTA) 4–  excess NaOH

(a) 4 isomers (Geometrical) 2+
[Mg(EDTA)] + 6H2O
(b) 2 isomers (Geometrical) (b) CuSO4 + 4KCN  K2[Cu(CN)4] + K2SO4
(c) 3 isomers (Geometrical) (c) Li2O + 2KCl  2LiCl + K2O
(d) 6 isomers (Geometrical) (d) [CoCl(NH3)5]+ + 5H+  Co2+ + 5NH4+ + Cl–
31. Which of the following complex ions will exhibit 38. The correct statement about the magnetic
optical isomerism :– [JEE–Main–2012, Online] properties of [Fe(CN) 6]3– and [FeF 6]3– is :
(en = 1,2–diamine ethane) (Z = 26). [J–MAIN–2014, Online]
(a) [Co(en)2Cl2]+ (b) [Zn(en)2]2+ (a) [Fe(CN)6] is paramagnetic, [FeF 6]3– is
3–

(c) [Co(NH3)4Cl2]+ (d) [Cr(NH3)2Cl2]+ diamagnetic.


32. Which of the following complex species is not (b) both arc diamagnetic.
expected to exhibit optical isomerism :– (c) [Fe(CN) 6 ] 3– is diamagnetic, [FeF 6 ] 3– is
[JEE–Main–2013] paramagnetic.
(a) [Co(en)3] 3+
(b) [Co(en)2Cl2]+ (d) both are paramagnetic
(c) [Co(NH3)3Cl3] (d) [Co(en)(NH3)2Cl2]+ 39. An octahedral complex of Co3+ is diamagnetic. The
hybridisation involved in the formation of the
33. Type of isomerism which exist between
complex is [J–MAIN–2014]
[Pd(C6H5)2(SCN)2] and [Pd(C6H5)2(NCS)2] is :–
(a) d2sp3 (b) sp3d
[JEE–Main–2002, Online]
(c) dsp2 (d) sp3d2
(a) Solvate isomerism
40. Which of the following name formula combinations
(b) Ionisation isomerism
is not correct? [J–MAIN–2014, Online]
(c) Linkage isomerism
Formula Name
(d) Coordination isomerism
(a) K[Cr(NH3)2.Cl4] Potassium diammine
34. Which of the following is diamagnetic ? Tetrachlorochromate
[JEE–MAIN–2013, Online] (III)
3–
(a) [CoF6] (b) [FeF6]3– (b) [Co(NH3)4(H2O)I]SO4 T e t r a a m m i n e
(c) [Fe(CN)6]3– (d) [Co(Ox)3]3– aquaiodo cobalt (III)
35. The magnetic moment of the complex anion sulphate
[Cr(NO)(NH3)(CN)4]2– is :– (c) [Mn(CN)5] 2–
Pentacyanomagnate
[JEE–Main–2013, Online] (II) ion
(a) 2.82 B.M. (b) 5.91 B.M. (d) K2[Pt(CN)4] Potassiumtetracya
(c) 1.73 B.M. (d) 3.87 B.M. noplatinate(II)
36. The octahedral complex of a metal ion M3+ with
four monodentate ligands L1, L2, L3 and L4 absorb 41. Consider the coordination compound,
wavelength in the region of red, green, yellow and [Co(NH3)6]Cl3. In the formation of this complex,
blue respectively. The increasing order of ligand the species which acts as the Lewis acid is :
strength of the four ligands is : [JEE–Main–2014] [J–MAIN–2014, Online]
(a) L3 < L2 < L4 < L1 (b) L1 < L2 < L4 < L3 (a) [Co(NH3)6]3+ (b) NH3
(c) L4 < L3 < L2 < L1 (d) L1 < L3 < L2 < L4 (c) Co 3+
(d) Cl–

www.jeebooks.in
2.40 COORDINATION COMPOUNDS
42. Among the following species the one which (a) (a) and (b) show geometrical and optical
causes the highest CFSE, 0as a ligand is: isomerism
[J–MAIN–2014, Online] (b) (b) and (c) show geometrical and optical

(a) CN (b) NH 3 isomerism
(c) CO (d) F – (c) (a) and (b) show only geometrical isomerism
43. Which one of the following complexes will most (d) (b) and (c) show only geometrical optical
likely absorb visible light ? isomerism
[J–MAIN–2014, Online] 50. Which one of the following complexes shows
(At. nos. Sc = 21, Ti = 22, V = 23, Zn = 30) :– optical isomerism :- [J–MAIN–2016]
(a) [Ti(NH3)6]4+ (b) [V(NH3)6]3+ (a) [Co(NH3)4Cl2]Cl
2+ 3+
(c) [Zn(NH3)4] (d) [Sc (H2O)6] (b) [Co(NH3)3Cl3]
44. Nickel(Z = 28) combines with a uninegative (c) cis [Co(en)2Cl2]Cl
monodentate ligand to form a diamagnetic complex
(d) trans[Co(en)2Cl2]Cl
[NiL4]2–. The hybridisation involved and the
number of unpaired electrons are respectively: (en = ethylenediamine)
[J–MAIN–2014, Online] 51. The pair having the same magnetic moment is :-
(a) sp3, zero (b) sp3; two [J–MAIN–2016]
(c) dsp2, one (d) dsp2, zero (At. No. : Cr = 24, Mn = 25, Fe = 26, Co = 27)
45. The number of geometrical isomers that can exist (a) [CoCl4]2– and [Fe(H2O)6]2+
for square planar [Pt(Cl)](py)(NH3)(NH2OH)]+ is (b) [Cr(H2O)6]2+ and [CoCl4]2+
(py = pyridine) :- [J–MAIN–2015]
(c) [Cr(H2O)6]2+ and [Fe(H2O)6]2+
(a) 4 (b) 6
(d) [Mn(H2O)6]2+ and [Cr(H2O)6]2+
(c) 2 (d) 3
52. Which one of the following complexes will consume
46. The colour of KMnO4 is due to :- highest equivalents of aqueous solution of
[J–MAIN–2015] Ag(NO3)? [J–MAIN–2016-Online]
(a) L  M charge transfer transition (a) [Cr(H2O)6]Cl3 (b) Na2[CrCl5(H2O)]
(b) – transition (c) Na3[CrCl6] (d) [Cr(H2O)5Cl]Cl2
(c) M  L charge transfer transition 53. Identify the correct trend given below :-
(d) d –d transition [J–MAIN–2016, Online]
47. Which of the following complex ions has electrons (At. No. : Ti = 22, Cr = 24 and Mo = 42)
that are symmetrically filled in both t2g and eg
(a) 0 of [Cr(H2O)6]2+ < [Mo(H2O)6]2+ and 0 of
orbitals ? [J–MAIN–2015, Online]
[Ti(H2O)6]3+ < [Ti(H2O)6]2+
(a) [CO(H2O)6]2+ (b) [CoCl6]3–
(b) 0 of [Cr(H2O)6]2+ < [Mo(H2O)6]2+ and 0 of
(c) [CoCl4]2– (d) [CoCl6]4–
[Ti(H2O)6]3+ > [Ti(H2O)6]2+
49. The correct statement on the isomerism associated
(c) 0 of [Cr(H2O)6]2+ > [Mo(H2O)6]2+ and 0 of
with the following complex ions,
[Ti(H2O)6]3+ > [Ti(H2O)6]2+
(a) [Ni(H2O)5NH3]2+ [J–MAIN–2015, Online]
2+ (d) 0 of [Cr(H2O)6]2+ > [Mo(H2O)6]2+ and 0 of
(b) [Ni(H2O)4(NH3)2] and
[Ti(H2O)6]3+ < [Ti(H2O)6]2+
(c) [Ni(H2O)3 (NH3)3]2+ is

www.jeebooks.in
COORDINATION COMPOUNDS 2.41
EXERCISE # V(B) JEE-ADVANCED Fill in the Blanks
9. The type of magnetism exhibited by [Mn(H2O)6]2+
True/False
ion is (paramagnetic) [JEE 1994]
1. Tho electron density in the xy plane in 3dx2 – y2 orbital
is zero. [JEE 1986] 10. The IUPAC name of [Co(NH3)6]Cl3, is
2. Amongst the following the lowest degree of [JEE 1994]
paramangetism per mole of the compound at 298 11. Identify the complexes which are expected to be
K will shown by ; [JEE1988] coloured [JEE 1994]
(a) MnSO4 . 4H2O (b) CuSO4 . 5H2O (a) Ti(NO3)4
(c) FeSO4 . 6H2O (d) NiSO4 . 6H2O (b) [Cu(NCCH3)4]+BF4–
True/False (c) [Cr(NH3)6]Cl3
3. Both potassium ferrocyanide and potassium (d) K3[VF6]
ferricyanide are diamagnetic. [JEE 1989]
12. Which of the following is formed when excess of
4. Mixture X = 0.02 mole of [Co(NH3)5SO4]Br and
KCN is added to aqueous solution of copper
0.02 mole of [Co(NH3)5Br]SO4 was prepared in
2 L of solution. [JEE 1991] sulphate ? [JEE 1996]
1 L of mixture X + excess AgNO3  Y (a) Cu[CN]2 (b) K2[Cu[CN]4]
1 L of mixture X + excess BaCl2  Z (c) K[Cu[CN]2] (d) K3[Cu(CN)4]
Number of moles of Y and Z are 13. Which of the following is an organometallic
(a) 0.01, 0.01 (b) 0.02, 0.01 compound [JEE 1997]
(c) 0.01, 0.02 (d) 0.02, 0.02 (a) Lithium methoxide
5. Amongst [Ni(CO)4], [Ni(CN)4]2– and [NiCl4]2– (b) Lithium acetate
[JEE 1991] (c) Lithium dimethylamide
2–
(a) [Ni(CO)4] and [Ni(CN)4] are diamagnetic (d) Methyl lithium
and [NiCl4]2–is paramagnetic
14. In nitroprusside ion, the iron and NO exist as FeII
(b) [NiCl4]2– and [Ni(CN)4]2– are diamagnetic and and NO+ rather than FeIII and NO. These forms
[Ni(CO)4] is paramagnetic
can be differentiated by [JEE 1998]
(c) [Ni(CO)4], [NiCl4]2– and [Ni(CN)4]2– are
diamagnetic. (a) estimating the concentration of iron
(d) [Ni(CO)4] is diamagnetic and [Ni(CN)4]2– , (b) measuring the concentration of CN–
[NiCl4]2–are paramagnetic (c) measuring the solid state magnetic moment
6. The compound which does not show (d) thermally decomposing the compound
paramagnetism is [JEE 1992] 15. The geometry of Ni(CO)4 and Ni(PPh3)2Cl2 are
(a) [Cu(NH3)4]Cl2 (b) [Ag(NH3)2]Cl [JEE 1999]
(c) NO (d) NO 2 (a) both square planar
7. The number of d–electron is [Cr(H2O)6]3+ [At No.. (b) tetrahedral and square planar
of Cr = 24] is [JEE 1993]
(c) both tetrahedral
(a) 2 (b) 3
(d) square planar and tetrahedral
(c) 4 (d) 5
8. Amongst the following ions which one has the 16. The complex ion which has no ‘d’ electrons in the
highest paramagnetism [JEE 1993] central atom is : [JEE 2001]
(a) [Cr(H2O)6] 3+
(b) [Fe(H2O)6]2+ (a) [MnO4]– (b) [Co(NH3)6]3+
(c) [Cu(H2O)6]2+ (d) [Zn(H2O)6]2+ (c) [Fe(CN)6]3– (d) [Cr(H2O)6]3+

www.jeebooks.in
2.42 COORDINATION COMPOUNDS
17. The correct order of hybridisation of the central (c) Tctracyanidonickel (II), tetrachloridonickel (II)
atom in the following species. [JEE2001] (d) Potassium tetracyanidonickcl (II), potassium
2–
NH3,[PtCl4] , PCl5 and BCl3, is [At No. Pt = 78] tetrachloridonickel (II)
(a) dsp2,sp3d,sp2 and sp3 24. Predict the magnetic nature of A and B.
(b) sp3,dsp2, sp3d, sp2 (a) Both are diamagnetic
(c) dsp2,sp2,sp3 and sp3d (b) A is diamagnetic and B is paramagnetic with
(d) dsp2, sp3,sp2 and sp3d two unpaired electrons
18. The species having tetrahedral shape is : (c) A is diamagnetic and B is paramagnetic with
[JEE2004] one unpaired electron
(a) [PdCl4] 2–
(b) [Ni(CN)4]2– (d) Both are paramagnetic
(c) [Pd(CN)4]2– (d) [NiCl4]2– 25. The hybridisation of A and B –
19. The pair of compounds having metals in their highest (a) dsp2, sp3 (b) sp3, sp3
oxidation state is [JEE2004] (c) dsp2, dsp2 (d) sp3d2, d2sp3
(a) MnO2, FeCl3 26. If the bond length of CO bond in carbon monoxide
(b) [MnO4]–, CrO2Cl2 is 1.128 Å, then what is the value of CO bond
(c) [Fe(CN)6]3–, [Co(CN)3] length in Fe(CO)5? [JEE 2006]
(d) [NiCl4]2– , [CoCl4]– (a) 1.15 Å (b) 1.128 Å
20. Spin only magnetic moment of the compound Hg (c) 1.72 Å (d) 1.118 Å
[Co(SCN)4] is [JEE2004] 27. Among the following metal carbonyls, the C–O
bond order is lowest in [JEE 2007]
(a) 3 (b) 15
+
(a) [Mn(CO)6] (b) [Fe(CO)5]
(c) 24 (d) 8
(c) [Cr(CO)6] (d) [V(CO)6]–
21. Which of the following pair is expected to exhibit
same colour in solution? [JEE2005] 28. Match the complexes in Column I with their
properties listed in Column II. Indicate your
(a) VOCl2 ; FeCl2 (b) CuCl2; VOCl2
answer by darkening the appropriate bubbles of
(c) MnCl2 ; FeCl2 (d) FeCl2;CuCl2 the 4 x 4 matrix given in the ORS. [JEE 2007]
22. Which type of isomerism is shown by Column I
Co(NH3)4Br2Cl? [JEE2005]
(a) [Co(NH3)4(H2O)2]Cl2
(a) Geometrical and Ionisation
(b) [Pt(NH3)2Cl2]
(b) Optical and Ionisation
(c) [Co(H2O)5Cl]Cl
(c) Geometrical and Optical
(d) [Ni(H2O)6]Cl2
(d) Geometrical only
Column II
Question No. 23 to 25 (3 questions)
(P) Geometrical isomers
The coordination number of Ni2+ is 4. [JEE2006]
(Q) Paramagnetic
NiCl2 + KCN (excess)  A (cyano complex)
(R) Diamagnetic
NiCl2 + KC1 (excess)  B (chloro complex)
(S) Metal ion with 2+ oxidation state
23. The IUPAC name of A and B are
29. Among the following, the coloured compound is
(a) Potassium tetracyanidonickelate (II), potassium
[JEE 2008]
tctrachloridonickelate (II)
(a) CuCl (b) K3[Cu(CN)4]
(b) Tetracyanidopotassiumnickelate (II),
tctcrachloridopolassiumnickelate (II) (c) CuF2 (d) [Cu(CH3CN)4]BF4

www.jeebooks.in
COORDINATION COMPOUNDS 2.43
30. The [UPAC name of [Ni(NH3)4] [NiCl4] is 36. The number of water molccule(s) directly bonded
[JEE2008] to the metal centre in CuSO4. 5H2Ois is.
(a) Tetrachloronickel (Il)–tetraamminenickel (II) [JEE 2009]
(b) Tetraamminenickel (Il)–tetrachloronickel (II) 37. The ionization isomer of [Cr(H2O)4Cl(NO2)]Cl is–
(c) Tetraamminenickel (II)–tetrachloronickelate (II) (a) [Cr(H2O)4(O2N)]Cl2 [JEE 2010]
(d) Tetrachloronickel (II)–tetraamminenickelate (0)
(b) [Cr(H2O)4Cl2](NO2)
31. Both [Ni(CO)4] and [Ni(CN)4]2– are diamagnetic.
(c) [Cr(H2O)4Cl(ONO)]Cl
The hybridisations of nickel in these complexes,
respectively, are [JEE 2008] (d) [Cr(H2O)4Cl2(NO2)].H2O
3
(a) sp , sp 3 3
(b) sp , dsp 2 38. Total number of geometrical isomers for the
2
(c) dsp , sp 3
(d) dsp2, dsp2 complex [RhCl(CO)(PPh3)(NH3)] is. [JEE2010]
32. Statement–1 : The geometrical isomers of the 39. The correct structure of ethylenediaminetetraacetic
complex [M(NH3)4Cl2] are optically inactive. acid (EDTA) is – [JEE2010]
Statement–2 : Both geometrical isomers of the (a) HOOC–CH2 CH2–COOH
complex [M(NH3)4Cl2] possess axis of symmetry. N–CH=CH–N
HOOC–CH 2 CH2–COOH
(a) Statement–1 is True, Statement–2 is True ; (b) HOOC COOH
Statement–2 is a correct explanation for N–CH2–CH2–N
HOOC COOH
Statement–1
(c) HOOC–CH2 CH2–COOH
(b) Statement–1 is True, Statement–2 is True ; N–CH2–CH2–N
Statement–2 is NOT a correct explanation for HOOC–CH2 CH2–COOH
Statement–1 COOH

(c) Statement–1 is True, Staicment–2 is False CH2


(d) HOOC–CH2 H
(d) Statement–1 is False, Statement–2 is True N–CH–CH–N
H CH2–COOH
33. Statement–1 : [Fe(H2O)5NO]SO4 is paramagnetic.
CH2
Statement–2 : Fe in [Fe(H2O)5NO]SO4 has three
HOOC
unpaired electrons. [JEE 2008]
40. Geometrical shapes of the complexes formed by
(a) Statement–1 is True. Statcment–2 is True ;
the reaction of Ni 2+ with F– ,CN– and H2O
Statemcnt–2 is a correct explanation for
respectively– [JEE 2011]
Statement–1
(b) Statement–1 is True, Statement–2 is True ; (a) octahedral, tetrahedral and square planar
Statement–2 is NOT a correct explanation for (b) tetrahedral, square planar and octahedral
Statcmcnt–1 (c) square planar, tetrahedral and octahedral
(c) Statement–1 is True. Statemcnt–2 is False (d) octahedral, square planar and octahedral
(d) Statement–1 is False. Statcmcnt–2 is True 41. Among the following complexes (K–P)
34. The spin only magnetic moment value (in Bohr [JEE 2011]
magneton units) of Cr(CO)6 is [JEE2009]
K 3 [Fe(CN) 6 ] (K), [Co(NH 3 ) 6 ]Cl 3 (L),
(a) 0 (b) 2.84 Na3[Co(oxalate)3](M), [Ni(H2O)6]Cl2 (N),
(c) 4.90 (d) 5.92
K2[Pt(CN)4] (O) and [Zn(H2O)6] (NO3)2(P)
35. The compound(s) that exhibit(s) geometrical
The diamagnetic complex are –
isomerism is (are): [JEE 2009]
(a) [Pt(cn)Cl2] (b) [Pt(en)2]Cl2 (a) K, L, M, N (b) K, M, O, P
(c) [Pt(en)2Cl2]Cl2 (d) [Pt(NH3)2Cl2] (c) L, M, O, P (d) L, M, N, O

www.jeebooks.in
2.44 COORDINATION COMPOUNDS
42. The volume (in mL) of 0.1 M AgNO3 required for 49. Match each coordination compound in List–I with
complete precipitation of chloride ions present in an appropriate pair and select the correct answer
30 mL of 0.01 M solution of [Cr(H2O)5Cl]Cl2, as using the code given below the lists,
silver chloride is close to. [JEE 2011] {en = H2NCH2CH2NH2 ' atomic numbers ; Ti =
43. As per IUPAC nomenclature, the name of the 22 ; Cr = 24 ; Co = 27 ; Pt = 78}
complex [Co(H2O)4(NH3)2]Cl3 is : [JEE 2011] List–I
(a) Tetraaquadiaminccobalt(III)chloride (P) [CrCNH3)4Cl2]Cl
(b) Tetraaquadiaminccobalt(III)chloride (Q) [Ti(H2O)5Cl](NO3)2
(c) Diaminetetraaquaccobalt(III)chloride (R) [Pt(en)(NH3)Cl]NO3
(d) Diamminetetraaquaccobalt(III)chloride (S) [Co(NH3)4(NO3)2]NO3
44. The colour of light absorbed by an aqueous solution List–II
of CuSO4 is [JEE 2012] (1) Paramagnetic and exhibits ionisation isomerism
(a) orange-red (b) blue-green (2) Diamagnetic and exhibits cis–trans isomerism
(c) yellow (d) violet (3) Paramagnetic and exhibits cis–trans isomerism
45. NiCl2 {P(C2H5)2(C6H5)}2 exhibits temperature (4) Diamagnetic and exhibits ionisation isomerism
dependent magnetic behaviour (paramagnetic / Code :
diamagnetic). The coordination geometries of Ni2+
P Q R S
in the paramagnetic and diamagnetic states are
respectively : [JEE 2012] (a) 4 2 3 1
(a) tetrahedral and tetrahedral (b) 3 1 4 2
(b) square planar and square planar (c) 2 1 3 4
(c) tetrahedral and square planar (d) 1 3 4 2
(d) square planar and tetrahedral 50. A list of speCles having the formula XZ4 is given
46. Consider the following complex ions P, Q and R , below : [JEEAdv.2014]

P = [FeF6]3–, Q = [V(H2O)6]2+ XeF4, SF4, SiF4, BF4~, BrF4~, [Cu(NH3)4]2+,


[FeCl4]2–, [CoCl4]2– and [PtCl4]2–.
and R = [Fe(H2O)6]2+
Defining shape on the basis of the location of X
The correct order of the complex ions, according
and Z atoms, the total number of species having a
to their spin–only magnetic moment values (in B.M) square planar shape is–
[JEE 2013]
Subjective :
(a) R < Q < P
51. Give reasons in two or three sentences only for
(b) Q < R < P the following :
(c) R < P < Q "The species [CuCl4]2– exists, while [CuI4]2~ does
(d) Q < P < R not exist" [JEE 1992]
48. The pair(s) of coordination complex/ion exhibiting 52. Write the IUPAC name of the following
the same kind of isomerism [JEE2013] compounds [JEE1995]
(a) [Cr(NH3)5Cl]Cl2 and [Cr(NH3)4Cl2]Cl (i) [Co(NH3)5ONO]Cl2
+ +
(b) [Co(NH3)4Cl2] and [Pt(NH3)2(H2O)Cl] (ii) K3[Cr(CN)6]
2– 2–
(c) [CoBr2Cl2] and[PtBr2Cl2] 53. Write the IUPAC name for [Cr(NH3)5CO3]Cl
(d) [Pt(NH3)3(NO3)] Cl and [Pt(NH3)3Cl] Br [JEE 1996]

www.jeebooks.in
COORDINATION COMPOUNDS 2.45
54. Write the formulae of the following complexes : (a) Draw its structure and show H–bonding
[JEE 1997] (b) Give oxidation state of Ni and its hybridisation
(i) Pentamminechlorocobalt (III) ion (c) Predict whether it is paramagnetic or
(ii) Lithium tetrahydridoaluminate (III) diamagnetic
55. A, B and C are three complexes of chromium (III) 61. For the octahedral complexes of Fe3+ in SCN–
with the empirical formula Hl2O6Cl3Cr. All the three (thiocyanato-S) and in CN– ligand environments,
complexes have water and chloride ion as ligands. the difference between the spin only magnetic
Complex A does not react with concentrated moment in Bohr magnetons (when approximated
H2SO4, whereas complexes B and C loose 6.75% to the nearest integer) is :
and 13.5% of their original mass, respectively, on [Atomic number of Fe = 26] [JEEAdv.2015]
treatment with concentrated H2SO4. Identify A, B 62. In the complex acetylbromidodicarbonylbis
and C. [JEE1989] (triethylphosphine)iron(II), the number of Fe–C
56. Draw the structures of [Co(NH3)6] , [Ni(CN)4]2–
3+
bond(s) is :- [JEEAdv.2015]
and [Ni(CO)4]. Write the hybridisation of atomic 63. Among the complex ions, [Co(NH2–CH2–CH2–
orbitals of the transition metal in each case. NH2)2Cl2]+, [CrCl2(C2O4)2]3– , [Fe(H2O)4(OH)2]+,
[JEH 2000] [Fe(NH 3 ) 2 (CN) 4 ] – , [Co(NH 2 –CH 2 –CH 2
57. A metal complex having composition Cr(NH3)4 –NH2)2(NH3)Cl]2+ and [Co(NH3)4(H2O)Cl]2+, the
Cl2Br has been isolated in two forms A and B. number of complex ion(s) that show(s) cis-trans
The form A reacts with AgNO3 to give a white isomerism is :- [JEEAdv.2016]
precipitate readily soluble in dilute aqueous 64. Among [Ni(CO)4], [NiCl4]2– , [Co(NH3)4Cl2]Cl,
ammonia, whereas B gives a pale yellow precipitate Na3[CoF6], Na2O2 and CsO2 , the total number of
soluble in concentrated ammonia. Write the formula paramagnetic compounds is :- [JEEAdv.2016]
of A and B and state the hybridisation of chromium
(a) 2 (b) 3
in each. Calculate their magnetic moments (spin–
only value). [JEE 2001] (c) 4 (d) 5
58. Deduce the structures of [NiCl4] and [Ni(CN)4]2~
2– 65. The number of geometric isomers possible for
considering the hybridisation of the metal ion. complex [CoL2Cl2]– (L = H2NCH2CH2O–) is :-
Calculate the magnetic moment (spin only) of the [JEEAdv.2016]
species. [JEE 2002] 66. The geometries of the ammonia complexes of Ni+2,
59. Write the IUPAC name of the compound Pt2+ and Zn2+ respectively are :-[JEEAdv.2016]
K2[Cr(NO)(CN)4(NH3)]. Spin magnetic moment (a) octahedral, square planar and tetrahedral
of the complex µ = 1.73 BM. Give the structure of (b) square planar, octahedral and tetrahedral
anion. [JEE 2003]
(c) tetrahedral, square planar and octahedral
60. NiCl2 in the presence of dimethyl glyoxime (DMG)
(d) octahedral, tetrahedral and square planar
gives a complex which precipitates in the presence
of NH4OH. giving a bright red colour.
[JEE 2004] 

www.jeebooks.in
METALLURGY
PROB L EM S B ASED ON GI VEN T OPI CS  Preparation of allys
 Types of ores  Amalgam
 Principal steps in the recovery of a metal form its  Different types of furnances used in metallurgy
ore
 Extraction of silver
 Concentration or dressing of ore
 Refining of Ag
 Gravity separation or levigation
 Extraction of Gold by Cyanide process
 Magnetis separation
 RefiningAu
 Froth floatation of oil floatation  Extraction of tin
 Chemical method of separation leaching  Refining of Sn
 Conversion of concentrated ore into its oxide  Extraction of Magnesium
 Calcination  Electrolytic reduction
 Roasting  Carbon reduction process
 Different reduction processes  Other processes
 Carbon reduction  Extraction of Aluminium
 Self reduction  Beneficiation of bauxide
 Thermite reduction (or Goldschmidt-Thermite  Electrolytic reduction of pure Al2O3
process)  Electrorefining of aluminium
 Metal replacement method (Hydrometallurgy)  Extraction of lead
 Electrolytic reduction  Carbon reduction
 Thermal decomposition method  Self reduction process

 Purification or refining of metal


 Refining of lead
 Extraction of copper
 Thermal refining
 Refining of blister copper
 Electrorefining
 Extraction of zinc
 Thermodynamics of rection process
 Extraction of iron
 Alloy and amalgams
 Purification of iron or preparation of wrought
 Classification of alloys iron
 Characteristic of allys  Steel making

www.jeebooks.in
CHAPTER
3
Metallurgy

EXERCISE # I
 Only one correct answer :
1. Which of the following is not sulfide ore ? 8. During leaching of alumina from Bauxite the leaching
(a) Zinc blende (b) Galena reagent used would be :
(c) Cinnabar (d) Baryte (a) Concentrated solution of Ba(OH)2
2. Which of the following is carbonate ore ? (b) Concentrated solution of NaOH or 40% KOH
(a) Siderite (b) Magnesite (c) Concentrated HCl
(c) Dolomite (d) All (d) Concentrated H2SO4
3. The salt which is least abundant as mineral is ? 9. The leaching of alumina from Bauxite is done by :
(a) Halide (a) 473-523 K & 35-36 bar pressure
(b) Nitrate (b) 300 K & 5-6 bar pressure
(c) Carbonate (c) 1000 K & 1 bar pressure
(d) sulfide (d) None
4. Which ore does not have water of crystallisation ? 10. In the metallurgy of silver and gold, the metal is
leached with the reagent :
(a) Diaspore (b) Bauxite
(a) NaOH (b) NaNH2
(c) Gibbsite (d) Pyrolusite
(c) NaCN (d) Zn metal
5. Which of the following is not halide ore ?
11. Which of the following ore is best concentrated by
(a) Cryolite
Froth floatation method ?
(b) Fluorospar
(a) Galena
(c) Horn silver
(b) Casseterite
(d) Limonite
(c) Magnetite
6. Sulphides ore is common in between :
(d) Malachite green
(a) Ag, Cu, Pb (b) Ag, Cu, Sn
12. Gravity separation is primarily used for separation
(c) Ag, Mg, Pb (d) Al, Cu, Pb of impurities from which ore
7. BO 1 O (a) Cassiterite (SnO2)
400ºC 2 2 + B
(b) Haematite (Fe2O3)
ACl2 + BCl2 ACl4 + B.
(c) Chromite (FeCr2O4)
If ACl2 is the reducing agent. B is the metal, then
(d) All
ore of A & B would be :
13. The metal that can’t be obtained by electrolysis of
(a) Siderite, Cinnabar
an aqueous solution of its salt is :
(b) Casseterite, Cinnabar
(a) Cu (b) Cr
(c) Hornsilver, Tinstone
(c) Ag (d) Ca
(d) None

www.jeebooks.in
3.4 METALLURGY
14. Which metallurgical process describe best the 23. During self reduction of Cu2S in the Ist step when
extraction of Mn from Mn3O4 by the use of O2 is added then.
aluminium. (a) Oxidation state of Cu+1 converted into Cu2+
(a) Hydrometallurgy (b) Oxidation state of S2– converted ento SO2
(b) Electrometallurgy (c) Oxidation state of Cu+1 & S2– both undergoes
(c) Pyrometallurgy change
(d) Amalgamation (d) Oxidation state of Cu+1 & S2– both remains
15. The main impurity present in red bauxite is : unchanged
(a) CuO (b) ZnO 24. Slag formed by reaction between impurities and
(c) Fe2O3 (d) SiO2 flux. If flux is basic then impuriteis must be
16. The main impurities present in white bauxite is : (a) Acidic in nature eg. SiO2
(a) Fe2O3 (b) TiO2 (b) Acidic in nature eg. CaO
(c) SiO2 (d) FeO (c) Basic in nature eg. MgO
17. Heating pyrites in air for oxidation of sulphur is (d) Basic in nature eg. P4O10
called : 25. Carbon reduction method is not applicable for
(a) Smelting (b) Roasting extraction of
(c) Calcination (d) Slagging (a) Tin from SnO2 (b) Iron from Fe2O3
18. The process of converting hydrated ore into (c) Al from Al2O3 (d) Pb from PbO
anhydrous ore is called : 26. In which reaction aluminium acts as reducing agent:
(a) Roasting (b) Smelting (a) Baeyer’s process
(c) Dressing (d) Calcination (b) Serpeck’s process
19. Copper matte consists (c) Thermite process
(a) Mixture of Cu2S + FeS (d) Hall’s process
(b) Mixture of Cu2S + FeO 27. Thermite reduction is not used for commerical
(c) Metallic copper with 98-99 % purity extraction of respective metal from which of the
following oxide :
(d) Metallic copper with 1-2 % purity
(a) Mn3O4 (b) TiO2
20. Zincite is example of
(c) Fe2O3 (d) Cr2O3
(a) Oxide ore (b) Halide ore
28. Spiegel (or spiegeleisen), used in the manufacture
(c) sulphide ore (d) Corbonate ore
of steel by the Bessemer process, is an alloy of :
21. Which method is based on the principle that
(a) Iron , chromium and carbon
impurties are more soluble in molten metal than in
solid state. (b) Iron , nickel and carbon
(a) Vapour phase refining (c) Iron , tungsten and carbon
(b) Zone refining (d) Iron, Manganese and carbon
(c) Poling 29. Which of the following statement is incorrect
regarding thermite welding.
(d) Levigation
(a) Al is used as reducing agent
22. Auto reduction is applicable for which of metals ?
(b) Fe is obtained in the form of molten metal
(a) Al, Fe (b) Cu, Sn
(c) This process is used for rail rod joining
(c) Cu, Hg (d) Ag, Au
(d) Fe2O3 + Al taken are in the ratio (5 : 4)

www.jeebooks.in
METALLURGY 3.5

ZrI4 ZrI4 38. Out of following oxides, unstable oxide at


30. Zr + 2I2 Zr + I2
temperature 200ºC is
The name of this method is :
(a) CaO (b) Al2O3
(a) Van Arkel method (b) Zone refinig
(c) HgO (d) Fe2O3
(c) Poling (d) Cupellation
39. Ti is prepared by the following method .
31. In Mond’s process during extraction of nickel, the
Heating
reagent added which gives complex is X & the Impure metal + X2 TiX4 Ti + 2X2  ; X
at high temp. 2
complex formed if is Y, then X & Y are respectively: Ti
(a) CN–, Ni(CN)42– (b) CO, Ni(CO)4 is :
(c) CN–, [Ni(CN)6]4– (d) CO, Ni(CO)5 (a) Cl2 (b) Fe
32. Froth floatation is based on the following principle: (c) Br2 (d) I2
(a) Differences in wetting ability of ore particles in 40. Which alkaline earth metals are present in
oil and impurity particles in water. Dolomite.
(b) Difference in specific gravity of ore particles (a) Na, Mg (b) K, Ca
and impurity particles in water. (c) Be, Ba (d) Ca, Mg
(c) Difference in magnetic property of ore particles Roasting
and impurity particles in water. 41. Zinc blende Metal oxide + Gas.
(Suffocating odour)
(d) All
What is the oxidation state of S in zinc blende and
33. Zinc blende and chalcopyrites are concentrated
gas ?
by :
(a) –2, +4 (b) –2, +6
(a) Leaching
(c) 0, +6 (d) 0, +4
(b) Levigation
42. Ellingham diagram normally describes
(c) Froth floatation
(d) Magnetic separation (a) Variation of Gº vs temperature for fromation
of oxide 2xM(s) + O2(g) 2MxO(s)
34. The oxidation state of metal in cuprite and fool’s
gold are respectively : (b) Variation of Hº vs temperature for formation
(a) +1 , +1 (b) +2, +2 and oxide 2xM(s) + O2 (g) 2MxO(s)
(c) +1, +2 (d) +2 , +1 (c) Variation of Sº vs temperature for formation
of oxide 2xM(s) + O2 (g) 2MxO(s)
35. Oxidation state of Fe in Haematite and Magnetite
respectively : (d) None
(a) II ; III (b) II ; (II + III) 43. In Ellingham diagram there is sudden increase in
slope at particular point for every metals which
(c) III ; III (d) III ; (II + III)
depicts
36. Observe the following partial roasting process,
(a) Phase change e.g., solid Liq. ; Liq gas
CuFeS2 + O2  A + FeO + SO2 ; A is :-
(b) The metals oxide decomposition temperature
(a) Cu2O (b) CuO
(c) Metal oxide boiling point
(c) Cu2S (d) CuS
(d) Metal oxide fusion point
37. Steps which are involved in the extraction of Cu
from its low grade ore. 44. Which reaction is most spontaneous ?
(a) Leaching by NaCN (a) Zn2+ + 2e Zn
(b) Metal displacement by Ag (b) Ag+ + e– Ag
2+
(c) Leaching with acid (c) Mg Mg + 2e–
(d) Metal displacement by iron (d) Li+1 + e Li

www.jeebooks.in
3.6 METALLURGY
45. The oxide of which metal having least thermal 52. In cupellation process :
stability. (a) Lead impurities are separated from Ag
(a) Hg (b) Fe (b) Silver impurities are separated from Pb
(c) Al (d) Mg (c) Cu impurities are separated from Sn
46. For which reaction S = 0. (d) Sn impurities are separated from Cu
(a) C(s) + O2(g) CO2(g) 53. Boiling point order :
1 (a) Cd > Zn > Fe (b) Fe > Zn > Cd
(b) Ca(s) + O (g) CaO(s)
2 2 (c) Zn > Fe > Cd (d) Zn > Cd > Fe
3 54. In liquation metal can be separated from impurities.
(c) 2Al(s) + O2(g) Al2O3
2 Here metal and impurities are of
1 (a) Higher melting point and lower metling point
(d) C(s) + O2 CO(g)
2 respectively
47. From Ellingham diagram in Blast furnace at (b) Higher boiling point and lower boiling point
673 K, best reducing agent is : respectively
(a) Coke
(c) Lower metling point and higher melting point
(b) CO respectively
(c) Mixture of (C+CO) (d) Lower boiling point and higher boiling point
(d) Can’t be predicted respectively
48. Sphalerite ore is used for extraction of which 55. In zone refining impurities get concentrated
metal?
(a) In solidified zone (b) Molten zone
(a) Fe (b) Cu
(c) Freezing zone (d) None
(c) Zn (d) Al
56. The chemical process in the production of steel
49. Distillation is a method of refining of metals. For from haematite ore involve :-
which metal it is applicable.
(a) reduction
(a) Low boiling point metals e.g., Pb, Sn
(b) oxidation
(b) High boiling poing metal e.g., Zn, Hg
(c) reduction followed by oxidation
(c) Low boiling point metal e.g., Zn, Hg
(d) oxidation followed by reduction
(d) High boiling point metal e.g., Pb, Sn
50. In the metallurgy of aluminium the electrolyte is : 57. The temperature T1 and T2 for Mond process
T1
(a) Fused matrix of Al2O3 with Na3AlF6 and CaF2 Ni + 4CO Ni(CO)4 ;
(b) Al2O3 only
T2
(c) Na3AlF6 Ni(CO)4 Ni + 4CO
(d) CaF2 (a) 330 - 350 K ; 600 - 630 K
51. Impurity has more tendency to be oxidised by (b) 330 - 350 K ; 450 - 470 K
atmospheric O2 than ore particles. This concept is (c) 450 - 470 K ; 600 - 630 K
applicable for : (d) 600 - 630 K ; 330 - 350 K
(a) Poling 58. In which of the following process roasting followed
(b) Cupellation process by self reduction takes place :
(c) Zone refining (a) Cu2S Cu (b) PbS PbO
(d) Electrolytic refining (c) CaCO3 Ca (d) None

www.jeebooks.in
METALLURGY 3.7
59. The pyrometallurgy requires 66. White bauxite is purified by.
(a) Electric current (b) Low temperature (a) Baeyer’s process (b) Hoop’s process
(c) High temperature (d) Atmospheric O2 (c) Serpeck’s process (d) Hall’s process
60. Which is correctly matched ? 67. Red bauxite is purified by.
(a) Anthracite – Ore of iron (a) Baeyer’s process (b) Hall’s process
(b) Fluorospor – Mineral of aluminium (c) Both A and B (d) Hoop’s process
(c) Cementite – A carbide of iron in steel 68. Aluminium is purified by.
(d) Pyrolusite – An ore of Mg (a) Baeyer’s process (b) Hall’s process
61. At 1100ºC which of the following reaction is (c) Serpeck’s process (d) Hoop’s process
thermodynamically most feasible. 69. The correct position of metal (from top to bottom)
(a) TiO2 + C Ti + CO2 is electrochemical series.
(b) CO2 + C 2CO (a) Li > K > Mg > Al
(c) Al2O3 + 3C 2Al + 3CO (b) Na > Mg > K > Li
(d) Cr2O3 + 3C 2Cr + 3CO (c) Li > K > Ca > Na
62. In Parke’s process, 2 layers are developed. (d) Mn > Mg>Al >Ka
Correct options is :- 70. In electrolytic reduction of Al2O3, the ratio of
(a) Upper layer consists of Pb Al2O3, Cryolite and fluorspar will be
(b) Upper layer consists of alloy of Zn-Ag (a) 1 : 1 :1 (b) 3 : 1 : 2
(c) Upper layer consists of Zn (c) 1 : 5 : 1 (d) 1 : 3 : 1
0(d)Upper layer consists of Ag-Pb-alloy 71. In Golds mith process Fe2O3 and Al mixture are in
63. Lower layer consists of the ratio
(a) Molten zinc (a) 1 : 2 (b) 5 : 1
(b) Molten silver (c) 3 : 2 (d) 3 : 1
(c) Molten lead 72. Ignition mixture which is used for thermite process
consists of
(d) Molten mixture of (Zn + Ag)
(a) KCl + BaO (b) KClO3 + BaO
64. In Baeyer’s process Fe2O3 does not react with
NaOH, but SiO2 does along with Al2O3 becuase. (c) KClO3 + BaO2 (d) KCl + BaO2
(a) Because Fe2O3 is basic oxide whereas SiO2 is 73. At approximate what temperature, zinc and carbon
acidic oxides and Al2O3 is amphoteric oxide have equal affinity for oxygen
(b) Al2O3 and SiO2 are basic oxides whereas (a) 500°C (b) 1000°C
Fe2O3 is acidic oxide (c) 1200°C (d) 1500°C
(c) Fe2O3 is basic oxides whereas SiO2 and Al2O3 74. In Castner’s process for extraction of sodium the
are example of acidic oxide elctrolyte is
(d) Al2O3 , Fe2O3 are example of basic oxide (A fused NaOH (b) NaOH + CaCl2
whereas SiO2 is example of acidic oxide (c) fused NaCl (d) fused NaCl +CaCl2
65. In froth floatation method sodium ethyl xanthate is 75. In Down’s process for extraction of sodium the
used as collector. Actually it elctrolyte is
(a) Helps sulfide ore to collect in water medium (a) fused NaOH
(b) Helps sulfide ore to collect on air bubble (b) NaOH + CaCl2
(c) Dissolves impurities in water (c) fused NaCl
(d) Dissolves impurities in air (d) fused NaCl +CaCl2

www.jeebooks.in
3.8 METALLURGY
76. Which of the following options are incorrect ? 82. Which of the following option are correct?
(A Si can’t reduce MgO at all. 1
(b) Al can’t reduce ZrO2 but Mg can reduce ZrO2 (a) Ca + O  CaO line sharply increases at
2 2
(c) Al can reduce Cr2O3 as well as TiO2 temperature at 1440°C
(d) H2 does not act as strong reducing agent at
1
high temperature (b) Mg + O  MgO line sharply increases at
2 2
77. Sodium cloride is added during electrolysis of fused
anhydrous magnesium chloride temperature at 1440°C
(a) To increase melting point of MgCl2 from 500°C 1
to 800°C (c) Hg + O  HgO line sharply increases at
2 2
(b) To reduced melting point of MgCl2 from 700°C temperature at 1440°C
to 400°C
(D All
(c) To increase melting point of MgCl2 from 600°C
83. Carnalite on electrolysis gives
to 900°C
(a) Ca and Cl2 (b) Al and Cl2
(d) To reduced melting point of MgCl2 from 900°C
to 600°C (c) Na & CO2 (d) Mg and Cl2
78. The point at which line of M + O2  MO cuts the 84. In which of the following extraction, no reducing
zero free energy line depicts agent is used
(a) decomposition temperature of oxide (a) Iron from Haematite
(b) decomposition temperature of metal (b) Tin from Casseterite
(c) fusion point of metal (c) Mercury from Cinnabar
(d) fusion point of slag (d) Zinc from zincblende
79. The elements which are at lowest line in Elingham 85. Railway wagon axles are made by heating iron rods
diagram has embeded in charcoal powder. This process is
(a) very low tendency to combine with O2 known as -
(b) very high tendency to combine with O2 (a) Sherardising (b) Anealing
(c) Oxide which are very unstable (c) Tempering (d) Case hardening
(d) None 86. Which of the following options is incorrect -
80. In Polling, basic impurities associated with metal (a) Reduction potential of alkali metal cation is
(a) gets reduced to form volatile elemental form much lower than that of H+
(b) gets oxidised to form slag which is immescible (b) Magnesia and quick lime are used as acidic
with molten metal flux
(c) gets oxidised to form volatile oxide which is (c) TiI4 is volatile stable compound
removed as scum (d) In Park’s process, silver impurity is removed
(d) gets reduced to form solidified element which from lead metal
is crystallised out of molten metal 87. At 1100°C which of the following reactions is
81. In Distillation process, zinc is seperated from Cd thermodynamically most favoured
at temperature T1 and zinc is separated from Pb
(a) TiO2 + C Ti + CO2
and Fe at temperaure T2 . T1 and T2 are respectively
(b) Al2O3 +3C 2Al + 3CO
(a) 767°C and 920°C (b) 667°C and 940°C
(c) Cr2O3 + 3C 2Cr + 3CO
(c) 657°C and 800°C (d) 957°C and 750°C
(d) CO2 + C 2CO

www.jeebooks.in
METALLURGY 3.9
88. In the extraction of copper from its sulphide ore, 96. Iron obtained from blast furnance is
the metal is formed by reduction of Cu2O with
(a) wrought iron (b) cast iron
(a) FeS (b) CO
(c) pig iron (d) steel
(c) Cu2S (d) SO2
97. High temperature (>1000°C) electrolytic reduction
89. Refractory materials are generally used in furnance
because - is necessary for extraction of
(a) they are chemically inert (a) Mg (b) Cu
(b) they can withstand high temperature (c) Pb (d) Al
(c) they do not contain impurities 98. From the Ellingham graph on carbon, which of the
(d) they decrease melting point of ore following statement is false?
90. Addition of high proportions of manganese makes (a) CO2 is more stable than CO at less than
still useful in making rails of rail roads, because 983 K
manganese - (b) CO reduces Fe2O3 to Fe at less than 983 K
(a) gives hardness to steel (c) CO is less stable than CO2 at more than
(b) help the formation of oxide of iron 983 K
(c) can remove oxygen and sulphur (d) CO redues Fe2O3 to Fe in the reduction zone
(d) can show highest oxidation state of +7 of blast furnace
91. Silica is added to roasted copper ores during 99. Coke powder is spreaded over the molten
extraction copper to remove - electrolyte in electrolytic reduction of Al2O3 due
(a) Cuprous sulphide (b) Cuprous oxide to :
(c) Ferrous oxide (d) Ferrous sulphide (a) prevent the heat radiation from the surface
92. Blister copper is refined by stirring molten impure (b) prevent the corrosion of graphite anode
metal with green logs to wood because such wood (c) prevent oxidation of molten aluminium by air
liberates hydrocarbon gases. This proces is called
and the metal contain impurities of - (d) both (a) & (b)
(a) Cupellation, CuO (b) Polling, CuO 100. Which of the following reaction cannot occur in
blast furnace during extraction of iron.
(c) Cupellation, Cu2O (d) Polling, Cu2O
93. The process which does not use a catalyst is - (a) CaO + SiO2  CaSiO3
(a) Contact process (b) Thermite process (b) Fe2O3 + 3CO  2Fe + 3CO2
(c) Ostwald’s process (d) Haber’s process 1
(c) FeO  Fe + O
94. Spelter may be purified by - 2 2
(a) Polling 1
(d) P2O5 + 5C  P + 5CO
(b) Heating with iodine 2 4
(c) Electrolysis process EXERCISE # II
(d) Fractional distillation  One or More Than One Correct Answer :
95. G° vs T plot in Ellingham diagram slopes 1. In which ore iron (Fe) can have (+2) oxidation
downward for which of the following reactions state :
(a) 2Mg + O2 2MgO (a) Haematite (Fe2O3)
(b) 2CO + O2 2CO2 (b) Iron pyrite (Fool’s gold) FeS2
(c) 2Zn + O2 2ZnO (c) Siderite (FeCO3)
(d) 2C + O2 2CO (d) Copper pyrite (CuFeS2)

www.jeebooks.in
3.10 METALLURGY
2. Which of the following ores having potassium ? 10. For aluminium extraction when electrolysis of
(a) Carnalite (b) Indian salt Petre molten mass Al2O3 is carried out then which
statement are correct?
(c) Gypsum (d) Cryolite
(a) Lining of carbon acts as cathode and graphite
3. Which of the following ores have copper ?
anode is used
(a) Chalcopyrite (b) Azurite (b) During electrolysis, the oxygen liberated reacts
(c) Malachite green (d) Limonite with carbon of anode to give CO and CO2
4. Which of the following changes is obsered when (c) This process is called Hall-Heroult process
ore is heated for calcination ? (d) CaF2 reduces the melting point of the mixture
(a) Fe2O3.3H2O Fe2O3 + 3H2O and increases electrical conductivity
(b) Al2O3.2H2O Al2O3 + 2H2O 11. Mercury can’t form amalgam with :
(c) PbCO3 PbO + CO2 (a) Fe (b) Ag
(d) ZnCO3 ZnO + CO2 (c) Au (d) Pt
5. Magnetic substance which are present in casseterite 12. The method used in metallurgy to refine the impure
for Sn extraction. metal is :
(a) FeWO4 (b) SnO2 (a) Chromatographic separation
(c) MnWO4 (d) Fe3O4 (b) Vapour phase refining
6. Which of the two metal form cyano complex during (c) Zone refining
its extraction? (d) Distillation
(a) Zn (b) Ag 13. Tin can be refined by
(c) Au (d) Cu (a) Poling
7. Electrolytic reduction is applicable for (b) Liquation
(a) Al from Al2O3(Molten) (c) Distillation
(b) Ag from Ag2S (d) Zone refining
(c) Na from NaOH 14. 4Au + 8CN– + 2H2O + O2 (g)
(d) Cu from CuFeS2 4[Au(CN)2]–(aq) + 4OH–(aq)
8. In zone refining method some elements are formed 2[Au(CN)2]–(aq) + Zn
in ultrapure form. The elements can be : 2Au(s) + [Zn(CN)4]2–(aq)
(a) Si (b) Ge Which of the following options are correct ?
(c) Ga (d) Fe (a) Both the reactions are the example of nonredox
9. Which of the following options are correct ? reaction
(a) Carbon cannot be used to produce magnesium (b) Both are examples of redox reactions
by chemical reduction of MgO because (c) In Ist reaction O2 acts as oxidising agent
magnesiumreacts with carbon to form carbide (d) In IInd reaction Zn acts as reducing agent
(b) In froath floatation method activator can 15. CO can’t be used as reducing agent
enhance the frothering tendency of sulphide ore (a) To prepare/extract Zn metal from ZnO
(c) CO is preferential reducing agent in iron (b) To Prepare Al metal from Al2O3
extraction from haematite ore (c) To prepare Fe metal from Fe2O3
(d) Na, K, Li are situated at the top of (d) To prepare Pb metal from PbS
electrochemical series.

www.jeebooks.in
METALLURGY 3.11
16. In Ellingham diagram (c) In blast furnace carbon does not reduce iron
(a) The elements in the higher line can reduce oxide oxides
of elements in the lower line (d) Iron scraps will be advisable and advantageous
(b) The elements in the lower line can reduce oxide w.r.t Zn scraps for reducing leached copper ore
of elemetns in higher line 22. Find the number of ores which can be concentrated
(c) Reduction of metals oxide is easier if the metals by magnetic separation method.
formed is in liqid state at the temperature of (a) Galena (b) Copper pyrites
reduction (c) Haematite (d) Siderite
(d) Reduction of metals oxide can take place of 23. Which of the following are the steps / reactions
room temperture. involved in extraction of iron :-
17. In Ellingham diagram Fe2O3 can be redcued by (a) 3FeO3 + CO 2FeO3 + CO2
which of following elements. (b) Fe3O4 + CO 3FeO3 + CO2
(a) Ca (b) Mg (c) FeO + CO Fe + CO2
(c) Hg (d) Al (d) Calcium silicate is produced as slag
18. Pick the correct statement(s) for aluminothermite 24. Which of the following statements are correct :-
process :-
(a) Anhydros MgCl2 can not be prepared by
(a) Reaction in endothermic heating MgCl2.6H2O
(b) It is used for welding of rail tracks (b) Anhydros CaCl2 is used in drying gases and
(c) To start the reaction a Mg ribbon is ignited organic compounds but not NH3 or ethyl
(d) Al is used as reductant alcohol due to formation of CaCl2.8NH3 and
19. Which of the following statement is / are correct :- CaCl2 .4C2H5OH
(a) Combination Lime stone and wolframite is non (c) Leblanc process is used for preparation of both
magnetic in nature Na2CO3 and H2CO3 but not solvay process.
(b) No external reducing agent is required for (d) KOH is preferably used compared to NaOH
extraction of Hg from HgO. for absorption to NaOH for absorption of CO2
(c) Poling method is mainly used when impure metal because KHCO3 formed is soluble where as
oxide having oxide as impurity NaHCO3 is sparingly soluble and therefore
choke the tubes of apparants used.
(d) For Cu extraction in bessemer converter
process, oxidation and reduction reaction takes 25. Which of the following reactions are example of
place. thermite reduction ?
20. Which of the following statements are not correct? (a) A thermite reaction may start, if a ship (having
aluminium parts) is hit by a missile
(a) Copper is extracted by self reduction method
(b) Fe2O3 + 2Al  2Fe + Al2O3
(b) Cast iron is the purest form of iron
Δ
(c) The composition of malachite ore is Ca(OH)2 . (c) Al2O3 + 3Mg  3MgO + 2Al
CaCO3 Δ
(d) Cupellation process is used for refining of Ag (d) Cr2O3 + 2Al  2Cr + Al2O3
and Au 26. Blister copper is :-
21. Select the statement which is correct. (a) Impure copper
(a) Elingham diagram M (b) Obtained by self reduction process in Bessemer
MO curves have ngative slope. converter
(b) MgO can be reduced by carbon at very high (c) pure 100% Cu
temperature (~800°C) (d) 50% Copper

www.jeebooks.in
3.12 METALLURGY
27. Select correct option :- 30. The compound with different metals as
(a) Al2O3 + 2Fe  Fe2O3 + 2Al H = +ve (a) Dolomite (b) Carnalite
Cu CO (c) Copper pyrite (d) Azurite
(b) FeO Fe3O4
reduction reduction 31. Choose incorrect statement.
CO
(a) Mg is liquid at less than 1500ºC and gas at
Fe2O3 2Fe above 1500ºC
reduction
(c) Out of two metal oxide FeO and CaO ; CaO (b) Si can’t reduce MgO at all
more easily combine with SiO2 (c) The equation of line in Ellingham diagram is given
(d) Impurities present in molten cast iron are by G = H – T S
possibly oxidised to Fe2O3 (d) The preferable temperature for Mg to be used
28. Which of the following statements are incorrect ? as a reducing agent for SiO2 is 2200ºC
(a) During metallurgy of zinc, spelter is obtained 32. When copper is purified by electrolytic refining of
by reduction process of ZnO the blister copper, the correct statement :
(b) Pig iron is obtained from blast furnance. (a) Pure Cu is deposited at cathode
(c) Liquation is only valid for purification of Bi not (b) Impurities settle as anode mud.
for Sn and Pb. (c) Acidified CuSO4 is used as electrolyte
(d) Pitch blende is ore of U (d) Pure Cu is deposited at anode
29. Which of the following is an example of amphoteric 33. Which of the following metals can be extraced by
oxide ? electrolytic reuction
(a) Al2O3 (b) PbO (a) Na (b) Mg
(c) ZnO (d) FeO (c) Ca (d) Ag

34. Which of the following diagram are not related to the extraction of Mg from sea water ?
Na2CO3 Calcination Carbon
(a Sea water MgCO3 MgO Reduction Mg )

lime HCl(aq.) 
(b) Sea water water
Mg(OH)2 MgCl2.6H2O With excess
of dry
HCl(g)

Electrolysis MgCl2
Mg in molted condition anhyd.

drying
(c) Sea water Sea salt Electrolysis Mg

Ca(OH)2 HCl(aq.) MgCl2 Electrolysis


(d) Sea water Mg(OH)2
solution in aq. solution Mg

35. “In electrolysis Mn+ (aq.), metal ions are discharged at negative electrodes (cathodes) and deposited there”
above method of extraction is not possible for :
(a) Sodium (b) Aluminium (c) Silver (d) Carbon

www.jeebooks.in
METALLURGY 3.13
36. The reduction processes employed to prepare lead 43. Which of the following statements are correct :-
metal can be (a) 24 carat Gold consists of 100 % Au
(a) Carbon reduction (b) 14 carat Gold consists of 54 % Au
(b) self reduction
(c) Stainless steel have Cr : 12-14%
(c) Aluminothermic reduction
(d) Amalgamation is carried out in Iron container
(d) Electrolytic reduction
44. Which of the following statements are correct :-
37. The reduction process employe to prepare iron
metal can be (a) The metallic impurities having higher oxidation
(a) Carbon reduction potential than that of metal to be refined are
separated in the form of anode mud.
(b) self reduction
(c) Aluminothermic reduction (b) Cathode is made of large slab of impure metal
in electrolytic refining
(d) Electrolytic reduction
(c) Anode is made of thin strip of pure metal in
38. The metallurgy/reduction process involved for
extraction for Cu can be electrolytic refining
(a) Electrometallurgy (d) Spinels are mineral of Al with molecular formula
(b) Pyrometallurgy with carbon reduction MgO.Al2O3
(c) Pyrometallurgy with Auto reduction 44. Which of the following can not be obtained by
(d) Hydrometallurgy electrolytic reduction of there compounds in
aqueous solution.
39. In Elingham Diagram, Fe2O3 can be reduced by
which of the following elements (a) Barium (b) Cadmium
(a) Ca (b) Mg (c) Potassium (d) Nickel
(c) Ag (d) Al 45. Which of the following are true or false electrolytic
40. The position of element from (top to bottom) in extraction of aluminium -
electrochemical series (a) anode material contains graphite
(a) Zn > Cr > Fe > Sn > Pb (b) cathode made of carbon
(b) H > Cu > Hg (c) cathode reacts with O2 released to form CO2
(c) Hg > Ag > Au (d) anode reacts with O2 released to form CO2
(d) Al > Mn > Zn > Cr
46. Which of the following employ downward
41. In Cupellation movement of ore -
(a) Ag impurity is removedfrom Pb metal (a) Gravity separation (b) Froath floatation
(b) Pb impurity is removed from Ag metal
(c) Bessemer converter(d) Blast furnance
(c) Zn impurity is removed from Ag metal
47. Metal which can be extracted by smelting process
(d) Zn impurity is removed from Pb metal
(a) Pb (b) Fe
42. Which of the following option are correct
(c) Zn (d) Mg
(a) In electrolytic refining of Au, AuCl3 solution is
used as electrolyte 48. Which of the following reactions are actually
(b) In electrolytic refining of Ni, Ni(NO3)2 solution employed for commercial extraction of metal -
is used as electrolyte (a) Fe2O3 + 2Al Al2O3 + 2Fe
(c) In electrolytic refining of Ag, AgNO3 + HNO3 (b) 2Na[Au(CN)2] + Zn Na2[Zn(CN)4] + 2Au
solution is used as electrolyte (c) Cr2O3 + 2Al Al2O3 + 2Cr
(d) In electrolytic refining of Zn, ZnSO4 + H2SO4
(d) Cu2S + Pb Cu + PbS
solution is used as electrolyte

www.jeebooks.in
3.14 METALLURGY
49. In the manufacturing of metallic sodium by fused 55. If at a temperature there is sudden rise in slope in
salt electrolysis method (Down’s process), small Elingham diagram , that temperature can indicate-
amount of CaCl2 is added. The purpose is - (a) Melting point of metal oxide
(a) To improve electrical conductance of electrolyte (b) Boiling point of metal oxide
(b) To stabilise metallic sodium (c) Melting point of metal
(c) To increase the temperature of elctrolysis (d) Boiling point of metal
(d) To decrease the melting point of NaCl 56. Which alloy has Cu and Zn as constituent metals -
50. Collectors are the substances which help in (a) Brass (b) Dutch metal
attachment of an ore particle to air bubble in froth. (c) Bronze (d) Bell metal
The example of collectors are
57. Correct statements are -
(a) Sodium pyrophosphate
(a) Black zack is ZnS
(b) Sodium nitropruside
(b) Ruby copper is Cu2O
(c) Sodium ethyl xanthate
(c) Parke’s process is based on distribution
(d) Sodium lauryl sulphate principle
51. B4C is used - (d) Stellite is an alloy having highest % of cobalt
(a) to extract boron metal among all metals present in it
(b) for making bullet-proof clothing 58. The crude metal zinc which is called spelter contains
(c) as an abrassive for polishing impurities like -
(d) as lubricants (a) Ag (b) Fe
52. Elingham diagram can be drawn for which of the (c) Cd (d) Pb
following 59. Which alloy has Cu and Sn as constituent metals -
(a) Sulphide (b) Oxide (a) Brass (b) Dutch metal
(c) Halides (d) Nitrates (c) Bronzene (d) Bell metal
53. Choose correct options - 60. Pitch blende is not the sorce of
(a) Reduction of metal oxide will be easier if the (a) Ba (b) U
metal formed is in the liquid state at the (c) Ge (d) Tl
temperature of reduction
(b) Reduction of metal oxide will be easier if the EXERCISE # III
metal formed is in the solid state at the  Linked Comprehension Type :
temperature of reduction Paragraph for Q.1 to Q.3
(c) Aluminium can reduce MgO above 1350°C
According to Lux concept of acids and bases, an acid
(d) Aluminium can not reduce MgO above 1500°C is an acceptor of oxide ions and a base is a donor of
54. Choose correct options - oxide ion. This concept is very much applicable in the
(a) Reduction of metal oxide take place at room fields of ceramic and metallurgy.
temperature only 1. CaO acts as flux for impurities like SiO2 to form
(b) A particular metal oxide can be reduced by slag CaSiO3. Here :-
many reducing agent , but temperature for (a) CaO acts as base because it is donor of oxide ion.
reduction is same for all reducing agent (b) CaO acts as base because it acceptor of oxide
(c) On increasing temperature metal oxide stability ion.
decreases (c) SiO2 acts as acid because it is donor of oxide ion.
(d) A metal oxide can be reduced by many reducing (d) SiO2 acts as acid because it is acceptor of oxide
agent but temperature for reduction is different ion.
for all reductants

www.jeebooks.in
METALLURGY 3.15
2. 6SiO2 + 2Ca3(PO4)2 6CaSiO3 + P4O10. Here temperature and air supply in order to burn the
(a) SiO2 and P4O10 acts as acid organic metal and drive of most of the moisture ,
(b) SiO2 and P4O10 acts as base carbondioxide, sulphur, arsenic. Ferrous oxide is
converted into ferric oxide. The roasted mass
(c) Ca3(PO4)2 and CaSiO3 acts as acid
contains ferric oxide Fe2O3.
(d) Ca3(PO4)2 and CaSiO3 acts as extremly strong
(ii) Smelting and reduction in blast furnace : The
base
roasted ore is mixed with coke and limestone (flux)
3. CaO + SiO2 CaSiO3 and charged into the blast furnace when the ferric
FeO + SiO2 FeSiO3 oxide is reduced to metallic iron.
MgO + SiO2 MgSiO3 7. Top most zone in blast furnace, at 1070 K / 800°C
CaSiO3 , FeSiO3 MgSiO3 all are :- which reaction can not take place :-
(a) Flux (a) 3Fe2O3 + CO 2Fe3O4 + CO2
(b) Gangue particle (b) Fe3O4 + CO 3FeO + CO2
(c) Slag (c) CaCO3 CaO + CO2
(d) Lewis acid (d) FeO + C Fe + CO
Paragraph for Q.4 to Q.6 8. The slag formed in blast furnace is :-
When mixture of metal in the liquid state solidify three (a) SiO2 (b) CaSiO3
types of alloys are formed : (c) CaCO3 (d) P 4O10
(i) Alloys in which the metals crystallise separately. 9. At temperature zone 1300°C to 1900°C which
(ii) Alloys in which the metals form solid solutions. reaction can not take place :-
(iii) Alloys in which the metals form the intermetallic (a) FeO + C Fe + CO
compound. (b) C + O2 CO2
4. Alnico ; Duralumin and Magnelium are alloy of :- (c) C + CO2 2CO
(a) Zn metal (b) Cu metal (d) 3Fe2O3 + CO 2Fe3O4 + CO2
(c) Al (d) As Paragraph for Q.10 to Q.13
5. Copper based alloy is :- Steel is produced from pig iron by first resoning
(a) Brass (b) Bronze mechanically all the impurities in the pig iron. e.g. e, Si,
(c) Bell metal (d) All Mn, S, P by oxidation and then adding the current
among of carbon. The operation is carried out in
6. Which of the following alloy having copper and
Bessemer converter or in a Siemens-Martin open hearth
zinc metal along with other metal :-
furnace.
(a) German silver
10. The two types of bessemer process are :
(b) Gun metal
(a) The oxidation bessemer process and reduction
(c) (a) and (b) bessemer process
(d) None (b) The redox bessemer process and non-redox
Paragraph for Q.7 to Q.9 bessemer process
Iron is extracted from its oxide and carbonate ores by (c) The acidic bessemer process and basic
reduction with coke in blast furnace. The process is bessemer process
carried out in two steps. (d) The neutral bessemer process and amphoteric
(i) Preliminary roasting or calcination : The ore is bessemer process
calcine with a little coal in heaps regulating the

www.jeebooks.in
3.16 METALLURGY
11. In acid bessemer process the slag obtained can 13. Which of the following statements are correct :
be:- (a) Silica bricks are used as lining in acidic bessemer
(a) MnSiO3 converter :-
(b) FeSiO3 (b) Calcined dolomite or magnesite is used as lining
(c) Ca3(PO4)2 in basic bessemer converter
(c) In acidic bessemer , the interaction between
(d) both (a) and (b)
the slag which is iron phosphate and the carbon
12. In basic bessemer process the slag obtained is containing iron continuously regenerates iron
called Thomas slag which is used as fertiliser. The phosphide according to the reaction.
Thomas slag is actually.
2P2O5 + 6Fe + 3O2 2Fe3(PO4)2
(a) MnSiO3
Fe3(PO4)2 + 2Fe3C + 3Fe 2Fe3P + 6 FeO + 2CO
(b) FeSiO3
(d) All
(c) Ca3(PO4)2
(d) CaSiO3
Paragraph for Q.14 to Q.16
G°/kJ mol–1 of O2

A
–800
Al 2O 3
3 O2
–900 A l+2
2
–1000
2MgO
O2
–1100 2Mg +

–1200
1400°C

0°C 400°C 800°C 1200°C 1600°C 2000°C

Temperature
14. The two equations are
4 2
Al + O2  Al2O3 .....(i)
3 3
2Mg + O2 2MgO .....(ii)
A is the point of intersection of the Al2O3 and MgO curves. Which of the following option is correct.
2 4
(a) Below temperature 1400°C, Al2O3 + 2Mg 2MgO + Al reaction is thermodynamically feasible.
3 3
4 2
(b) Above temperature 1400°C, Al + 2MgO  Al2O3 + 2Mg reaction is thermodynamically feasible.
3 3
2 4
(c) At temperature 1400°C, G° = 0 for the reaction Al2O3 + 2Mg → 2MgO + Al
3 3
(d) All

www.jeebooks.in
METALLURGY 3.17
15. Al metal can’t be used for reduction of MgO 18. Which step is not involved in the extraction of pure
because :- metal from ore-(Y)
(a) This reaction is nonspontaneous (a) Froath flotation
(b) This reaction is economical (b) Smelting
(c) This reaction is noneconomical (c) Self reduction
(d) None (d) Distillation
16. Al2O3 , MgO are examples of Paragraph for Q.19 & Q.20
(a) amphoteric , amphoteric oxide (I) FeCr2O4 + NaOH + air a + Fe2O3
(b) amphoteric , basic oxide (II) (a) + (b)  Na2Cr2O7
(c) basic , amphoteric oxide (III) Na2Cr2O7 + X 

 Cr2O3
(d) basic , basic oxide
(IV) Cr2O3 + Y  
 Cr
Paragraph for Q.17 & Q.18
19. Compound A and B are -
Zinc ore (X) 
calcination
 Solid(S) + Gas - 1 (a) Na2CrO4 , H2SO4
Zinc ore (Y) 
roasting Solid(S) + Gas - 2
(b) Na2Cr2O7 , HCl
 with C
Solid (S) 
At 1673 K
 Metal (M) + Gas - 3 (c) Na2CrO5 , H2SO4
combustion
Gas - 3   Gas - 1 (d) None
with O 2
20. X and Y are
17. Which substance produces colourless, odourless
gas with dil. H2SO4 - (a) C and Al
(b) Al and C
(I) Ore(X) (II) Ore (Y)
(c) C in both
(III) Metal(M) (IV) Solid(S)
(d) Al in both
(a) I, III (b) I, II, III
(c) II, III (d) I, IV
Matrix Match Type :
21. Match the column-I with column-II :-
Column - I : (Property) Column - II : (Element/ Compound)
(a) Explosive (P) Cu
(b) Self-reduction (Q) Fe3O4
(c) Ferrimagnetic material (R) Cu(CH3COO)2.Cu(OH)2
(d) Verdigris (S) Pb(NO3)2
22. Column - I : (Process) Column - II : (Compounds formed during reaction)
(a) Mac-Arther forest cyanide process (P) [Ag(CN)2]–1
(b) Mond process (Q) TiI4
(c) Van-Arkel method (R) Ni(CO)4
(d) Blast Furnace (S) CaSiO3(slag)
23. Column - I : (Process involved) Column - II : (Pure metal extracted)
(a) Bessemerisation (P) Cr
(b) Electrolytic refining using aqueous salt as an electrolyte (Q) Au
(c) Extracted by the formation of soluble complex (R) Ag
(d) Reaction which involves Mg ribbon and (S) Fe
(BaO2 + KClO3) mixture to generate ignition temperature
(T) Cu

www.jeebooks.in
3.18 METALLURGY
24. Column - I Column - II
(Elements) (Characterstic feature of elements and their oxide)
(a) Mg (P) Highest value of | G| range for oxide formation for these elements
(b) Al (Q) Lowest value of | G| range for oxide formation for these elements
(c) Fe (R) Can reduce at least one of metal oxides of given elements
(d) Hg (S) Strongest reducing agent among these
(T) Its metal oxides has lowest thermal stability among oxide of these metal
25. Column - I Column - II
Chemical changing metallurgy Purpose of metallurgy of specified metal
(a) M2O3 + C + Electrical energy M + CO / CO2 (P) Purification of copper

(b) (M + CO + Fe) + CO Fe(s) (Q) Benefication of bauxite
50°C
+ CO(s) + [vapoures of carbonyl complex of M]
(c) (Fe2O3.MgO)g + OH–(aq.) Fe2O3 + [M(OH)4]–1 sol. (R) Extraction of metal from alumina
(d) Fe.M.Ag.Au + Electrical oxidation (S) Separation of nickel from impurities

in aq. solution Fe2+(aq.) + Ag(s)


of metal (M) + Ml2+(aq.) + Au(s)
26. Match the column :-
Column - I Column - II
(Process involved) (Pure metal extracted)
(a) Poling (P) Sn
(b) Cupellation (Q) Ag
(c) Zone refining (R) Cu
(d) Parke’s process (S) Ge
(T) Pb
27. Match the column :-
Column - I Column - II
(Elements) (Correct information regarding extraction process)
(a) Ni (P) A solution of lead silicofluoride PbSiF6 containing 8-10% of H2SiF6 is used
as electrolyte for
(b) Pb (Q) Complex formation reaction is involved in extraction of
(c) Cu (R) Self reduction
(d) Hg (S) Carbon monoxide is used for extraction of metal
(T) d-block element

www.jeebooks.in
METALLURGY 3.19
28. Column - I Column - II
(Metal extracted from its ore) (Process involved for extraction of metal)
(a) Zinc from ZnCO3 (P) Calcination
(b) Lead from PbS (Q) Removal of iron
(c) Cu from CuFeS2 (R) Froath floatation process
(d) Tin from cassiterite (S) Poling
(T) Gravity separation
29. Consider following extraction process and match Column-I with Column-II :
Wolframite
(Impurity)

Cassiterite
Step-I
+
Wolframite

Step-II Step-III
Cassiterite Metal (M) M
(Ore) + Metal
(Impurity of metal
oxide MO2) Step-IV

Pure
Metal M
Column - I Column - II
(a) Magnetic seperation (P) Step-I
(b) Liquation (Q) Step-II
(c) Poling (R) Step-III
(d) Carbon reduction (S) Step-IV
30 Match the column :-
Column - I Column - II
(a) Downs cell (P) Fused MgCl2
(b) Dow sea water process (Q) Fused Al2O3 + Na3AlF6
(c) Hall - Heroult (R) Fused KHF2
(d) Moissan process (S) Fused 40 % NaCl and 60% CaCl2
31 Match the column :-
Column - I Column - II
Ore Process for benefactions
(a) ZnS (P) Magnetic separation
(b) FeCr2O4 (Q) Froath floatation
(c) SnO2 (R) Gravity separation
(d) Cu2S.Fe2S3 (S) Roasting

www.jeebooks.in
3.20 METALLURGY
32 Match the column :-
Column - I Column - II
Conversion Process involved
(a) Cu2S  Cu2O (P) Roasting
(b) CaCO CaO (Q) Calcination
(c) ZnS  Zn (R) Carbon reduction
(d) PbS  Pb (S) Self reduction
33. Match the column :-
Column - I Column - II
Definition Term
(a) Naturally occuring chemical substance in the earth’s (P) Metallurgy
crust obtainable by mining
(b) Substance from which metal can be extracted (Q) Gangue
economically and conventionally
(c) Undesired earthy substance contaminated with desired (R) Ore
earthy substance ore
(d) Scientific and technological process used for isolation of metal (S) Mineral
34. Chemical changing metallurgy :-
Column - I Column - II
Phenomenon Term
(a) Calcium phosphate (P) Blister copper
(b) Hard steel is heated to bright redness and then (Q) Pig iron
allowed to cool very slowly
(c) Many impurities C/Si/P/S (R) Thomas slag
(d) SO2 evolution take place and pores are developed (S) Anealing
35. Match the column:-
Column - I Column - II
Different types of iron Features
(a) Cast iron (P) Purest form of iron with C % 0.1 - 0.15%
(b) Wrought iron (Q) Molten iron from blast furnance having more than 4% carbon
(c) Steel (R) Carbon % varies from 0.15 - 1.5%
(d) Pig iron (S) Carbon % varies from 2 - 4%

EXERCISE # IV
 Integer Type :
1. If a mixture of ores consist of :
(i) Zinc blend (ii) Argentie (iii) Haematite (iv) Chromite (v) Dolomite
(vi) FeWO4 (vii) Cassiterite
Then find out number of ores which are collected in container when allowed to move towards magnetic roller.

www.jeebooks.in
METALLURGY 3.21
2. Calcination of how many ore produce CO2. 9. How many of the following terms are associated
(a) Limonite (b) Siderite for purificaton of iron.
(c) Malachite (d) Cerrusite (a) Puddling process
(e) Azurite (f) Dolomite (b) Thomas slag
3. Of the following metal that can not be obtained by (c) Cupellation
electrolysis of the aqueous solution of their salts (d) Bessemerisation
using inert electrode : (e) Amalgamation
Ag ; Cu ; Cr ; Mg ; Al ; Na ; Ba ; Ca
10. Find the number of ores in which any metal is
4. Find the number of compounds which are added present in +1 oxidation state.
to acidic impurities to form slag.
Copper pyrite, Chalcocite, Chile Salt petre,
(a) SiO2 ; MgO ; CaO ; CaCO 3 ; MgCO3 ; Cryolite, Cuprite, Sylvine
P2O5 ; B2O3
11. Find the number substance which contain metal is
5. How many metals are commercially obtained by in its elemental form mainly.
auto reduction.
Blister copper, Pig iron, Wrought iron, Steel,
Fe ; Na ; Mg ; Mg ; Ca ; Pb ; Sn ; Cu ; Hg ; Au
Copper matte, Alumina, Spelter
6. How many metals are commercially obtained by
12. Find the number of ores which can be roasted or
carbon reduction of their oxide .
calcined followed by reduction with carbon to give
Fe ; Al ; Mg ; Ca ; Sn ; Pb ; Zn respective metal.
7. How many metals can be commercially extracted
Fe2O3 . 3H2O ; ZnS ; ZnCO3 ; PbS (low grade)
by hydrometallurgy i.e., displacement reaction
takes place in aqueous solution. 13. Find the number of impurities which are deposited
as anode mud in the electrorefining of copper.
Au ; Ag ; In ; Zn ; Pb ; Al ; Cu
Antimony ; Selenium ; Tellurium ; Silver ; Gold ;
8. How many statements are correct?
Platinum
(i) Graphite rod is used as anode but not diamond
14. Find the number of ores which can be concentrated
because moblie electrons are present in graphite
by magnetic separation.
layer which helps in electrical conductivity.
(ii) If the impurity in a metal has greater affinity for Galena ; Copper pyrites ; Haematite ; Siderite
oxygen then the purification of metal may be 15. Find the number of ores in which roasting process
carried out by cupellation. is used in metallurgy of corresponding metal.
(iii) Fe and Pt can’t form alloy with Hg. Galena ; Haematite ; Calamine ; Zinc Blende ;
(iv)During roasting of sulfide ore at high temperature Cinnaber, Horn Silver ; Lime stone.
oxide is formed and at low temperature sulphate 16. Find the number of steps in the following used
is formed. during the extraction of spelter from zinc blend.
(v) During Sn/Pb/iron metallurgy CaSiO3 is formed Poling ; Electrorefining ; Roasting ; Froath floatation
as slag. ; Smelting ; Magnetic separation
(vi)Wolframite is ferro magnetic, therefore attracted 17. The number of following pairs is correctly matched:
by magnet. (i) Van Arkel method - Zirconium
(vii)Magnetic siderite, chromite, Wolframite are of (ii) Amalgamation - Lead
Fe(II) but limonite, Haematite are of Fe(III).
(iii) Distillation method - Zinc
(viii)Bauxite is considered as oxide as well as
(iv) Poling process - Copper
hydroxide of Al.
(ix)In cyanide extraction process of silver from (v) Mond process - Titanium
argentite ore the oxidising and reduing agent (vi) Van Arkel method - Zirconium
used as O2 and Zn.

www.jeebooks.in
3.22 METALLURGY
18. Find the number of curves which are wrongly 24. Find the number of unstable oxide out of the
presented in the Ellingham diagram. following oxide which undergo decomposition
O below 200°C.
Ni
(+)ve  CaO ; Al2O3 ; MgO ; Fe2O3
Ni
O
PbPbO 25. Find the number of oxide out the of the following
(–)ve) Cu oxide which undergo decomposition below 400°C.
C
uO
CCO CaO ; Al2O3 ; MgO ; Fe2O3 ; Ag2O and HgO
G° (kj/mol)
 CO 26. What is the coordination number of aluminium in
C
MgO O mineral Cryolite
Mg
27. How many water of crystallisation present in the
ore Carnalite.
T(°C) 
19. Find the number of following reactions which are 28. In iron metallurgy for preparation pig iron from
involved in roasting process : Haematite ore the number of reducing agent
involed.

(i) CaCO3 . MgCO3 CaO + MgO + CO2 29. Find out number of ores which are examples of
 oxy salt ore but not oxide ore ?
(ii) 2Cu2S + 3O2 2Cu2O +2SO2 Epsom salt ; Glauber salt ; Gypsum ; Anglesite ;
 Rutile ; Cuprite ; Plaster of paris
(iii) S8 + 8O2 8SO2
30. Find the number of ores in which atleast one metal
 is in +3 oxidation state :-
(iv)P4 + 5O2 P4O10
 Bauxite ; Diaspore ; Gibbsite ; Limonite ; Cryolite;
(v) 4As + 3O2 2As2O3 Haematite ; Magnetite ; Chromite
 31. Find the number of ores in which atleast one metal
(vi)2ZnS + 3O2 2ZnO + 2SO2
is in +2 oxidation state :-
20. How many reactions can show slag formaton
Baryte ; Fluorspar ; Chromite ; Magnetite ; Iron
process from the given reaction ?
pyrite ; Copper pyrite ; Carnalite
(i) SiO2 + CaO CaSiO3
32. % of silver in German silver -
(ii) MgCO3 + SiO2 MgSiO3 + CO2
33. Number of metals present in Alnico alloy -
(iii) FeO + SiO2 FeSiO3
34. Number of metals present in Devardas alloy -
(iv)Cr2O3 + 2Al Al2O3 + 2Cr
35. For how many elements amalgamation process is
(v) CaO + P2O5 Ca3(PO4)2 applicable for purification of metals -
21. How many elements can be purified by liquation Ag ; Au ; Cu ; Zn
method ?
36. For the reaction Al2O3. 2H2O + xNaOH + H2O
Sn ; Pb ; Bi ; Zn ; Cd ; Fe  xNa[Al(OH)4]
22. How many elements can be purified by Zone The value of x
refining method ?
37. Find the total number of acidic refracories out of
Ge ; Si ; Ga ; Se ; Te ; Tl ; the following compounds.
22. How many elements can be purified by distillation? CaO ; Quartz ; Dolomite ; Bone ash ; Magnesite.
Zn ; Pb ; Cd ; Hg ; Mg ; Na ; Ka 38. Among the following number of compound that
23. Find out the number of layers developed in Hoop’s would require electrolysis process to get their
process which is used for electrolytic refining of respective metal is .
aluminium. Al2O3, MgCl2, Fe2O3

www.jeebooks.in
METALLURGY 3.23
39. Find out the number of layers developed in Park’s 6. The substance used as froth stablisers in froth -
process which is used for purification of lead metal.. floatation process is :-
40. How many statements are correct [Jee-Mains -2012(online)]
(i) Steel are Brittle (a) Copper sulphat
(ii) Cast iorn is soft (b) Aniline
(iii) Pewter is an Alloy of Pb and Sn (c) Sodium cyanide
EXCERCISE # V(A) (JEE-MAIN) (d) Potassium ethyl xanthante
1. Aluminium is extracted by the electrolysis of :- 7. Which of the oxide groups among the following
[AIEEE - 2002] cannot be reduced by carbon :-
(a) Bauxite [Jee-Mains - 2012(online)]
(a) Fe2O3 , ZnO (b) PbO , Fe2O3
(b) Alumina
(c) Cu2O , SnO2 (d) CaO, K2O
(c) Alumina mixed with molten cryolite
8. In Goldschmidt alumino thermic process which of
(d) Molten cryolite
the following reducing agents is used :-
2. Pyrolusite is a / an :- [AIEEE - 2002] [Jee-Mains - 2013(online)]
(a) Oxide ore (b) Sulphide ore (a) Calcium (b) Coke
(c) Carbide ore (d) Not an ore (c) Sodium (d) Al-power
3. Which one of the following ores is best 9. Calcination is the process in which :-
concentrated by froth - flotation method :- (1) Ore is heated strongly below its melting point
[AIEEE - 2004] in the presence of excess of air and is used for
(a) Galena (b) Cassiterite the conversion of carbonates and hydrated
(c) Magnetite (d) Malachite oxide ores to their respective oxides.
4. Which one of the following factors is of no (2) Ore is heated strongly below its melting point
in the absence or limited supply of air and is
significance for roasting sulphide ores to the oxides
used for conversion of sulphide ores to their
and not subjecting the sulphide ores to carbon
respective oxides.
redction directly ? [AIEEE - 2008]
(3) Ore is heated strongly below its melting point
(a) Metal sulphides are thermodynamically more either in the limited or absence of air and is
stable than CS2 used to converte carbonates and hydrated
(b) CO2 is thermodynamically more stable than oxide ores to their respective oxides.
CS2 (4) Ore is heated strongly above its melting point
(c) Metal sulphides are less stable than the CS2 in the limited supply of air to convert sulphide
(d) CO2 is more volatile than CS2 ores to their respective oxides.
10. The metal that cannot be obtained by electrolysis
5. Which method of purification is represented by the
of an aqueous solution of its salts is :-
following equation :- [AIEEE - 2012]
[Jee-Mains - 2014]
Ti(s) + 2I2(g) 523 K TiI4(g) 1700 K Ti(s) + 2I2(s)
(a) Cu (b) Cr
(a) Van Arkel (c) Ag (d) Ca
(b) Zone refining 11. The form of iron obtained from blast furnace is :-
(c) Cupellation [Jee-Mains - 2014(On line)]
(d) Poling (a) Steel (b) Wrought iron
(c) Cast Iron (d) Pig iron

www.jeebooks.in
3.24 METALLURGY
12. In the context of the Hall-Heroult process for the 6. The chemical process in the production of steel
extraction of Al, which of the following statements from haematite ore involve :- [2000 Qualifying]
is false? [Jee-Mains - 2015] (a) reduction
3+
(a) Al is reduced at the cathode to form Al (b) oxidation
(b) Na3AlF6 serves as the electrolyte (c) reduction followed by oxidation
(c) CO and CO2 are produced in the process (d) oxidation followed by reduction
(d) Al2O3 is mixed with CaF2 which lowers the 7. Electrolytic reduction of alumina to aluminium by
melting point of the mixture and brings Hall - Heroult process is carried out :-
conductivity. [2000 Qualifying]
EXCERCISE # V(B) (ADVANCED) (a) in the presence of NaCl
1. In extractive metallurgy of zinc partial fsion of ZnO (b) in the presence of fluorite
with coke is called ........... and reduction of the (c) in the presence of cryolite which forms a melt
ore to the molten metal is called ......... [Jee’88] with lower melting temperature
(d) in the presence of cryolite which forms a melt
2. Carnallite does not contain :-
with higher melting temperature
(a) K 8. The chemical composition of “slag” formed
(b) Ca during the smelting process in the extraction of
(c) Mg copper is :- [2001 Qualifing]
(d) Cl (a) Cu2O + FeS (b) FeSiO3
(c) CuFeS2 (d) Cu2S + FeO
3. During initial treatment, preferential wetting of ore
9. Which of the following process is used in extractive
by oil and gangue by water takes place in :-
metallurgy of magnesium ? [2002 Qualifing]
(a) Levigation (gravity separation)
(a) Fused salt electrolysis
(b) Froth floatation (b) Self reduction
(c) Leaching (c) Aqueous solution electrolysis
(d) Bessemerisation (d) Thermite reduction
4. Which of the following is true for calcination of 10. In the process of extraction of gold,
metal ore ? [2003 Qualifing]
(a) It makes the ore more porous O
Roasted gold ore + CN – + H 2 O 2

(b) The ores is heated to a temperature when fusion –


[X] + OH [X] + Zn [Y] + Au
just begins Identity the complexes [X] and [Y] :-
(c) Hydrated salts lose their water of crystallisation (a) X = [Au(CN–)2]– , Y = [Zn(CN)4]2–
(d) Sulphur in sulphides is oxidised to SO2 (b) X = [Au(CN)4]3– , Y = [Zn(CN)4]2–
(e) Heating with carbon leads to better calcination (c) X = [Au(CN)2]– , Y = [Zn(CN)6]4–
5. In the commercial electrochemical process for (d) X = [Au(CN)4]– , Y = [Zn(CN)4]2–
aluminium extraction, the electrolyte used as :- 11. The methods chiefly used for the extraction of lead
[Jee-1999] and tin from their ores are respectively :-
[JEE - 2004]
(a) Al(OH)3 in NaOH solution
(a) self reduction and carbon reduction
(b) An aqueous solution of Al2(SO4)3
(b) self reduction and electrolytic reduction
(c) A molten mixture of Al2O3 and Na3AlF6 (c) carbon reduction and self reduction
(d) A molten mixture of AlO(OH) and Al(OH)3 (d) cyanide process and carbon reduction

www.jeebooks.in
METALLURGY 3.25
12. Which ore contains both iron and copper ? 18. Match the extraction processes listed in column I
[JEE - 2004] with metals listed in column II :- [JEE-2006]
Column-I Column-II
(a) Cuprite (b) Chalcocite
(a) Self reduction (P) Lead
(c) Chalcopyrite (d) Malachite
(b) Carbon reduction (Q) Silver
13. Extraction for zinc from zinc blende is achieved
(c) Complex formation and (R) Copper
by:- [JEE - 2007]
displacement by metal
(a) electrolytic reduction
(d) Decomposition of iodide (S) Boron
(b) roasting followed by reduction with carbon
19. Match the conversions in column I with the type(s)
(c) roasting followed by reduction with another of reaction(s) given in column II. Indicate your
metal answer by barking the appropriate bubbles of the
(d) roasting followed by self-reduction 4 × 4 matric given in the ORS :- [JEE-2008]
14. Native silver metal forms a water soluble complex Column-I Column-II
with a dilute aqueous solution of NaCN in the (a) PbS PbO (P) Roasting
presence of :- [JEE-2008] (b) CaCO3 CaO (Q) Calcination
(a) Nitrogen (b) Oxygen (c) ZnS Zn (R) Carbon reduction
(c) Carbon dioxide (d) Argon (d) Cu2S Cu (S) Self reduction
Passage for Q. 15 to 16 20. In extractive metallurgy of zinc partial fusion of ZnO
Copper is the most nobel of the first row transition with coke is called .............. and reduction of the
metals and occurs in small deposits in several countries. ore the molten metal is called ............ (smelting,
Ores of copper include chalcanthite (CuSO4 . 5H2O), calcining, roasting, sintering) :- [JEE-1988]
atacamite (Cu2Cl(OH)3), cuprite (Cu2O), copper 21. Extraction of metal from the ore cassiterite
glance (Cu 2S) and malachite (Cu 2(OH)2CO 3). involves:- [JEE2011]
However, 80% of the world copper production comes (a) carbon reduction of an oxide ore
from the ore chalcopyrite (CuFeS2). The extraction of (b) self-reduction of a sulphide ore
copper from chalcopyrite involves partial roasting ,
(c) removal of copper impurity
removal of iron and self-reduction. [JEE-2010]
(d) removal of iron impurity
15. Partial roasting of chalcopyrite produces :-
22. Oxidation states of the metal in the minerals
(a) Cu2S and FeO haematite and magnetite, respectively, are:-
(b) Cu2O and FeO [JEE-2011]
(c) CuS and Fe2O3 (a) II, III in haematite and III in magnetite
(d) Cu2O and Fe2O3 (b) II, III in haematite and II in magnetite
16. Iron is removed from chalcopyrite as :- (c) II in haematite and II, III in magnetite
(a) FeO (d) III in haematite and II, III in magnetite
(b) FeS 23. In the cyanide extraction process of silver from
(c) Fe2O3 argentite ore, the oxidizing and reducing agents used
are :- [JEE-2012]
(d) FeSiO3
(a) O2 and CO respectively
17. In self-reduction, the reducing species is :-
(b) O2 and Zn dust respectively
(a) S (b) O 2–
(c) HNO3 and Zn dust respectively
(c) S 2– (d) SO2
(d) HNO3 and CO respectively

www.jeebooks.in
3.26 METALLURGY
24. Sulphide ores are common for the metals :- 27. Copper is purified by electrolytic refining of blister
[JEE-2013] copper. The correct statement (s) about this
(a) Ag, Cu and Pb (b) Ag, Cu and Sn process is (are) :- [JEE-Adv.2015]
(c) Ag, Mg and Pb (d) Al, Cu and Pb (a) Impure Cu strip is used as cathode
25. The carbon - based reduction method is NOT used (b) Acidified aqueous Cu 2 SO 4 is used as
for the extraction of :- [JEE-2013] electrolyte
(a) Tin from SnO2 (c) Pure Cu deposite at cathode
(b) Iron from Fe2O3 (d) Impurities settle as anode - mud
(c) Aluminium from Al2O3 28. Match the anionic species given in column I that
(d) Magnesium from MgCO3 . CaCO3 are present in the ore(s) given in column II :-
[JEE-Adv. 2015]
26. Upon heating with Cu2S, the reagent(s) that give
copper metal is / are :- [JEE-2014] Column-I Column-II
(a) CuFeS2 (b) CuO (a) Carbonate (P) Siderite
(c) Cu2O (d) CuSO4 (b) Sulphide (Q) Malachite
(c) Hydroxide (R) Bauxite
(d) Oxide (S) Calamine
(T) Argentite



www.jeebooks.in
QUALITATIVE SALT ANALYSIS
PROB L EM S B ASED ON GI VEN T OPI CS  Tests for nitrate (NO3–) radical
Tests for acid radicals  Tests for sulphate (SO42–) radical
 Action of dilute acids  Tests for borate (BO33–) radical
 Tests for CO32– / HCO3– and SO32– / HSO3– radicals  Tests for phosphate (PO43–) radical
 Distinction between carbonate and bicarbonate  Tests for chromate (CrO 42–) and dichromate
 Distinction of carbonate and bicarbonate when (CrO72–) radicals
both are present together  Tests for permagnate (MnO 4–) and magnate
 Distinction between sulphide and bisulphite (MnO42–) radicals
Tests for basic radical
 Some other tests for SO32– ions
 Dry tests for basic radical
 Tests for sulphide (S2–) radical
 Tests for thiosulphate (S2O32–) radical
 Heating effect on the dry sample

 Tests for nitrite (NO2–) radical  Flame test


 Tests for Acetate, Formate and Oxalate radicals  Borax bead test
 Specific test for acetate (cacodyl oxide test)  Sodium carbonate bead test
 Specific test for formate (mercury (II) formate  Wet tests for basic radicals
test)  Classification of cations (group analysis)
 Specific test for oxalate  Some general test for cations
 Tests for Halide (Cl– , Br– , I– ) radicals  Test for group V cations
 Specific test for Cl– (chromyl chloride test)  Test for group VI and zero group cations
 Specific test for Br– and I– (layer test)  Specific tests for some cations

 Other test for Br– Heating effect

 Other test for I–

www.jeebooks.in
CHAPTER
4
Qualitative Salt Analysis

EXERCISE # I
 Only one correct answer : 6. Black (A) + H2SO4 
1. Which of the following compound(s) slowly (B) (smell of rotten egg) + (C)
disappears with prolonged passage of CO2 in (C) + K3[Fe(CN)6]  Blue (D)
aqueous solution ? A would be
(a) BaCO3 (b) CaCO3 (a) HgS (b) FeS
(c) Both (a) and (b) (d) None of these (c) Bi2S3 (d) CuSO3
2. (I) Salt of ‘X’ + CaCl2(excess) ‘Y’ + Filtrate 7. (I) Salt ‘A’ + dil. H2SO4 ‘B’ gas [‘B’ has
(II) Filtrate + NH3 ‘Y’ suffocating order.]
(III) Filtrate + Boiling ‘Y’ /O2
(II) Salt ‘A’ + Pb(OAc)2 C()
Here salt ‘X’ is :- ‘D’() [‘C’ & ‘D’ both have white ppt.]
(a) mixure of HCO3– and CO32– Have ‘A’ and ‘D’ are :-
(b) only CO32– (a) HSO3– , PbSO4 (b) SO32– , PbSO3
(c) only HCO3– (c) SO32– , PbSO4 (d) CO32– , PbSO3
(d) None of these 8. Aqueous solution of a gas ‘X’ decolourises an
Hg2(NO3)2 dil.H2SO4
acidified K2Cr2O7 solution and on passing H2S,
3.
(C) (A) (B) the solutions gives white turbidity ‘Y’. Here ‘X’
yellow ppt. (Salt) Odourless gas and ‘Y’ are :-
(a) SO2 & S (b) CO2 & S
 CaCl2 sol.
(c) SO2 & Cr(OH)3 (d) CO2 & Cr(OH)3
(D)
dil. H2SO4
White ppt. 9. Na2S2O3 Na2SO4 + S() + 'B' (gas)
Salt ‘A’ is :
'B' + Ba(OH)2 'C'() + H2O
(a) Na2CO3 (b) Na2SO3
(white ppt.)
(c) Na2C2O4 (d) Na2SO4
4. What colour of ppt. is formed when KHCO3 reacts 'B'
with excess AgNO3 followed by boiling ? Clear solution 'D'
(a) white (b) green Here‘B’ and ‘D’ are :-
(c) brown (d) none of these (a) H2S & BaSO3
5. Which of the following does not form volatile (b) SO2 & BaSO3
product with conc. H2SO4 ?
(c) H2S & Ba(HSO3)2
(a) CO32– (b) SO42–
(d) SO2 & Ba(HSO3)2
(c) Cl– (d) NO3–

www.jeebooks.in
4.4 QUALITATIVE SALT ANALYSIS
10. The sulphide ion gives purple colour with sodium (b) Because it oxidises Fe2+ to Fe3+ so that Fe3+
nitroprusside. The purple colour is due to the can be completely precipitated as its hydroxide
formation of :- (c) Because precipitation of cations belonging to
(a) Na2[Fe(CN)3(NOS)3] IIIrd group by group reagent takes place only
(b) Na4[Fe(CN)4(NOS)2] in presence of conc. HNO3
(d) None of these
(c) Na4[Fe(CN)5(NOS)]
16. Sodium nitroprusside produces violet coloration
(d) Na4[Fe(NOS)6]
when react with Na2S due to :-
11. Which of the following statement is correct, about
(a) Oxidation of Fe2+ to Fe3+
“on passing H2S gas through acidified K2Cr2O7
solution” ? (b) Reduction of Fe2+ to Fe1+
(c) Formation of soluble complex without
(a) White turbidity is due to SO32–
changing oxidation state of iron
(b) White turbidity is due to SO42–
(d) Precipitation of iron sulphide from sodium
(c) White turbidity is due to S nitroprusside
(d) White turbidity is due to Cr3+ 17. Which of the following reagent is preferred to
12. ‘X” + CuSO4  I2 + White ppt. [Y] give vinegar-like odour on warming with acetate
ion?
Na2S2O3 (a) dil. H2SO4
NaI + [Z] colourless compound. (b) conc. CH3CO2H
Here compound [Z] is - (c) (conc. H2SO4 + C2H5OH)
(a) Cu2S2O3 (b) Cu2I2 (d) conc. HNO3
(c) Na2S4O6 (d) Na4[Cu6(S2O3)5] 18. Anion that can not decolorise acidic solution of
KMnO4.
13. FeCl3 + ‘X’(salt solution)  ‘Y’ (dark violet
colouration) (a) HCO3– (b) HSO3–
(c) SO32– (d) S2–
on standing Na 2S 2O3
19. Aq. solution of ‘A’ white ppt.
solution
[colour disappears]
which is not ‘A’ from the following :-
Here ‘X’ and ‘Y’ are -
(a) Pb(OAc)2 (b) dil. HCl
(a) Na2S2O3 & FeS4O6
(c) [Ni(en)3](NO3)2 (d) CuSO4
(b) Na2S2O3 & [Fe(S2O3)2]– 20. Which of the following reagent is used with KI
(c) Na2SO3 & FeSO4 solution to give pure HI ?
(d) Na2S & (FeS + S) (a) conc. H2SO4 (b) dil. H2SO4
14. Which of the following compound is insoluble in (c) conc. H3PO4 (d) dil. HCl
water ?
21. When ‘A’ salt solution is added carefully to a
(a) KNO2 (b) AgNO2 saturated solution of Fe(II) sulphate acidified with
(c) Ba(NO2)2 (d) NaNO2 dilute H2SO4 a brown ring is formed at the junction
15. Why few drops conc. HNO3 is added to IInd group of two liquids. The compound A is
filterate before proceding for IIIrd group radicals? (a) NaNO3 (b) NaNO2
(a) Because it oxidises the dissolved H2S to (c) NaCl (d) All of these
colloidal sulphur

www.jeebooks.in
QUALITATIVE SALT ANALYSIS 4.5
22. An inorganic salt solution ‘M’ gives white 28. Silver is not produced when :
precipitate with Pb(OAC)2 solution. The ppt. (a) AgNO3 solution is treated with Cu rod
dissolves on warming but becomes needle like (b) AgNO3 (ammoniacal solution) reacts with
crystal on cooling. The anionic part of salt ‘M’ is :- glucose
(a) Chloride (b) Bromide (c) AgNO3 solution + excess KCN is reacted
(c) Iodide (d) Sulphite with Zn dust
23. The mixture of salts NaCl, NaBr, NaI on adding (d) AgNO3 solution is treated with Na2S2O3 &
conc. H2SO4 and followed by warming gives gases followed by heating
respectively. 29. Which of the following compound(s) give black
(a) Cl2, Br2, I2 (b) HCl, HBr, HI precipitate with ammonia solution ?
(c) HCl, Br2, I2 (d) CrO2Cl2 , Br2, I2 (a) Hg2Cl2 (b) AgCl
24. A metal Mand its compound can give the following (c) PbCl2 (d) None of these
observable changesin a consequence of reactions 30. Which of the following cation gives black
dilute Colourless  aqueous  White  precipitate with H2S gas as well as soluble with

HNO3
  
NaOH  
solution   precipitate  excess yellow sodium disulphide ?
excess  Colourless  H 2S  White  (a) Hg22+ (b) Ag+
     
NaOH(aq)
soluton 
NaOH(aq.)
 precipitate  (c) Cd2+ (d) Both (a) & (b)
(a) Mg (b) Pb 31. Salt solution ['X'] dil. HCl white ppt.
(c) Zn (d) Sn ['Y']
25. Salt 'A' + dil. H2SO4 No action

+ conc. H2SO4
ppt. is washed by decantation
and dilute ammonia is added.
['B']
Brown fumes gas
Salt ‘A’ and gas ‘B’ are :- white ppt. [Z]
(a) NaNO2 and NO2 (b) NaNO3 and NO2 Here white ppt. of [Y] is :-
(c) NaBr and HBr (d) NaCl and Cl2 (a) PbCl2 (b) AgCl
26. NaNO3(s) + conc. H2SO4 + Cu. turning (c) Pb(OH)2 (d) Hg2Cl2
on heating
32. With ammonium sulphide (NH4)2S, Al3+ gives :-
(a) White precipitate of Al2S3
Brown fumes of 'X' + Blue colour of 'Y' (b) White precipitate of Al(OH)3
Here ‘Y’ is :-
(c) White precipitate of Al(OH)2(CH3CO2)
(a) NO2 (b) Cu(NO3)2
(d) None
(c) Ag2SO4 (d) Na2SO4
33. A white powder (A) on heating gave a non
27. Which of the following ion gives white precipitate combustible gas and a white residue.The residue
with ferric chloride solution ? on heating turns yellow. The residue dissolve in dil
(a) F– HCl and the solution gives a white ppt with
(b) S2O32– K4 [Fe(CN)6]. A would be
(c) CO32– (a) CaCO3 (b) ZnCO3
(d) CH3COO– (c) CaSO3 (d) CuCO3

www.jeebooks.in
4.6 QUALITATIVE SALT ANALYSIS
34. Which of the following compound is not coloured 40. What colour is observed when phenol phthalein
yellow :- indicator is added to soluble carbonate
(a) (NH4)3[As(Mo3O10)4] (a) White (b) Blue
(b) BaCrO4 (c) Pink (d) Green
(c) Zn2[Fe(CN)6] 41. ZnCl2 reacts with excess of NH3 solution to
(d) K3[Co(NO2)6] produce
35. 3% H2O2 on heating converts :- (a) a precipitate on Zn(OH)2
(a) Black precipitate PbS into yellow precipitate (b) a complex ion [Zn(NH3)4]2+ with tetrahedral
PbSO4 geometry
(b) Black precipitate PbS into white precipitate (c) a complex ion [Zn(NH3)4]2+ with linear
PbSO4 geometry
(c) Black precipitate PbS into colorless solution (d) a complex ion [Zn(NH3)4]2+ with square
PbSO4 planar geometry
(d) White precipitate PbS into colorless solution 42. Which of the following compound(s) turns
PbSO4 brownish - black precipitate, on boiling
36. Thr color of BiI3 precipitate is :- with Na2CO3 to their respectively aqueous salt
solution ?
(a) Blue (b) Brownish
(a) Pb(OH)2.2PbCO2 (b) Hg2CO3
(c) Black (d) White
(c) Ag2CO3 (d) (NH4)2CO3
37. Sodium carbonate when reacts with mercuric
chloride then 43. One of the product formed, when Bi2O3 is added
to an alkaline solution of sodiun hypochlorite, is
(a) reddish brown precipitate is formed.
(a) Bi(OH)3 (b) BiO3–
(b) yellow precipitate is formed.
(c) BiOCl (d) Bi2Cl3
(c) white precipitate is formed.
44. A salt on treatment with dil HCl gives a pugent
(d) black precipitate is formed.
smelling gas and a yellow ppt. The salt gives green
38. NO O2 A D fame when burnt. The salt solution gives a yellow
NaOH ppt. with potassium chromate. The salt is
B+C (a) BaS2O3 (b) PbS2O3
A,B,C,D are respectively :- (c) CuSO4 (d) NiSO4
(a) NO2, N2O3, NaNO2, NaNO3 45. A light blue coloured compound (a) on heating
(b) N2O3, NO2, NaN3 , NaNO3 gives a black compound (b) which reacts with
glucose to give red compound (c). (a), (b) and (c)
(c) NO2, NaNO2, NaNO3, N2O4
are respectively.
(d) NO2, N2O4, NaNO2, NaNO3
(a) CuSO4.5H2O
39. K2[Ni(CN)4] solution on treatment with sodium
(b) Cu(OH)2, Cu2O,CuO
hydroxide and bromine water followed by heating
gives a black residue (A). The compound A is (c) Cu(OH)2, CuO, Cu2O
(a) NiO (d) [Cu(NH3)]SO4, CuO, Cu2O
(b) NaCN 46. Which of the following oxide gives hydrogen
peroxide on reaction with a dilute acid :-
(c) Ni2O3
(a) PbO2 (b) Na2O2
(d) None of these
(c) MnO2 (d) TiO2

www.jeebooks.in
QUALITATIVE SALT ANALYSIS 4.7
47. A + HCl  B + HNO3 precipitate B is soluble 55. A metal ion ‘X’ reacts with NaOH to gives a white
ppt.
ppt. Precipitate can be oxidised by 4-6 drops of
in hot water (33.4 gl– at 100°C) conc. H2O2 , when yellowish brown colour solution
A + KI  Yellow ppt. Identify ‘A’ is formed. The metal ion ‘X’ is :-
(a) Hg2+ (b) Bi3+
(a) Pb(NO3)2 (b) AgNO3 (c) Cu2+ (d) Pb2+
(c) Hg2(NO3)2 (d) None of these 56. Which of the following metal sulphide is soluble in
48. M + N2  Nitride  both hot and conc. HNO3 and KCN :-
H 2O
 NH3 in above reaction
metal M may be :- (a) CuS (b) CdS
(a) only Li (b) only Mg (c) Ag2S (d) PbS
(c) only Cs (d) Li and Mg both 57. Which of the following metal cation gives yellow
49. On the electrolysis of HCl, hydrogen gets liberated ppt. with pyrogallol (10% freshly prepared) :-
at :- (a) Pb (b) Cu2+
(a) Anode (b) Cathode (c) Bi+3 (d) All the correct
(c) In air (d) Cannot be defined 58. Mixing of which will result in a white ppt. which
50. A + moderately concentrated (8M) HNO3  turns blue on oxidation :-
B + NO + S + H2O; (a) CuSO4 + HgCl2
B + NH4 OH  Deep blue solution; Identify ‘B’ (b) MgCl2 + Cu
(a) Pb(NO3)2 (b) Bi(NO3)3 (c) Fe2(SO4)3 , K3[Fe(CN)6]
(c) Cu(NO3)2 (d) Cd(NO3)2 (d) FeSO4 , K4[Fe(CN)6]
51. Which of the following thermal decomposition 59. With ‘X’ cation [Fe(CN)6]4– gives Prussian blue
yields a basic as well as an acidic oxide :- colouration due to the formation of ferr-ferro
(a) Na2CO3 (b) CaCO3 cyanide, Fe4[Fe(CN)6]3 , while with NH4SCN,
(c) NaNO3 (d) All the correct ‘X’ cation gives .........
Na 2C 2O 4 solution (a) Deep red, [Fe(H2O)5(SCN)]2+
52. CaCO3(s) + AcOH  
Comment on the product of this reaction :- (b) Blue , Fe4[Fe(CN)6]3
(a) CaCO3 remains unaffected (c) Brown, [Fe(H2O)5(NO)]2+
(b) CaC2O4 will be precipitated as white ppt. (d) Green, [Cr(NH3)6]3+
(c) Ca(OAc)2 will be precipitated as white ppt. 60. Turnbull’s blue is formed when Fe+2 ions are added
to K3[Fe(CN)6]. Turnbull’s blue is :-
(d) Clear solution
53. HCO2– and CH3CO2– ion can be distinguished by :- (a) K2Fe[Fe(CN)6]
(a) Ethanol in conc. H2SO4 (b) Fe3[Fe(CN)6]2
(b) HgCl2 (c) Fe2[Fe(CN)6]
(c) FeCl3 (d) All of these
(d) AgNO3 61. Find the ion that can be precipitated by the reagents
54. Addition of SnCl2 to HgCl2 gives ppt. of :- NH4Cl and aqueous NH3/NH4OH.
(a) Black turning to white silky (a) Ca2+
(b) White silky turning to greyish black (b) Al3+
(c) White silky turning to red (c) Mg2+
(d) Black silky turning to green (d) Zn2+

www.jeebooks.in
4.8 QUALITATIVE SALT ANALYSIS
62. Anion (A1) / Acid radical Cation (B1) / Basic radical 67. X + H2S 
HCl Y ; Y + (NH 4 )2 S  Soluble;
ppt. 2
(YAS)
conc. HSO
2 4 NaOH ‘X’ may contain
Gas I Gas II (i) As3+ (ii) Sb3+
Gas I and Gas II reacts to give white fume. (iii) Sn2+ (iv) Cd2+
A1 and B1 are respectively :- (a) Only (iv) (b) (ii) or (iv)
(a) SO32– , Ca2+ (b) S2– , Ba2+ (c) (i), (ii) or (iii) (d) Any of the four
(c) I– , Ag+ (d) Cl– , NH4+ B + CO + H O ;
68. A + CH3COOH  (soluble) 2 2
63. Idenfity wrong statement with CuSO4 :-
(a) CuSO4 reacts with KI to give violet vapour I2 B + (NH4)2C2O4 White ppt.
which further reacts with I2 to give brown A and B may contain
solution of KI3. (a) Ni2+ (b) Ba2+
(b) CuSO4 on heating give CuO (c) Sr2+ (d) Ca2+
(c) CuSO4 reacts with KCl to give greenish yellow 69. Which of the following oxide of lead satisfy the
vapour Cl2 observation?
(d) It’s tartarate complex reacts with NaOH and (1) It is an oxidising agent
glucose to give reddish brown precipitate of (2) It is insolule in acid
Cu2O.
(3) In ice cold medium when conc.HCl is added
64. A white powder (a) when strongly heated it gives
to the oxide, yellow hexa-chloroplumbate
off a colourless, odourless gas (b) which turns lime
(IV) ion[PbCl6]–2 is formed.
water milky (c). A on strong heating give solid
residue (e) which is yellow when hot but turns (a) PbO (b) PbO2
white on cooling A is :- (c) Pb2O3 (d) PbO3
(a) Ag2CO3 (b) PbCO3 70. Arrange the following hydroxy compound in order
(c) HgCO3 (d) ZnCO3 of increasing acid strength
65. When can produce Rinmann’s green colour with CrO2(OH)2(I) ; Cr(OH)2 (II); Cr(OH)3(III)
cobalt nitrate Co(NO3)2 solution :- (a) I < II < III (b) II < III < I
(a) ZnO (b) ZnSO4 (c) III < II < I (d) III < I < II
(c) 3Zn(OH)2 . ZnCO3 (d) All 71. A mixture contains maganous sulphide and cobalt
66. A yellow solid (a) is soluble in water. Its aqueous sulphide.Dilute hydrochloric acid is added. Which
solution turns blue litmus red. Aqueous solution of of the following statement is correct?
A in strong heating gives a dark brown residue (b) (a) Both are soluble in dil HCl
and a compound C which gives white fumes with (b) Both are insoluble in dil HCl
NH3 gas. When dry Cl2 gas is passed over a
(c) MnS is soluble in HCl while CoS is insoluble
heated mixture of (b) and carbon, compound D is
formed. D absorbs water vapours to give (d) CoS is soluble in HCl while MnS is insoluble
compound A. A is :- 72. To a solution containing metal cations A and B,
(a) FeCl2 K2CrO4 is added separately when A gives red
(b) AlCl3 coloured precipitate while B gave a yellow
precipitate. A and B are
(c) FeCl3 . 6H2O
(a) Cu+2, Pb+2 (b) Ag+,Ba+2
(d) Fe2O3 . H2O
(c) Zn+2,Cu+2 (d) Pb+2Cu+2

www.jeebooks.in
QUALITATIVE SALT ANALYSIS 4.9
73. KI gives a precipitate with all the cations given 80. In the previous question, further if the precipitate
below. Choose the cation turns black with NH4OH, it is
(a) Ag+, Hg22+, Pb+2 (b) Cu+2, Zn+2, Ni+2 (a) PbCl2 (b) AgCl
(c) Na+,Ca+2, Mg+2 (d) Ag+,Ca+2, Sr+2 (c) Hg2Cl2 (d) (b) or (c)
74. NH4SCN can be used to test,one or more out of 81. Iron is precipitated as Fe(OH)3 in the third group
Fe+3, Co+2 and Cu+2 and not as Fe(OH)2 because
(a) Fe+3 only (b) Co+2, and Cu+2 (a) Fe(OH)2 is light green
(c) Fe+3, Cu+2 (d) all (b) Fe(OH)3 is reddish brown
75. A light coloured crystalline solid A has 27.55% (c) Fe(OH) 2 is partialy soluble and is not
H2O. A gives the following reactions. completely precipitated
BaCl2
(i) A 
Solution
white precipitate insoluble in (d) Fe(OH)2 is oxidant
conc. HNO3 82. Cu2+ ions will be reduced to Cu+ ion by addition
K 3 [Fe(CN)6 ]
(ii) A   a dark- blue precipitate of an aqueous solution of
Solution
K 2HgI4 (a) KF (b) KCl
(iii) A 
NaOH
Brown precipitate
(c) KI (d) KOH
Identify A
83. Which of the following gives black precipitate on
(a) Fe2(SO4)3 (NH4)2SO4.8H2O passing H2S through it
(b) FeSO4(NH4)2SO4.6H2O (a) acidfied zinc nitrate solution
(c) (NH4)2FeSO4.6H2O (b) ammoniacal barium chloride solution
(d) FeSO4(NH4)2SO4. 5H2O (c) magnesium nitrate solution
76. Which of the following compounds change the (d) copper nitrate solution
salmon-coloured precipitate of zinc nitroprusside
84. A mixture containing Cu2+ and Ni2+ can be
into red ?
separated for identification by
(a) Na2CO3 (b) NaHCO3
(a) passing H2S in acid medium
(c) Moist-SO2 (d) Moist-CO2
(b) passing H2S in alkaline medium
77. The sulphide which isinsoluble in both ammonium
(c) passing H2S in neutral medium
sulphide and HNO3 is
(d) all
(a) Bi2S3 (b) CuS
(c) HgS (d) FeS 85. Identify the correct order of solubility of Na2S,CuS
and ZnS in aqueous medium.
78. To an acidified Cr2O72–solution,Na2O2 is added,a
blue colour is observed. The blue colour changes (a) CuS > ZnS > Na2S
to green when H2SO4is added.What are these blue (b) ZnS > Na2S > CuS
and green coloured compounds? (c) Na2S > CuS > ZnS
(a) Cr2O3,Cr2(SO4)3 (b) CrO3,CrO(O2)2 (d) Na2S > ZnS > CuS
(c) CrO3,Cr2(SO4)3 (d) CrO(O2)2, Cr2(SO4)3 86. Which of the following reagent cause the separation
79. A colourless solution contains a metal nitrate. A of precipitate of Fe(OH)3 and Cr(OH)3 in IIIrd
little solution of sodium chloride is added to it when group analysis ?
a cloudy white precipitate appears.Idenfity the (a) dil H2SO4
precipitate (b) Aq.NH3
(a) PbCl2 (b) AgCl (c) NaOH +O2
(c) Hg2Cl2 (d) any of the three (d) NaOH/H2O

www.jeebooks.in
4.10 QUALITATIVE SALT ANALYSIS
87. In the qualitative analys is,NH4Cl is added before 95. On passing H2S gas in II group, sometimes the
NH4OH to solution turns milky. Itindcates the presence of
(a) decrease OH– ions conc. (a) oxidizing agent (b) acdic salt
(b) increase OH– ion conc. (c) thiosulphate (d) reducing agent
(c) form HCl 96. Sodium sulphide react with sodium nitropruside to
(d) form complexes of group II radicals form a purple coloured compound.During the
88. Mercurous ions is represent as reaction,the oxidation state of iron
(a) Hg+ (b) Hg2+ (a) change from+ 2 to + 3
(c) Hg3+ (d) Hg22+ (b) changes from +3 to +2
89. Mark the correct statement (c) change from +2 to +4
(a) I group basic radical precipitate as chlorides (d) remain unchanged
(b) IV group basic radical precipitate as sulphides 97. An inorganic salt when heated evolves coloured
gas which bleaches moist litmus paper. The evolved
(c) V group basic radical precipitate as carbonates
gas is
(d) all the above statements are corect
(a) NO2 (b) Cl2
90. Which of the following is not precipitated as
sulphides by passing H2S in presence of conc. (c) N2O (d) I2
HCl? 98. When hot and concentraed KOH reacts with Cl2,
(a) Copper (b) Arsenic the product is
(c) Chromium (d) Lead (a) KClO (b) KClO4
91. The cation present in slightly acidic solution are (c) KClO3 (d) None of these
Fe3+, Ni2+ and Cu2+ . Which reagent, when added 99. In the reaction
excess to this solution will identify and separate 2KI +H2O+O3  2KOH +O2 + A
Fe3+ in one step the compound A is:
(a) 2M HCl (b) 6 M NH3 (a) KIO3 (b) I2O5
(c) 6 MNaOH (d) H2S gas (c) HIO3 (d) I2
92. During analysis of basic radicals of a mixture 100. An aqueous solution contains Hg2+, Hg22+, Pb2+
contaning cations of group V, if a solution of and Cd 2+ . The addition of 2(M) HCl will
NaHCO3 is added instead of Na2CO3 along with precipitate
NH4OH this would result in formation of
(a) Hg2Cl2 only.
(a) white ppt (b) yellow ppt
(b) PbCl2 and CdCl2
(c) brick red ppt (d) no ppt at all
(c) PbCl2 and HgCl2
93. The metal whose salts do not give the borax bead
test is - (d) PbCl2, Hg2Cl2 and HgCl2
(a) Cr (b) Ni 101. White precipitate (X) + O2 Brown precipitate
(c) Pb (d) Mn (Hydroxide)
94. In qualitative analysis of group I radicals, a white (X) is -
precipitate is formed which is insoluble in boiling (a) Mn(OH)2
water but when treated with NH4OH it turns black, (b) Fe(OH)2
the precipitate may be. (c) Ni(OH)2
(a) PbCl2 (b) AgCl (d) Cr(OH)3
(c) HgCl2 (d) Hg2Cl2

www.jeebooks.in
QUALITATIVE SALT ANALYSIS 4.11

102. Green precipitate (X) + H2O2 + OH Yellow solution 111. Passing H2S gas in a solution mixture containing
(Hydroxide) Co2+ , Zn2+ , Pb2+ , Cu2+ in an acidified solution
(X) is- precipitates -
(a) Mn(OH)2 (b) Fe(OH)2 (a) CoS, PbS (b) ZnS, CoS
(c) Ni(OH)2 (d) Cr(OH)3 (c) PbS, CuS (d) CuS, ZnS
2+
 O
112. Aqueous solution of Ni + HCl(excess) P1
103. Green precipitate  Black precipitate (X)
Aqueous solution of Zn2+ + HCl(excess) P2
(Hydroxide)
P1 and P2 differs in -
(X) is-
(a) Shape (b) Magnetic moment
(a) Mn(OH)2 (b) Fe(OH)2
(c) Hybridisation (d) Colour
(c) Ni(OH)3 (d) Fe(OH)3
( NH 4 )2 S
104. Which ion gives yellow precipitate on addition of 113. A metal ion   Pink precipitate
KNO2 solution ? Metal ion can be -
(a) Ni2+ (b) Zn2+ (a) Mn2+ (b) Ni2+
(c) Mn2+ (d) Co2+ (c) Co2+ (d) both (B) and (C)
105. In a mixture containing Al3+ , Ba2+ , Mg2+ , Cr3+ ; if ( NH 4 )2 S
114. A metal ion   Black precipitate
NH4Cl is added in ammonical medium, the ion pair
get precipitated is Metal ion can be -
(a) Ba2+ , Cr3+ (b) Al3+ , Mg2+ (a) Mn2+ (b) Ni2+
(c) Al3+ , Cr3+ (d) Mg2+ , Cr3+ (c) Co2+ (d) both (B) and (C)
106. Correct Ksp value of alkaline earth sulphate 115. A compound gives scarlet red precipitate with KI
(a) BaSO4 > SrSO4 > CaSO4 solution. The compound may have -
(b) SrSO4 > CaSO4 > BaSO4 (a) Fe2+ (b) Bi3+
(c) BaSO4 > CaSO4 > SrSO4 (c) Hg2+ (d) Hg22+
(d) CaSO4 > SrSO4 > BaSO4 116. A compound gives green precipitate with KI
solution. The compound may have -
107. Chromate of which metal ion is most soluble -
(a) Fe2+ (b) Bi3+
(a) BaCrO4 (b) CaCrO4
(c) SrCrO4 (d) All (c) Hg2+ (d) Hg22+
108. Oxalate of which metal ion is least soluble - 117. Which metal ion gives green precipitate with KCN
solution -
(a) BaC2O4 (b) CaC2O4
(a) Fe2+
(c) SrC2O4 (d) All
(b) Ag+
109. Ni2+ + NH3 product(soluble)
(c) Hg2+
Product is -
(d) Ni2+
(a) Brown solution
(b) sp3d2 hybridised 118. FeCl3 + sodium acetate solution  deep red
colouration (X1)
(c) Inner orbital octahedral complex
X1 is -
(d) Diamagnetic
(a) Fe(OH)2CH3CO2
110. Zn2+ + NH3 product(soluble)
(b) [Fe(OH)2(CH3CO2)4]3–
Product is -
(c) [Fe3(OH)2(CH3CO2)6]+1
(a) Colourless solution (b) sp3 hybridised
(d) [Fe(OH)(CH3CO2)]+1
(c) Diamagnetic (d) All

www.jeebooks.in
4.12 QUALITATIVE SALT ANALYSIS
119. FeCl3 + sodium acetate solution  7. When salt solution ‘A’ is treated with dilute HCl, it
deep red colouration (X1) gives off gas ‘B’ which is passed through a filter
paper, moistened with potassium iodate and starch
X1 
boiling
 X2 (reddish brown precipitate)
solution turns deep blue colouration. The correct
X2 is - statement about ‘A’ and ‘B’ is /are :-
(a) Fe(OH)2CH3CO2 (a) A = Na2SO3 & B = SO2
(b) [Fe(OH)2(CH3CO2)4]3– (b) A = Na2SO3 & B = CO2
(c) [Fe3(OH)2(CH3CO2)6]+1 (c) A = Na2S & B = H2S
(d) [Fe(OH)(CH3CO2)]+1 (d) A = NaHSO3 & B = SO2
120. Red colouration is not observed when Fe2+ reacts 8. Observe the following experiment :-
with -
(a) NH4SCN (b) DMG
(c) 2,2’-bipyridyl (d) 1,10-phenanthrolin AgNO3 Na2SO3

EXERCISE # II Na2SO3 AgNO3


 One or More Than One Correct Answer : (I) (II)
1. Which of the following compound gives ‘X’ gas The correct statement about set up (I) and (II) are :-
with addition of dilute HCl. The gas ‘X’, is passed (a) In set up (I) initially no precipitate is formed
into lime water or baryta water , turns turbidity. but on addition of excess of reagent and
(a) Na2CO3 (b) Na2SO3 followed by boiling gives greyish back ppt.
(c) Na2S2O3 (d) BaCO3 (b) In set up (II) initially white precipitate is formed
2. Which of the following anion(s) gives white ppt. as well as on boiling gives greyish back ppt.
with CaCl2 in aqueous solution? (c) In set up (II) the precipitate is soluble is dilute
(a) CO32– (b) S2– nitric acid , and gives gas with suffocating
smell of burning sulphur which turns lime water
(c) SO32– (d) CHCOO–
milky.
3. Which of the following acid is/are used to evolve
(d) In set up (I) the precipitate is formed with
CO2 gas from carbonate salt?
excess AgNO3 which is soluble in ammonia
(a) dil. HCl (b) dil. CH3COOH solution.
(c) dil. HCN (d) dil. H3BO3 9. Which of the following compounds(s) gives green
4. Which of the following reagent(s) form ppt. with solution, when treated with K2Cr2O7 and dilute
Na2CO3 ? H2SO4 ?
(a) BaCl2 (b) CuSO4 (a) KNO3 (b) Na2S
(c) AgNO3 (d) HgCl2 (c) NaHCO3 (d) Na2SO3
5. The precipitate of Ag2CO3 dissolve in 10. CO2 & SO2 gas can be distinguished by
(a) dil. HNO3 (b) KCN solution (a) KMnO4, H+ (b) Ca(OH)2 solution
(c) NH3 solution (d) None of these (c) Baryta water (d) K2Cr2O7, H+
6. Which of the following interfering radical form 11. Which of the following compounds give precipitate
volatile product wich conc. H2SO4 ? with sulphide solution ?
(a) PO43– (b) F– (a) BaCl2 (b) KI3
(c) C2O42– (d) CO32– (c) Silver nitrate (d) Cd(OAc)2

www.jeebooks.in
QUALITATIVE SALT ANALYSIS 4.13
12. Which of the following(s) is /are required to give 17. Which of the following compounds give red azo
water soluble dye stuff, methylene blue in strongly dye with HNO2
acid solution with H2S. NH2.CH3COOH
N(CH3)2 SO3H
(a)
(a) (b)
SO3H
NH2 NH2.CH3COOH
(c) FeCl3 (d) FeCl2
(b)
13. Which of the following reagents are used to give
NH2
blue ring test of solution of Na2S2O3 ?
(c) (KMnO4 + dil. H2SO4)
(a) (NH4)2MoO4 (b) CuSO4
(d) (FeSO4 + dil. CH3COOH)
(c) conc. H2SO4 (d) [Ni(en)3](NO3)2
18. The addition of dilute HCl acid to a solid nitrite in
14. Which of the following statement is / are correct ? cold yields a transient, pale-blue liquid is obtained
(a) In presence of sulphate, sulphite can be due to presence of following compound(s) :-
detected by treating dil. HCl (a) free nitrous acid
(b) Both sulphite and sulphate form white (b) anhydride of nitrous acid
precipitate with BaCl2
(c) free nitric acid
(c) Both sulphite and carbonate can be detected
(d) condensed form of N2O5 & NO2
by treating acidified KMnO4
19. Which is the following reagent(s) are used to
(d) Both sulphite and carbonate can be detected
distuinguish nitrite and nitrate salt solution ?
by treating acidified K2Cr2O7
(a) Ag+ solution (b) FeSO4 + dil. H2SO4
15. Methylene blue is an indicator widely used in
acid-base titration, it is formed on mixing :- (c) KMnO4 (d) sulphamic acid
NH2 20. Which of following reagent(s) make distinction of
acetate from oxalates and formates ?
in ........... + .......... (a) BaCl2 (b) CaCl2
(c) HgCl2 (d) FeCl3
NMe2
21. Salt solution ‘X” + FeCl3(aq.) 
(a) Na2SO3 (b) Na2S
‘Y’(deep red colouration) + 2H+
(c) Fe2(SO4)3 (d) FeSO4
16. Select the correct statements :- /on boiling

(a) The hydrogen carbonates of the alkali metal


‘Z’ (Brownish red ppt.)
are soluble in water, but are less soluble than
the corresponding normal carbonates. Correct statement(s) about the above experiment
is / are :-
(b) Solution of carbonate forms reddish brown
precipitate basic mercury(II) carbonate (a) Aqueous solution of ‘X’ salt gives deep red
(3HgO.HgCO3) when react with Mercury(II) coluration, with FeCl3 owing to formation of a
chloride. complex ion ‘Y’ is [Fe3(OH)2(CH3COO)6]+.
(c) When AgNO3 react with sulphate ion no visible (b) Z is Fe(OH)2(CH3CO2)
change occurs initially because of the formation (c) On boiling the red solution of ‘Y’ gets
of soluble sulphitoargentate ion. decomposed into brownish red precipitate of
(d) On standing the precipitate BaSO3 is slowly ‘Z’ is basic iron (III) acetate.
oxidised to the sulphate. (d) Z is Fe(OH)3.

www.jeebooks.in
4.14 QUALITATIVE SALT ANALYSIS
22. Which of the following ions interfere chromyl 28. When solid NaF is treated with conc. H2SO4 in a
chloride test :- test tube and followed by warming, then dimeric
(a) NO3– (b) Br– form of gas ‘M’ is evolved, by which the test tube
(c) I– (d) NO2– acquires a greasy appearance on its walls after
hydrolysis which of the following compounds
23. KI(solid) + conc. H2SO4 (variable concentration) is /are fomrmed.
On warming (a) SiF4 (b) H2SiF6
(c) SiO2.H2O (d) H2SiO3
'X" + K2SO4 / KHSO4 + H2O
29. Salt soluton of 'X' + FeCl2 (aq.)
Here ‘X’ may be -
(a) KI3 (I2 + KI) (b) H2S 'Y' complex ion (deep red solution)
(c) S (d) SO2 on boiling

24. Cl (salt solution) + conc. H2SO4 'Z' [Brownish - red ppt.]
The correct statement(s) about the flow diagram
On warming
is/are :-
only 'X' (gas) + ..... (a) CH3COONa is the salt solution ‘X’
The correct statement about ‘X’ gas is :- (b) ‘Y’ is the [Fe(H2O)5(SCN)2+] ion
(a) It is recognized by its pungent odour and (c) ‘Z’ is the basic acetate of iron (III)
production of white fumes, on blowing across (d) Oxidation state of iron in ‘Y’ and ‘Z’ remains
the mouth of the test tube. constant.
(b) It is recognized by formation of white clouds 30. Which of the following compounds give white
of ammonia chloride when a glass rod precipitate with dilute HCl ?
moistened with ammonia solution is held near (a) Pb(NO3)2 (b) AgNO3
the mouth of the test tube.
(c) Na2S2O3 (d) CH3COONa
(c) It is recognised by turning litmus paper blue.
31. Which of the following anionic radicals give white
(d) ‘X’ gas is Cl2 with greenish yellow fumes. precipitate with magnesium nitrate reagent or
25. Which of the following compound gives white magnesia mixture ?
precipitate with silver nitrate solution ? (a) HPO42– (b) CO32–
(a) NaCl (b) Na2CO3
(c) CH3COO– (d) NO3–
(c) Na2SO3 (d) Na2S2O3
32. Which of the following compound(s) give white
26. Which of the following salts show chromyl chloride precipitate with silver nitrate solution ?
test ?
(a) Na2CrO4 (b) KSCN
(a) KCl (b) CrCl3
(c) KCN (d) Na2S2O3
(c) HgCl2 (d) FeCl3
33. An aqueous solution of a substance gives a white
27. When mixture of CuCO 3 . Cu(OH) 2(s) and precipitate on treatment with NaCl solution. Which
Pb(NO3)2(s) is added in dil. HCl solution -
dissolves on heating. When H2S gas is passed
(a) Produces coloured precipitate and colourless through the hot acidic solution, a black precipitate
solution. is obtained. The cations which are not associated
(b) Produce coloured solution & white ppt. to this test.
(c) Evolve CO2 gas (a) Mg2+ (b) Ag+
(d) Evolve NO2 gas (c) Hg22+ (d) Pb2+

www.jeebooks.in
QUALITATIVE SALT ANALYSIS 4.15
34. Depict correct flame colouration through cobalt 38. Following observation are given :-
glass for different ions. Unknown salt [X]
(a) For K+ : Crimson colour
NaOH
(b) For Ca2+ : Light green colour
(c) For Sr2+ : Purple colour White ppt.[Y]
(d) For Ba2+ : Bluish green colour
excess NaOH
35. Select the correct statement(s) :-
(a) Alkaline solution of potassium Soluble complex of [Y]
tetra-iodomercurate(II) gives brown ppt. with
ammonium chloride solution. H2O2 S2O8
2–

(b) On warming mixture of NH4Cl and NaOH


solution, evolved gas form deep blue complex Black precipitate [Z]
with CuSO4. The correct statement is / are :-
(c) On warming mixture of NH4Cl and NaOH (a) White precipitate of Y is Pb(OH)2
solution, evolved gas turns Hg(NO3)2 solution (b) The precipitate [Y] dissolves in excess NaOH
black.
and forms tetrahdroxido plumbate(II) ion
(d) NH4+ + NaOH  
 only gas ‘X’ ; evolved (c) Soluble complex of [Y] gets oxidised by H2O2
gas ‘X’ turns moistened blue litmus paper red. into black ppt.(Z) of PbO2
36. Which of the following reagents give white (d) Black precipitate of [Z] is (Hg + HgO)
precipitate with MgSO4 solution , but readily 39. Which of the following cations form precipitate with
soluble in ammonium salts ? K4[Fe(CN)6] ?
(a) NaOH (a) Zn2+
(b) (NH4)2CO3 (b) Cu+2
(c) Na2CO3 (c) Cd2+
(d) Na2HPO4
(d) Al3+
37. Which of the following reaction is / are represented
40. How many of the following reactions produce blue
correctly ?
coluration or blue ppt.
'A '  
(a) Pb(OAc)2 + dil. H2SO4  (white NH3 in
(a) CuSO4 solution  
ppt.) excess

K 4 [ Fe(CN)6 ]
soluble in hot and conc. H2SO4 (b) CuSO4  
(b) Hg(NO3)2 + Na2CO3(aq)  excess
(c) NiCl2 solution 
NH 4OH


'B'    black ppt. K 3 [Fe(CN) 6 ]
( yellow ppt.) (d) FeSO4  

'C '  41. The correct statement about Cr2+ and Mn3+ is /are
(c) 2AgNO3 + 2NH3(aq.) 
 H 2O
 (brown ppt.)
(a) Cr2+ is a reducing agent
soluble in NH3 (b) Mn3+ is an oxidising agent
+
(d) 4[Ag(NH3)2] + N2H4 . H2SO4  (c) Both Cr2+ and Mn3+ exhibits d4 configuration
4‘D’ () + N2() + 6NH4+ + 2NH3 + SO42– (d) When Cr2+ is used an reducing agent, the Cr3+
‘D’ soluble in conc. HNO3 attains d5 electronic configuration

www.jeebooks.in
4.16 QUALITATIVE SALT ANALYSIS
42. Which of the following statements is/are incorrect? 49. MnO2 + conc. H2SO4
X3(Greenish yellow gas)
(a) In thiourea test for nitrite, a green coloured
Salt (X1) Flame Test Golden yellow flame
soluton is obatined.
(b) It is not necessary to carry out the chromyl AgNO3 K2Cr2O 7 2 4
X4(Reddish brown gas)
chloride test in a dry test tube.
X2(white ppt. insoluble
(c) In Pb(NO3)2, the brown ring test can be in HNO)3
performed with its water extract. Correct statements are :-
(d) Suspension of CdCO3 gives black precipitate (a) X2 is AgBr (b) X3 is Cl2
with sodium sulphide solution. (c) X1 is NaCl (d) X4 is Br2
43. Which of the following salt(s) produces red 50. Which of the following pair of species can be
coloration on flame :- separated by HCl ?
(a) BaCl2 (b) SrCl2 (a) Hg(NO3)2 and Hg2(NO3)2
(c) NaCl (d) LiCl (b) AgNO3 and Hg(NO3)2
44. HgS is insoluble in :- (c) Fe(OH)3 and Al(OH)3
(a) Water (d) HgS and MnS
(b) Hot dilute nitric acid 51. In which reaction(s) blood red coloration is
(c) Alkali hydroxides observed in product :-
(d) Ammonium sulphide (colourless) H 2O 2
(a FeSO4  KCN
.....   Product )
45. Which of the following ppt. is soluble in both aqua KCN Fe 3

regia and sodium disulphide (yellow) :- (b Na 2S2O3   .....   Product )
Ferric
(a) (Hg + HgS) (b) HgS (c) NH 4SCN 
alum
 Product
(c) CuS (d) Ag2S (d) NO2  thiourea Fe 2
.....   Product
46. Using ammonia solution we can distinuish :-
52. Which is dissolved by HCl ?
(a) Cu2+. Bi3+ (b) Cu2+. Cd2+
(a) ZnS (b) MnS
(c) Bi3+. Cd2+ (d) Ba2+. Ca2+
(c) BaSO3 (d) BaSO4
47. Compound ['P'] + KI 53. When H2S gas is passed through HCl containing
aqueous solution of CuCl2 , HgCl2 , BiCl3 , and
Precipitate of 'Q" CoCl2 , it precipitate out :-
excess KI (a) CuS
Soluble complex of 'R' (b) HgS
Compound ‘P’ may be :- (c) Bi2S3
(a) CuSO4 (b) Hg(NO3)2 (d) CoS
(c) Bi(NO3)3 (d) CdSO4 54. An aqueous solution of FeSO4 . Al2(SO4)3 and
chrome alum is heated with excess of Na2O2 and
48. Which of the following reaction(s) give(s) blue
filtered. Which can be obtained as product :-
precipitate ?
(a) Green filtrate
(a) Hg2+ + Co2+ + 4SCN–(aq.) 
(b) Green Residue
(b) CuSO + excess NH3 
(c) Yellow filtrate
(c) CuSO4 + excess NaOH 
(d) Brown residue
(d) Cu2+ + KSCN 

www.jeebooks.in
QUALITATIVE SALT ANALYSIS 4.17
55. Acidified Chromate solution + H2O2 61. n-factor of the metal centre in aqueous
In presence permanganate solution

or organicsolvent
Product (P)
Correct option :- (a) 3 in neutral medium (b) 5 in neutral medium
(c) 3 in alkaline medium(d) 5 in acidic medium
(a) P is permanent deep blue coloured compound 
(b) Oxidation state of Cr in product P is +3 62. 2Cr 3  3O Br  10OH   2CrO 24  3Br   5H 2O
(c) Oxidation state of Cr in product P is +6 Which element change its oxidation state in this
reaction
(d) Product has peroxy linkage
(a) Cr in Cr3+ (b) O in OH–
56. How many reactions, products are correctly given :-
(c) O in OBr– (d) Br in OBr–
CrO
(a) RCH2OH  3
in conc.H 2SO 4
 RCO2H 63. At 25°C, stability order
PCC
(b) RCH2OH   RCH = O (a) CuCl > CuCl2
(b) CuSCN > Cu(SCN)2
(c) R2CHOH 
H 2CrO 4 R2 C = O
(c) CuCN > Cu(CN)2
(d) R3COH 
H 2CrO 4 R2C = O + ROH (d) Cu2(NO3)2 > Cu(NO3)2
57. For which of the ammonium salt, when they 64. Which ion gives colourless solution with excess
undergo heating , non redox reaction can take NH4OH solution ?
place :- (a) Ag+ (b) Cu2+
(a) (NH4)2CO3 (b) NH4NO3 (c) Ni2+ (d) Cd2+
(c) (NH4)2Cr2O7 (d) CH3CO2NH4 65. Ni2+ + CN–  Product (soluble)
58. Which reactions are not applicable for obtaining (excess)
greenish yellow colour Cl2 :- Product is -
(a) NaCl + conc. H2SO4  (a) Yellow colour solution
(b) NH3 + HCl  (b) Square planar
(c) NaCl + MnO2 + H2SO4  (c) dsp2 hybridisation of central atom
(d) Diamagnetic
(d) Au + H+ + NO3– + Cl– 
59. The equilbrium Al(OH)3 + OH–  [Al(OH)4]– EXERCISE # III
in aqueous medium shifts towards the left in the  Linked Comprehension Type :
presence of : Paragraph for Q.1 to Q.3
(a) NH4Cl ‘X’ + AgNO3 ‘Y’ (white ppt.)
(b) dilute H2SO4 (not in excess) ‘Y’ is soluble in nitric acid and ammonia. The ppt.
(c) NaOH becomes yellowish brown upon addition of excess
(d) carbon dioxide gas reagent owing to the formation of ‘Z’.
60. Correct statements are :- If aqueous suspension of ‘Y’ is boiled then it gives
same product ‘Z’.
(a) The compound formed in the borax bead test
of Cu2+ ion in oxidising flame is Cu(BO2)2 1. Here ‘X’ and ‘Y’ are :-
(a) CO32– & Ag2O (b) CO32– & Ag2CO3
(b) NH4NO3 leaves no residue on heating
(c) HCO3– & Ag2CO3 (d) HCO3– & Ag2O
(c) Mg when burnt in air and the ash on moistening
smells of ammonia 2. Here ‘Z’ is
(d) CuSO4 decolourises on addition of excess (a) Ag2CO3 (b) Ag
KCN, the product is [Cu(CN)4]3– (c) Ag2O (d) All of these

www.jeebooks.in
4.18 QUALITATIVE SALT ANALYSIS
Paragraph for Q.3 to Q.4 Paragraph for Q.8 to Q.10
 excess SO2 Na2CO3
Solid(A) Acidicgas(B)+ Neutral liquid(C) Step-I : Na2CO3(aq.) 'A' 'B'
(16.8gm) (4.4gm) (1.8gm)
BaCl2 /O2
+ Solid (D) Step-II : 'B' 'D' 'E'
BaCl2 sol.
Aqueous solution of (D) KMnO4/H
+
in excess
(Solid ‘E’ of 19.7 gm)
'C' BaCl2 'E'
3. Identify ‘A’ ? 8. Compound ‘E’ and ‘B’ are
(a) KHCO3 (b) NaHCO3 (a) NaHCO3 and Na2CO3
(c) K2CO3 (d) Na2CO3 (b) NaHCO3 and Na2SO3
4. Identify ‘E’ ? (c) NaHSO3 and Na2SO3
(a) BaCO3 (b) Na2CO3 (d) NaHSO3 and Na2CO3
(c) CaCO3 (d) K2CO3 9. Here ‘e’ is insoluble in dilute HCl will be
Paragraph for Q.5 to Q.7 (a) BaCO3 (b) BaSO3
Step-(I) : Salt ‘A’ + Zn + dil H2SO4  (c) BaSO4 (d) BaS2O3
‘X’(g) (Rotten egg’s smell) 10. The compound ‘D’ treated with dilute H2SO4 gives
filter paper moistened gas with pungent smell and turns lime water milky
Step-(II) : ‘X’ (gas) + with Pb(OAc)2  turbidity. The compound ‘D’ is -
‘Y’ (Black colour) (a) BaCO3 (b) BaSO3
Step-(III) : ‘X’(gas) + H2SO4  ‘Z’ (white turbidity) (c) BaSO4 (d) Na2SO3
5. Salt ‘A’ and gas ‘X’ may be? Paragraph for Q.11 to Q.12
boil
(a) Na2SO3 and SO2 (b) Na2S and SO2 SO3–2 + (A) (B)
(c) Na2SO3 and H2S (d) NaHSO3 and SO2
Br2 water dil. HCl
6. Here ‘Y’ & ‘Z’ are
(a) PbS & PbSO4 carbon (C) gas + (A)
(D) (E)
(b) PbS & S
(c) ZnS & PbSO4 dil. HCl
(d) ZnS & ZnSO4 (F) gas C(gas) (A)
7. The incorrect statement about the above 11. Identify type of reaction between F(gas) with C(gas) :-
experimental observation is/are :-
(a) Ion exchange
(a) In step-1, salt ‘A’ gets reduced to ‘X’ gas
(b) Disproportionation
which turns lime water milky.
(c) Comproportionation
(b) ‘X’ gas also turns filter paper moistened with
cadmium acetate to yellow coloration spot. (d) None of these
(c) ‘Y’ is insoluble in water but with O3 it gets 12. Identify D
completely oxidised and changed to white (a) S–2
colour precipitate. (b) SO3–2
(d) In step-3, white turbidity is formed due to (c) S2O3–2
redox reaction (comproportionation) of ‘X’ (d) SO4–2
gas & H2SO3

www.jeebooks.in
QUALITATIVE SALT ANALYSIS 4.19
Paragraph for Q.13 to Q.15 Paragraph for Q.18 to Q.19
Salt solution 'A' + KCN KSCN + 'B' (solution) The compound ‘A’ is colourless solid. It gives following
tests.
Fe3+
Step-I : The addition of compound ‘A’ solution of
Blood red colouration (C) KI, followed by acidification with dilte H2SO4 , result

in the liberation of compound ‘B’, which may be
F (excess) identified by blue colour produced with starch paste.
Colourless (D) Step-II : The acidified KMnO4 solution decolourized,
13. Identify ‘A’ and ‘B’ :- by a solution of compound ‘A’ , but no gas is evolved.
(a) SO32– & (S + SO42–) (b) S2O32– & SO32– 18. Compound ‘A’ and ‘B’ are
(c) S2– & S2O32– (d) S2– & SO32– (a) Na2S2O3 & S (b) NaNO2 & NO
14. The magnetic moment of complex ‘C’ is (c) NaNO2 & NO2 (d) Na2S2O3 & I2
(a) 1.7 B.M. (b) 3.87 B.M. 19. Which of the following reaction(s) is / are
incorrectly matched about step-I and step-II.
(c) 2.83 B.M. (d) 5.92 B.M.
(a) 2NaNO3 + 3KI + 2H2SO4 KI3 + 2NO
15. The compound ‘D’ is + 2H2O + 2SO42– + 2K+ + 2Na+
(a) Na3[FeF6] (b) [Fe(H2O)5(SCN)]2+ (b) 5NaNO 2 + 2KMnO 4 + 3H 2 SO 4 
(c) Na3[Fe(CN)3F3] (d) FeS4O6 5NaNO3 + 2MnSO4 + K2SO4 + 3H2O
Paragraph for Q.16 to Q.17 (c) Step-(I) & Step-(II) both give redox reation
(I) (Na2S + I2 + Na2SO3) (d) Step-(I) & Step-(II) both give precipitation
boil reation
Compound [A] + 2NaI
Paragraph for Q.20 to Q.21
(II) A(aq.) + AgNO3 white ppt. [B] (unstable)
Observe the following experiment
turning dark on standing dil. HCl S() + 'B' (acid)
'A'
Black compound (C) [Salt solution]
(III) A(aq.) + FeCl3 dark-violet colouration [E] Set-I Set-II
on warming
brought filter paper
moistened with
[F] colour disappears on standing K2Cr2O7/H
+

'D'(green) 'C' (gas)


16. Compound ‘A’ is :-
20. The incorrect statement about set-II is / are :-
(a) Na2SO4 (b) Na2S2O3
(a) The acidified liquid soon becomes turbide
(c) Na2S4O6 (d) Na2SO3 owing to formation of sulphur and H2SO4.
17. The correct statement about the above observation (b) The sulphur first forms a colloidal solution, which
is / are :- is gradually coagulated by free acid present side
(a) Aqueous solution of ‘A’ on treatment with reactions also occur giving rise to thionic acids.
AgNO3 gives ‘B’ white ppt. of Ag2S2O3 (c) On warming the solution of set up-II, the ‘C’
(b) Aqueous solution of ‘A’ on treatment with gas evolved is H2S. Which is recognised by
AgNO3 gives white ppt. of Ag2SO3 its action upon filter paper moistened with
acidified K2Cr2O7.
(c) Compound ‘C’ is Ag2S which is black in colour
(d) On warming the solution of set up-II, the ‘C’
(d) The violet coloration of compound [E] is gas evolved is SO2. Which is recognised by
[Fe(S2O3)2]– , which turns into Fe2+ and its action upon filter paper moistened with
S4O62–. acidified K2Cr2O7.

www.jeebooks.in
4.20 QUALITATIVE SALT ANALYSIS
21. Which of the following reaction is / are correctly 25. The incorrect statement about above experiment is
matched about set up - I and set up -II. (a) ‘Y’ is AgCl and soluble in dil. HNO3
(a) Na2S2O3 + 2HCl  (b) ‘Z’ is Ag3AsO3
S() + H2SO3 + 2NaCl (c) ‘Y’ is soluble in Na 2 S 2 O 3 and forms
 Na3[Ag(S2O3)2]
(b) Na2S2O3 + 2HCl (d) AgCl forms fulminating silver (Ag3N) with
S() + SO2() + 2NaCl + H2O NH3 on standing for a long time
(c) 3SO2 + Cr2O72– + 2H+ 2Cr3+ + 3SO42– + H2O Paragraph for Q.26 to Q.27
 (I) Borax + conc. H2SO4  'A'
(d) S + H2SO3 H2S2O3
(white fumes)
Paragraph for Q.22 to Q.23 conc. HCl
[A] () (white ppt.)
Solid 'X' salt + MnO2 (solid)
(II) Borax + conc. H2SO4 + CH3OH
conc. H2SO4 added dropwise on ignition in
and on warming porcelain basin

'Y' (coloured gases) ['B' burns with green edge flames]


26. Compound ‘A’ is :-
+ [KI - starch paper]
(a) HCl (b) H3BO3
[Turns KI-starch paper blue] (c) SO2 (d) B2O3
22. Here salt ‘X’ may be :- 27. Correct statement about ‘B’ is / are :-
(a) NaCl (b) NaBr (a) green edge flame occurs due to formation of
methyl borate
(c) NaI (d) None of these
(b) ‘B’ is B(OCH3)3
23. Which of the following statement(s) are correct ?
(c) On burning , the paste of borax + CaF2 and
(a) MnO2 oxidizes halides ions into dihalgon
conc. H2SO4 on bunsen flame, gives same
(b) Reducing nature of halides ions follow the green edged flame.
order, F– < Cl– < Br– < I– (d) Borax gets hydrolysed into boric acid in acidic
(c) ‘Y’ gas can be I2, which turns KI-starch paper medium and then form ester of methyl borate
into blue colour. with methyl alcohol.
(d) ‘Y’ gas can be Cl2 and Br2 which oxidises of Paragraph for Q.28 to Q.29
KI to I2 & turns starch paper blue. KMnO4 + 'X' salt solution
Paragraph for Q.24 to Q.25
BaCl2 (aq.) is added
– Na3AsO 3
X (salt solution) + AgNO3 'Y'ppt. 'Z' (yellow ppt.)
('Y' pink ppt. + pink solution)
Soluble in NHand
3

Na2S 2O3 H2O2 is added

24. ‘X’ salt is :- ('Z' pink ppt. + colourless solution)


(a) NaCl 28. The compound of ‘X’ and ‘Y’are :-
(b) NaBr (a) Na2SO3, BaSO4
(c) Na2CO3 (b) Na2SO4, BaSO4
(d) Na2SO3 (c) Na2S2O3, MnO(OH)2
(d) Na2SO4, (BaSO4 + MnO(OH)2)

www.jeebooks.in
QUALITATIVE SALT ANALYSIS 4.21
29. Which of the following statement is / are incorrect Paragraph for Q.33 to Q.34
about of ‘Y’ and ‘Z’ ? dil. HCl
Salt [A] white ppt.
(a) The pink colour is due to adsorption of [B]
KMnO4 on BaSO4 surface H2S is passed
(b) H2O2 can not reduce adsorbed KMnO4 on with saturated
KCl
BaSO4 surface on dilution
E + [D] Black ppt. Red ppt.
(c) The compound ‘Y’ also formed when BaSO3 [C]
is exposed to air and product is added in
KMnO4 solution +H2S
(d) ‘Y’ and ‘Z’ are nonidentical precipitate
Paragraph for Q.30 to Q.32 Black ppt.
[D]
Mixture of unknown salt
33. Compound ‘D’ is :-
dil. HCl (a) (Hg + HgS) (b) Ag2S
Mixtur of white ppt.[A] (c) PbS (d) Pb2SCl2
34. The correct statement about [C] is -
aq. suspension is
heated and filtered (a) Red precipitate is formed due to formation of
ead sulphochloride
Residue Filtrate (hot sol.) (b) Red precipitate is formed due to formation of
lead sulphochloride
Added
on cooling (c) Red precipitate is formed due to formation of
NH3(aq.)
lead sulphochloride
White needle ppt.
[B] (d) Red precipitate is formed due to formation of
Filtrate Residue lead sulphochloride
[D] (black ppt.)
(soluble [C] Paragraph for Q.35 to Q.37
complex) Q R Square
Tetrahedral M
excess 1 excess planar
dilute nitric acid
Q R
(X2 ) Tetrahedral M Tetrahedral (X1)
excess 2 excess
White ppt.
S(stoichiometric amount)
[E]
S
30. Mixture of white ppt. [A] may contain. White ppt. Precipitate dissolved
excess
(I) PbCl2 (II) AgCl 35. M1 , Q & R respectively are :-
(III) Hg2Cl2 (a) Zn2+, KCl, KCN (b) Ni2+, KCl, KCN
(a) Only I (b) Only II, III (c) Ni2+, KCN, HCl (d) Zn2+, KCN, KCl
(c) Only III (d) I, II and III 36. M2 , X1 & X2 respectively are :-
31. Compound ‘B’ is (a) Zn2+, Zn(CN)42–, ZnCl42–
(a) PbCl2 (b) AgCl (b) Ni2+, Ni(CN)42–, BiCl42–
(c) Hg2Cl2 (d) (Hg + HgCl2) (c) Zn2+, ZnCl42–, Zn(CN)42–
32. Co-ordination number of central metal ion in filtrate (d) Ni2+, NiCl42–, Zn(CN)42–
(soluble complex) ‘D’ is - 37. S is :-
(a) 4 (b) 2 (a) H2S (b) NaOH
(c) 6 (d) 5 (c) HNO3 (d) KI

www.jeebooks.in
4.22 QUALITATIVE SALT ANALYSIS
Paragraph for Q.38 to Q.40 Paragraph for Q.44 to Q.46
Aqueous solution of CaCl 2+SrCl2+BaCl2 The trends in properties of alkaline earth metal salts
(NH4)2CrO4 can be explained by using Fajan’s rule. It is associated
A (yellow precipitate) with incorporation of covalent character in ionic bond.
CH2CO2H & filtered It states that smaller cations form more covalent
compound and ionic character explains the solubility,
thermal stability, hydrolysis and other properties of
B(yellow residue) C(filtrate) these salts. Some exceptions are also there and are
excess of (NH4)2SO4 explained by change in lattice energy.
& filtered
44. Correct order of increasing thermal stability of
alkaline earth metal sulphates is :-
(White (filtrate) E F(White precipitate)
residue) D
(NH4)2Cr2O4 (a) SrSO4 < CaSO4 < MgSO4 < BeSO4
38. Yellow precipitate A contains :- (b) BeSO4 < MgSO4 < CaSO4 < SrSO4
(a) BaCrO4 only (b) BaCrO4 + SrCrO4 (c) CaSO4 < BeSO4 < MgSO4 < SrSO4
(c) CaCrO4 only (d) CaCrO4 + BaCrO4 (d) MgSO4 < BeSO4 < CaSO4< SrSO4
39. E(aq.) is :- 45. Which salts does not show hydrolysis at all ?
(a) (NH4)2[CaSO4] (b) (NH4)2[Ca(SO4)2] (a) MgCl2 (b) BeCl2
(c) (NH4)2[Ca(SO4)4] (d) Ca(SO4)2 (c) AlCl3 (d) BaCl2
40. D gives :- 46. Whcih alkaline earth metal form superoxide ?
(a) Apple green colour in flame test (a) Be
(b) Brick red colour in flame test (b) Mg
(c) Golden yellow colour in flame test (c) Ca
(d) Crimson red colour in flame test (d) None
Paragraph for Q.41 to Q.43 Paragraph for Q.47 to Q.48
The elements Mg, Cr, Sr and Ba react with hydrogen Bi3+(aq) + KI Black ppt. 'A'
to form hydrides ‘MH2’. Beryllium hydride is difficult excess KI
to prepare and less stable than the others.
Orange solution 'B"
41. Hydride, which act as a reducing agent :-
on dilution with H2O
(a) CaH2 (b) SrH2
aqueous suspenstion
(c) BaH2 (d) All Orange ppt. 'C' is heated
'A'()

42. Which of the following hydride is polymeric and 47. Orange solution ‘B’ is due to formation of :-
covalent in nature in solid ? (a) BiI3
(a) BeH2 (b) SrH2 (b) [BiI4]–
(c) BaH2 (d) All (c) Bi(OH)2I
43. On hydrolysis with water and dilute acids, alkaline (d) BiOI
earth metal hydrides produces :- 48. Compound ‘C’ is :-
(a) O2 gas (a) BiI3
(b) H2 gas (b) BiOI
(c) O3 gas (c) Bi(OH)2I
(d) CO2 gas (d) [BiI4]–

www.jeebooks.in
QUALITATIVE SALT ANALYSIS 4.23
Paragraph for Q.49 to Q.51 Paragraph for Q.52 to Q.53
Following observations are given : soluble
Unknown salt
K [Fe(CN)6]
4 BaCl2 (solid) 'A' in H2O
(F) (A) (B) + (C)
Chocolate Blue White ppt. NH4OH/NaOH soln.

Brown ppt. solution
Colourless gas Brown ppt.
KI
+brown residue (insoluble in NaOH soluton)
Hypo
(D) solution (E) Dissolved in
Brown White ppt. conc. HCl
solution K4[Fe(CN)6]
49. Correct statement :- Yellow solution Blue ppt.
52. Identify A :-
(a) Compound F is Cu2Fe(CN)6
(a) FeSO4 (b) Fe2(SO4)3
(b) Compound E is CuI
(c) FeCl3 (d) CrCl3
(c) Both (A) & (B)
53. What is the formula of brown ppt. :-
(d) None of these
(a) Cr(OH)3 (b) Fe(OH)2
50. Brown color solution is due to presence of :-
(c) FeCl3 (d) Fe(OH)3
(a) Cu2[Fe(CN)6] (b) Cu2I2
Paragraph for Q.54 to Q.55
(c) KI3 (d) CuSO4
NH4OH Greenish ppt. (B)
51. The incorrect statement(s) about above flow Aqueous green solution of (A)
diagram is / are :-
Na2O2
(a) Adding an excess of hyposolution to the
brown solution, ‘d’ is reduced to colourless Bright yellow Aq. Pb(NO3)2 Orange Acidification Yellow
solution and white precipitate becomes visible. (ppt.) (D) +Base colour solution (C)
(b) F is Cu3[Fe(CN)6]2 54. Identify A :-
(c) The reduction of hyposolution yield (a) AlCl3 (b) Cr(OH)3
tetrathionate ions (c) CrCl3 (d) FeCl3
(d) The co-ordination number of iron in the 55. Identify D :-
chocolate brown ppt.(F) is changed than the (a) Na2CrO4 (b) PbCrO4
K4[Fe(CN)6] (c) Na2MnO4 (d) Fe(OH)3
Paragraph for Q.56 to Q.58
 
Compound 'A' Initially swelled Amorphous powder
strong heating

Lilac flame in the flame test

excess NaOH H2O2


Compound 'A' 'B' (No change in colour) 'C' (Yellow solution)
(in aq. solution)

56. Compound ‘A’ is having water of crystallization 57. The compound ‘B’ is having oxidation state of
by the number of (a) Zero (b) II
(a) 10 (b) 20 (c) III (d) IV
(c) 24 (d) 36

www.jeebooks.in
4.24 QUALITATIVE SALT ANALYSIS
58. The hybridization ‘B’ of compound ‘C’ is Paragraph for Q.65 to Q.68
(a) sp3 (b) sp3d (I) (X)  
 glassy transparent bead (Y)
(c) d2sp3 (d) d3s on platinum wire
Paragraph for Q.59 to Q.62 (Y) + CuSO4 coloured bead (Z)

Light green colour compound (A)   (II) (X) + conc. H2SO4 + CH3CH2OH  ignite

white residue (B)
high green flame (W)
(B) temp
C  D  E
(III) Aqueous solution of (X) is alkaline.
D and E are acidic in nature. When A is passed through
65. X is :-
HgCl2 , yellow precipitate is observed. D is passed
through H2S , white turbidity is obtained :- (a) NaNH4 HPO4 . 4H2O
59. D and E are :- (b) Na2B4O7 . 10H2O
(a) SO2 , SO3 (b) SO2 , CO2 (c) CuSO4 . 5H2O
(c) CO2 , CO (d) SO2 , CO (d) None of these
60. Yellow precipitate is :- 66. Y is :-
(a) HgO (a) NaPO3 (b) NaBO2
(b) Basic Hg (I) sulphate (c) NaBO2 + B2O3 (d) None of these
(c) Basic Hg (II) sulphate 67. Z is :-
(d) HgI2 (a) Cu3(PO4)2 (b) CuSO4
61. C is soluble in :- (c) Cu(BO2)2 (d) None of these
(a) conc. HCl (b) dil. H2SO4 68. W is :-
(c) AcOH (d) Na2CO3 (a) (CH3)3BO3 (b) (C2H5)3BO3
62. Number of water of crystallization in A is :- (c) H3BO3 (d) None of these
(a) 0 (b) 2 Paragraph for Q.69 to Q.71
(c) 7 (d) 5 Microcosmic salt is used to detect different metal ions.
Paragraph for Q.63 to Q.64 Microcosmic salt is first heated to give glassy bead of
An aqueous solution of a mixture of two inorganic salts X. X when reacts with different metal oxide, different
, when treated with dilute HCl, gave a precipitate (P) colour is observed.
and a filtrate (Q). The precipitate (P) was found to 69. Microcosmic salt is :-
dissolve in hot water. The filtrate (Q) remained (a) Na(NH4) HPO4.4H2O
unchanged, when treated with H2S in a acidic medium.
(b) Na2HPO4.4H2O
However, it gave a precipitate (R) with H2S in an
ammonical medium. The precipitate (R) gave a (c) Na3PO4.4H2O
coloured solution (S), when treated with H2O2 in an (d) Na(NH4) HPO4.2H2O
aqueous NaOH medium. 70. The correct statement about microcosmic salt :-
63. The precipitate P contain :- (a) It is sodium ammonium hydrogen phosphate
(a) Pb2+ (b) Hg22+ tetrahydrate
(c) Ag+ (d) Hg2+ (b) It gives canary yellow precipitate with
64. The colour solution S contains :- ammonium molybdate solution
(a) Fe2(SO4)3 (b) CuSO4 (c) Both (A) and (B)
(c) ZnSO4 (d) Na2CrO4 (d) None of these

www.jeebooks.in
QUALITATIVE SALT ANALYSIS 4.25
71. Microcosmic salt 
NaOH / 
 Basic gas (Z) :- Paragraph for Q.76 to Q.78
Aqueous solution of Z gives :- Dissolved
Crysalline salt (A)  
in water
(a) Blue colour precipitate with CuSO4 solution
KI (aq) A yellow ppt. insoluble in conc.
(b) Brown precipitate which is called iodide of
HNO3 and also in NH3(aq)
millons base with K2[HgI4]
(c) White precipitate with ZnCl 2 which is
dissolved when Z is added in excess K2CrO4(aq)
Brick red ppt.
(d) All of these
Paragraph for Q.72 to Q.75
Identification of cations can also be done using a dry K2CrO4(aq)
test called sodium carbonate bead test which is similar A black ppt.
to borax - bead test :-
With MnO Cl2
A B conc.H2SO4
KNO3 Brown ring.
fuse and heat in FeSO4
Na2CO3 76. Crystalline salt A is
oxidizing flame
With Cr2O3 H SO (a) AgNO3 (b) Pb(NO3)2
C 2 4 D
72. The hybridisation and colour of A with molecular (c) AgNO2 (d) Hg(NO3)2
formula ? 77. Black ppt. and yellow ppt. are respectivly
(a) Na2MnO4 , d3s , Green (a) PbS, PbI2 (b) HgS, HgI2
(b) NaMnO4 , sp3 , Purple (c) Ag2S, AgI (d) (Hg + HgS), Hg2I2
(c) Mn2O7 , d3s , Red 78. Which of the following options are correct
(d) Na2Cr2O7 , d3s , Orange regarding A?
73. The hybridisation and colour of B with molecular (a) A turns our skin black
formula ? (b) A gives white precipitate with NaOH which
(a) Na2MnO4 , d3s , Green produces silver mirror with glucose solution
(b) NaMnO4 , sp3 , Purple (c) A gives white precipitate with KCN which is
(c) Mn2O7 , d3s , Red soluble in excess KCN
(d) Na2Cr2O7 , d3s , Orange (d) All
(d) None of these Paragraph for Q.79 to Q.81
74. C and its colour is :- A white salt (A) does not dissolve in water, but dissolve
(a) Na2CrO4 , orange in dilute nitric acid solution. Passing H2S gas through
acidic solution of A, a black precipirate A 1 was
(b) Na2Cr2O7 , orange
produced which was insoluble in water, dilute HNO3
(c) Na2Cr2O7 , yellow and alkali but sdissolved in concentrated solution of
(d) Na2CrO4 , yellow Na2S as well as in aqua-regia. Adding SnCl2-dropwise
75. D and number of unpaired electron in D are :- to the solution of A, gave a white , silky precipitate
(a) Na2CO3 , 1 (B), which turned into black precipitate (C) on adding
(b) NaMnO4 , 2 excess reagent. Addition of KI to the acidified solution
of A gave a red coloured precipirate (D) in the
(c) Na2Cr2O7 , 0
beginning, which is dissolved in excess reagent.
(d) Na2CrO4 , 0

www.jeebooks.in
4.26 QUALITATIVE SALT ANALYSIS
79. A is called Extract solution in concentrated H2SO4 when mixture
(a) Calomel (b) Corrosive sublimate mixed with K2Cr2O7, gave a dense brown fumes on
boiling, that turned aqueous NaOH solution yellow.
(c) Brine solution (d) Rock salt
Extract solution when treated with Hg(NO3)2 a yellw
80. The formula of B and D are respectively precipitate (E) was formed.
(a) Hg2Cl2, HgI2 (b) HgCl2, HgI2 85. The acidic radical present in the mixture
(c) Pb2Cl2, PbI2 (d) Hg2Cl2, Hg2I2 (a) A consists of Cl–, B consists of SO32–
81. Black ppt. A1 and black ppt. C are respectively (b) A consists of Br–, B consists of SO32–
(a) Ag2S, Ag (b) HgS, Hg (c) A consists of Cl–, B consists of SO42–
(c) Hg2S, Hg (d) PbS, Pb (d) A consists of Cl–, B consists of CO32–
Paragraph for Q.82 to Q.84 86. Yellow precipitate is
An inorganic compound (A), transparent like glass is (a) Hg(OH)2. HgCO3 (b) HgO
a strong reducing agent. Its hydrolysis in water gives (c) HgSO4.2HgO (d) HgCl2
white turbidity (). Aqueous solution of (A) gives white 87. The basic radical present in the mixture
precipitate (C) with NaOH (aq) which is soluble in (a) A consists of Fe3+, B consists of Pb2+
excess NaOH.(A) also reduce I2 and gives chromyl (b) A consists of Fe2+, B consists of Pb2+
chloride test.
(c) A consists of Fe3+, B consists of Zn2+
A+ corrosive sublimate  Grey mass + D.
(d) A consists of Fe2+, B consists of Zn2+
82. Grey mass and D are respectively
Paragraph for Q.88 to Q.90
(a) Hg and SnCl2 (b) Ag and Cu2Cl2 Substance (A) is a yellowish-white deliquescent oslid
(c) Hg and SnCl4 (d) Hg and Cu2Cl2 whihc sublimes and has a vapour density of 133. (A)
83. What is the correct option regarding A. reacted violently with water forming solution (B). A
(a) A reduces auric chloride to produce purple of sample of (B) gave a curdy white precipitate (C) on
cassius. addition of diliute HNO3 and AgNO3 solution, but is
readily dissolved on the addition of dilute NH4OH,
(b) A reduces ferric chloride to iron. thourhg a geleatinous white precipirate was formed in
(c) A reduces cupric chloride to red deposit its place. (D) was filtered off and dissolved in excess
copper NaOH, forming a clear solution (E). When CO2 was
(d) All passed into (E), comound (D) was reprecipitated.
84. A is Qualitative analysis of solution (B) gave a white
gelationous precipitate in Group III. When 0.1333 g of
(a) CdCl2 (b) CuCl2
(A) was dissolved in water and treated with
(c) SnCl2 (d) SnCl4 8-hydroxyquinoline. 0.4594 g of precipirate was obtained.
Paragraph for Q.85 to Q.87 88. A is
A mixture consists of an yellow salt (A) and a white (a) FeCl3 (b) CrCl3
salt (B), both anhydrous. Salt mixture was dissolved (c) ZnCl2 (d) AlCl3
in water and few drops of HCl was added to obtain a
89. The coordination number of metal ion in E
clear, yellow coloured soluton. The solution was
(a) 1 (b) 2
then treated with NH 3/NH4Cl solution when a
reddish-brown precipirate (C), insoluble in NaOH (c) 3 (d) 4
solution, was produced. Solution was filtred off and 90. The same galetinous white precipitate was formed
filtrate was treated with aqueous NaOH solution when when (A) reacts with
a white precipirate (D), soluble in excess reagent was (a) (NH4)2 S (b) Na2S
formed. (c) Na2CO3 (d) All

www.jeebooks.in
QUALITATIVE SALT ANALYSIS 4.27
Paragraph for Q.91 to Q.93 94. C and D are respectively :-
A white powder turns yellow (B) on heating and (a) Fe(OH)3 , CoS (b) Fe(OH)3 , MnS
evolves a gas which turns lime water milky, as well as (c) Fe(OH)2 , CoS (d) Fe(OH)2 , MnS
water vapour. The yellow residue turns white on cooling 95. The anion present in B -
but turns yellow again when heated B reats with dilute (a) NO3– (b) Br–
sulphuric acid to give a colourless solution (C). If dilute
(c) Cl– (d) NO2–
NaOH is added to C, a white precipitate (D) is formed
initialy, which dissolves on adding excess base. With Paragraph for Q.96 to Q.97
dilute ammonia soluton C gives a white precipitate Three black powder A, B and C have to be identified.
which dissolves in excess ammonia, giving a clear A dissolves in dilute HCl to give a blue solution which
solution (E) due to complex formation. becomes deep blue with the addition of excess of
NH4OH. The solution of A in dilute HNO 3, on
91. A is
evaporation yields blue crystals which, on strong
(a) PbCO3 (b) CaCO3 heating reagents A. B dissolves completely, on boiling
(c) ZnCO3 (d) NiCO3 with concentrated HNO3 yielding coious brown fumes.
92. The cordination number of metal ion in the complex The resulting solution after proper dilution yields no
is positive test for any cation. C does not dissolve in dilute
HCl, but goes into solution on boiling with concentrated
(a) 2 (b) 4
HCl and a crystal of KClO3. Evaporation of this
(c) 6 (d) 5 solution yields colourless crystals which dissolve in
93. The shape of the complex is water. The solution yields a scarlet precipitate when a
(a) Square planer little KI solution in added but redissolves in excess of
KI solution.
(b) Tetrahedral
96. A & B is -
(c) Inner orbital Octahedral
(a) Hg2O, Copper (b) CuO, Coke powder
(d) Outer orbital Octahedral
(c) Ag , Hg (d) Cu, Hg
Paragraph for Q.94 to Q.95
97. C is -
A solid mixture consists of a reddish brown metal oxide
(a) PbS (b) HgS
(A) and a white hydrated salt B. The mixture was
dissolved in dilute HNO3 and a portion of this solution (c) Ag2S (d) Bi2S3
was treated with NH3 solution, just to make the solution Paragraph for Q.98 to Q.100
neutral and then finally treated with NH3/NH4Cl buffer A yellow deliquescent solid (a) is soluble in water,
solution when a reddish brown gelatinous precipitate alcohol and ether . Its aqueous solution turns blue litmus
(C) was obtaind. C was then dissolved in dilute HCl red. (a) gives following reactons :
and few drops of NH4SCN solution was added when (i) On strong heating it gives a dark brown residue
a deep blood red colouration was observed. Filtrate (b) water and a compound, (c) which gives white
obtained after removal of C was then treated with fumes with NH3 gas. Compound (c) in aqueous
excess of (NH4)2S solution when a pink coloured solution decomposes carbonates to evolve CO2
precipitate (D) was formed. D was then dissolved in gas.
dilute HNO3 and treated with NaBiO3 solution. A deep (ii) When dry chlorine gas is passed over a heated
purple coloured solution was formed. mixture of (b) and carbon , one another compound
In a separate experiment a pinch of original salt mixture (d) is formed. The colour of solid (d) is black red.
was mixed with solid K2Cr2O7 and then dissolved in It dissolves in water giving a yellow solution.
concentrated H2SO4 and finally boilled. A deep red (iii) When aqueous solution of (d) is heated, a brown
fumes of (E) was obtained which made aqueous precipitate (e) is formed. The hydrolysis can be
solution of NaOH yellow, when passed through it. checked by the addition of HCl to aqueous solution.

www.jeebooks.in
4.28 QUALITATIVE SALT ANALYSIS
(iv) (d) absorbs water vapours to give compound (a). 103. Match the column-I with column-II :-
(d) also absorbs NH3 gas to form an addition Column - I Column - II
compound (F).
(Reagents) (Compound)
(v) A solution of (d) reacts with ammonium thicoyanate
to produce a red colour of compound (G). (a) Catalytic test [NaN3 + KI3] (P) Na2S
(vi) A solution of (d) reacts with K4[Fe(CN)6] solution (b) Sodium nitroprusside test (Q) Na2S2O3
to give a deep blue colour (H). A solution of (d) (c) Blood red colour with FeCl3 (R) NaSCN
liberated iodine form KI solution. H2S is oxidized
(d) Black ppt. with AgNO3 at (S) Na2CO3
to free sulphur and SO2 to sulphuric acid.
room temperature
98. A is -
(a) FeCl3 (b) AlCl3 (T) Na2SO3
(c) FeCl3 (d) SbCl3 104. Match the column-I with column-II :-
99. Red colour compound (G) and deep blue colour Column - I Column - II
(H) are respectively - (Acidic radical) (Observations)
(a) [Fe(H2O)5SCN]2+, Fe4[Fe(CN)6]3 (a) NO2– (P) Gives brown ring test
(b) [Fe(H2O)5SCN]1+ , Fe3[Fe(CN)6]2 (b) NO3– (Q) Gives brown ppt. with
(c) [Fe(SCN)2], Fe2[Fe(CN)6] Nesslar’s reagent
(d) [Sb(SCN)3], Fe4[Fe(CN)6]3
(c) NH4+ (R) Gives bloor red colouration
100. Brown precipitate (E) and addition compound F with KSCN
are -
(d) Fe3+ (S) Gives red colouration with
(a) Fe(OH)3, [Fe(NH3)6]3+
CH3COONa
(b) Fe(OH)2, [Fe(NH3)6]2+
(T) Radical is reduced to
(c) Fe(OH)3, [Fe(NH3)6]2+
ammonia with Zn + conc.
(d) Fe(OH)2, [Fe(NH3)6]3+
NaOH
Matrix Match Type :
105. Match the column-I with column-II :-
101. Match the column-I with column-II :-
Column - I
Column - I Column - II
(Reactants) (Products) (Acidic radical)
(a) Zn + dil. H2SO4 (P) Metallic salt (a) Cl– salt solution
(b) Fe + dil. HNO3 (Q) SO2 (b) Br– salt solution
(c) Zn + conc. H2SO4 (R) Oxyacid (c) I– salt solution
(d) P + conc. H2SO4 (S) N2O (d) NO2– salt solution
(T) H2 or H2O Column - II
102. Match the column-I with column-II :- (Observations)
Column - I Column - II
(P) Gives positive layer test (with chlorine water)
(Reactant) (Product)
(Q) Yellow color with excess chlorine water in
(a) Ag + conc. H2SO4 (P) Acidic oxide
organic layer
(b) Ag + conc. HNO3 (Q) Metal nitrate
(R) Brown fumes with conc. H2SO4
(c) As + conc. H2SO4 (R) Metal sulphate
(d) Sn + conc. HNO3 (S) Oxy acid (S) Ppt. formed with conc. AgNO3
(T) Water (T) Coloured gas with K2Cr2O7 + conc. H2SO4

www.jeebooks.in
QUALITATIVE SALT ANALYSIS 4.29
106. Match the column-I with column-II :- Column - II (Property / Test)
Column - I Column - II (P) Produces soluble complex with excess of
(Reagents) (Anion gives ppt. KCN without any change of oxidation state
change in colour) of metal ion
(a) FeCl3(aq.) (P) SO42–(aq.) (Q) Produces coloured soluble complex with
excess of KCN
(b) BaCl2(aq.) (Q) CH3COO–(aq.)
(R) Produces no ppt. with excess of KI
(c) AgNO3(aq.) (R) PO43–(aq.)
(S) Produces coloured ppt. with NaOH
(d) (NH4)2MoO4(aq.) (S) S2O32–(aq.) (T) Produces colourless soluble complex with
107. Column-I lists some of group reagents, excess of NH4OH
give characteristics colour / precipitate 110. Match the column-I with column-II :-
mentioned cation in column-II. Match each Column - I Column - II
entry of column-I with those given in (Basic radical) (Property / Test)
column-II :- 3+
(a) Fe (P) Produces white ppt. with
Column - I Column - II K4[Fe(CN)6]
(Group reagents) (Cations) (b) Zn2+ (Q) Produces soluble complex in
(a) H2S in NH4OH(aq.) (P) Bi3+(aq.) excess of NaOH solution.
3+
(b) H2S in HCl(aq.) (Q) Pb2+(aq.) (c) Cr (R) Give redox reaction with
Na2S2O3 solution.
(c) (NH4)2CO3 in NH4Cl(aq.) (R) Ca2+(aq.) 3+
(d) Al (S) Produces corresponding
(d) NH4OH in NH4Cl (S) Mn2+(aq.) hydroxide with aqueous Na2S
(T) Al3+(aq.) (T) Produces reddish brown ppt.
108. Match the coloum of precipitate listed in Column-I with CH3CO2– solution
with the reagent(s) listed in column-II. 111. Match the column-I with column-II :-
Column - I (Observations) Column - I (Reagent) Column - II (Name
(a) Ag+ gives black / brown ppt. with of product / colour)
(b) Hg22+ gives black ppt. with (a) CH3CO2– + As2O3 (P) Thenard Blue
(c) Pb2+ gives yellow ppt. with (b) Na-K tartarate+CuSO4 (Q) Tollens reagent
(d) Mg2+ gives pink mass (c) NH4OH + AgNO3 (R) Fehling solution
(d) Al2O3 + Co(NO3)2 (S) Cacodyl oxide -
Column - II (Reagents)
Extremely
(P) Co(NO3)2 in charcoal cavity nauseating odour
(Q) KI 112. Match the column-I with column-II :-
(R) H2S (saturated in H2O) Column - I Column - II
(S) K2CrO4 solution (Decolourise acidic (Given salt)
(T) NaOH solution KMnO4 solution)
109. Match the coloum :- (a) Only acidic radical (P) Sn(NO3)2
Column - I (Basic radical) (b) Only basic radical (Q) KNO2
(a) Cu2+ (c) Both radical (R) FeC2O4
(b) Fe3+ (d) Neither acidic nor (S) Na2CO3
(c) Cd2+ basic radical
(d) Ag+ (T) BaSO3

www.jeebooks.in
4.30 QUALITATIVE SALT ANALYSIS
113. Match the column-I with column-II :- 117. Match the colour of ppt. listed in column-I with
Column - I Column - II the reagent(s) in column-II :-
Color of the sublimate Compound present Column - I (Observation/Test)
(a) Bi3+ gives black ppt. with
(a) White (P) NH4+, HgCl2, Hg2Cl2 (b) Cu2+ gives black ppt. with
(b) Yellow (Q) As2O3 (c) Cd2+ gives white ppt. / bluish - white pp. with
(c) Orange (R) Sb2O3 (d) Ag+ gives black ppt. with
Column - II (Reagents)
(d) Black (S) HgS
(P) Saturated solution of H2S in H2O
114. Match the column-I with column-II :-
(Q) KSCN(aq.)
Column - I Column - II
(R) K4[Fe(CN)6] (aq.)
(Anion) (Property)
(S) dil. HCl
(a) Peroxides (P) Diamagnetic
(T) NaH (aq.)
(b) Superoxides (Q) Paramagnetic 118. Column - I Column - II
(c) Oxides (R) Bond order is 1.5 Pair of Same colour Colour
(d) Hydroxides (S) Total number of precipitate
electrons are 17 (a) CdS, SnS2 (P) Brownish red
(T) Total number of (b) PbS, Ni(OH)3 (Q) Black
electrons are 18 (c) Fe(OH)3, Ag3AsO4 (R) Yellow
115. Column - I Column - II (d) Ni(OH)2, Cu3[Fe(CN)6]2 (S) Green
Precipitate Dissolving solution 119. Match the column-I with column-II :-
(a) Black CuS (P) NaOH solution Column - I Column - II
(Effect on heating) (Metal)
(b) White AgCl (Q) Sodium peroxide
(a) Amphoteric metal oxide (P) Pb
solution
(c) Green Cr(OH)3 (R) 50% HNO3 (b) Metal acetate 
 acetone (Q) Zn
+ metal carbonate
(d) White Zn(OH)2 (S) Excess NH3 solution
(T) KCN solution (c) Metal carbonate 

 (R) Na
116. Column - I (Compound of same metal with metal oxide+CO2
different oxidation state) (d) Metal nitrate 

 (S) Li
2 1 metal oxide+NO2
(a) Cu(SCN) 2 ,Cu SCN
120. Which of anions in the column-I shows one ore
(b) Cu(CN)2 , CuCN
more observatons from the column-II :-
(c) HgI2 , Hg2I2 Column - I Column - II
(d) HgO, Hg2O (Radical) (Characteristic feature)
Column - II (Colour of the ppt) (a) S2– (P) White ppt. with AgNO3
(Q) Yellow, white (b) NO2 –
(Q) Evolution of pungent smell gas
(R) Red, Green with (Al + conc. NaOH)
2–
(P) Black, white (c) SO3 (R) Brown fumes with conc.
H2SO4 (hot)
(S) Yellow, Black
(d) CH3COO– (S) Decolourises acidified KMnO4

www.jeebooks.in
QUALITATIVE SALT ANALYSIS 4.31
121. Match the column-I with column-II :- 124. Match the column-I with column-II :-
Column - I Column - I Column - II
Colour of borax bead Metal
(a) µ = 15B.M. test(cold) present
(b) O.N. of metal = +2 Oxidising Reducing
(c) Octahedral complex flame flame
(d) d2sp3 hybridised metal ion (a) Green Green (P) Cu
(b) Violet Colourless (Q) Fe
Column - II
(c) Yellow Green (R) Mn
(P) Schweizer’s salt
(d) Blue Red (S) Cr
(Q) Potassium ferrocyanide
125. Match the column-I with column-II :-
(R) Potassium ferricyanide
Column - I Column - II
(S) Brown ring complex Colour of borax bead Metal
(T) Sodium nitroprusside test(cold) present
122. Match the column-I with column-II :- Oxidising Reducing
flame flame
Column - I (Group reagent)
(a) Blue Blue (P) Co
(a) NH4Cl + NH4OH
(b) Colourless Blue (Q) Ti
(b) NH4Cl + NH4OH + (NH4)2CO3
(c) Brown Grey (R) Ni
(c) HCl.(dil.) + H2S
(d) Amethyst Colourless (S) Mn
(d) NH4Cl + NH4OH + H2S
Column - II (Group ion precipitated) EXERCISE # IV
(P) IInd group  Integer Type :
(Q) IIIrd group 1. Find the number of reagent(s) which form white
(R) IVth group ppt. with CO32– ion.
(S) Vth group BaCl2, CuSO4, Hg2Cl2, Pb(CH3COO)2, CaCl2
2. Find the number of reagent(s) which oxidises
123. Match the column-I with column-II :-
HCO3– ion solution.
Column - I (Colour of precipitate)
MnO4– / H+, Cr2O72– / H+, Cl2 water, Br2 water, I2
(a) Black ppt. water
(b) Canary yellow ppt. 3. Find the number of carbonate having Ksp value
(c) Brick red ppt. greater than Li2CO3
(d) White ppt. Na2CO3, K2CO3, Rb2CO3, Cs2CO3
Column - II(Reactions) 4. Find out the number of anions that can not
decolorise acidic solution of KMnO4.
(P) Bi(OH)3 + [Sn(OH)4]2– 
NO2– ; NO3– ; Cl– ; I– ; C2O42– ; HCO2– ; CO32– ;
(Q) Na2S + Hg(NO3)2  S2– ; SO32–
(R) Na3PO4+(NH4)2 MoO4  5. Find out the total number of acidic radical, to
(S) AgNO3 + K2CrO4 which if dilute HCl is added then volatile gases
(T) HC CH + AgNO3 + NH4OH  are evolved. CO 3 2– , SO 3 2– , S 2– , S 2 O 3 2– ,
NO3–, CN–

www.jeebooks.in
4.32 QUALITATIVE SALT ANALYSIS
6. Among the following, how many compound(s) (5) AgNO3 oxidizes suspension of sulphite
destroye nitrite solution and N2 gas is evolved. solution to sulphate.
AgNO 3, NH 4Cl, (NH 2CONH 2 + dil. HCl), (6) Bromine water oxixises BaSO3 to BaSO4.
Sulphamic acid, (thiourea + dil. CH3COOH), 14. Salt solutution ‘A’ + dil. HCl only ‘B’ gas
(FeSO4 + dil. CH3COOH) ; MnO4– + H+ (suffocating odour of burning sulphur)
7. How many compounds of Mn 2+ are pink
BaCl2
precipitate :-
MnS ; MnO(OH)2 ; Mn(NH4)PO4 ; KMnO4, Mn ‘C’ [white ppt.] soluble in dil. HCl and gives
(OH)2 ‘B’ gas
8. How many of the following reagents can be used
to distinguish between SO2 and CO2 ? Reagent 'M'

Ca(OH)2 , (starch + KIO3), (K2Cr2O7 + H2SO4), white ppt. of [D] (insoluble in dil. HCl)
(KMnO4 + H2SO4), Ba(OH)2, H2O2, FeCl3,
Find the total number of reagents ‘M’ :
Zn[Fe(CN)5(NO)]
9. The total reagents are used to distinguish between (conc. HNO3), O2 , H2O2 , N2 , NH3 , (Br2 + H2O)
sulphite and bisulphite acidic radical. 15. In Ni(DMG)2 ; how many following statements are
Litmus paper, AgNO3, Pb(OAc)2, CaCl2, SrCl2, correct ?
BaCl2, (K2Cr2O7 + H2SO4), (KMnO4 + H2SO4), (a) Ni(DMG)2 is red in colour
(H2O2 followed by litmus paper) (b) Both 6 and 5 membered chelation takes place
10. Find the total number of acidic radical, to which if (c) H bonding is present which is intramolecular
conc. H2SO4 is added then volatile gases are (d) It is diamagnetic
evolved with specific odour. CO32–, SO32–, HCO2–,
(e) Ni(+2) has coordination no. 4
CH3CO2–.
(f) In (DMG), N is the donor site
11. Find the total number of metals , which makes a
thin protective layer of its oxide on treatment with 16. Observe the following reaction,
conc. HNO3. slightly
[Ni(en)3](NO3)2 + ‘X’(aq)
alk. medium
Al, Zn, Sn, Cu, Pt, Cr, Au, Ag
‘Y’() violet ppt.
12. The total number of anionic species gives coloured
Among the following , how many compounds
gas with conc. H2SO4.
decompose the nickel ethylenediamine nitrate
CO32–, HCO3–, SO32–, NO32–, S2O32–, NO2–, reagent with the precipitation of NiS.
CH3COO–, SO42–
Na2SO3 , Na2SO4 , Na2S2O3 , Na2S4O6 , H2S,
13. Among the following find out the total number of (NH4)2S, NaSCN
incorrect statement(s) -
17. Na S O + Reagent [A]  white ppt. of [B]
2 2 3
(1) BO33– , PO43– , F– & SiO44– anions are called
ng

+ex S 2O 3

as interfering radicals,
Na 2
armi

cess

(2) HgCl2 reacts with carbonate ion and gives


on w

basic mercuty(II) carbonate which does not


give test with bicarbonate. [Black ppt. of [Soluble complex
metal sulphite] of [B]
(3) SO32– and HSO3– can be distinguished by
addition of neutral H2O2 solution against litmus Among the following how many reagents ‘A’ are
paper. used to give black precipitate of metal sulphide ?
(4) Zinc and sulphuric acid oxidized sulphite to AgNO 3, BaCl 2, Pb(OAC) 2 , HgCl 2, CaCl 2 ,
sulphate. CuSO4, FeCl3

www.jeebooks.in
QUALITATIVE SALT ANALYSIS 4.33
18. Among the following how many compound(s) PbSO3 ; [PbCO3.Pb(OH)2] ; PbCrO4 ; AgCl ;
destroy nitrite solution and does not form traces Ag2S ; Ag2O
of nitrate ? 28. Among the following , total number of reagent(s)
AgNO 3, NH 4Cl, (NH 2CONH 2 + dil. HCl), which gives yellow precipitate with ammonium
sulphamic acid, (thiourea + dil. CH3COOH), chloride solution.
(FeSO4 + dil. CH3COOH) Na3[Co(NO2)6]; H2[PtCl6] ; (NaH.C4H4O6) ;
Na 2S2O3
19. Salt of iron   Violet complex
NaClO4 ; NaOH ; O2N N=N–Cl
(X) (Y)

On standing
 Green solution 29. Colourless salt (A) NaOH(excess)  
 Gas(B)
(Z)
giving white fumes with HCl + solution (C)
Write oxidation number of Fe in (Z).
20. Find the number of cation when treated with aq. (C) + Zn 
 NaOH (B)
solution with aq. solution of Na2CO3 , ppt. of 
(A)   Gas(D) + liquid (E)
carbonate formed ,
D, E both are triatomic
Hg+2 , Ba+2 , Cu+2 , Ag+ , Pb+2
(a) – NH4NO3 ;
21. Among the following compounds insoluble in water
is/are (b) – NH3 ;
PbI2, HgI2, Hg2OCl2, NaCl, KI, AgCl (c) – (NaNO3 + NaOH) ;
22. The total number of following ions which interfere (d) – N2O ;
with the brown ring test for NO3– is /are (e) – H2O
Br–, I–, CO32–, NO2–, ClO3–, CrO4–2 A to E compounds are given. How many are
23. 'X' (s) (salt) + K2Cr2O7 (s) + conc. H2SO4 correctly given ?
30. Read the following reaction carefully :
on heating
(A) Cu + HNO4(dil.)  NO + A
+NaOH (B) Pb(NO3)2 
heat PbO + B
'Z' 'Y' (gas only)
(yellow solution) (C) CuSO4 + NH4OH(excess)  C
The maximum number of difference in oxidation (D) AgCl + NH4OH  D
state between central atom of ‘Y’ and ‘Z’ is :-
A,B,C,D are mixture of compounds which are
24. Among the following total number of compounds,
given as follows :-
which give(s) white precipitate with calcium
chloride solution. (a) – Cu(NO3)2 , H2O ;
Na2S2O3 ; Na2CO3 ; Na2C2O4 ; NaBr ; NaI (b) – NO2 , O2 ;
25. The total number of reagent(s) give white (c) – [Cu(NH3)4] SO4 + H2O ;
precipitate with solution of sodium oxalates. (d) – Ag(NH2)2Cl + H2O
(MnO(OH)2 + H2SO4), CaCl2 , AgNO3 , BaCl2 , How many are correctly given ?
KMnO4 , (K2Cr2O7 + H2SO4) 31. How many hydroxides are white precipitate ?
26. Among the following total number of anions, which (i) Zn(OH)2 (ii) Cd(OH)2
decolourise the potassium paramagnate solution
(iii) Mn(OH)2 (iv) Al(OH)3
in acid medium.
(v) Ni(OH)2 (vi) Co(OH)2
C2O42– ; NO2– ; (BaCl2 + SO42–) ; SO32–, NO3–
(vii) Pb(OH)2 (viii) Fe(OH)3
27. Find out the total number of compounds, which
can be dissolved by both dil. HNO3 and NaOH. (ix) Cu(OH)2

www.jeebooks.in
4.34 QUALITATIVE SALT ANALYSIS
32. How many hydroxide precipitate are soluble in –
Cl (excess)
P
excess NaOH ? 38. Ni
2+

(i) Zn(OH)2 (ii) Cd(OH)2 NH3(excess) Q


(iii) Mn(OH)2 (iv) Al(OH)3 Coordination number of Ni2+ in P and Q are
(v) Ni(OH)2 (vi) Co(OH)2 respectively a and b. Find out b – a :-
(vii) Pb(OH)2 (viii) Fe(OH)3 39. How many of the following will dissolve in
(ix) Cu(OH)2 concentrated NaOH as well as in concentrated
33. How many hydroxide precipitate are soluble in HCl.
excess NH4OH ? Fe(OH) 2 ; Fe(OH) 3 ; Cr(OH) 3 ; Al(OH) 3 ;
(i) Zn(OH)2 (ii) Cd(OH)2 Zn(OH)2 ; Sn(OH)2
(iii) Mn(OH)2 (iv) Al(OH)3 40. In how many of the following reactions, one of the
products is obtained as a yellow precipitate?
(v) Ni(OH)2 (vi) Co(OH)2
Ba2+ CrO42–  Product
(vii) Pb(OH)2 (viii) Fe(OH)3
Hg2+ + Co2+ + 4SCN–  Product
(ix) Cu(OH)2
BI3 + C6H3 (OH)3  Product
34. NaCl + Solid K2Cr2O7 + conc. H2SO4(few drops)
Mn(OH)2 + O2  Product
X(Reddish-brown fumes)
Zn2+ + 2H PO42–  Product
How many statements are correct regarding X :-
NH4+ + [PtCl6]2–  Product
(i) No axial d-orbitals is involved in hybridization
of X 41. Na2S + Na2[Fe(CN)5NO]  X (Purple color)
(ii) One s-orbital is involved in hybridization of X How many statements are correct regarding X :-
(iii) The complex does not have any unpaired (i) Six ligands are present in compound X
electron. (ii) Ambidentate ligand is present in X
(iv) Three non axial d-orbitals are involved in (iii) Two d-orbitals are involved in hybridization
hybridization of X of X
(v) Magnetic moment of X is zero (iv) Magnetic moment of X is zero
35. A solution containing several unknown cations is (v) Two axial d-orbitals are involved in
treated with dil. HCl and no ppt. forms. The ppt. hybridization of X
is filtered and the filterate at pH 1.0 is treated with (vi) The total possible linkage isomer of X is four
H2S, no ppt. forms. At pH 8.0 H2S causes the 42. How many of the following pair of ions can be
formation of a ppt., the filterate form which gives separated by H2S in dilute HCl.
no ppt. on treatment with Na2CO3. Which group
Mn2+, Cd2+ ; Cr3+, Cu2+ ; As3+, Sn2+ ; Sb3+, Pb2+;
of cations are present in the original solution.
Bi3+, Sn4+ ; Hg2+, Fe3+
36. When NaOH is added to K2Cr2O7 solution , it
43. Find the number compounds which are
becomes yellow. The change in oxidaton state of
producing the oxides of their metal on thermal
Cr in this phenomenon is ..........
decomposition
37. Amongst the following, the total number of
Mg3N2 , Mg(OH)2 , SrCO3 , Ba(NO3)2 , KClO3
compounds soluble in concentrated NH3 solution
is : 44. Among the following , total number of cations tend
to form soluble complex with excess NH4OH(aq.)
BaSO 4 , Ni(OH) 2 , Zn 3 (PO 4 ) 2 , Ag 2 CrO 4 ,
and excess NaCN(aq).
PbSO 3 , Al(OH) 3, Mn(OH) 2 , Bi(OH) 2 NO 3,
Cu(OH)2.CuSO4 Pb2+ ; Cd2+ ; Hg2+ ; Bi3+ ; Cu2+ ; Ag+

www.jeebooks.in
QUALITATIVE SALT ANALYSIS 4.35
45. Among the following , find out the total numebr of 2. Which products are expected from the
black metal sulphides :- disproportionation of hypochlorous acid :-
CuS ; NiS ; CoS ; ZnS ; As2S3 [AIEEE - 2002]
46. Find the number of reagents in which Ag2S is (a) HClO3 and Cl2O (b) HClO2 and HClO
insoluble :- (c) HCl and Cl2O (d) HCl and HClO3
hot conc. HNO 3 ; (NH 4)2S ; NH 3 ; KCN ; 3. A metal M readily forms its sulphate MSO4 which
Na2S2O3 is water soluble. It forms oxide MO which
47. Among the following , total number of becomes inert on heating. It forms insoluble
compound(s) reduce(s) the yellow solution of hydroxide which is soluble in NaOH. The metal
FeCl3. M is :- [AIEEE - 2002]
Tin(II) chloride; Potassium iodide; Hydroxylamine; (a) Mg (b) Ba
Hydrazin sulphate; Ascorbic acid; H y d r o g e n (c) Ca (d) Be
sulphide; 4. Which statement is correct ? [AIEEE - 2003]
Sulphur dioxide; Potassium dichromate; Cconc. (a) Fe3+ ions give deep green precipitate with
nitric acid K4[Fe(CN)6]
48. Find out the total number of reagent(s) which (b) On heating K+, Ca2+ and HCO3– ions, we get
converts chromium(III) ion to chromate ion :- a precipitate of K2[Ca(CO3)2]
H2O2 solution; (NaBO3 . 4H2O + H2O2); NaOBr; (c) Manganese salts give a voilet borax bead test
FeSO4; NaOH; K2S2O8 in reducing flame
49. How many reagents are used to identify chromium (d) From a mixed precipitate of AgCl and AgI
after oxidation to chromate ion ? ammonia solution dissolves only AgCl.
BaCl2; CaCl2; (Acidified H2O2 - Amyl alcohol); 5. What would happen when a solution of potassium
AgNO3 chromate is treated with an excess of dilute nitric
50. Find total number of reagents in which oxidation acid :- [AIEEE - 2003]
3+ 2–
of Fe2+ ion to Fe3+ takes place : (a) Cr and Cr2O7 are formed
(a) On exposure to air (b) Cr2O72– and H2O are formed
(b) On reaction with SnCl2 (c) Cr2O72– is reduced to +3 state of Cr
(c) On reaction with MnO4– / H+ (d) Cr2O72– is oxidised to +7 state of Cr
(d) On reaction with Cr2O72– / H+ 6. Ammonia forms the complex in [Cu(NH3)4]2+ with
(e) On addition of concentrated HNO3 copper ions in alkaline solutions but not in acidic
solution. What is the reason for it :-
(f) On addition with H2O2
[AIEEE - 2003]
(g) On reaction with KI
(a) In acidic solutions hydration protects copper
EXERCISE # V(A) (JEE-MAIN) ions
(b) In acidic solutions protons coordinate with
1. How do we differentiate between Fe3+ and Cr3+ ammonia molecules forming NH4+ ion and
in qualitative analysis gp. III :- [AIEEE - 2002] NH3 molecules are not available.
(a) By taking excess of NH4OH (c) In alkaline solutions insoluble Cu(OH)2 is
(b) By increasing NH4+ ion concentration precipitated which is soluble in excess of any
(c) By decreasing OH– ion concentration alkali
(d) Both (b) and (c) (d) Copper hydroxide is an amphoteric
substance

www.jeebooks.in
4.36 QUALITATIVE SALT ANALYSIS
7. Excess of KI reacts with CuSO4 solution and then 14. Fire extinguishers contain H2SO4 and which one
Na2S2O3 solutions is added to it. Which of the of the following :-
statement is incorrect for this reaction :- (a) CaCO3 [Jee(Main)-2012, Online_P-1]
(a) Evolved I2 is reduced [AIEEE - 2004] (b) NaHCO3 and Na2CO3
(b) CuI2 is formed (c) Na2CO3
(c) Na2S2O3 is oxidised (d) NaHCO3
(d) Cu2I2 is formed 15. Copper wire test for halogens is known as :-
8. Calomel on reation with NH4OH gives :- [Jee(Main)-2012, Online_P-2]
[AIEEE - 2004] (a) Duma’s Test (b) Beilstein’s Test
(a) HgNH2Cl (b) NH2 – Hg – Hg – Cl (c) Lassigne’s Test (d) Liebig’s Test
(c) Hg2O (d) HgO 16. The standard potentials of Ag+ / Ag, Hg22+ / 2Hg,
9. One mole of magnesium nitride on reaction with Cu2+ / Cu and Mg2+ / Mg electrodes are 0.80,
excess of water gives :- [AIEEE - 2004] 0.79, 0.34 and –2.37 V, respectively. An aqueous
solution which contains one mole per litre of the
(a) Two mole of HNO3 (b) Two mole of NH3
salts of each of the four metals is electrolyzed. With
(c) 1 mole of NH3 (d) 1 mole of HNO3 increasing voltage , the correct sequence of
10. The products obtained on heating LiNO3 will be :- deposition of the metals at the cathode is :-
[AIEEE - 2004] [Jee(Main)-2012, Online_P-3]
(a) LiNO2 + O2 (b) Li2O + NO2 + O2 (a) Cu, Hg, Ag only
(c) Li3N + O2 (d) Li2O + NO + O2 (b) Mg, Cu, Hg, Ag
11. What is the best description of the change that (c) Ag, Hg, Cu only
occurs when Na2O(s) is dissolved in water ? (d) Ag, Hg, Cu, Mg
[AIEEE - 2011] 17. Beilstein test is used for estimation of which one of
(a) Oxidation number of sodium decreases following elements :-
(b) Oxide ion accepts sharing in a pair of electrons [Jee(Main)-2012, Online_P-3]
(c) Oxide ion donates a pair of electron (a) S (b) Cl
(d) Oxidation number of oxygen increases (c) C and H (d) N
12. Which of the following on thermal - decomposition 18. In the following balanced reaction, XMnO4– +
Z
yields a basic as well as an acidic oxide. YC2O42– + ZH+ = XMn2+ + 2YCO2 + H2O the
2
(a) NH4NO3 [AIEEE - 2012] values of X, Y and Z respectively are :-
(b) NaNO3 [Jee(Main)-2012, Online_P-4]
(c) KClO3 (a) 8, 2, 5 (b) 5, 2, 16
(d) CaCO3 (c) 2, 5, 16 (d) 5, 8, 4
13. The correct statement for the molecule, CsI3, is :- 19. A metal M on heating in nitrogen gas gives Y. Y
[Jee(Main) - 2014] on treatment with H 2O gives a colourless
(a) It contains Cs3+ and I– ions gas which when passed through CuSO4 solution
gives a blue colour , Y is :-
(b) It contains Cs+ , I– and lattice I2 molecule
[Jee(Main)-2012, Online_P-4]
(c) It is a covalent molecule
(a) NH3 (b) MgO
(d) It contains Cs+ and I3– ions
(c) Mg3N2 (d) Mg(NO3)2

www.jeebooks.in
QUALITATIVE SALT ANALYSIS 4.37
20. Electrode potential (E°) are given below :- 25. Values of dissociation constant , Ka are given as
Cu+ / Cu = + 0.52 V, Fe3+ / Fe2+ = +0.77 V, follows :- [Jee(Main)-2013, Online_P-2]
1 Acid Ka
I (s) / I– = + 0.54 V,, Ag+ / Ag = +0.88 V. HCN 6.2 × 10–10
2 2
Based on the above potential , strongest oxidizing HF 7.2 × 10–4
agent will be :- [Jee(Main)-2013, Online_P-1] HNO2 4.0 × 10–4
(a) Cu+ (b) Fe3+ Correct order of increasing base strength of the
(c) Ag+ (d) I2 base CN–, F– and NO2– will be
21. Potassium dichromate when heated with (a) NO2– < CN– < F– (b) F– < CN– < NO2–
concentrated sulphuric acid and a soluble (c) NO2– < F– < CN– (d) F– < NO2– < CN–
chloride , gives brown red vapours of :- 26. Identify incorrect statement :-
[Jee(Main)-2013, Online_P-1] [Jee(Main)-2013, Online_P-3]
(a) CrO3 (b) Cr2O3 (a) Copper (I) compounds are colourless except
(c) CrCl3 (d) CrO2Cl2 where colour result from charge transfer
22. Given : [Jee(Main)-2013, Online_P-1] (b) Copper (I) compounds are diamagnetic
X Na2 HAsO3 + Y NaBrO3 + ZHCl  (c) Cu2S is black
NaBr + H3AsO4 + NaCl (d) Cu2O is colourless
The values of X, Y and Z in the above redox 27. Given :- [Jee(Main)-2013, Online_P-3]
reaction are respectively :-
E1/o 2Cl –
o
 1.36V, E Cr 3  0.74V ;
2 / Cl / Cr
(a) 2,1,3 (b) 3,1,6
(c) 2,1,2 (d) 3,1,4 E oCr O2– / Cr3  1.33V, E oMnO / Mn 2  1.51V
2 7 4

23. Sodium carbonate cannot be used in place of The correct order of reducing power of the species
(NH4)2CO3 for the identification of Ca2+, Br2+ and (Cr, Cr3+ , Mn2+ and Cl–) will be :-
Sr2+ ion (in group V) during mixture analysis (a) Mn2+ < Cl– < Cr3+ < Cr
because :- [Jee(Main)-2013, Online_P-1]
(b) Cr3+ < Cl– < Mn2+ < Cr
(a) Sodium ions will react with acid radicals
(c) Cr3+ < Cl– < Cr < Mn2+
(b) Concentration of CO32– ions is very low
(d) Mn2+ < Cr3+ < Cl– < Cr
(c) Mg2+ ions will also be precipitated
28. Which one of the following cannot function as an
(d) Na+ ions will interfere with the detection of oxidising agent ? [Jee(Main)-2013, Online_P-4]
Ca2+, Ba2+, Sr2+ ions :-
(a) NO3–(aq)
24. Which of the following statement is incorrect :-
(b) I–
[Jee(Main)-2013, Online_P-2]
(c) Cr2O72–
(a) Fe ion also gives blood red colour with SCN–
2+
(d) S(s)
ions
29. Which of the following statement about Na2O2 is
(b) Cupric ion reacts with excess of ammonia
not correct :- [Jee(Main)-2014, Online_P-2]
solution to give deep blue colour of
[Cu(NH3)4]2+ ion (a) Na2O2 oxidises Cr3+ to CrO42– in acid medium
(c) Fe3+ ion gives blood red colour with SCN– ion (b) It is diamagnetic in nature
(d) On passing H2S into Na2ZnO2 solution , a (c) It is the super oxide of sodium
white ppt of ZnS is formed (d) It is a derivative of H2O2

www.jeebooks.in
4.38 QUALITATIVE SALT ANALYSIS
30. Given :- [Jee(Main)-2014, Online_P-2] 35. Consider the reaction :-
Fe3+(aq) + e– Fe2+(aq) ; E° = +0.77 V H2SO3(aq) + Sn4+(aq) + H2O(l) Sn2+(aq) + HSO4–
Al3+(aq) + 3e– Al(s) ; E° = –1.66 V (aq)
+ 3H+(aq)
Br2(aq) + 2e– 2Br–(aq) ; E° = +1.09 V Which of the following statement is correct :-
Considering the electrode potentials, which of the [Jee(Main)-2014, Online_P-3]
following represents the correct order of reducing (a) H2SO3 is the reducing agent because it
power ? undergoes oxidation
(a) Al < Fe2+ < Br– (b) Al < Br– < Fe2+ (b) H2SO3 is the reducing agent because it
(c) Fe2+ < Al < Br– (d) Br– < Fe2+ < Al undergoes reduction
31. Consider the following equilibrium :- (c) Sn4+ is the reducing agent because it undergoes
oxidation
AgCl+ 2NH3   [Ag(NH3)2]+ + Cl–
(d) Sn 4+ is the oxidizing agent because it
White precipitate of AgCl appears on adding which
undergoes oxidation
of the following ? [Jee(Main)-2014, Online_P-2]
36. How many electrons are involved in the following
(a) NH3 (b) Aqueous NaCl
redox reaction :- [Jee(Main)-2014, Online_P-4]
(c) Aqueous NH4Cl (d) Aqueous HNO3
Cr2O72– + Fe2+ + C2O42– Cr3+ + Fe3+ + CO2
32. Copper becomes green when exposed to moist (Unbalanced)
air for a long period. This is due to :-
(a) 3
[Jee(Main)-2014, Online_P-3]
(b) 4
(a) the formation of a layer of cupric oxide on the
(c) 5
surface of copper
(d) 6
(b) the formation of basic copper sulphate layer
on the surface of the metal 37. Amongst the following , identify the species with
an atom in +6 oxidation state :-
(c) the formation of a layer of cupric hydroxide
on the surface of copper (a) [MnO4]– [Jee(Main)-2014, Online_P-4]
(d) the formation of a layer of basic carbonate of (b) [Cr(CN)6]3–
copper on the surface of copper (c) Cr2O3
33. When one of the following exhibits the largest (d) CrO2Cl2
number of oxidation states ?
EXERCISE # V(B)
[Jee(Main)-2014, Online_P-3]
(JEE-ADVANCED)
(a) Mn(25) (b) V(23)
1. Which of the following statement(s) is (are) correct
(c) Cr(24) (d) Ti(22)
with reference to the ferrous and ferric ions :
34. Hydrogen peroxide acts both as an oxidising and [Jee 1998]
as a reducing agent depending upon the nature of
(a) Fe3+ gives brown colour with potassium
the reacting species. In which of the following cases
ferricyanide
H 2 O 2 acts as a reducing agent in acidic
medium ? [Jee(Main)-2014, Online_P-3] (b) Fe2+ gives blue precipitate with potassium
– ferricyanide
(a) MnO4
(c) Fe 3+ gives red colour with potassium
(b) SO32–
thiocyanate
(c) KI
(d) Fe2+ gives brown colour with ammonium
(d) Cr2O72– thiocyanate

www.jeebooks.in
QUALITATIVE SALT ANALYSIS 4.39
2. Which of the following statements(s) is / are 8. A metal nitrate reacts with KI to give a black
correct. When a mixture of NaCl and K2Cr2O7 is precipitate which on addition of excess of KI
gently warmed with conc. H2SO4? [Jee 1998] convert into orange colour solution. The cation of
(a) A deep red vapours is evolved metal nitrate is :- [Jee 2005]
2+ 3+
(b) The vapours when passed into NaOH solution (a) Hg (b) Bi
gives a yellow solution Na2CrO4 (c) Pb2+ (d) Cu2+
(c) Chlorine gas is evolved 9. A solution when diluted with H2O and boiled , it
(d) Chromyl chloride is formed gives a white precipitate. On addition of excess
NH4Cl / NH4OH, the volume of precipitate
3. An aqueous solution of a substance gives a white decreases leaving behinde a white gelatinous
precipitate on treatment with dilute hydrochloric precipitate. Identify the precipitate which dissolves
acid, which dissolves on heating. When hydrogen in NH4OH / NH4Cl? [Jee 2006]
sulphide is passed through the hot acidic
(a) Zn(OH)2 (b) Al(OH)3
solution, a black precipitate is obtained. The
substance is a : [Jee 2000] (c) Mg(OH)2 (d) Ca(OH)2
(a) Hg2+ salt (b) Cr2+ salt 10. CuSO4 decolorises on addition of KCN, the
product is [Jee 2006]
(c) Ag+ salt (d) Pb2+ salt
(a) [Cu(CN)4]2–
4. A gas ‘X’ is passed through water to form a
(b) Cu2+ get reduced to form [Cu(CN)4]3–
saturated solution. The aqueous solution on
treatment with silver nitrate gives a white (c) Cu(CN)2
precipitate. The saturated aqueous solution also (d) CuCN
dissolves magnesium ribbon with evolution of a 11. Consider a titration of potassium dichromate
colourless gas ‘Y’. Identify ‘X’ and ‘Y’ : solution with acidified Mohr’s slat solution using
[Jee 2002 (Mains)] diphenylamine as indicator. The number of moles
of Mohr’s salt required per mole of dichromate
(a) X = CO2, Y = Cl2 (b) X = Cl2, Y = CO2
is :- [Jee - 2007]
(c) X = Cl2, Y = H2 (d) X = H2, Y = Cl2
(a) 3 (b) 4
5. [X] + H2SO4  [Y] a colourless gas with irritating
(c) 5 (d) 6
smell :- [Jee-2003]
12. The species present in solution when CO2 is
[Y] + K2Cr2O7 + H2SO4  green solution
dissolved in water are? [JEE - 2007]
[X] and [Y] are :- – 2–
(a) CO2 , H2CO3 , HCO3 , CO3
(a) SO32– , SO2 (b) Cl– , HCl
(b) H2CO3 , CO32–
(c) S2– , H2S (d) CO32– , CO2 (c) CO32– , HCO3–
6. A sodium salt of an unknown anion when treated (d) CO2 , H2CO3
with MgCl2 give white precipitate only on boiling.
13. Sodium fusion extract , obtained from anilione , on
The anion is :- [Jee 2004]
treatment with iron (II) sulphate and H2SO4 in
(a) SO42– (b) HCO3– presence of air gives a Prussian blue precipitate.
(c) CO32– (d) NO3– The blue colour is due to the formation of :-
7. (NH4)2Cr2O7 on heating gives a gas which is also (a) Fe4[Fe(CN)6]3 [Jee 2007]
given by :- [Jee 2004] (b) Fe3[Fe(CN)6]2
(a) hating NH4NO2 (b) heating NH4NO3 (c) Fe4[Fe(CN)6]2
(c) Mg3N2 + H2O (d) NaNO2 + H2O2 (d) Fe3[Fe(CN)6]3

www.jeebooks.in
4.40 QUALITATIVE SALT ANALYSIS
14. Column - I Column - II 20. Match each of the reactions given column I
[Jee 2007] with the corresponding product(s) given in
(A) O2–  O2 + O22– (P) Redox reaction column II :- [JEE-2009]
(B) CrO42– + H+  (Q) One of the products Column-I Column-II
has trigonal structure (A) Cu + dil. HNO3 (P) NO
(C) MnO4– + NO2– (R) Dimeric bridged (B) Cu + conc. HNO3 (Q) NO2
+ H+  tetrahedral metal ion (C) Zn + dil. HNO3 (R) N2O
(D) NO3– + H2SO4 (S) Disproportionation (D) Zn + conc. HNO3 (S) Cu(NO3)2
+ Fe2+  (T) Zn(NO3)2
15. A solution of a metal ion when treated with KI 21. Passing H2S gas into a mixture of Mn2+, Ni2+, Cu2+
gives a red precipitate which dissolves in excess and Hg2+ ions in an acidified aqueous solution
KI to give a colourless solution. Moreover , the precipitates :- [JEE-2011]
solution of metal ion on treatment with a solution (a) CuS and HgS (b) MnS and CuS
of cobalt (II) thiocyanate gives rise to a deep blue
crystalline precipitate. The metal ion is :- (c) MnS and NiS (d) NiS and HgS
(a) Pb2+ (b) Hg2+ [Jee 2007] 22. Reduction of the metal centre in aqueous
permangante ion involves [JEE-2011]
(c) Cu2+ (d) Co2+
(a) 3 electrons in neutral medium
16. A solution of colourless salt H on boiling with
excess NaOH produces a non-flammable gas. (b) 5 electrons in neutral medium
The gas evolution cases after sometime. (c) 3 electrons in alkaline medium
Upon addition of Zn dust to the same solution, (d) 5 electrons in acidic medium
the gas evolution restarts. The colourless salt(s) H
is (are) :- [Jee 2007] 23. The equilibrium :- [JEE-2011]
(a) NH4NO3 (b) NH4NO2  Cu° + CuII
2CuI 
(c) NH4Cl (d) (NH4)2SO4 in aqueous medium at 25° shifts towards the left in
Passage for Q. 17 to 19 the presence of :-
p-Amino-N, N-dimethylaniline is added to a strongly (a) NO3– (b) Cl–
acidic solution of X. The resulting solution is treated
(c) SCN– (d) CN–
with a few drops of aqueous solution of Y to yield blue
coloration due to the formation of methylene blue. Passage for Q. 24 to 26
Treatment of the aqueous solution of Y with the reagent When a metal rod M is dipped into an aqueous
potassium hexacyanoferrate(II) leads to the formation colourless concentrated solution of compound N, the
of an intense blue precipitate. The precipitate dissolves solution turns light blue. Addition of aqueous NaCl to
on excess addition of the reagent. Similarly, treatment te blue solution gives a white precipitate O. Addition
of the solution of Y with the solution of potassium of aqueous NH3 dissolves O and gives in intense blue
hexacyanoferrate(II) leads to a brown coloration due solution. [JEE-2011]
to the formation of Z. [JEE-2009]
24. The metal rod M is :-
18. The compound Y is :-
(a) Fe (b) Cu
(a) MgCl2 (b) FeCl2
(c) Ni (d) Co
(c) FeCl3 (d) ZnCl2
25. The compound N is :-
19. The compound Z is :-
(a) AgNO3 (b) Zn(NO3)2
(a) Mg2[Fe(CN)6] (b) Fe[Fe(CN)6]
(c) Fe4[Fe(CN)6]3 (d) K2Zn3[Fe(CN)6]2 (c) Al(NO3)3 (d) Pb(NO3)2

www.jeebooks.in
QUALITATIVE SALT ANALYSIS 4.41
26. The final solution contains :- 28. The reaction of white phosphorus with aqueous
(a) [Pb(NH3)4]2+ and [CoCl4]2– NaOH gives phosphine along with another
(b) [Al(NH3)4]3+ and [Cu(NH3)4]2+ phosphorus containing compound. The
(c) [Ag(NH3)4]2+ and [Cu(NH3)4]2+ reaction type : the oxidation states of
phosphorus in phosphine and the other product
(d) [Ag(NH3)2]2+ and [Ni(NH3)6]2+
are respectively : [JEE-2012]
27. Which of the following hydrogen halides react(s)
with AgNO3(aq) to give a precipitate that dissolves (a) redox reaction : –3 and –5
in Na2S2O3(aq) :- [JEE-2012] (b) redox reaction : +3 and +5
(a) HCl (b) HF (c) disproportionation reaction : –3 and –5
(c) HBr (d) HI (d) disproportionation reaction : –3 and +3
29. For the given aqueous reactions, which of the statement(s) is (are) true :- [JEE-2012]
dilute H 2SO 4
excess KI + K3[Fe(CN)6] brownish-yellow solution

ZnSO4

white precipitate + brownish - yellow solution

Na2S2O3

colourless solution
(a) The first reaction is a redox reaction (b) White precipitate is Zn3[Fe(CN)6]2
(c) Addition of filtrate to starch solution gives blue colour (d) White precipitate is soluble in NaOH solution
30. Upon treatment with ammonical H2S , the metal ion that precipitate as a sulfide is [JEE-2013]
(a) Fe(III) (b) Al(III)
(c) Mg(II) (d) Zn(II)
Passage for Q. 31 & 32
An aqueous solution of a mixture of two inorganic salts, when treated with dilute HCl, gave a precipitate (P) and
a filtrate (Q). The precipitate (P) was found to dissolve in hot water. The filtrate (Q) remained unchanged , when
treated with H2S in a dilute mineral acid medium. However, it gave a precipitate (R) with H2S in an ammonical
medium. The precipitate R gave a coloured solution (S), when treated with H2O2 in an aqueous NaOH medium.
31. The coloured solution (S) contains :- [JEE-2013]
(a) Fe2(SO4)3 (b) CuSO4
(c) ZnSO4 (d) Na2CrO4
32. The precipitate (P) contains :- [JEE-2013]
2+ 2+
(a) Pb (b) Hg2
(c) Ag+ (d) Hg2+

www.jeebooks.in
4.42 QUALITATIVE SALT ANALYSIS
33. Consider the following list of reagents :- [JEE-ADV.2014]
Acidified K2Cr2O7 , alkaline KMnO4 , CuSO4 , H2O2 , Cl2 , O3 , FeCl3 , HNO3 and Na2S2O3
The total number of reagents that can oxidise aqueous iodide to iodine is :
34. Among PbS, CuS, HgS, MnS, Ag2S, NiS, CoS, Bi2S3 and SnS2 the total number of black coloured
sulphide is. [JEE-ADV. 2014]
Passage for Q. 35 & 36
An aqueous solution of metal ion M1 reacts separately with reagent Q and R in excess to give tetrahedral and
square planar complexes , respectively . An aqueous solution of another metal ion M2 always forms tetrahedral
complex with these reagents. Aqueous solution of M2 on reaction with reagent S gives white precipitate which
dissolves in excess of S. The reaction are summarized in the sheme given below : [JEE-ADV.2014]
SCHEME :
Q R
Tetrahedral M1 Square planar
excess excess
Q R
Tetrahedral M2 Tetrahedral
excess excess

S, Stoichiometric amount

S
White precipitate Precipitate dissolves
excess
35. M1, Q and R respectively are :-
(a) Zn2+ , KCN and HCl (b) Ni2+ , HCl and KCN
(c) Cd2+ , KCN and HCl (d) Co2+ , HCl and KCN
36. Reagent S is :-
(a) K4[Fe(CN)6] (b) Na2HPO4
(c) K2CrO4 (d) KOH
37. Fe3+ is reduced to Fe2+ by using :- [JEE-Adv.-2015]
(a) H2O2 in presence of NaOH (b) Na2O2 in water
(c) H2O2 in presence of H2SO4 (d) Na2O2 in presence of H2SO4
38. The paris of ions where BOTH the ions are precipitated upon passing H2S gas in presence of dilute HCl, is
(are) :- [JEE-ADV.2015]
(a) Ba2+ , Zn2+ (b) Bi3+, Fe3+
(c) Cu2+ , Pb2+ (d) Hg2+ , Bi3+



www.jeebooks.in
THE s-BLOCK ELEMENTS AND THEIR COMPOUNDS
PROB L EM S B ASED ON GI VEN T OPI CS The Chlor-Alkali industry
Group 1-the Alkali metals  Sodium hydroxide
 General properties  Lebanc process
 Electronic structure  Weldon and deacon processes
 Size of atoms and ions  Electrolytic processes
 Density  Diaphragm cell
 Ionization energy  Mercury catode cell
 Electronegativity and bond type  Quantities
 Born - Haber cycle : Energy chages in the formation  Sodium carbonate
of ionic compounds  The solvay (or Ammonia - Soda) process
 Structures of te metals, Hardness and Cohesive Group 2-the Alkaline earth elements
energy  General properties
 Melting and bouling points  Electronic structure
 Flame coloures and spectra  Size of atoms and ions
 Colour of compounds  Ionisation energy
 Chemical properties  Electronegativity
 Reaction with water  Hydration energies
 Reaction with air  Solubility and lattice energy
 Reaction with dinitrogen  Solutions of the metals in liquid ammonia
 Oxides, Hydroxies, Peroxides and Superoxides  Anamalous behaviour of Beryllium
 Reaction with air  Chemical properties
 Normal oxides - monoxides  Reaction with water
 Hydroxides  Hydroxides
 Peroxides and Superoxides  Hardness of water
 Sulphides  Reaction with acid and bases
 Oxosalts - Carbonates, Bicarbonates, Nitrates,  Oxides and pertoxides
Nitrites and Sulphates  Sulphates
 Halides and Polyhalides  Nitrates
 Hydrides  Hydrides
 Solubility and hydration  Halides
 Solution of metals in liquid ammonia  Nitrides
 Comounds with carbon  Carbides
 Complexes, crowns and crypts  Complexes
 Biological importance  Biological role of Mg2+ and Ca2+
 Difference between Lithium and the other group 1
elements

www.jeebooks.in
CHAPTER
5
The s-Block Elements and
Their Compounds
EXERCISE # I
 Only one correct answer : 8. Alkali metals can be kept in :
1. Sodium pentasulphide is (Na2S5). Anionic part S52– (a) H2O (b) CH3OH
is : (c) CH3CO2H (d) Kerosene oil
(a) Tetrahedral (b) Bent 9. The ionic mobility of alkali metals in aqueous solution
(c) Trigonal planar (d) Trigonal bipyramidal is maximum for
2. When Na reacts with NH3 gas at 360ºC , the gas (a) Rb+ (b) K+
evolved is : (c) Na+ (d) Li+
(a) N 2 (b) N 2 O 10. The products obtained on heating LiNO3 will be
(c) NO (d) H2 (a) LiNO2 + O2 (b) Li2O + NO2 + O2
3. Aqueous solution of Na2CO3 is : (c) Li3N + O2 (d) Li2O + NO + O2
(a) Acidic (b) Basic 11. When Li is combusted in air the mixture of product
(c) Neutral (d) None is :
4. 2Na + 2NH3 2NaNH2 + H2 (a) Li2O + Li2O2 (b) Li3N only
In this reaction NH3 acts as (c) Li2O + Li3N (d) Li3N + LiO2
(a) Reducing agent (b) Dehydrating reagent 12. Which Na is exposed to air the mixture of product
(c) Basic gas (d) Oxidising reagent is :
5. A soft metal (A) gives violet flame test. When burnt (a) Na2O + Na3N (b) Na2O + NaO2
in excess O2, A give yellow powder B which on (c) Na2O + Na2O2 (d) Na3N + NaO2
reaction with water gives alkaline solution A & B 13. What happened when to the aqueous solution of
are respectively : Na2O, red litmus paper is added :
(a) Na ; Na2O2 (b) K ; K2O2 (a) No change in colour
(c) K ; KO2 (d) Li ; Li2O (b) Red litmus paper becomes blue (permanent)
6. Which of the following pair of compounds does (c) Red litmus paper becomes blue but after
not coexist ? sometime the colour disappered
(a) NaOH + Na2CO3 (d) Red litmus paper becomes green
(b) NaOH + NaHCO3 14. What happened when to the aqueous solution of
(c) NaOH + Na Na2O2 , red litmus paper is added
(d) NaOH + NaNH2 (a) No change in colour
7. Which of the following substance is not related to (b) Red litmus paper becomes blue (permanent)
the Solvay process ? (c) Red litmus paper becomes blue but after
(a) CaCO3 (b) NaHCO3 sometime the colour disappered
(c) NH3 (d) KCl (d) Red litmus paper becomes green

www.jeebooks.in
5.4 THE s-BLOCK ELEMENTS AND THEIR COMPOUNDS
15. Weakest base among the following : 24. To prepare BeF2 the best route is :
(a) Ca(OH)2 (b) Ba(OH)2 (a) Thermal decomposition of (NH4)2BeF4
(c) Be(OH)2 (d) KOH 600ºC
(b) BeO + C + F2 BeF2 + CO
16. If I2 and I’2 is the second ionisation potentials of
(c) CaF2 + BeO  BeF2 + CaO
alkali and alkaline earth metal for same period
respectively then. (d) Be + F2  BeF2
(a) I2 = I’2 (b) I2 > I’2 25. Cement clinker is :
(c) I’2 > I2 (d) I’2 > > > I2 (a) Fused mass of clay & gypsum
17. Bleaching powder, a mixture of CaCl2 + Ca(OCl)2 (b) Fused mass of lime & gypsum
is obtained by passing Cl2 gas with. (c) Fused mass of clay & lime
(a) Ca(OH)2 (b) CaO (d) Fused mass of Ca(OH)2 & CaSO4.1/2H2O
(c) CaCO3 (d) CaCl2 26. The milk of magnesia used as antacid is chemically
18. Hydroxide of first element of group-IA and group- (a) Mg(OH)2
IIA are respectively :
(b) MgO
(a) LiOH, Be(OH)2
(c) MgCl2
(b) NaOH, Mg(OH)2
(d) MgO + MgCl2
(c) KOH, Ca(OH)2
27. Which of the following are amphoteric in nature ?
(d) RbOH, Be(OH)2
(a) BeO (b) Be(OH)2
19. The correct order of 2nd ionisation potential for
Mg, Sr and K is : (c) Al(OH)3 (d) All
(a) K > Sr > Mg (b) Mg > Sr > K 28. MgBr2, MgI2 are soluble in acetone because of
(c) Sr > Mg > K (d) K > Mg > Sr (a) Ionic character
20. The formula of soda ash is : (b) Metallic nature
(a) Na2CO3.10H2O (b) Na2CO3.2H2O (c) Covalent nature
(c) Na2CO3.H2O (d) Na2CO3 (d) Coordination compound
21. The substance that gives brick red flame and breaks 29. White heavy precipitate is formed when BaCl2 is
down on heating to give O2 and a brown gas is added to a clear solution of compound A. If
(a) Magnesium nitrate precipitate is insoluble in dilute HCl then anionic
(b) Calcium nitrate part of compound A is :
(c) Barium nitrate (a) Bicarbonate (b) Carbonate
(d) Sodium nitrate (c) Nitrite (d) Sulphate
22. In the synthesis of Na2CO3, the recovery of Li in
30. 2-Butyne P
Liq. NH3 1
ammonia is done by heating NH4Cl with Ca(OH)2.
+ H2
The by product obtained in this process. 2-Butyne BaSO + Pd P2
4
(a) CaCl2 (b) NaCl
P1 & P2 are related as :
(c) NaOH (d) NaHCO3
(a) Structural isomers
23. Which of the following does not form hydride by
direct heating with H2 (dihydrogen) : (b) Optical isomers
(a) Be (b) Mg (c) Geometrical isomers
(c) Sr (d) Ba (d) Not isomers

www.jeebooks.in
THE s-BLOCK ELEMENTS AND THEIR COMPOUNDS 5.5
31. Which of the following group of elements have 39. The dilute solution of all alkali metal in liquid
chemical properties that is almost similar : ammonia are blue in colour. The blue colour of the
(a) Rb, Cs, Ca (b) Be, Al, Ba solution.
(c) Li, Mg, K (d) Mg, Sr, Ba (a) Due to excitation of free electrons to high energy
32. CsI3, KI3, TlI3 contains : level (sp transition). This type of transition
gives absorption band near 1500º A in red
region of spectrum therefore the solution 1ooks
(a) Monopositive cation Cs+, K+, Tl+ & triiodide
blue in transmitted light.
ion I3–
(b) Due to excitation of free electrons to high energy
(b) Cs+1, K+1, Tl3+ & I3– in CsI3, I3– in KI3 & I–1 in
level (sp transition). This type of transition
TlI3
gives absorption band near 1500º A in yellow
(c) Ca2+, K+, Tl3+ & iodide ion I– region of spectrum therefore the solution 1ooks
(d) Cs+, K3+, Tl+3 ion and I3– ion blue in transmitted light.
33. Among the alkaline earth metal halide given below (c) Due to excitation of free electrons to high energy
which is most covalent in nature ? level (sd transition). This type of transition
(a) SrCl2 (b) BeCl2 gives absorption band near 1500º A in red
(c) MgCl2 (d) CaCl2 region of spectrum therefore the solution 1ooks
34. Which of the following reacts most vigorously with blue in transmitted light.
water ? (d) None of these
(a) Li (b) K 40. When calcium carbide (CaC 2) reacts with
(c) Rb (d) Na atmospheric N2 at very high temperature, the
35. Among nitrate of alkali metal which one can be product mixture formed is :
decomposed to its oxide most easily ? (a) Ca(CN)2 + C (b) CaCN2 + C
(a) NaNO 3 (b) KNO 3 (c) CaNC2 + C (d) Ca3N2 + C
(c) LiNO3 (d) RbNO3 41. The correct formula of hydrate of halide of alkaline
36. Among the carbonate of alkali metals which has earth metal :
highest thermal stability ? (a) MgCl2.2H2O ; CaCl2 ; 4H2O ; SrCl2.6H2O ;
(a) Cs2CO3 (b) Rb2CO3 BaCl2.8H2O
(c) K2CO 3 (d) Na2CO3 (b) MgCl2.8H2O ; CaCl2.4H2O ; SrCl2.2H2O ;
BaCl2.6H2O
37. X + C + Cl2 Chloride salt + CO
(c) MgCl2.8H2O ; CaCl2.6H2O ; SrCl2.6H2O ;
X can be :
BaCl2.2H2O
(a) Na2O, CaO (b) SrO, Cs2O
(d) None
(c) BeO, Al2O3 (d) MgO, K2O
42. X(salt) + dil.H2SO4  Y (gas)
38. CaCO3 can be prepared by :
Y(gas) + Ca(OH)2  Z (miky ppt.)
(a) By passing CO2 through slaked lime Ca(OH)2
The salt X may be :
+ CO2  CaCO3 + H2O
(a) NaNO 2
(b) By the addition of sodium carbonate to calcium
chloride CaCl2 + Na2CO3  CaCO3 + 2NaCl (b) Na2SO3
(c) Both (a) & (b) (c) Na2CO3
(d) None (d) Both (b) & (c)

www.jeebooks.in
5.6 THE s-BLOCK ELEMENTS AND THEIR COMPOUNDS
43. Percentage loss on mass on heating mixture of 48. Which of the alkali metal produce hydrated
Na2CO3 & CaCO3 containing equal mass of the perchlorate (MClO4.3H2O)
two components will be : (a) K (b) Rb
(a) 44% (b) 22% (c) Cs (d) Li
(c) 38% (d) 80% 49. Correct order of weight percentage of calcium for
44. If there three electron been possible to be placed same amount of gypsum & plaster of paris.
in an orbital the position of potassium in periodic (a) (% of Ca)gypsum > (% of Ca)plaster of paris
table would be in which block : (b) (% of Ca)gypsum < (% of Ca)plaster of paris
(a) s (b) p (c) (% of Ca)gypsum = (% of Ca)plaster of paris
(c) d (d) f (d) None of these
45. EºCl /Cl–  +1.36 volt EºAg+/Ag  +0.79 volt 50. Aqueous solution of NaCl & fused NaH when
2
EºI /I–  +0.53 volt EºNa+/Na  –2.71 volt undergo electrolysis, the same gas is evolved at
2
EºLi+/Li  –3.04 volt cathode for NaCl (aq.) & at anode for NaH. What
is that gas ?
Correct order of reducing strength.
– – (a) O 2 (b) Cl2
(a) Li > Cl > Ag > I > Na
– – (c) H2 (d) H2O vapours
(b) Cl > Ag > I > Na > Li
– –
51. Fused NaCl when undergo electrolysis using
(c) Li > Na > I > Ag > Cl proper electrode.
(d) None (a) Na is deposited at anode and Cl2 is evolved at
46. Beryllium & Magnesium are kinetically inert to cathode
oxygen & water (b) Na is deposited at cathode and Cl2 is evolved
(a) Because of the formation of an oxide film on at anode
their surface (c) Na is deposited at cathode and O2 is evolved
(b) Because of their large size at anode
(c) Because of large hydration energy of Be2+ & (d) Na is deposited at anode and H2 is evolved at
Mg2+ cathode
(d) All 52. Calcium is obtained from its ore by :
47. Baking powder is used to cause the bread to swell. (a) Carbon reduction
Baking powder is a (b) Reduction byAluminium
(a) Mixture of sodium potassium tartarate & (c) Electrolytic reduction
sodium carbonate and during reaction CO2 gas (d) Reduction by hydrogen gas
evolved cause the bread to swell 53. Which of the following oxalate is most insoluble ?
(b) Mixture of potassium hydrogen tartarate & (a) BeC2O4
sodium carbonate and during reaction CO2 gas
(b) K2C 2O 4
evolved cause the bread to swell
(c) CaC2O4
(c) Mixture of sodium potassium tartarate &
sodium bicarbonate and during reaction CO2 (d) Na2C2O4
gas evolved cause the bread to swell 54. Which alkaline earth metal has highest melting
point ?
(d) Mixture of potassium hydrogen tartarate &
sodium bicarbonate and during reaction CO2 (a) Be (b) Mg
gas evolved cause the bread to swell (c) Ca (d) Sr

www.jeebooks.in
THE s-BLOCK ELEMENTS AND THEIR COMPOUNDS 5.7
55. Which alkaline earth metal has lowest boiling 62. Lattic energy order
point ? (a) LiF > NaF > KF > RbF > CsF
(a) Be (b) Mg (b) LiF < NaF < KF < RbF < CsF
(c) Ca (d) Sr (c) NaF> KF > LiF > RbF > CsF
56. Fire extinguiser contains. (d) RbF > CsF > KF > NaF > LiF
(a) CaCO3 solution 63. If NaOH is added to an aqueous solution of Al3+
(b) H2SO4 and NaHCO3 solution ions, a white precipitate appears and on adding
(c) NaHCO3 solution excess of NaOH, the precipitatae dissolves. In the
(d) CaCO3 and H2SO4 solution solution, aluminium exists in the
57. Crown ethers & cryptands form (a) anionic part
(a) Complex with alkali metals (b) cationic part
(b) Hydroxides of alkali metals (c) both in anionic and cationic part
(c) Salts of alkali metals used for inorganic (d) colloidal form
quantitative analysis 64. When NaNO3 is treated with Na:
(d) Organic salts of alkali metals (a) NaNO2 and O2 are formed
58. The hydroxides of alkaline earth metal which has (b) Na2O and N2 are formed
lowest value of solubility product at normal (c) Na2O2 and N2 are formed
temperature (25°C) (d) Na2O and Na3N are formed
(a) Ca(OH)2 (b) Mg(OH)2 65. What happens when a mixture of sodium sulphate,
(c) Sr(OH)2 (d) Be(OH)2 lime stone and coke is heated in a furnace?
Co2in (a) Black ash containing sodium carbonate and
59. Na + Al2O3  High temperature X 
water
 Y, the
calcium sulphide is formed
compound Y is (b) Carbon dioxide gas is liberated
(a) NaAlO2 (b) NaHCO3 (c) Sodium carbonate and calcium sulphate is
(c) Na2CO3 (d) Na2O2 formed
60. Which of the following group of element have (d) (a) and (b) both
chemical properties that are most similar 66. Alkali metals possesss metallic lustre when freshly
(a) Na, K, Ca cut because :
(b) Mg, Sr, Ba (a) they have a hard surface and light is reflectd back
(c) Be, Al, Ba (b) their crystal structure contains ordeered
(d) Be, Cs, Ra arrangement of constituent atoms
61. The commercial method of preperation of (c) they contain loosely bound electrons which
potassium by reduction of molten KCl with metallic absorb the photons and then re-emit.
sodium at 50°C is based on the fact that (d) they are obtained from the minerals on which
(a) potassium is solid and sodium is liquid at 850°C light has been falling for years.
(b) sodium has less affinity to chloride ions than 67. Low solubility of CsI in water is due to
potassium (a) smaller hydration enthalpy of Cs+
(c) potassium being more volatile so it distills off (b) smaller hydration enthalpy of I–
thus shifting the reaction forward (c) lower lattice enthalpy of its two ions
(d) reactivity of sodium is more tham potassium (d) (a) and (b) both

www.jeebooks.in
5.8 THE s-BLOCK ELEMENTS AND THEIR COMPOUNDS
68. When ionic hydrides react with water, the products EXERCISE # II
are :
 One or More Than One Correct Answer :
(a) acidic solutions and hydrogen gas
1. When KO2 reacts with water then the products
(b) acidic solutions and oxygen gas are.
(c) basic solutions and hydrogen gas
(a) KOH (b) H2O2
(d) basic solutions and oxygen gas
(c) H2O (d) O 2
69. The stability of K2O, K2O2 and KO2 is in order
2. Which of the following are correct ?
K2O < K2O2 < KO2. This increasing stability as
the size of anion increases is due to stabilisation of: (a) Na3PO4 : Boneash
(a) larger cations by smaller anions through lattice (b) CaF2 : Fluorospar
energy effects. (c) NaNH2 : Sodamide
(b) larger cations by larger anions through lattice (d) KOH : Caustic potash
energy effects. 3. The hydration energy of Be2+ is :
(c) smaller cations by smaller anions through (a) More than that of Mg2+
melting point.
(b) More than that of Li+
(d) smaller cations by larger anions through
(c) More than that of K+
hydration energy effect.
70. Sodium dichromate on heating with coke yields: (d) More than that of Al3+
(a) sodium carbonate (b) sodium chromite 4. Which of the following reaction giving MgSO4 as
one of the product ?
(c) sodium chromate (d) chromic anhydride
(a) Mg reacts with concentrated solution of H2SO4
71. Barium cabide + N2 gas  Product (P) . P is
(b) Mg reacts with dilute H2SO4
(a) Ba(CN)2 (b) BaCN2
(c) BaN3 (d) Ba3N2 (c) MgO react with dilute H2SO4
72. During heating of sodium nitrate at low temperature (d) Dolomite react with dilute H2SO4
& at very high temperature what is the common 5. Which of the following reaction give hydrogen gas
product. as a product ?
(a) Na2O (b) N 2 O (a) Mg + dilute HCl 
(c) NO 2 (d) O 2 (b) Mg + dilute H2SO4 
73. Crhomate of which alkali metals is not dissolved in (c) Mg + Conc. H2SO4 
acetic acid (d) Mg + very dilute HNO3 
(a) Ca (b) Sr 6. Which of the following reactions are spontaneous ?
(c) Ba (d) all (a) MgCl2 + Ca(OH)2  Mg(OH)2 + CaCl2
74. Number of Cl atoms directly attched to each
(b) BeSO4 + BaCl2  BeCl2 + BaSO4
beryllium atom in solid BeCl2 is
(c) Ca(HCO3)2 + Ca(OH)2  CaCO3 + 2H2O
(a) 2 (b) 3
(d) CaCO3 + CO2 + H2O  Ca(HCO3)2
(c) 4 (d) 5
7. Which of the compounds are coloured and give
75. Of the following the commonly used in the
violet colour to oxidising flame?
laboratory desiccator is .
(a) K2CO 3 (b) CaCl2 (a) KO 2 (b) KCl
(c) NaCl (d) CaCO3 (c) KO 3 (d) KNO 3

www.jeebooks.in
THE s-BLOCK ELEMENTS AND THEIR COMPOUNDS 5.9
8. Which of the compounds do not have peroxy 18. Correct order of basic strength :
linkage and give golden yellow colour to oxidising (a) Cs2O > Rb2O > K2O > Na2O > Li2O
flame.
(b) BaO > SrO > CaO > MgO > BeO
(a) Na2O2 (b) NaNO 2
(c) Cs2O > K2O > CaO > MgO > NiO
(c) BaO2 (d) Na2SO3
(d) NaNH2 > NaOH > NaI
9. Which of the halide of beryllium can’t be
19. Which of the following reactions give correct
polymerised ?
products?
(a) BeF2 (b) BeCl2
(a) Na + N2  Na3N
(c) BeBr2 (d) BeI2
(b) K + N2  K3N
10. Which of the following are example of
organometallic compound ? (c) Li + N2  Li3N
(a) LiCH3 (b) NaOCH3 (d) Mg + N2  Mg3N2
(c) BeH2 (d) CH3MgBr 20. Which of the compound has polyhalide ion ?
11. Mg2+ is smaller than (a) KI3 (b) AlI3
(a) Ba2+ (b) Be2+ (c) ClF3 (d) RbICl2
(c) Li+ (d) Na+ 21. Correct statements :
12. Rb+ is larger than : (a) Lithium is stored in paraffin wax
(a) Sr2+ (b) Ca2+ (b) Na is stored in kerosen oil
(c) K+ (d) Cs+ (c) Na2HPO4 is stored in NaOH solution
13. CaCO3 can’t react with : (d) All
(a) Ca(OH)2 (b) NaOH 22. Which salt react with NaOH to give coloured
(c) HCl (d) H2SO4 precipitate ?
14. Ca(HCO3)2 can react with : (a) Ni(NO3)2 (b) FeCl2
(a) Ca(OH)2 (b) NaOH (c) CuSO4 (d) SnCl2
(c) HCl (d) HNO3 23. Which compounds (oxide) react with NaOH to
15. BeH2 can be prepared by give products by disproportionation product?
(a) Reaction of BeCl2 with HCl (a) NO 2 (b) Cl2O7
(b) Reaction of BeCl2 with LiAlH4 (c) SO2 (d) Cl2O6
(c) Direct combination of Be and H2 24. Which of the following compound when react with
NaOH to give salt by nonredox reaction?
(d) Reaction of BeCl2 with LiH
(a) Acetic acid ( CH3–C–OH )
16. Which of the following sulphates are sufficiently
soluble ? O
(a) BeSO4 (b) MgSO4 O
(c) BaSO4 (d) SrSO4 (b) Methyl sulphonic acid ( CH3–S–OH )
17. Potassium formate when allowed to heat in air, the
products formed are : O
(a) K2CO 3 (b) CO 2 (c) Carbon dioxide
(c) H2O (d) K2(CO2)2 (d) Sulphur trioxide

www.jeebooks.in
5.10 THE s-BLOCK ELEMENTS AND THEIR COMPOUNDS
25. Which of the following statemets are not correct? 31. Which of the following reactions are correctly
(a) K2CO3 can’t be prepared by Solvay process matched ?
because KHCO3 is insoluble in H2O (a) Be2C + D2O  CD4 + Be(OD)2
(b) (Li+) ionic mobility in aq. phase > (Na+) ionic (b) NaH + H2O  NaOH + H2
mobility in aq. phase
(c) 2NaHCO3 Na2CO3 + H2O + CO2
(c) Lithium is the strongest reducing alakli metals
in aqueous solution (d) Na2CO3 Na2O + CO2
(d) KOH is used as absorber of CO2 gas 32. Select the correct statement for s-block elements
26. Correct option regarding NaHCO3 : and their compounds.
(a) It reacts with ethanol to evolve CO2 (a) All the element form basic oxide
(b) It reacts with acetic acid to evolve CO2 (b) Potassium ion are the most abundant cations
(c) The bicarbonate ion are held together by H- within cell fluids
bonding in crystal structure (c) Most covalent halide is BeI2 among all alkaline

O earth metals.
O O (d) Mg react with very dilute HNO3 (2%) to give
H C H C O O
O O H C N2O gas

O O

33. Which of the following statement is incorrect for
(d) On heating it gives mixture of Na2O + H2O + Na2O2 ?
CO 2
(a) It absorbs CO2
27. In group-I A (alkali metals), correct options are :
(b) At room temperature it produces O2 with water
(a) Lithium fluoride is most stable fluoride
(c) It produces NO2 with NH3
(b) Lithium iodides has maximum covalent
character out of all halides of alkali metals (d) It converts green solution of Cr3+ to yellow
solution
(c) Lithium carbonate is least thermally stable
carbonate 34. Which of the following order is /are correct ?
(d) Lithium oxide is most basic oxide (a) Na+ < K+ (Ionic radius)
28. Which of the following are paramagnetic? (b) Na+ < K+ (Hydration energy)
(a) KO 2 (b) KO 3 (c) Na+ < K+ (Polarising power)
(c) Na2O2 (d) RbO2 (d) Na+ < K+ (Electrical conductance in aqueous
29. Which of the following options are correct ? solution)
(a) Lowest melting point among alkaline earth metal 35. Which of the following element reacts with NaOH
is Mg solution to give H2 gas?
(b) Highest boiling point among alkaline earth metal (a) Zn (b) Al
is Be (c) Sn (d) Pb
(c) Lowest melting point among alkali metal is Cs 36. which salt reacts with NaOH to give white
(d) Highest boiling point among alkali metal is Li precipitate which is soluble in excess NaOH ?
30. Which of the following reactions are correctly (a) ZnCl2 (b) AlCl3
matched ? (c) CuSO4 (d) FeSO4
(a) CaC2 + N2  Ca(CN)2 37. Which of the following options are correct ?
(b) BaC2 + N2  Ba(CN)2 (a) LiI is less soluble than KI in ethanol
(c) CaC2 + N2  CaCN2 + C (b) Alkali metals can be prepared by electrolysis
(d) BaC2 + N2  BeCN2 + C of their fused chloride

www.jeebooks.in
THE s-BLOCK ELEMENTS AND THEIR COMPOUNDS 5.11
(c) Sodium is found to be more useful than 43. Which of the following compounds on hydrolysis
potassium give NH3 gas ?
(d) Lithium salts are commonly hydrated and those (a) Ca3N2 (b) CaCN2
to the other alkali ions are usually anhydrous (c) Mg3N2 (d) Li3N
38. Which of the following compounds of potassium 44. The various silicates & aluminates which constitute
are correctly matched ? more than 90% of the cement are :
(a) Carnalite : KCl.MgCl2.6H2O (a) Tricalcium silicate 3CaO.SiO2
(b) Langbeinite : K2SO4.MgSO4 (b) Dicalcium silicate 2CaO.SiO2
(c) Sylvine : KCl (c) Tricalcium aluminate 3CaO.Al2O3
(d) Caustic Potash : KOH (d) Tetracalcium alumino ferrite 4CaO.Al2O3.Fe2O3
39. Correct decreasing order of different properties : 45. Be is amphoteric in nature. That’s why Be reacts
(a) Ksp : Ba(OH) 2 > Sr(OH) 2 > Ca(OH) 2 > with several acids like H2SO4 ; HNO3. It also
Mg(OH)2 > Be(OH)2 reacts with NaOH. How many reactions of Be
with acids/base are correct ?
(b) Electronegativity : Be > Mg > Ca > Ba
(a) Be + H2SO4  BeSO4 + H2
(c) Heat of hydration : Ba2+ > Sr2+ > Ca2+ > Be2+
(dilute)
(d) Basic strength : CsOH > KOH > LiOH
(b) Be + 2HNO3  Be(NO3)2 + H2
40. Select correct statements.
(hot)
(a) NaNO3 , KNO3 both are soluble
(c) Be + 2NaOH  Na2BeO2 + H2
(b) NaClO4 is water soluble but KClO4 is insoluble
(d) Be + 2H2SO4  BeSO4 + SO2 + 2H2O
(c) Na 3 [Co(NO 2 ) 6 ] is water soluble but
(conc.)
K3[Co(NO2)6] is insoluble
46. Which of the following are polymeric in nature ?
(d) NaCl & KCl both are insoluble
(a) BeCl2 (vapour phase)
41. 2M(NO3)2  2MO + 4NO2 + O2
(b) BeCl2 (solid)
This decomposition pattern is applicable for which (c) BeH2(solid)
of alkaline earth metal.
(d) BeH2 (vapour phase)
(a) Be (b) Mg
47. Presence of which ion makes water hard ?
(c) Ca (d) Sr
(a) Na+ (b) K+
42. M + (x + y) NH3  [M(NH3)x]2+ + 2[e(NH3)y]–1
(c) Mg2+ (d) Ca2+
...(1)
48. The pair of reagents that yield paramagnetic species
M + (x + y) NH3  [M(NH3)x]+1 + [e(NH3)y]–1
is/are :
..(2)
(a) Na & excess of NH3
Which options are correct.]
(b) K & excess of O2
(a) Equation (2) is applicable for alkali metal
(c) Cu & dilute HNO3
(b) Equation (1) is applicable for alkaline earth
(d) O2 & ethyl anthraquinol
metal
49. Which of the following are ionic carbide ?
(c) Both the solution in product side is blue in colour
(a) Be2C (b) Al4C3
(d) Both the solution in product side are
paramagnatic (c) Mg2C3 (d) CaC2

www.jeebooks.in
5.12 THE s-BLOCK ELEMENTS AND THEIR COMPOUNDS
50. Which of the following statements are correct ? (c) Microcosmic salt is NaNH4 HPO4. 4H2O
(a) The carbonate of lithium i.e., Li 2 CO 3 (d) BeCl2 and AlCl3 both are dimeric in liquid state
decomposes on heating to form lithium oxide 57. Which of the following options are incorrect
Li2O (a) CaF2 undergoes disproportionation to give
(b) Beryllium carbonate is kept in the atmosphere CaF3 and CaF
of carbon dioxide (b) The product mixture formed is called lithophone
(c) Be2+, Li+ both have high polarising power when BaS reacts with ZnSO4 solution.
(d) Be2+, Li+ both have high degree of hydration (c) BeCl2can exists in dimeric and polymeric state
51. Which of the following statements are true ? but AlCl3 alwasy exists as dimeric state.
(a) BeCl2 is linear molecule in the vapour phsase (d) 2BeCl2 + LiACH4  2BeH2 + LiCl + AlCl3, in
but it is polymeric in the solid state this reaction LiAlH4 act as oxidising regent.
(b) Oxide of Be and Al are amphoteric 58. Which of the following disproportionate (s) on
(c) Above 393 K gypsum converted into dead heating with sodium hydroxide?
burnt plaster (a) P 4 (b) S 8
(d) CaO is basic oxide, so it reacts with acidic (c) Cl2 (d) B
oxide like SO3 & P4O10 59. Hydrated sodium aluminium silicate, the zeolite, is
52. Which of the following carbide on reaction with treated with hard water, the sodium ions are
water to give CH4 gas ? exchanged with :
(a) Be2C (b) Mg2C3 (a) Ca2+ ions (b) Mg2+ ions
(c) Al4C3 (d) CaC2 (c) H+ions (d) SO42– ions
53. Which of the following options are correct 60. Which of the following statements are correct?
(a) BeCl2 fumes in moist air. (a) potassium and caesium are useful in
(b) Anhydrous calcium chloride can be used to dry photoelectric cells
NH3 (b) the dehydration of hydrated chloride , bromide,
(c) Beryliium halides dissolve in organic solvent iodide of Ca, Sr, Ba can be achieved on heating.
(d) Mg can not form complex (c) KOH can be prepared by Le-Blanc process
54. Which of the following options are incorrect (d) Both LiCl & MgCl2 are deliquescent.
(a) Lithium salts are hygroscopic 61. Which of the following alkaline metals can not exist
(b) BaSO4 is reduced by carbon to give BaS & in elemental form
CO (a) Li (b) Na
(c) Dry KOH reacts with ozone to yield black solid (c) K (d) Si
(d) LiI is more soluble than RbI in water. 62. Which of the following are example of water
55. Alkaline earth metals which does not react with soluble compound
cold water is
(a) NaClO4 (b) KClO4
(a) Mg (b) Ba
(c) Na3[Co(NO2)6] (d) K3[Co(NO2)6]
(c) Be (d) Ca
63. Which of the following statement(s) is/are correct
56. Which of the following statements are correct? with respect to alkali metal bicarbonate?
(a) Chloride of Be & Al are soluble in organic (a) In crystalline sodium bicarbonate, the HCO3–
solvents and are strong lewis acid. anions form an infinite chain through H-bonding
(b) Be & Al have strong tendency to form (b) Bicarbonates do not produce any colour with
complexes with F–.
phenophtalein indicator.

www.jeebooks.in
THE s-BLOCK ELEMENTS AND THEIR COMPOUNDS 5.13
(c) All bicarbonate except that of lithium are found 3. A1 + H2O  Alkaline solution
in solid state A2 + H2O  Alkaline solution + A3 + A4 ;
(d) KHCO3 is formed when CO2 (in excess) reacts A4 A3 formed as one of the product
with KO2(medium water)
A3 & A4 are respectively :
64. The more commonly used baking powder contains
(a) H2 & O2 (b) H2O2 ; O2
about 30% NaHCO3, 20% NaAl(SO4)2 10%
Ca(H2PO4)2 and 40% starch. Which of the (c) H2O2 ; H2 (d) O2 ; H2O2
following statement(s) is/are correct? Paragraph for Q.No. 04 to 05
(a) Ca(H2PO4)2 is acidic when moistened it reacts Baryta water has chemical formula Ba(OH)2. It is white
with NaHCO3 evolving CO2 gas solid. It melting point is 78ºC ; Its boiling point is 780ºC.
(b) NaAl(SO4)2 slows down the decomposition 4. Which of the following option is correct regarding
reaction of NaHCO3 so that CO2 is evolved Ba(OH)2?
more slowely. (a) Ba(OH)2 form barium sulphate and barium
(c) Sartch functions as filler. phosphate with sulphuric and phosphoric acid,
(d) NaAl(SO4)2 and and starch both acts as filler.ss which are insoluble
65. Which of the following statements are correct? (b) Ba(OH)2 reacts with H2S to give barium
sulphide
(a) Disssolution of alkali metal hydroxide in water
is exothermic. (c) Ba(OH)2 decomposes at 800ºC to give BaO
(b) Sea shells, dolomite, a marble statue and (d) All
calcined gypsum, all contains CaCO3. 5. The reaction of barium hydroxide octahydrate with
(c) Lime water is a clear solution of Ca(OH)2 in ammonium chloride is often used as a classroom
water chemistry demonstration because :
(d) Baryta water is a clear slution of Ca(OH)2 in (a) It produces very high temperature enough to
water boil water
(b) It produces very low temperatue enough to
EXERCISE # III freeze water
 Linked Comprehension Type : (c) The above reaction is highly exothermic
Paragraph for Q.No. 01 to 03 (d) The above reaction is highly endothermic
An element gives violet flame colour to the flame test Paragraph for Q.No. 06 to 08
A(excess) + O2 180ºC A1 Element (metal)
H2O dil. HNO3
A D E
A + O2(excess)  A2 (Brick red flame)

A1 is colourless but A2 is chrome yellow colour powder. +Cl2 + CO2


1. A1 & A2 are respectively : B C
1000ºC
(a) KO2 ; KO3 (b) K2O ; KO2 (used for
bleaching purpose)
+HCl (dilute)
(c) K2O ; KO3 (d) KO3 ; K2O
2. Correct statement is : + CaF2 heated G
H
electrolysis HCl/260ºC
(a) A1 & A2 both are diamagnetic 700ºC
(b) A1 & A2 both are paramagnetic 6. A is :
(c) A1 is diamagnetic but A2 is paramagnetic (a) Mg(OH)2 (b) Ca(OH)2
(d) A2 is paramagnetic but A1 is diamagnetic (c) CaO (d) CaH2

www.jeebooks.in
5.14 THE s-BLOCK ELEMENTS AND THEIR COMPOUNDS
7. B & G are respecitvely : 13. Q & S are respectively :
(a) MgCl2 ; MgCl2.6H2O (a) CaCO3, Ca(HCO3)2
(b) CaCl2 ; CaCl2.6H2O (b) CaCO3, Ca(OH)2
(c) Ca(OCl)Cl ; CaCl2.6H2O (c) Ca(HCO3)2, CaCl2
(d) CaCl2 ; Ca(OCl)Cl (d) Ca(OH)2, Ca(HCO3)2
8. C & D are respecitvely : Paragraph for Q.No. 14 to 15
A colourlesss crystalline substance (A) gives crimson
(a) Ca(HCO3)2 ; CaCO3
colour to the flame. On heating (A) gives a brown gas
(b) CaCO3 ; CaO as leaving a white residue(C). C is dissolved in HCl
(c) Ca(HCO3)2 ; CaO heated with (NH4)2SO4 gives white precipitate(D).
(d) Ca(NO3)2 ; CaSO4 14. A is
Paragraph for Q.No. 09 to 11 (a) Ba(NO3)2 (b) Ca(NO3)2
Sodium chloride is abundantly found in nature as rock (c) Mg(NO3)2 (d) Sr(NO3)2
salt. Sea water consists of 2.5% NaCl. It is used as 15. D is
preservative for food particles. (a) BaSO4 (b) CaSO4
9. NaCl + Conc. H2SO4  X + HCl (c) MgSO4 (d) SrSO4
X + Coke  X’ Paragraph for Q.No. 16 to 17
X & X’ are respecitvely : A white insoluble solid (A) is attacked by acid liberating
a colourless, odourless gas (B) leaving a clear solution
(a) Na2SO4 ; Na2SO3
C. The solution C gives a white precipitate (D) on
(b) NaHSO4 ; NaHSO3 addition of dilute H2SO4. C gives yellow precipitate E
(c) Na2SO4 ; Na2S on addition of K2CrO4. Gas B is bubbled through a
(d) Na2SO4 ; SO2 suspension of CaCO 3 causes solution of that
compound.
10. NaCl when reacts with mixture of MnO2 &
concentrated H2SO4 the gas evolved is 16. A and B is
(a) BaCO3, CO2 (b) SrCO3, CO2
(a) HCl (b) Cl2
(c) Ba (NO3)2, NO2 (d) MgCO3, CO2
(c) SO2 (d) SO3\
17. C and D are respectively
11. Electrolysis of aqueous solution of NaCl results.
(a) SrSO4, SrCrO4 (b) MgSO4, MgCrO4
(a) H2 evolution at cathode
(c) BaSO4, BaCrO4 (d) BeSO4, BeCrO4
(b) Cl2 evolution at anode
Paragraph for Q.No. 18 to 20
(c) Basic Solution
A metal (A) burns in O2 to give a orange powder B
(d) All which reacts with water liberates O2 and gives a
Paragraph for Q.No. 12 to 13 solution C. B is strong oxidising agent & is used in
P + CO2 Q (Milky cloud) respirators of the rebreather type. When 1.42 gm B is
heated with 0.32 gm of sulphur in an evacuated sealed
P + Na2CO3 Q+R tube, 1.74 gm of a white crystalline solid(D) is formed.
Q + CO2 (excess) S (clear solution) D forms a hydrated double salt E with Al2(SO4)3.
12. P & R are respectively. 18. A & B can be
(a) Ca(OH)2, NaOH (b) CaCl2, NaCl (a) Na, Na2O (b) K, K2O
(c) NaOH, CaCl2 (d) Ca(OH)2, NaCl (c) Na, Na2O2 (d) K, KO2

www.jeebooks.in
THE s-BLOCK ELEMENTS AND THEIR COMPOUNDS 5.15
19. Solution C is 25. Which of the following has the highest solubility in
(a) Acidic (b) Basic water?
(c) Neutral (d) Amphoteric (a) LiOH (b) KOH
20. E is (c) CsOH (d) RbOH
(a) K2SO4. Al2(SO4)3.24 H2O Paragraph for Q.No. 26 to 28
(b) Na2SO4. Al2(SO4)3.24 H2O On reaction with cold water, an element (A) reacts
quitely to give colourless, odourless gas B and a
(c) K2SO4. Al2(SO4)3.6 H2O solution C. Lithium reacted with B yielding a solid
(d) Na2SO4. Al2(SO4)3.6 H2O product D which reacts with water to give a strong
Paragraph for Q.No. 21 to 23 basic solution E. When CO2 is bubbled through solution
0.347 gm of metal (A) was dissolved in HNO3. The C, an initial precipitate F was formed. Precipitate F.
solution give a red colouration to a nonluminous flame Precipitate F effervesced with concentrated HCl and
and on evaporation gave 0.747 gm metal oxide B. A gave red colour to Bunsen flame. When F was heated
also reacts with N2 forming a compound C and with with carbon at 1000°C, acaustic white solid G was
hydrogen forming D. On reacting 0.159 gm D with formed which when heated with carbon at 1000°C
water, a gas E was evolved & a sparingly soluble gave a solid J of some commercial importance.
compound F was formed which gave a strongly basic 26. G and J are
solution and requires 200 ml of 0.1(M) HCl to (a) CaCO3, CaO (b) CaO, CaC2
neutralise it. (c) CaO, CaCO3 (d) CaO, Ca2C3
21. A is 27. Element A and colourless gas B are respectively.
(a) Li (b) Be (a) Mg, N2 (b) Ca, CO2
(c) Na (d) K (c) Ca, H2 (d) Mg, O2
22. Sparingly soluble compound F is 28. Precipitate G
(a) Be(OH)2 (b) LiOH (a) will be dissolved on passing through more CO2
(c) NaOH (d) KOH gas
23. B, C and D are respectively (b) will be ignited on passing through more CO2 gas
(a) BeO; Be3N2; BeH2 (c) will remain precipitate on passing through more
CO2 gas
(b) K2O; KN3; KH
(d) No reaction
(c) Na2O; NaN3; NaH
Matrix Match Type :
(d) Li2O; LiN3; LiH
29. Column - I Column - II
Paragraph for Q.No. 24 to 25
(Compunds) (Use)
The elements of group 1 describe, more clearly than
(A) Na2CO3.10H2O (P) Laundering ; Cleaning ;
any other gruopp of elements.These metals are highly
(Washing soda) Important laboratory
electropositive, strong reducing agent, have stable oxo
reagent for qualitative
salts.
analysis
24. Sodium hydroxide is manufactured by the
(B) NaCl (Q) It is used in preparation
electrolysis of brine solution. The reaction by- of mortar, a building
products are : material
(a) Cl2 and H2 (c) Ca(OH)2 (R) It is used as an antacid,
(b) Cl2 and Na-Hg mild abrassive in tooth
(c) Cl2 and NaCl paste
(d) Cl2 and O2 (D) CaCO3 (S) It is used as table salt

www.jeebooks.in
5.16 THE s-BLOCK ELEMENTS AND THEIR COMPOUNDS
30. Column - I Column - II 33. Column-I Column-II
(Compunds) (Use) (Compound) (Charasteristic Feature)
(A) NaHCO3 (P) Used as in fire (A) Chile saltpetre (P) Ba2+ is present
extinguishers (B) Lithophone (Q) Used as pigment
(B) NaOH (Q) Used as laboratory (BaSO4 + ZnS)
reagent (C) Brine solution (R) Na+ is present
(c) CaO (R) Used for manufacturing (D) Oxone (S) –O–O– linkage is
cement present
(D) CaSO4.1/2H2O (S) Used for immobilising (T) N–O linkage is present
the affected part of
34. Column-I Column-II
organ where there is a
bone fracture (Compound) (Charasteristic Feature)
31. Column - I (A) Ca(HCO3)2 (P) Exists in solid form
(Compunds/Aqueous solution) (B) CaCO3 (Q) With dilute acid it gives
CO2 gas
(A) K 2 O
(C) NaHCO3 (R) Does not react with
(B) K 2O 2
K2Cr2O7 solution
(C) N 2O 5
(D) Na2CO3 (S) Water insoluble
(D) NH3
35. Column-I Column-II
Column - II
(Compounds) (Average oxidation state
(Colour change on addition of litmus paper) of oxygen)
(P) Red litmus paper turns blue (A) KO 3 (P) –2
(Q) Blue litmus paper turns red 1
(R) Red litmus paper turns blue at first and then (B) K 2O 2 (Q) –
3
colourless (C) SO3 (R) –1
(S) Blue litmus paper remains blue 1
(D) O2[PtF6] (S) +
32. Column-I 2
(Heating effect) 36. Column - I Column - II
(Compounds) (Gas on hydrolysis)
(A) CaSO4.2H2O 120º C
(A) Mg3N2 (P) NH3
(B) CaSO4.2H2O 200º C
(B) CaC2 (Q) C2H2
(c) CaSO4.2H2O 1100º C (C) BeH2 (R) H2
(D) CaCO3 (D) Mg2C3 (S) CH3–C=CH
37. Column - I Column - II
Column-II
(A) Hydrolith (P) Contains Ca2+
(Products obtained in reaction)
(B) Gypsum (Q) Used as fertiliser
(P) Plaster of paris
(c) Nitrolim (R) Reacts with water to
(Q) Dead burnt plaster give H2
(R) Quick lime (D) Dolomite (S) On heating it is
(S) Gas evolved acidic in nature converted into plaster
of paris

www.jeebooks.in
THE s-BLOCK ELEMENTS AND THEIR COMPOUNDS 5.17
38. Column - I Column - II 42. Column - I
(Process/Experiment) (Compound) (Hydrolysis)
(A) Solvay process (P) Na2O (A) Mg2C3 + nH2O Mg(OH)2 + Hydrocarbon
(B) Castner process (Q) Na2CO3 (B) CaC2 + nH2O Ca(OH)2 + Hydrocarbon
(C) Aq. solution is basic (R) NaOH (C) Al4C3 + nH2O Al(OH)2 + Hydrocarbon
towards litmus
(D) Be2C + nH2O Be(OH)2 + Hydrocarbon
(D) On heating it gives (S) NaHCO3
CO2 gas Column - II
39. Column - I Column - II (n/Hydrocarbon)
(Element) (Characterstic feature) (P) 4 ; CH3C CH
(A) Cs (P) Electronegativity is (Q) 2 ; HC CH
same as that of Al (R) 12 ; CH4
(B) Li (Q) Strongest reducing (S) 4 ; CH4
alkali metal
43. Column - I Column - II
(C) Ba (R) 6s2 configuration
(A) BeCl2 (P) Hydroxo complex in
(D) Be (S) Blue colour towards aqueous solution
oxidising flame
(B) AlCl3 (Q) Fumes in moist air
40. Column - I
(c) MgCl2 (R) Lewis Acid
(A) Rb2CO3 > K2CO3 > Na2CO3
(B) SrSO4 > CaSO4 > MgSO4 (D) BaCl2 (S) Soluble in water
(C) Rb > K > Na (T) Hydrated salt formation
(D) Be > Mg > Ca 44. Column - I Column - II
Column - II (A) Be (P) Reacts with NaOH to
(P) solubility of salts in water give H2 gas
(Q) Thermal stability of salts (B) Al (Q) Forms polymeric halide
(R) Softness of metals (c) B (R) Forms ionic hydride
(S) Hydration energy of metals (D) Ca (S) Present in colemnite
(T) Ionisation energy of metals ore
41. Column - I (T) Forms amphoteric
(A) Calcium hypochlorite oxide.
(B) Calcium sulphate hemihydrate 45. Column - I Column - II
(C) Potassium superoxide (A) Pearl Ash (P) Insoluble in water
(D) Calcium oxide (B) MgCl2 (Q) Can exist only in
Column - II aqueous solution
(P) Used as air purifier in submarine (c) CaSO4 (R) Cause parmanent
(Q) Used for ‘softening’ water hardness
(R) A constituent of bleaching powder (D) CaCO3 (S) Contains K+
(S) Plaster of Paris (T) Its hydrated salts can
(T) Used in preparation of sodium carbonate from not be made anhydrous
caustic soda by heating.

www.jeebooks.in
5.18 THE s-BLOCK ELEMENTS AND THEIR COMPOUNDS
EXERCISE # IV (c) Alkali metals & alkaline earth metal are example
 Integer Type : of strong reducing agents
1. Find the number of s-block elements which can (d) Photoelecric effect is maximum for ceasium
produce ammoniated cation and ammoniated (e) LiF is less soluble because of high lattice energy
electron in liquid ammonia : (f) CsI is less soluble due to smaller hydration
Li, Na, K, Rb, Cs, Ca, Sr, enthalpy of Cs+ & I–
2. How many of the following metal chloride impart 10. Epsom salt : MgSO4.n1H2O
characteristic colour to the oxidisiing flame? Gypsum : CaSO4 .n2H2O
BaCl2, LiCl, NaCl, KCl, BeCl2, MgCl2, CaCl2, Dead burnt plaster : CaSO4.n3H2O
SrCl2 Find out n1 + n2 + n3
3. Find out number of bicarbonate(s) which exist in 11. How many of the following contain hydride ion ?
solid form. (a) Hydrolith (CaH2) (b) NaH
(a) Ca(HCO3)2 (b) Be(HCO3)2 (c) LiAlH4 (d) NaBH4
(c) NaHCO3 (d) KHCO3 (e) HCl (f) H2 gas
(e) NH4HCO3 12. Find the total number of species which would
4. What is the atomic number difference between undergo disproportionation in alkaline medium
alkali metal of 2nd period & alkaline earth metal of (NaOH).
4th period. NO2 ; Cr2O77– ; Cl2O6 ; Br2 ; P4 ; S8 ; Al2O3 ;
5. Find out number of Cl-atoms which are directly 13. Find number of substance which produce H2 gas
bonded to each berylium in solid state of BeCl2. when react with cold water.
6. Decomposition temperature of carbonate are given. Ca; CaH2 ; Mg ; MgCl2 ; Ba ; LiAlH4
How many of them are correct ? 14. How many alkali metal on reaction with excess O2
BeCO3  25ºC ; MgCO3  540ºC ; CaCO3 give superoxide as product ?
 900º C ; SrCO3  1290ºC ; BaCO3  (a) Li (b) Na
1360ºC.
(c) K (d) Rb
7. 2NaOH + Cl2 Cold X1 + Y + Z
(e) Cs
6NaOH + 3Cl2 80º C X2 + Y + Z
15. How many compounds do not react with sodium
What is the difference of oxidation state of chlorine metal ?
atom of anionic part of X1 & X2 :
(a) Ethanol (C2H5OH)
8. NaF in predominantly ionic and if the cationic part
(b) Ethyne (HCCH)
of the compound is replaced by potassium ion,
then which of the properties decrease in the formed (c) Dimethyl ether (MeOMe)
compound. (d) Phenol (PhOH)
(i) Melting point (ii) Lattice energy (e) 2-butyne(CH3–CC–CH3)
(iii) Thermal stability (iv) Solubility in water 16. (Compounds) (Oxidation state of
(v) Covalent character central atom)
9. How many of the statements are correct ? (A) BaO2 (P) X
(a) Fr is radioactive (B) KO 2 (Q) Y
(b) Superoxide ion is stable in presence of large (c) CO 2 (R) Z
cations like Cs+, K+ (D) CrO5 (S) W
Find the value of X + Y + Z – W :

www.jeebooks.in
THE s-BLOCK ELEMENTS AND THEIR COMPOUNDS 5.19
17. What is the maximum coordination number of (a) Inter-ionic attraction
beryllium? (b) Entropy of solution formation
18. Find out the number of Be–Cl–Be bonds present (c) Lattice energy of solids
in product formed when BeCl 2 reacts with (d) Hydration energy of cations
concentrated diethyl ether solution. 5. The substance not likely to contain CaCO3 is :-
19. Beryllium gives a compound with the following [AIEEE - 2003]
percentage composition: (a) Sea shells (b) Dolomite
Be, 6.1%; N,37.8%; Cl,48%; H8.1%. One mole (c) A marble same (d) Calcined gypsum
of the compound had a mass of 148 g. Mol.wt. 6. One mole of magnesium nitride on reaction with
(Be) = 9 gmol–1. If the empirical formula = excess of water gives :- [AIEEE - 2004]
BeNxClyH12 then find out x + y.
(a) Two mole of HNO3
20. Find out the number of Be–O bonds present in
(b) Two mole of NH3
product formed when BeCl 2 reacts with
concentrated diethyl ether solution. (c) 1 mole of NH3
21. Number of alkali metal which impart colour to the (d) 1 mole of HNO3
flame. 7. Berylium and aluminium exhibit many properties
Li, Na, K, Rb, Cs, Be, Mg which are similar. But the two elements differ in :-
[AIEEE - 2004]
EXERCISE-V(A) (JEE-MAIN)
(a) Exhibiting maximum covalency in compounds
1. A metal M readily forms its sulphate MSO4 which
(b) Forming polymeric hydrides
is water soluble. It forms oxide MO which
becomes inert on heating. It forms insoluble (c) Forming covalent halides
hydroxide which is soluble in NaOH. The metal (d) Exhibiting amphoteric nature in their oxides
M is :- [AIEEE - 2002] 8. The ionic mobility of alkali metal ions in aqueous
(a) Mg (b) Ba solution is maximum for :- [AIEEE - 2006]
(c) Ca (d) Be (a) Rb+ (b) Li+
2. KO2 is used space and submarines because it :- (c) Na+ (d) K +
[AIEEE - 2002] 9. Thr prodcts obtained on heating LiNO3 will be :-
(a) Absorbs CO2 and increase O2 concentration [AIEEE - 2011]
(b) Absorbs moisture (a) LiNO2 + O2 (b) Li2O + NO2 + O2
(c) Absorbs CO2 (c) Li3N + O2 (d) Li2O + NO + O2
(d) Produce ozone 10. What is the best description of the change that
occurs when Na2O(s) is dissolved in water ?
3. In curing cement plasters , water is sprinklet from [AIEEE - 2011]
time to time. This helps in :- [AIEEE - 2003] (a) Oxidation number of sodium decreases
(a) Hydrating sand and gravel mixed with cement
(b) Oxide ion accepts sharing in a pair of electrons
(b) Converting sand into silicate
(c) Oxide ion donates a pair of electrons
(c) Developing interlocking needle like crystals of
(d) Oxidation number of oxygen increases
hydrated silicates
11. Which of the following on thermal - decomposition
(d) Keeping it cool
yields a basic as well as an acidic oxide ?
4. The solubilities of carbonates decreases down the [AIEEE - 2012]
magnesium group due to decrease in :- (a) NH4NO3 (b) NaNO3
[AIEEE - 2003]
(c) KClO3 (d) CaCO3

www.jeebooks.in
5.20 THE s-BLOCK ELEMENTS AND THEIR COMPOUNDS
12. Fire extinguishers contain H2SO4 and which one 19. The correct order of thermal stability of hydroxides
of the following :- [Jee-Main - 2012 (Online)] is :- [Jee-Main - 2015 (Online)]
(a) CaCO3 (b) NaHCO3 and Na2CO3 (a) Ba(OH)2 < Sr(OH)2 < Ca(OH)2 < Mg(OH)2
(c) Na2CO3 (d) NaHCO3 (b) Mg(OH)2 < Sr(OH)2 < Ca(OH)2 < Ba(OH)2
13. Based on lattice energy and other considerations, (c) Mg(OH)2< Ca(OH)2< Sr(OH)2 < Ba(OH)2
which one of the following alkali metal chloride is (d) Ba(OH)2 < Ca(OH)2 < Sr(OH)2 < Mg(OH)2
expected to have the highest melting point ? 20. Which of the alkaline earth metal halides gives
[Jee-Main - 2012 (Online)] below is essentially covalent in nature :-
(a) RbCl (b) LiCl [Jee-Mains - 2015 (Online)]
(c) KCl (d) NaCl (a) SrCl2 (b) CaCl2
(c) BeCl2 (d) MgCl2
14. Which one of the following will react most
vigorously with water ? 21. Which one of the following alkaline earth metal
sulphates has its hydration enthalpy greater than
[Jee-Main - 2012 (Online)]
its lattice enthalpy :- [Jee-Mains - 2015]
(a) Li (b) K
(a) BaSO4 (b) SrSO2
(c) Rb (d) Na (c) CaSO4 (d) BeSO4
15. A metal M on heating in nitrogen gas gives Y. Y on
treatment with H2O gives a colourless gas which
EXERCISE-V(B) (JEE-ADVANCED)
when passed through CuSO4 solution gives a blue 1. The species that do not contain peroxide linkage
colour, Y is :- [Jee-Main - 2012 (Online)] are :- [JEE-1992]
(a) NH3 (b) MgO (a) PbO2 (b) H2O2
(c) Mg3N2 (d) Mg(NO3)2 (c) SrO2 (d) BaO2
16. The correct statement for the molecule, CsI3 is :- 2. Read the following statement and explanation and
[Jee-Main - 2014] answer as per the options givesn below :-
(a) It contains Cs3+ and I– ions Statement-1 : The alkali metals can form ionic
hydrides which contain the hydride ion H–.
(b) It contains Cs+ , I– and lattice I2 molecule
Statement-2 : The alkali metals have low
(c) It is a covalent molecule electronegativity ; their hydrides conduct conduct
(d) It contains Cs3+ and I3– ions electricity when fused and liberate hydrogen at the
17. Which of the following statements about Na2O2 is anode. [JEE-1994]
not correct? [Jee-Main - 2014 (Online)] (a) Both 1 and 2 are true and 2 is the correct
(a) Na2O2 oxidises Cr3+ to CrO42– in acid medium explanation of 1.
(b) It is diamagnetic in nature (b) Both 1 and 2 are true but 2 is not the correct
explanation of 1.
(c) It is the super oxide of sodium
(c) 1 is true but 2 is true.
(d) It is a derivative of H2O2
(d) 1 is false but 2 is true.
18. Amongst LiCl, RbCl, BeCl 2 and MgCl2 the
compounds with the greatest and the least ionic 3. The following compounds have been arranged in
order of their increasing thermal stabilities. Identify
character, respectively are :-
the correct order. [JEE-1996]
[Jee-Main - 2014 (Online)]
K2CO3(I) MgCO3(II) CaCO3(III) BeCO3(IV)
(a) RbCl and MgCl2 (b) LiCl and RbCl
(a) I < II < III < IV (b) IV < II < III < I
(c) MgCl2 and BeCl2 (d) RbCl and BeCl2
(c) IV < II < I < III (d) II < IV < III < I

www.jeebooks.in
THE s-BLOCK ELEMENTS AND THEIR COMPOUNDS 5.21
4. Property of all the alkaline earth metals that increase ASSERATION AND REASON
with their atomic number? [JEE-1997] 7. The questions contains statement-1 (asseration)
(a) ionisation energy and statement-2 (reason) and has 4 choices (a),
(b) solubility of their hydroxides (b), (c) and (d) out of which only one is correct.
(c) solubility of their sulphate Statement-1 : Alkali metals dissolves in liquid
(d) electronegativity ammonia to give blue solutions. because.
5. Highly pure dilute solution of sodium in liquid Statement-2 : Alkali metals is liquid ammonia give
ammoia - [JEE-1998] solvated species of the type [M(NH3)n]+
(a) shows blue colour (M = alkali metals) [JEE-2007]
(b) exhibits electrical conductivity (a) Both 1 and 2 are true and 2 is the correct
explanation of 1.
(c) produces sodium amide
(b) Both 1 and 2 are true but 2 is not the correct
(d) produces hydrogen gas
explanation of 1.
6. The set representing the correct order of first
(c) 1 is true but 2 is true.
ionization potential is :- [JEE-2001S]
(d) 1 is false but 2 is true.
(a) K > Na > Li
8. The compound(s) formed upon combustion of
(b) Be > Mg > Ca
sodium metal in excess air is (are) [JEE-2009]
(c) B > C > N (a) Na2O2 (b) Na2O
(d) Ge > Si > C (c) NaO 2 (d) NaOH



www.jeebooks.in
THE p-BLOCK ELEMENTS AND THEIR COMPOUNDS
PROB L EM S B ASED ON GI VEN T OPI CS  Tetrahydridoborates (Borohydries)
The group 13 elements  Halides
 Oxidation state of types of bonds  Trihalides
 The (+III) oxidation state  Dihalides
 The (+I) oxidation state - the ‘inert pair effect’  Monohalides
 General properties  Complexes
 Melting points, boiling poinds an structurs  Differences between boron and the other elements
 Size of atoms and ions  Boron hydrides
 Electropositive characters  Compounds known
 Ionization energy  Preparation
 Reaction of boron  Reaction of the boranes
 Reaction of the other elements  Hydroboration
 Reation with water and air  Reaction with ammonia
 Reaction with acids and alkalis  Some other reactions of boranes
 Reaction with dioxygen  Structures of the boranes
 Reaction with the halogens and sulphate  Organometallic compounds
 Alums The group 14 elements

 Cement  Structure and allotropy of the elements

 Compounds of boron and oxygen  Differences between carbon , Silicon and the
remaining elements
 Boron sesquioxide and the borates
 Carbon dating
 Acidic properties of H3BO3 or B(OH)3
 Physical properties
 Structures of borates
 Covalent radii
 Borax
 Ionization energy
 Sodium peroxoborate
 Melting points
 Isopolyacids of B, Si and P
 Metallic and non metallic character
 Qualitative analysis of boron compounds
 Four - covalent compounds
 Fluoboric acid
 Chemical reactivity
 Borides
 Insert pair effect
 The other group 13 oxides
 Standard reduction potentials (volts)
 Amphoteric behaviour - aluminates
 Graphite compounds

www.jeebooks.in
6.2 THE p-BLOCK ELEMENTS AND THEIR COMPOUNDS
 Carbides  Halides
 Salt - like carbides  Tetrahalides
 Interstitial carbides  Catenated halides
 Covalent carbides  Dihalides
 Oxygen compounds of carbon  Cluster compounds

 Carbon monooxide CO  Reaction mechanisms


 Organic derivaties
 Carbon dioxide CO2
The group 15 elements
 Carbon suboxides
 General properties and structures of the elements
 Carbonates
 Nitrogen
 The carbon cycle
 Phosphorus
 Sulphides of carbon
 Bond type
 Oxides of silicon
 Metallic and non - metallic character
 Oxides of germanium, Tin and Lead
 Reactivity
 Silicates
 Hydrides
 Occurrence in the earth crust
 Ammonia NH3
 Soluble silicates
 Phosphine PH3
 Principles of silicate structures
 Arsine AsH3 , stibind SbH3 and bismuthin BiH3
 Classification of silicates
 Structure of the hydrides
 Orthosilicates (neso - silicates)
 Donor properties
 Pyrosilicates (soro-silicates , disilicates)
 Hydrazine N2H4
 Cyclic silicates
 Hydroxylamine NH2OH
 Chain silicates
 Liquid ammonia as a solvent
 Sheet silicates (Phyllo - silicates)
 Hydrogen azide and the azides
 Three - dimensional silicates  Nitrogen fixation
 Glass  Cyanamide process
 Organosilicon compounds and the silicones  Haber-Bosch process
 Organosilicon compounds  Fertilizers
 Preparation of organosilicon compounds  Urea
 Silicons  Phosphate fertilizers
 Hydrides of silicon  Halides
 Complexes  Trihalides
 Internal  bonding using d-orbitals  Pentahalides

www.jeebooks.in
THE p-BLOCK ELEMENTS AND THEIR COMPOUNDS 6.3

 Oxides of nitrogen  Bond lengths and p-d bonding


 Nitrous oxide N2O  Differences between oxygen and the othe
 Nitric oxide NO elements

 Nitrogen sesquioxide N2O3  Gemneral properties of oxides

 Nitrogen dioxide NO2 and dinitrogen tetroxide  Normal oxides


N 2O 4  Peroxides
 Dinitrogen pentoxide N2O5  suboxides
 Oxoacids of nitrogen  Basic oxides
 Nitrous acid HNO2  Amphoteric oxides
 Nitric acid HNO3  Acid oxides
 Oxides of phosphorus, Arsenic and Bismuth  Neutral oxides
 Trioxides  Oxides of sulphur
 Pentoxides  Dioxides SO2
 Other oxides  Trioxides SO3
 Oxoacid of phosphorus  Other oxides
 The phosphoric acid series  Detergents
 Thhe phosphorous acid series  Oxoacid of sulphur
 Major uses of phosphate  Sulphurous acid series
The group 16 elements - Chalcogens  Sulphuric acid series
 General properties  Thionic acid series
 Electronic structure and oxidation state  Peroxoacid series
 Acid rain and SO2  Oxohalides
 Uses of sulphur  Thionyl compounds
 Structures and allotropy of elements  Sulphuryl compounds
 Oxygen  Hydrides
 Ozone  Water
 Sulphur  Other hydrides
 Physical properties  Peroxides and polysulphides
 Chemical reactivity
 Hydrogen peroxides
 Standard reduction potentials (volts)
 Halides
 Oxidation state (+II), (+IV), (+VI)  Organo derivaties

www.jeebooks.in
6.4 THE p-BLOCK ELEMENTS AND THEIR COMPOUNDS
The group 17 elements - The haloges  Oxoacids
 Extraction and uses of the elements  Hypohalous acids HOX
 Fluorine  Halous acids HXO2
 Chlorine  Halic acid HXO3
 Bromine  Perhalic acid HXO4
 Iodine  Strength of the oxoacids
 General properties  Interhalogen compounds
 Size of atoms and ions  AX compounds
 Ionization energy
 AX3 compounds
 Types of bonds formed and oxidation staes
 AX5 compounds
 Melting and boiling point
 AX7 compounds
 Bond energy in X2 molecules
 Polyhalides
 Oxidizing power
 Basic properties of the halogens
 Reaction with water
 Pseudohalogens and Pseudohalides
 Reactivity of the element
The group 18 elements - Noble gases
 Hydrogen halides HX
 Occurence and recovery of the elements
 HF
 Use of the elements
 HCl
 Physical properties
 HBr and HI  Chemical properties of Helium
 Halides
 Chemical propterties of the noble gases
 Ionic halides
 Molecular ions formed under excited conditions
 Molecular covalent halides
 Clathrate compounds
 Bridging halides
 Chemistry of Xenon
 Preparation of anhydrous halides
 Xenon fluoride complexes
 Halogen oxides
 Structures and bonding in xenon compound
 Oxygen difluoride OF2
 XeF2
 Dioxygen difluoride O2F2
 XeF4
 Dichlorine monoxide Cl2O
 XeF6
 Chlorine dioxide ClO2
 Valediction
 Chlorine perchlorate Cl.ClO4
 Dichlorine hexoxide Cl2O6
 Dichlorine heptoxide Cl2O7

www.jeebooks.in
THE p-BLOCK ELEMENTS AND THEIR COMPOUNDS 6.5

CHAPTER
6
The p-Block Elements and
Their Compounds
EXERCISE # I
 Only one correct answer : 6. The correct boiling point order of stannic halides
1. The two main allotropes of tin are :- is :-
(a) SnF4 > SnCl4 > SnBr4 > SnI4
(a) grey tin, black tin (b) white tin, red tin
(b) SnI4 > SnBr4 > SnCl4 > SnF4
(c) white tin, black tin (d) white tin, grey tin
(c) SnCl4 > SnI4 > SnF4 > SnBr4
2. The colour of SnS and SnS2 are respectively :- (d) SnF4 > SnI4 > SnBr4 > SnCl4
(a) Black and White 7. Solution of SnCl2 and SnCl4 can be differentiated
(b) Brown and Yellow by different reagents, because they behave in
(c) Black and Colourless different way with different reagents. With HgCl2
which of the following option is correct :-
(d) Brown and Red
(a) SnCl2 react with HgCl2 to give white ppt. of
3. Tin particles crumble to powder during severe Hg2Cl2 which is further reduced by SnCl2 to
winters which is said “Tin plague”. It is due to :- give black precipitate
(a) at low temperature, transformation of white tin (b) SnCl4 react with HgCl2 to give white ppt. of
into grey tin which is accompanied by volume Hg2Cl2
expansion (c) SnCl4 does not react with HgCl2
(b) at low temperature, transformation of grey tin (d) both (a) and (c) are correct
into black tin which is accompanied by volume 8. Which of the following statement is correct ?
expansion (a) SnCl2 does not react with FeCl3
(c) Consuming grey tin causes disease (b) SnCl4 does react with FeCl3 to give FeCl62–
(d) None (c) SnCl2 reduces FeCl3 to FeCl2 & then when
K3[Fe(CN)6] is added, then blue colour is
4. SnO is example of amphoteric oxide, supported
observed.
by which of the following reaction :-
(d) None of these
(a) SnO + 2HCl  SnCl2 + H2O
9. A mass of arborescent crystals (lead tree) slowly
(b) SnO + 2NaOH  Na2SnO2 + H2O deposits when :-
(c) Both (a) & (b) (a) Zinc rod is hung in a solution of lead carbonate
(d) SnO + H2  Sn + H2O (b) Iron rod is hung in a solution of lead nitrate
5. Concentrated nitric acid when reacts with tin, a (c) Copper rod is hung in a solution of lead acetate
white precipitate is formed. The precipitate is :- (d) Zinc rod is hung in a solution of lead acetate
(a) H2SnO3 10. Lead is soluble in :-
(b) H2SnO2 (a) dilute HCl
(c) H2Sn5O11.4H2O (b) dilute H2SO4
(c) cold concentrated HCl
(d) SnO2.6H2O
(d) warm dilute HNO3
www.jeebooks.in
6.6 THE p-BLOCK ELEMENTS AND THEIR COMPOUNDS
11. Plumbo solvency - a serious menace where lead 17. H2O2 converts black precipitate PbS into :-
piping is used for water supply. It is caused by. (a) Elemental Pb
(a) Water soluble PbCO3 (b) Red colour Pb3O4
(b) Water insoluble PbCO3 (c) Grey colour PbO
(c) Slightly water soluble Pb(OH)2 (d) White coloured precipitate PbSO4
(d) Water soluble Pb(OH)2.PbCO3 18. The colour of PbO2 (lead dioxide) :-
12. Pb3O4 when reacts with HCl, the products formed (a) Yellow (b) Scarlet red
are :-
(c) Chocolate brown (d) Black
(a) PbCl2 , H2O , Cl2
19. The colour of Pb3O4 :-
(b) PbCl2 , H2O
(a) Yellow (b) Scarlet red
(c) PbCl2 , H2O , Cl2 , O2
(c) Chocolate brown (d) Black
(d) PbO2 , Cl2 , H2O
20. When lead is heated in air, a yellow powder A is
13. Pb3O4 when reacts with HNO3, the products formed. When A is fused, cooled and powdered,
formed are :- a reddish yellow crystalline varity B is formed, A
(a) Pb(NO3)2 , H2O , NO2 and B are respectivley :-
(b) Pb(NO3)2 , H2O (a) Massicot & Litharge
(c) Pb(NO3)2 , O2 , H2O (b) Red lead & lead dioxide
(d) Pb(NO3)2 , PbO2 , H2O (c) Litharge & Massicot
14. TEL (tetraethyl lead) is a colourless liquid which (d) Lead dioxide & Red lead
has boiling point 82°C. It is prepared by action
21. When litharge is calcinated for about 48 hrs in a
of :-
current of air at a temperature almost equal to
(a) ethyl alcohol with lead chloride 340°C, then the product formed is :-
(b) ethyl chloride vapour on sodilum lead alloy (a) PbO2 (b) Pb2O3
(c) ethyl ether vapour with lead metal (c) Pb3O4 (d) PbO
(d) ethyl benzene with lead chloride 22. The correct formula of white lead which is used
15. The correct melting point order of plumbous halide:- as pigment is :-
(a) PbF2 > PbCl2 > PbI2 > PbBr2 (a) PbCO3.Pb(OH)2 (b) PbCO3.2Pb(OH)2
(b) PbI2 > PbBr2 > PbCl2 > PbF2 (c) 2PbCO3.Pb(OH)2 (d) 3PbCO3.Pb(OH)2
(c) PbCl2 > PbF2 > PbBr2 > PbI2 23. Lithium aluminium hydride when reacts with
(d) PbF2 < PbCl2 < PbBr2 < PbI2 stannic chloride in ether medium at very low
16. CH3CH–OH   H2O + Product(P). temperature, the product formed :-
lead tetraacetate
(a) Sn2H6 (b) SnH2
(c) SnH4 (d) Sn3H8
CH3CH–OH
P is:- 24. PbO2 (Pb in +4 oxidation state) oxidises MnSO4
(a) CH3CO2H (b) CH3 – C – CH3 solution in acidic medium into :-
(a) pink solution of HMnO4
O (b) black MnO2 ppt.
(c) CH3 – CH = O (d) CH3 – C O
(c) reddish brown solution of MnO(OH)2
H3C – C O (d) Mn3O8

www.jeebooks.in
THE p-BLOCK ELEMENTS AND THEIR COMPOUNDS 6.7
25. Na2[SnO2] (Sn in +2 oxidation state) reduces 32. Anhydrous aluminium chloride is obtained by :-
Bi(NO3)3 into :- (a) strongly heating to 1000°C a mixture of alumina
(a) Black colour precipitate of Bi and coke in a current of chlorine
(b) Red colour precipitate of Bi (b) by passing dry chlorine or hydrogen chloride
(c) Brown colour precipitate of Bi over heated aluminium turnings
(c) both (a) & (b)
(d) None of these
(d) None
26. The lewis acidic strength order among bromo
halides :- 33. Zinc and copper reacts with concentrated nitric
acid to given
(a) BF3 > BCl3 > BBr3 > BI3
(a) NO 2 (b) NO
(b) BI3 > BBr3 > BCl3 > BF3
(c) N 2 O (d) N 2
(c) BBr3 > BCl3 > BF3 > BI3
34. B2H6 is :-
(d) BF3 > BI3 > BBr3 > BCl3
(a) electron rich molecule and paramagnetic
27. Orthoborates are :-
(b) electron rich molecule and paramagnetic
(a) planar unit (b) pyramidal unit
(c) electron defficient molecule and paramagnetic
(c) tetrahedral unit (d) angular unit
(d) electron defficient molecule and diamagnetic
B2 H 6 (THF)
 35. Zinc reacts with dilute nitric acid to give
28. H O  OH 
2 2
Product(P). P is
(a) NO 2 (b) NO
(c) N 2 O (d) N 2
(a) (b) 36. Aluminium triethyl in conjuction with titanium
OH OH
chloride TiCl4 is called:-
(c) OH (d) OH (a) Al-titanium alloy (b) Alum
(c) Goldsmith mixture (d) Zeigler natta catalyst
29. Which of the following reaction, product are
correctly predicted :- 37. Copper reacts with dilute nitric acid to give
(a) NO 2 (b) NO
(a) B3N3H6 + HCl  (XS)
(c) N 2 O (d) N 2
(b) B3N3H3Cl3 + NaBH4  B3N3H6 + NaCl +
B2H6 38. Alums containing thallium one of the types :-
(c) B3N3H6 + 9H2O H3BO3 + NH3 + H2 (a) K2SO4 .Tl(SO4)3.24H2O
(b) Tl2SO4.Al2(SO4)3.24H2O
(d) All
(c) K2SO4.Al2(SO4)3.24H2O
30. The oxidation stae of B, N in B 3N3H6 are
respectively :- (d) Tl(SO4).K2SO4.24H2O
(a) +3, –3 (b) +1, –1 39. A silvery white metal liquefies on a hot summer
day but is solid an cold days. It is readily attacked
(c) –3, +3 (d) –1, +1
by dilute acid liberating hydrogen. A is :-
31. B2H6 can not be methylated beyond :- (a) B (b) Ag
(a) Me2B2H4 (c) Al (d) Mg
(b) Me3B2H3 40. Which of the following is orange coloured solid :-
(c) Me4B2H2 (a) CCl4 (b) SiBr4
(d) Me5B2H (c) SnI4 (d) SbCl3

www.jeebooks.in
6.8 THE p-BLOCK ELEMENTS AND THEIR COMPOUNDS
41. A gas at low temperature does not react of the 48. A gas exists in three allotropic forms  is:-
compounds. It is inert in nature. The combustion (a) SO 2 (b) SO3
of this gas is exceptionally an endothermic reaction. (c) CO2 (d) NH3
The gas is :-
49. KOCN + (NH4)2SO4  

(a) O 2 (b) N 2
In the following reaction one of the product is :-
(c) CO (d) H2
(a) NH4NO2 (b) NH4NO3
42. An explosive compound (a) when reacts with
(c) NH2CONH2 (d) both N2 & O2
water, it give NH4OH and HCl. A is :-
50. Which element become passive due to protective
(a) TNG (b) TNT layer of oxide formation, when reacts with
(c) NCl3 (d) HNO3 concentrated nitric acid
43. Three allotropes A , B and C of phosphorus are (a) Fe (b) Cr
transfered into each other in the following (c) Al (d) all of these
reaction :- 51. The ratio of 2C–2e and 3C–2e bonds in diborane
470K 570K
A 
1200atm
 B 
CO 2atm C is :-
(a) white, black, red (b) black, white, red (a) 0 (b) 1
(c) red, black, white (d) red, white, black (c) 2 (d) 3
44. Which is incorrectly matched :- 52. Sulphur dioxide reacts with hydrogen sulphide to
give sulphur precipitate. Here SO2 and H2S acts
 Cs+ + Br –
(a) CsBr 3  as:-
3

 Ag+ + BrO –


(b) AgBrO3  (a) oxidising and reducing reagent respectively
3
(b) reducing and oxidising reagent respectively
 I3+ + (IO3–)3
(c) I4O9 
(c) base and acid respectively
 IO – + IO +
(d) I2O4  (d) acid and base respectively
2 2
45. Copper reacts with concentrated nitric acid to give 53. SO2 and Cl2 bleaches coloured substance :-
(a) NO 2 (b) NO (a) by reduction and oxidation respectively
(c) N 2 O (d) N 2 (b) by reduction and reduction respectively
46. C(OH)4 is unstable but Si(OH) 4 is a stable (c) by oxidation and reduction respectively
compound because :- (d) by oxidation and oxidation respectively
(a) C = O bond energy is very low 54. Zeolite which is hydrated sodium aluminium silicate
(b) C = O bond energy is very high is used as water softner because :-
(c) Si = O bond energy is very high (a) the potassium ions of zeolite are exchanged
with barium and strontium ions when hard
(d) All of these water is passed through it
47. When a salt X reacts with ozone in aqueous (b) the sodium ion of zeolite are exchanged with
medium, a compound Y is produced ozone also barium and strontium ions when hard water is
reacts with Y and produces compound Z. Z acts passed through it
as an oxidising reagent.Then X,Y, Z can be :- (c) the potassium ions of zeolite are exchanged
(a) X = HI ; Y = I2 ; Z = HIO3 with barium and strontium ions when hard
(b) X = KI ; Y = I2 ; Z = HIO3 water is passed through it
(d) the sodium ions of zeolite are exchanged with
(c) X = KI ; Y = I2 ; Z = HOI
calcium and magnesium ions when hard water
(d) X = KI ; Y = I2 ; Z = HIO4 is passed through it

www.jeebooks.in
THE p-BLOCK ELEMENTS AND THEIR COMPOUNDS 6.9
55. Sulphur dioxide acts as more powerful reducing 62. Among F–, Cl–, Br–, I– :-
agent in :- (a) I– is the least stable base
(a) acidic medium (b) neutral medium (b) I– is the weakest reducing reagent
(c) alkaline medium (d) None (c) I– is the most basic
56. The number of moles of H2O2 required to convert (d) I– is the strongest reducing reagent
1 mole of PbS into PbSO4. 63. Which of the following product is formed when
(a) 1 (b) 2 sulphur dioxide gas is passed through sodium
(c) 3 (d) 4 chlorate in strongly acidic solution :-
 N 2 gas (a) NaClO4 (b) ClO2
57. Metal (M)  Solids(S)
(c) NaSO3 (d) SO3
S+ H2O  G(gas)
64. The oxide of chlorine which has odd number
G + CuSO4 solution  Blue colour solution(X)
electrons and paramagnetic in nature is :-
Blue colour solution is :- (a) Cl2O7 (b) Cl2O
(a) [Cu(NH3)6]SO4 (b) [Cu(NH3)4]SO4 (c) ClO2 (d) Cl2O6
(c) [Cu(NH3)2]SO4 (d) [Cu(NO3)4]SO4 65. The colour of vapour obtained when an iodide
58. Xe + F2 P1Xe + F2 P2 Xe salt is heated with concentrated H2SO4 is :-
+ F2 P3 (a) pale yellow (b) violet
(2:1 volume ratio)(1:5 volume ratio)(1:20 volume (c) reddish brown (d) greenish yellow
ratio) 66. HI can be prepared by which of the following
P1 , P2 , P3 are respectively :- methods :-
(a) XeF2 , XeF4 , XeF6 (b) XeF2 , XeF6 , XeF4 (a) PI3 + H2O  (b) H2 + I2  Pt

(c) XeF6 , XeF4 , XeF2 (d) XeF4 , XeF2 , XeF6 (c) KI + H2SO4  (d) I2 + H2S 
59. XeO3 in strong alkaline medium (above pH 10.5) 67. What is the maximum coordination number of
forms :- iodine in the mixture of IF7 and CsF :
(a) HXeO4– (perxenate ion) (a) 2 (b) 4
(b) XeO64– (xenate ion) (c) 6 (d) 8
(c) HXeO4– (xenate ion) 68. For which halogen, reaction with water will be
(d) XeO64– (perxenate ion) most exothermic :-
(a) Fluorine (b) Chlorine
60. Among all noble gases, only Xe reacts with F2 to
give many Xenon fluoride because Xenon :- (c) Bromine (d) Iodine
(a) has largest size among all noble gases 69. A greenish yellow gas reacts with KOH to give
potassium halide which can be used in fire works
(b) has the lowest ionisation enthalpy among all and safety matches. The gas and half Fe
noble gases respectively are :-
(c) has highest heat of vaporisation among all noble (a) Br2 ; KBrO3 (b) Cl2 ; KClO3
gases (c) I2 ; NaIO3 (d) None
(d) is the most readily available among all noble 70. How many statements are correct regarding neon:-
gases (a) Neon is diatomic in nature
61. ClO3 is the mixed anhydride of :- (b) Neon does not form clathrate with quinol
(a) HClO2 and HClO3 (b) HClO2 and HClO4 (c) Neon lights are visible even in fog and moist
(c) HOCl and HClO3 (d) HClO3 and HClO4 (d) All of these

www.jeebooks.in
6.10 THE p-BLOCK ELEMENTS AND THEIR COMPOUNDS
71. XeF6 when reacts with cesium fluoride it acts as:- 78. A given silicate is amphibole :
(a) Bronsted acid (b) Bronsted base II III
Na 2 Fe x Fe y [Si8O22] (OH)2
(c) Lewis acid (d) Lewis base
The value of x + y is
72. A black solid X reacts with NH3 to form a mild
explosive which decomposes to give violet (a) 5 (b) 7
coloured gas. X also reacts with H2 to give an acid (c) 3 (d) 9
Y. Y can also be prepared by heating its salt with 79. On contact process, one of the step is
H2PO4. X and Y are :- 2SO2(g) + O2(g) 2SO3(g)
(a) Cl2 , HCl (b) F2 , HF This reaction is carried out :-
(c) Br2, HBr (d) I2, KI (a) at 500 K and 9 bar pressure with Pt/Rh as
73. Fluorine is obtained by the interaction of K2MnF6 catalyst
with Lewis acid SbF5 because :- (b) at 1200 K and 5 bar pressure with Pt/Rh as
(a) In the first step, K2MnF6 is converted into MnF4 catalyst
(b) In the second step, MnF4 is dissociated into (c) at 500 K and 7 bar pressure with V2O5 as
MnF2 and F2 catalyst
(c) Both (a) & (b) (d) at 720 K and 2 bar pressure with V2O5 as
(d) None of these catalyst
74. Which of the following product is formed when 80. In ostwald’s process , one of the step is
one mole periodate ion (IO4–) reacts with excess 4NH3(g) + 5O2(g)  4NO(g) + 6H2O(g)
of iodide ions followed by the acidification of This reaction is carried out :-
solution ? (a) at 500 K and 9 bar pressure with Pt/Rh as
(a) IO– (b) IO3– catalyst
(c) I2 (d) I2O5 (b) at 1200 K and 5 bar pressure with Pt/Rh as
75. Which of the following options is incorrect ? catalyst
(a) Sodium hypochlorite is used as bleaching and (c) at 500 K and 7 bar pressure with V2O5 as
sterilising agent. catalyst
(b) All halogens except F2 are more reactive than (d) at 720 K and 2 bar pressure with V2O5 as
interhalogens. catalyst
(c) Finely divided iron can not form ferric chloride 81. 2MX + H2SO4 2HX + M2SO4
with hydrochloric acid. MX can be :-
(d) HNO3 oxidises sulphur to H2SO4 but only (a) MI (b) MCl
oxidises selenium to H2SeO3. (c) MBr (d) MI
76. Ester of boric acid B(OC2H5)3 imparts :- 82. In a mixture of HNO3 and H2SO4, HNO3 acts
(a) Blue colour to the burner flame as:-
(b) Green colour to the burner flame (a) base (b) acid
(c) Red colour to the burner flame (c) oxidising reagent (d) reducing agent
(d) Brown colour to the burner flame 83. Br2 + 2X–(aq) X2 + 2Br–(aq)
77. The repeating structural units in silicone is :- above reacation is correct when X– is :-
(a) R2SiO– (b) SiO2 (a) F– , Cl– (b) I– only
(c) ROSiOR (d) –SiR2OOR (c) F– , Cl– , I– (d) Cl– , I–

www.jeebooks.in
THE p-BLOCK ELEMENTS AND THEIR COMPOUNDS 6.11
84. Which acid on heating produces phosphoric acid 92. H2O2 when reacts with KIO 4 and NH 2OH
by disproportionation reaction ? respectively, it acts as :-
(a) Phosphoric acid H3PO4 (a) reducing reagent & oxidising reagent
(b) Phosphorus acid H3PO3 (b) reducing reagent & reducing reagent
(c) Peroxy monophosphoric acid (c) oxidising reagent & oxidising reagent
(d) Metaphosphoric acid (d) oxidising reagent & reducing reagent
85. In copper sulphate pentahydrate how many anion 93. When titanium oxide reacts with H2O2 , it gives :-
water is present. (a) red colour due to formation of titanium oxide
(a) 0 (b) 1 (b) yellow orange colour due to formation of
(c) 2 (d) 3 titanium oxide/ titanium sulphate
86. Temporary hardness of water is caused by (c) black colour due to formation of titanium oxide
presence of (d) violet colour due to formation of titanium oxide
(a) Soulble nitrate 94. Chemical reagent added to remove temporary
(b) Soulble nitrite hardness of water is -
(c) Insoulble carbonate (a) Na2CO3 (b) Ca(OH)2
(d) Soulble bicarbonate of Ca and Mg (c) MgSO4 (d) CaSO4
87. The method which is not applicable for 95. Ozonolysis is a reaction when O3 gas is passed
manufacturing ammonia? through :-
(a) NH2 – C – NH2 + H2O (a) saturated hydrocarbon
O (b) unsaturated hydrocarbon
(b) NH4Cl + Ca(OH)2  (c) cyclohexane ring
(c) ZnSO4 + NH4OH  (d) cyclopentane ring
(d) (NH4)2SO4 + NaOH  96. Ozone oxidises moist sulphur to :-
X NaOH / 
 HCl
 Y  White fume (a) SO2 (b) SO3
88.  Colourlesssolid 
(c) H2SO4 (d) H2SO3
CaCl2 KMnO 4
X   Z Z  Colourless solution 97. On heating ozone, its volume :-
 White ppt.

Salt X is :- (a) decreases to half


(a) BaC2O4 (b) (NH4)2CO3 (b) becomes double
(c) CaC2On (d) (NH4)2C2O4 (c) increases to 3/2 times
89. Ammonia can be dried by :- (d) remain unchanged
(a) concentrated H2SO4 98. The laboratory test “trailing of mercury” is
(b) P4O10 applicable to identify :-
(c) CaO (a) O 2 (b) H2O
(d) anhydrous CaCl2 (c) H2O2 (d) O 3
90. HNO3 when reacts with P4O10 it gives :- 99. Which is responsible for depleting ozone layer
(a) NO 2 (b) N2O 3 which protects from ultraviolet light ?
(c) N2O 4 (d) N2O 5 (a) Chloroform
91. H2O2 is not an example of :- (b) Chlorofluoro carbon
(a) reducing reagent (b) oxidising reagent (c) Carbon tetrachloride
(c) dehydrating reagent (d) Bleaching reagent (d) Methyl chloride

www.jeebooks.in
6.12 THE p-BLOCK ELEMENTS AND THEIR COMPOUNDS
100. Aqueous solution of bleaching powder is source 108. O-O bond length :-
of :- (a) O2 < O3 < H2O2 (b) O3 < O2 < H2O2
(a) O2 + Cl– (b) OH– + Cl2 (c) H2O2 < O3 < O2 (d) O2 < H2O2 < O3
(c) OH– + Cl– (d) O3 + Cl– 109. The permutit process is based on
Ca (OCl)Cl
101. CH3COCH3   CHCl3 + Product (a) exchange of basic radical or metal ion. e.g.
aqueous solution
Ca2+ and Mg2+ by Na+
(P). P is :-
(b) exchange of acidic radical like HCO3– / CO32–
(a) CH3CH3 (b) CH3CHOHCH3
by SO42– , NO3–
(c) (CH3CO2)2Ca (d) (HCO2)2Ca
(c) exchange of both acidic and basic radical
102. The incorrect statements are :-
(d) None
(a) Bleaching powder is strong oxidising reagent
110. Pair of metals does not dissolve in nitric acid :-
due to evolution of nascent oxygen.
(a) Cr, Al (b) Ag, Au
(b) Bleaching powder is strong oxidising reagent
due to evolution of nascent chlorine. (c) Al, Ag (d) Pt, Hg
(c) Bleaching powder reacts with methanol to give 111. Ammonia on reaction with sodium hypochlorite
calcium formate gives :-
(d) Bleaching powder oxidises NH3 into NCl3 (a) NO (b) NH4Cl
103. A pale yellow solid (A) having crown shape is (c) NH2–NH2 (d) HNO2
heated with concentrated H 2SO4 , it gives 112. NH4Cl(s) is heated in test tube. Vapours are
suffocating smell of gas B, which is passed through brought in contact with red litmus paper, which
moistened with starch iodide paper and turn it into
changes to blue and then red. It is because of :-
blue. The gas B and solid A are respectively.
(a) formation of NH4OH and HCl
(a) SO3 & S8 (b) SO2 & S8
(c) SO2 & S2N2 (d) SO3 & SO2 (b) formation of NH3 and HOCl
104. H2O2 acts as oxidising reagent when it reacts with:- (c) greater diffusion of NH3 than HCl
(a) Potassium ferrocyanide in acidic medium (d) greater diffusion of HCl from NH3
(b) Potassium ferrocyanide in basic medium 113. Which of the following gives mixture of SO2 and
(c) Potassium ferricyanide in acidic medium SO3 on heating :-
(d) Potassium ferricyanide in basic medium (a) ZnSO4 (b) CuSO4
105. H2O2 acts as reducing reagent when it reacts with:- (c) Fe(SO4)3 (d) FeSO4
(a) Potassium ferrocyanide in acidic medium 114. ZSM–5–A type of zeolite is used :-
(b) Potassium ferrocyanide in basic medium (a) to convert starch into alcohol
(c) Potassium ferricyanide in acidic medium (b) to convert alcohols into gasoline
(d) Potassium ferricyanide in basic medium. (c) to convert petroleum products into branched
106. Ca(OCl)Cl when reacts with FeSO4 solution, hydrocarbon
Ca(OCl)Cl is converted into :- (d) to convert hydrocarbon into alcohol
(a) Cl2 (b) CaCl2 115. In alumino silicates
(c) Ca(OCl)2 (d) Ca(OH)2 (a) Aluminium atom replaces few silicon atoms in
107. Calgon is used for water treatment. Calgon is:- three dimensional network of silicon dioxide
(a) Na2[Na4(PO3)6] (b) Na4[Ca(PO3)6] (b) Silicon atoms replaces few aluminium atom in
(c) Na2[Ca2(PO3)6] (d) Na4[Na2(PO3)6] three dimensional network of silicon dioxide

www.jeebooks.in
THE p-BLOCK ELEMENTS AND THEIR COMPOUNDS 6.13
(c) Aluminium atom replaces few silicon atoms in 124. A certain salt X, gives the following results
three dimensional network of alumina (i) Its aqueous solution is alkaline to litmus
(d) Silicon atoms replaces few aluminium atom in (ii) It swells up to a glassy mateiral Y on strong
three dimensional network of alumina heating
116. Concentrated HNO3 reacts with iodine to give :- (iii) When concentrated H2SO4 is added to a hot
(a) HI (b) HOI solution od X, white crystal of an acid Z
(c) HIO4 (d) HIO3 separates out.
117. Blood pH is maintained between 7.26 to 7.42. It X is -
is due to - (a) Na2B4O7 (b) Ca2B6O11
(a) H2SO3 / HSO3– buffer system (c) B2O3.TiO2 (d) NaBH4
(b) H2SO4 / HSO4– buffer system
125. Z is
(c) H2CO3 / HCO3– buffer system
(a) HBO2 (b) H3BO3
(d) HF / F– buffer system
(c) H4B2O7 (d) None
118. 2E(s) + N2(g)  
 (s) 126. When metal X is treated with sodium hydroxide,
2E(s) + 3X2(g) 
 EX(s) a white precipitate (a) is obtained, which is soluble
E belongs to :- in excess of NaOH to give soluble complex (b).
(a) Group 13 element (b) Group 14 element Compound (a) is soluble in dilute HCl to form
(c) Group 15 element (d) Group 16 element compound (c). The compound (a) when heated
strongly gives (d) , which is used to extract metal.
119. S2O32– + Pb2+   PbS2O3 (Precipitate P1) X is :-
+ 2–
PbS 2O 3 + H 2 O    2H + SO 4 + PbS (a) B (b) Al
(Precipitate P2)
(c) Tl (d) Zn
PbS + H2O2   H2O + PbSO4 (Precipitate P3)
Correct option - 127. B and D are respectively -
(a) P1 is black colour precipitate (a) Tl(OH)2 ; Tl2O (b) Zn(OH)2 ; ZnO
(b) P2 is white colour precipitate (c) Al(OH)3 ; Al2O3 (d) B(OH)3 ; B2O3
128. Cl Cl Cl
(c) Precipitate P1 is soluble in excess thiosulphate Al   Al 
(d) P3 is black colour precipitate Cl Cl Cl
120. Strongest acid is (a)  > >
(a) HI (b) HBr (b)  > >
(c) HCl (d) HF + SbF5 (c)  > >
121. The least atomic radius is observed in (d)  > >
(a) Al (b) Ga 129. In alum like M I M III  SO4  2 .12H 2O :-
(c) In (d) Tl
(a) Six water molecule are octahedrally
122. The highest electronegativity is observed in
coordinated around MI
(a) Al (b) Ga
(b) Six water molecule are octahedrally
(c) In (d) Tl
coordinated around MIII
123. The highest and least oxidation state of element in
group 16 will be - (c) These water molecule are characterised as
lattice water
(a) +6, +2 (b) +4, –2
(d) All
(c) +5, –3 (d) +6, –2

www.jeebooks.in
6.14 THE p-BLOCK ELEMENTS AND THEIR COMPOUNDS
130. BF3 + 3LiAlH4  2B2H6 + 3LiF + 3AlF3 137. X  
 Na2SO4 + Na2S5
Correct option for this reaction :- X is formed as one of the product when sulphur is
(a) This reaction is example of redox reaction boiled with caustic alkali. X is -
where LiAlH4 acts as reducing agent. (a) Na2SO3 (b) Na2S4O6
(b) This reaction is example of redox reaction (c) Na2S2O3 (d) Na2S
where BF3 acts as reducing agent. 138. Roasting salt cake with coke results -
(c) This reaction is example of non redox reaction (a) formation of mixture of CO2 & S
(d) Oxidation state of hydrogen atom in reactant (b) formation of mixture of CO2 & Na2S
side is different w.r.t. oxidation state of (c) formation of mixture of CO & Na2S
hydrogen atom in product side.
(d) formation of mixture of CS2 & Na2O
131. NH3  N2 , This conversion is carried out in
139. FeCl3 when reacts with sodium thiosulphate , it
presence of -
gives intense purple colour complex X and then
(a) Bleaching powder (b) CuO on standing the purple colour disappears. What is
(c) Both (a) and (b) (d) O2 (air) X and colour is disappeared because of -
132. N2H4  N2 , This conversion is carried out in (a) X = [Fe(S2O3)3]3– ; colour is disappeared due
presence of - to formation of Fe2+
(a) I2 (b) KIO3 (b) X = [Fe(S2O3)2]–1 ; colour is disappeared due
(c) Fe3+ (d) All to formation of Fe2+
133. The bond order of S–O bond in SO2 is - (c) X = [Fe(S2O3)2]–1 ; colour is disappeared due
(a) 2 (b) 1.5 to formation of [Fe(S2O3)2]
(c) 1 (d) 2.5 (d) X = [Fe(S2O3)3]3– ; colour is disappeared due
to formation of [Fe(S2O3)2]4–
134. Which compound when reacts with SO2 , non-
437K
redox reaction takes place ? 140. Cl2 + F2   X1
(a) Cl2 (g) (b) Fe3+ solution (equal volume)
573K
(c) KMnO4, H+ (d) Na2CO3 solution Cl2 + 3F2  X2
135. Which oxy acid of sulphur is not possible ? (excess)
(a) H2S2O7 I2 + Cl2   X3
(b) H2SO5 (excess)
(c) H2SO6 Which of the following statements are correct -
(d) H2S2O6 (a) X2 is colourless gas (b) X3 is orange solid
136. For the following reaction, (c) X1 is colourless gas (d) All
C12H22O11(s) + H2SO4 + 1/2 O2(g)  11C(s) 141. Pb3O4 can not oxidise :-
+ CO2(g) + 12H2O + SO2(g) (a) HCl (b) H2SO4
which of the following statement is correct - (c) HNO3 (d) All
(a) Here H2SO4 acts as dehydrating as well as 142. A scarlet red powder when reacts with HNO3 , it
oxidising reagent gives chocolate brown precipitate. Scarlet red
(b) The reaction is endothermic in nature powder is :-
(c) The solution turns colourless (a) PbO2 (b) PbO
(d) Colourless and odourless gas is evolved (c) Pb3O4 (d) Pb2O3

www.jeebooks.in
THE p-BLOCK ELEMENTS AND THEIR COMPOUNDS 6.15
143. A scarlet red powder when reacts with HNO3 , it 2. Which of the following reactions are correctly
gives chocolate brown precipitate. Chocolate given ?
brown precipitate is :- (a) SnO2 + 2H2SO4  Sn(SO4)2 + 2H2O
(a) PbO2 (b) PbO (b) SnO2 + 2NaOH  Na2SnO2 + H2O
(c) Pb3O4 (d) Pb2O3 (c) SnCl4 + Hg  Hg2Cl2 + SnCl2
144. The number of lone pair on Cl in HOCl, HClO2, (d) SnCl2 + Zn  ZnCl2 + Sn
HClO3, HClO4 are respectively :-
3. Which of the following names are correct :-
(a) 1,2,3,4 (b) 0,1,2,3
(a) Crystalline SnS2 : Mosaic Gold
(c) 4,3,2,1 (d) 3,2,1,0
(b) SnCl4.5H2O : Butter of tin
145. Number of Cl = O bonds present in HOCl,
(c) SnO2 : Cassiterite
HClO2, HClO3, HClO4 are respectively :- :-
(d) SnH4 : Stannane
(a) 1,2,3,4 (b) 0,1,2,3
(c) 4,3,2,1 (d) 3,2,1,0 4. SnS2 is soluble in :-
146. Boric acid can act as strong acid by addition of :- (a) concentrated HCl (b) YAS
(a) Cis-1, 2-diol (b) Borax (c) H2O (d) NH3
(c) Trans-1,2-diol (d) Na2HPO4 5. Which of the following are example of mixed
oxide?
147. In aqueous solution, orthoboric acid acts as :-
(a) Proton donor acid (b) Lewis acid (a) PbO (b) Pb2O3
(c) Triprotic acid (d) Strong acid (c) Pb3O4 (d) Pb2O2
148. The transition temperature for rhombic sulphur to 6. Which of the following reaction are correct ?
monoclinic sulphur is :- (a) PbO2 + 4HCl PbCl2 + 2H2O + Cl2
(a) 95.5°C (b) 112°C (b) 2PbO2 + 4H2SO4 2PbSO4 + 2H2O + O2
(c) 160.5°C (d) 190°C (c) PbO2 + 2NaOH Na2PbO3 + H2O
149. Correct bond dissociation energy order :- (d) 2PbO2   2PbO + O2
730 C
(a) N N > PP (b) N–N > P–P
7. Which can be formed as yellow precipitate ?
(c) O–O < S–S (d) All
(a) PbSO4 (b) PbI2
150. Chlorofluoro carbon (freons) is represented as
(c) PbCrO4 (d) Pb(OH)2 . PbCO3
CFmCln. It can be :-
8. Which of the following options are correct ?
(a) CF2Cl2 (b) CFCl3
(a) PbS is insoluble in hot concentrated HCl
(c) CClF3 (d) All
(b) PbS is insoluble in boiling dilute HNO3
EXERCISE # II (c) White lead is used as pigment
 One or More Than One Correct Answer : (d) Pb(OH)2 is dissolved in acid as well as in base
1. Which of the following statements are correct 9. Which of the following reaction give NH3 gas :-
regarding SnCl2 ? (a) Boron nitride is fused with potassium
(a) SnCl2 conducts electricity hydroxide
(b) SnCl2 is dissolved in water to form turbidity of (b) Boron nitride reacts with F2 gas
stannous oxychloride (c) Boron nitride undergoes hydrolysis
(c) SnCl2 is ionic solid (d) Boron imide undergoes decomposition
(d) SnCl2 is covalent angular molecule

www.jeebooks.in
6.16 THE p-BLOCK ELEMENTS AND THEIR COMPOUNDS
10. Which of the following compound forms chelating 15. The pair that yield same gaseous product on
oxy derivative with B(OH)4– :- reaction with water are :-
OH (a) AlN, CaCN2 (b) Ca & CaH2
(a) Catechol ; (c) F2 & XeF4 (d) K & KO2
OH
CO2H 16. HNO3 :-
(b) ; Salicylic acid (a) oxidises S to H2SO4
OH (b) oxidises P4 to H3PO4
OH
(c) oxidises Se to H2SeO3
(c) Quinol ;
HO (d) oxidises I2 to HIO3
CO2H 17. Which of the following options are correct ?
(d) ; p-hydroxy benzoic acid (a) HF is the only hydra acid of halogen which
HO
does not form any precipitate with AgNO3.
11. The oxidation state of hydrogen atom in B3N3H6:-
(b) Oxidising property of halogens F2 > Cl2 >
(a) All hydrogens have same oxidation state of +1
Br2 > I2
(b) All hydrogens have same oxidation state of –1
(c) The bond dissociation energy of halogen Cl2 >
(c) Hydrogens attached with boron have oxidation
Br2 > F2 > I2
state of –1
(d) Hydrogens attached with nitrogen have (d) The bond dissociation energy of hydrogen
oxidation state of +1 halide HF > HCl > HBr > HI
12. Which of the following statements are incorrect ? 18. Astatin is the element below iodine in the group
(a) B–F bond length in BF3 is less than B–F bond VII A of the priodic table. Which of the following
length in NOBF4 statements are not true for astatine ?
(b) N–O bond length in NO is more than N–O (a) It is more electronegatives than iodine
bond length in NOClO4 (b) It will exhibit only – 1 oxidation state
(c) The terminal B–H bond length in B2H6 is more (c) Intermolecular forces between the astatine
than C–H bond length in C2H6 molecules will be more than that between
(d) The terminal B–H bond length in B2H6 is more iodine molecules.
than C–H bond length in C2H4
(d) It is less electronegatives than iodine
13. I2 is soluble in KI solution but is insoluble in water
19. Which statements are correct about halogens ?
because :-
(a) I2 is soft lewis acid and I– is hard lewis base (a) They are all diatomic and form univalent ions
(b) I2 is hard lewis acid and H2O is hard lewis base (b) Halogens have the smallest atomic radii in their
respective periods
(c) I2 is soft lewis acid and I– is soft lewis base
(d) I2 is soft lewis acid and H2O is hard lewis base (c) They are all diatomic and form divalent ions
14. Select correct statements :- (d) They are all reducing agent
(a) Borax is used as buffer 20. Which of the following reactions are feasible ?
(b) 1(M) borax solution reacts with equal volume (a) 2KCl + I2 2KI + Cl2
of 2(M) HCl solution (b) 2KI + Cl2 2KCl + I2
(c) Titration of borax can be made using methyl (c) MnO2 + 4HCl MnCl2 + Cl2 + 2H2O
orange as the indicator (d) MnO2 + 2NaBr + 2H2SO4  Br2 + 2Na+
(d) Coloured bead is obtained in borax bead test
+ Mn2+ + 2 SO42– + 2H2O
contains metaborate

www.jeebooks.in
THE p-BLOCK ELEMENTS AND THEIR COMPOUNDS 6.17
21. Which of the following statements are correct ? 27. Which of the following reactions give NH3 gas ?
(a) At ordinary temperature, the rate of disproport- (a) Zn + NaOH + NaNO3  

ionation of hypohalites of chlorine , bromine (b) (NH4)2CO3 

and iodine follows the order IO– > BrO– > ClO–.
(c) NH4ClO4 

(b) Fluorine can not be prepared in aqueous
medium by electrolysis, since it decomposes (d) Al + NaOH + NaNO3  

water with liberation of ozonised oxygen. 28. Which compound is not known ?
(c) HI is a stronger acid than HBr because of the (a) NX5 (b) BF63– (c) HPO2 (d) NOBr3
low dissociation energy of HI. 29. Which of the following statements are correct -
(d) In aqueous solution chlorine is a stronger (a) In ammonium nitrate, anionic part is sp2
oxidising agent than fluorine. hybridised
22. The products formed when potassium chlorate (b) In ammonium nitrate, cationic part is sp3
reacts with iodine. hybridised
(a) KIO4 (b) KOI (c) KIO3 (d) Cl2 (c) In ammonium nitrite , cationic part is sp2
hybridised
23. Correct order regarding HNO3 and H2SO4 :-
(d) In ammonium nitrite , anionic part is sp2
(a) acidic strength HNO3 > H2SO4
hybridised
(b) boiling point H2SO4 > HNO3 30. Which of the following can not coexist with solution?
(c) oxidising power HNO3 > H2SO4 (a) H2S + SO2 (b) Fe2S3 + HCl
(d) Volatility HNO3 > H2SO4 (c) HNO2 + KI (d) Na2CO3 + NaHCO3
24. Which of the following option are correct :- 31. Which of the reaction produces PH3 as one of the
(a) Zn when reacts with very dilute HNO3 , it gives product?
Zn(NO3)2 and NH4NO3 (a) Ca3P2 + H2O  
catalyst
(b) 2SO2(g) + O2(g)  2SO3(g) , it is an (b) P4(white) + NaOH + H2O 

example of endothrmic reaction
(c) PH4I + KOH 
(c) I2 produces HIO3 and NO2 with concentrated
(d) H3PO2  

HNO3
 H 2O
(d) Ag when reacts with concentrated HNO3 , it 32. A (Calcium imide)   B + C(gas)
gives AgNO3 and NO2 Ca (OCl)Cl
C (gas )   D(gas)
25. Bronze when treated with hot concentrated HNO3:- D (gas)  Mg  H 2O
E   C(gas)
(a) Cu and Zn both readily dissolves Correct option :-
(b) Copper is dissolved to give blue solution (a) C gas is N2O (b) D gas is N2O3
(c) Sn is dissolved to give colourless solution (c) C gas is N2O4 (d) D gas is N2
Sn(NO2)2 33. F2 reacts with H2O to give :-
(d) Tin is oxidised to metastannic acid in the form (a) HOF (b) HF
of white precipitate (c) O 2 (d) O 3
26. Which of the following statements are incorrect ? 34. Which of the following statements are correct ?
(a) To carry out brown ring test for nitrate ion, (a) The hydride of group 15 elements act as
dilute H2SO4 should be used. oxidising agents
(b) To carry out brown ring test for nitrite ion, dilute (b) The hydride of group 15 elements act as
H2SO4 should be used. reducing agents
(c) Devarda’s alloy is alloy of aluminium, copper (c) The reducing power increases is going from
and zinc where zinc percentage is maximum NH3 to BiH3
(d) Copper strip is added in concentrated H2SO4, (d) The basic character decreases in going from
the solution become blue NH3 to SbH3

www.jeebooks.in
6.18 THE p-BLOCK ELEMENTS AND THEIR COMPOUNDS
HCl H 2O 2
35. CH3 – CH = CH2   P1 43. NH3 when reacts with Cl2 , then :-
CH3 – CH = CH2 
HBr  H 2O 2
 P2 (a) NCl3 is formed when Cl2 is present in excess
Correct statements :- (b) N2 is formed when Cl2 is present in excess
(a) P1 is CH3CH2CH2Cl (c) N2 is formed when NH3 is present in excess
(b) P1 is CH3CHClCH3 (d) NCl3 is formed when NH3 is present in excess
(c) P2 is CH3CH2CH2Br 44. SO2 + Cl2 Compound X1
(d) P2 is CH3CHBrCH3 CO + Cl2 Compound X2
36. Which of the followings are diamagnetic ? Correct statement regarding X1 and X2 ?
(a) H2O2 (b) O 3 (a) X1 is used to chlorinate alkane, eg. ethane to
(c) SO2 (d) KO 3 ethyl chloride
37. Which of the followings reagent can be used to (b) X2 is poisonous gas
change the colour when SO2 gas is passed ? (c) The oxidation state of sulphur in compound
(a) Bromine water X1 is +4
(b) Acidified FeCl3 solution (d) The oxidation state of carbon in compound
(c) Chlorine water X2 is +4
(d) Acidified K2Cr2O7 solution 45. Dioxygen can be obtained in the laboratory by :-
38. Ozone can oxidise :- (a) Heating potassium chlorate in presence of
MnO2 at high temperature
(a) K2MnO4 (b) KMnO4
(b) Thermal decomposition of oxide of metal which
(c) S(moist) (d) Ag
is low in the electrochemical series, eg. Ag
39. Aqueous solution of which ion gives black and Hg.
precipitate in acidic medium when H2S gas is
passed through it :- (c) Thermal decomposition of H2O2
(a) Bi 3+ (b) Pb2+ (d) Thermal decomposition of NaNO2
(c) Cu2+ (d) Ag+ 46. Which of the following statements are correct :-
40. Aqueous solution of which ion gives black (a) SiF4 reacts with F– to give SiF62–
precipitate in basic medium when H2S gas is (b) SiCl4 reacts with Cl– to give SiCl62–
passed through it :- (c) O–O bond in O2F2 is shorter as compared to
(a) Fe2+ (b) Fe3+ O–O bond in H2O2
(c) Co2+ (d) Ni2+ (d) O–F bond in O2F2 is longer as compared to
41. H2O2 can be prepared by :- O–F bond in OF2
(a) Oxidation of 2-ethyl anthraquinol by air 47. X + Coke powder 
 2000 C
 Y + 2CO
(b) Electrolysis of 50% sulphuric acid (3D-silicate)
(c) CO2 gas is passed through aqueous solution Y + Coke powder   2000 C
 Z
of barium peroxide What are the correct statements regarding Z ?
(d) Na2O2 is added to dilute H2SO4 (a) All atoms of Z are sp3 hybridised
42. When ammonia is gas is passed through suspension (b) Z exhibits tetrahedral geometry
of gypsum in water and then passing in
(c) Z is yellow solid at room temperature and
carbondioxide then the products formed can be :-
colourless solid when impurity is present.
(a) H2SO4 (b) (NH4)2CO3
(d) Z is extremely hard substance used as
(c) CaCO3 (d) (NH4)2SO4
abrassive

www.jeebooks.in
THE p-BLOCK ELEMENTS AND THEIR COMPOUNDS 6.19
48. CO gas is absorbed by aqueous suspension of 55. F2 reacts with alkali to give -
cuprous chloride forming the complex like (a) HOF (b) O 3
[CuCl(CO)(H2O)2]. (c) F2 O (d) O 2
Correct statement regarding this complex :- 56. Ca2B6O11 + Na2CO3  
 [X] + CaCO3 +
(a) complex is tetrahedral NaBO2 (Unbalanced equation)
(b) complex is diamagnetic Correct statement fo [X] :-
(c) geometrical isomerism is not possible for this (a) Structure of anion of crystalline (X) has one
complex boron atom sp3 hybridised and other three
(d) Chlorine atoms is separated by an angle 90° boron atoms sp2 hybridised
with respect to both water molecule (b) (X) with NaOH(aq.) gives a compound which
49. Which of the following ammonium salt give on reaction with H2O2 in alkaline medium yields
bronsted acid on heating ? a compound used as brightner in soaps
(a) NH4NO3 (b) (NH4)2SO4 (c) Hydrolysis of (X) with HCl or H2SO4 yields a
(c) (NH4)2MoO4 (d) NH4Cl compound which on reaction with HF gives
fluoroboric acid
50. An element (X) has electronic configuration
1s22s22p3. Hydride of X is :- (d) [X] on heating with cobalt salt in oxidising flame
gives blue coloured bead
(a) Water soluble
57. Which of the following reaction is correctly
(b) Colourless gas
matched with product ?
(c) Odourless gas
(a) C2H5OH + PCl5 C2H5Cl + POCl3 + HCl
(d) Freezing point is 198.4K and 239.7K
(b) CH3CO2H + PCl5 CH3COCl + POCl3 + HCl
respectively
(c) 3CH3CO2H + PCl3 3CH3COCl + H3PO3
51. An element (X) has electronic configuration
1s22s22p2. Hydride of X is :- (d) Sn + PCl5 SnCl2 + PCl3
(a) Colourless gas (b) Pungent odour gas 58. O3 + 2H+ + 2e–  H2O + O2
E° = 1.65 volt
(c) Water insoluble (d) Water soluble
O2 + 4H+ + 4e–  2H2O E° = 0.814 volt
52. An element (a) has outermost electronic
configuration 4s24p5. Hydride of A is :- This value indicates :-
(a) Strong base (b) Strong acid (a) O2 & O3 are reducing reagent
(c) Brown colour gas (d) Colourless gas (b) O2 & O3 are oxidising reagent
53. Gun powder consists of :- (c) O3 is stronger oxidising reagent than O2
(a) NaNO 3 (b) KNO 3 (d) O2 is stronger oxidising reagent than O3
(c) Sulphur (d) Charcoal 59. Correct statements regarding H2O2 :-
54. Whcih of the following statements are correct ? (a) O–O bond length is 1.49Å
(a) When dilute H2SO4 is added with potassium (b) Boiling point is 152°C
ferrocyanide, then CO gas is evolved (c) HOO = 97°
(b) When concentrated H2SO4 is added with (d) Dipole moment is 2.01 Debye
potassium ferrocyanide, then HCN gas is evolved 60. In which of the following reactions , PCl3 is formed
(c) Urea, an nitrogenous organic matter when as one of the product ?
undergoes reaction with water, ammonia gas (a) Reaction between P4 and SOCl2
is evolved. (b) Reaction between P4 and SO2Cl2
(d) For H 2SO4 , first dissociation constant (c) Reaction between Ag and PCl5
(Ka > 10) is very high (d) Reaction between PH3 and Ca(OCl)Cl
1

www.jeebooks.in
6.20 THE p-BLOCK ELEMENTS AND THEIR COMPOUNDS
61. Correct statements regarding O3 :- (c) reaction between boron trifluoride and NaH
(a) It is deep blue in liquid form at 450 K
(b) It is violet black in solid form (d) Oxidation of sodium borohydride with I2
(c) It has fishy smell 69. Which of the following options are correct ?
(d) Decomposition of O3 O2 , entropy increases (a) Concentrated HNO3 can be transported in
62. Which of the following reaction give amorphous aluminium container
boron as product ? (b) Graphite is used as lubricant
(a) B2O3 + 3Mg 3MgO + B (c) Diamond is used as abrassive
(b) 3K + KBF4  4KaF + B (d) Aluminium alloys are used to make aircraft body
(Potassium borofluoride) 70. Which structure exist with central atom sp3d2
hybridised ?
(c) B2O3 + 3H22B + 3H2O
(a) SiF62– (b) SiCl62–
(d) 2BCl3 + 3H2 2B + 6HCl
(c) GeCl62– (d) SnOH62–
63. AlF3 is different from other aluminium halide. This
is illustrated by :- 71. Which of the following options are correct ?
(a) AlF3 is only ionic halide but rest other halides (a) N2(g) + 3H2(g)  2NH3(g), it is an example
are covalent in nature of endothermic reaction -
(b) AlF3 hydrolyses most readily but rest other (b) In CO2 molecule , resonance is possible
halides do not undergo hydrolysis (c) H2O is less polar than H2O2
(c) AlF3 is unaffected water but other halides are (d) Number of chlorine atom present in mustard
easily hydrolysed gas is two
(d) AlF3 has least boiling point among all aluminium 72. Which of the following gases are supporter of
halides combustion ?
64. Br2 when disproportionates in hot and concentrated (a) N 2 O (b) CO2
NaOH solution, the products formed are :- (c) CO (d) O 2
(a) NaBr (b) NaOBr 73. BaSO3 + H2O2  BaSO4 + H2O
(c) NaBrO3 (d) NaClO4 Precipitate P1Precipitate P2
65. Which of the following are example of amphoteric Correct option :-
oxide ?
(a) P1 is white colour precipitate
(a) CO (b) SnO
(b) P2 is white colour precipitate
(c) GeO (d) PbO
(c) P1 is soluble in dilute HCl
66. Which of the following are example of acidic oxide?
(d) P2 is insoluble in dilute HCl and dilute HNO3
(a) SiO2 (b) PbO2
74. Which of the following options are correct
(c) SnO2 (d) GeO2 regarding N2O5 ?
67. Which of the following compound when dissolve (a) The total number of lone pair of electron in
in water, it gives alkaline solution ? N2O5 is 10
(a) Borax (b) Sodium sulphide (b) The total number of lone pair of electron in
(c) K2CO3 (d) HCl N2O5 is 12
68. B2H6 is prepared by (c) The total number of lone pairs of electron on
(a) direct combination between elemental boron nitrogen atom in N2O5 is 2
and gaseous H2 (d) The total number of lone pairs of electron on
(b) reaction between boron trifluoride and LiAlH4 nitrogen atom in N2O5 is 0
in ether medium

www.jeebooks.in
THE p-BLOCK ELEMENTS AND THEIR COMPOUNDS 6.21
75. Which of the following options are correct A,B,C can be :-
regarding member of group 14 elements (a) A,B,C can be NH3, N2O, N2O4
(a) Among all elements , carbon forms the most (b) E can be HNO3
acidic oxide (c) A,B,C can be NH3, NO, NO2
(b) Among all elements , lead is most commonly (d) E can be HNO2
found in +2 oxidation state
80. An inorganic salt ,
(c) Among all elements , silicon and germanium
(a) + KOH B gas + C(solution)
both are semiconductor
B gas + CuSO4 solution Black precipitate
(d) Among all elements , tin has maximum melting
point C (solution) + CuSO4 White precipitate in
brown solution.
76. Which property is not characterised by silicone ?
Correct statements are :-
(a) High thermal stability
(a) A is NH4Br
(b) Chemically resistant towards oxidation
(b) White precipitate is CuBr
(c) Water attracting in nature due to presence of
polar group (c) A is PH4I
(d) Water repelling due to presence of non polar (d) White precipitate is CuI
alkyle group
77. An aqueous solution of gas (x) shows the following
EXERCISE # III
reaction.  Linked Comprehension Type :
(a) it turns rod litmus paper blue Passage for Q.1 to Q.3 :
(b) when added in excess of copper sulphate Lead was known in ancient Egypt and Babylonia. It
solution or deep blue colour is obtained was largely used by Romans for water pipes. Native
(c) on addition of the FeCl3 solution, a brown lead is occasionally found in traces but the cheilt are
precipitate, soluble in dilute HNO3 is obtained in galena PbS.
Correct options ? 1. The sulphate mineral of lead is called :-
(a) x is PH3 (a) Cerussite (b) Crocoisite
(b) x is NH3 (c) Lamarcite (d) Anglesite
(c) Brown precipitate is Fe(OH)3 2. Lead is extracted from galena. Which of the
(d) Blue solution is [Cn(NH 3)6]2+ following process in not involved for lead
extraction.
78. Correct statements regardidng carbide ?
(a) Froath floatation (b) Calcination
(a) WC is example of interstitial carbide
(c) Roasting (d) Smelting
(b) Mg2C3 is used in propyne preparation
Passage for Q.3 to Q.5 :
(c) B4C is used as abrasssive
100 C
(d) Cu 2 C 2 & Ag 2 C 2 are both white colour Orthoboric acid   P1
precipitate Orthophosphoric acid  220 C
 P4

79. A + O2   B(colourless gas) Orthoboric acid 
140 C
 P2
air Orthophosphoric acid  320
 P5
B   C (Brown colour gas)
very high temp.
B + C D Orthoboric acid heating
 P3
D + H2O E Orthophosphoric acid 
high
 P6
temperature heating
E + KI solution Gas B formed.

www.jeebooks.in
6.22 THE p-BLOCK ELEMENTS AND THEIR COMPOUNDS
3. P1 & P4 are respectively :- 9. A is :-
(a) H2B4O7 , HPO3 (b) HBO2 , HPO3 (a) NO (b) CO
(c) H2B4O7 , H4P2O7 (d) HBO2 , H4P2O7 (c) CO2 (d) SO2
4. P2 & P5 are respectively :- 10. B is :-
(a) H2B4O7 , HPO3 (b) HBO2 , HPO3 (a) Ni(CO)4 (b) Fe(CO)5
(c) H2B4O7 , H4P2O7 (d) HBO2 , H4P2O7 (c) Ni2(CO)10 (d) Ni(NO)4
5. P3 & P6 are respectively :- 11. C is :-
(a) B2O5 , P2O5 (b) B2O3 , P4O8 (a) (CO2Na)2 (b) CH3CO2Na
(c) B2O3 , P2O5 (d) B2O3 , P2O3 (c) HCO2Na (d) Na2CO3
Passage for Q.6 to Q.7 : Passage for Q.12 to Q.13 :
A colourless solid A is formed on adding an ethereal A colourless solid (a) dissolves in water. The aqueous
solution of aluminium chloride to lithium hydride solution gives a white precipitate (b) when NaOH or
suspended in ether. A when reacts with BCl3 & SnCl4, NH4OH is added. B dissolves in excess NaOH but
products B and C are formed respectively. not in excess NH4OH.BaCl2 solution is added to A , a
B is electron difficient but C is not. A is used as very white precipitate is formed which is insoluble in dilute
strong reducing agnent and can be used widely in HCl.
organic chemistry. Solid A, when heated with Na2CO3 in a charcoal cavity,
6. A when reacts with CH3CH = O & CH3CO2H , followed by addition of a few drops of cobalt nitrate
the products are formed respectively :- solution in charcoal cavity and further heating (by blow
(a) CH3CH2OH , CH3CH = O pipe flame) leaves a blue residue C.
(b) CH3CH2OH , CH3CH2OH 12. A is :-
(c) CH3CH = O , CH3CHO (a) Al2O3 (b) AlCl3
(d) CH3CH2OH , CH3CH2CH2OH (c) Al(NO3)3 (d) Al2(SO4)3
7. A reacts vigrongly with water to get :- 13. C is :-
(a) LiOH, Al(OH)3 , H2O (a) CoAl2O3 (b) CoAl2O4
(b) LiOH, Al(OH)3 , H2 (c) CoAl2O6 (d) CO2(SO4)3
(c) LiOH, AlH3 Passage for Q.14 to Q.16 :
(d) LiH, Al(OH)3 Finely powdered colemnite is treated with boiling
8. CH3CH2CN + A  P1 sodium carbonate solution when insoluble X1 is
CH3CH2NC + A  P2 precipitated and a mixture of X2 & X3 remains in the
solution. The insoluble X1 is separated by filtration. To
P1 & P2 are related as :- have further yield, CO2 gas is passed through X3 , where
(a) functional isomers X3 is converted into X2.
(b) geometrical isomers 14. X1 is :-
(c) chain isomers (a) CaCO3 (b) NaBO2
(d) positional isomers (c) Na2B4O7 (d) Na2CO3
Passage for Q.9 to Q.11 : 15. X2 is :-
A is colourless flammable gas which reduces CuO (a) CaCO3 (b) NaBO2
and when passed through a warm nickel tube forms a
(c) Na2B4O7 (d) Na2CO3
volatile liquid B. At high temperature and pressure,
(a) reacts with aqueous NaOH to form a salt C. A 16. X3 is :-
reacts in stage with Cl2 to give finally a highly explosive (a) CaCO3 (b) NaBO2
liquid D. (c) Na2B4O7 (d) Na2CO3

www.jeebooks.in
THE p-BLOCK ELEMENTS AND THEIR COMPOUNDS 6.23
Passage for Q.17 to Q.19 : 22. Which of the following statements are correct
A (organic compound) + O2 B+C+D. regarding C :-
A does not react with AgNO3 solution. A mixture of (a) Central atom of anion of compound C has sp3
70% A and 30% ether is used as an anaesthetic. B is hybridisation
neutral oxide which turns anhydrous CuSO4 blue. (b) Anion of compound C is tetrahedral in shape
Compound C turns lime water milky and produces an (c) Both (a) and (b)
acidic solution with water. D is pungent smelling gas :- (d) None of these
17. A and D are respectively :- Passage for Q.23 to Q.25 :
(a) CH2Cl2 , Cl2 (b) CHCl3 , COCl2 White crystalline solid X1 react with H2 to form a highly
associated liquid X2 & monoatomic , colourless gas
(c) CHCl3 , Cl2 (d) CH2Cl2 , COCl2
X3. X2 is used for etching glass.
18. B and C are respectively :- X1 + H2O X2 + X4 (diatomic gas) + X3
(a) N2O and CO (b) N2O and CO2 The ionisation energies of X3 & X4 are almost equal.
(c) H2O and CO (d) H2O and CO2 electrolysis in
X2 
molten state
 X6 (most reactive gas)
1.Cl2  NaOH
19. E 
 HCO2 H  A . E can be:-
2.H
X6 + X3 X1 (2 : 1)
(a) CH3OH 23. Correct statement regarding X4.
(b) CH3CH2OH (a) X4 oxidises acidified FeSO4 to Fe2(SO4)3
(c) CH3CHO (b) X4 oxidises acidified SnCl2 to SnCl4
(d) both (b) and (c) (c) X4 oxidises sulphurous acid to sulphuric acid
Passage for Q.20 to Q.22 : (d) All
A red liquid (a) when heated with concentrated hot 24. The compound X1 when reacts with sulphur, it
NaOH gives a mixture of two salts (b) & (c) in the forms a compound where hybridisation state of
sulphur atom is :-
solution. The mixture when acidification with H2SO4
and distillation produces the red liquid (a) again. C (a) sp3 (b) sp3d
has central atom with oxidation state +5. (c) sp3d2 (d) sp3d3
25. Highly associated liquid X2 when reacts with boric
20. Which is correct regarding liquid A ?
acid it gives :-
(a) it is sparingly soluble in water but readily (a) BF3 (b) BCl3
soluble in ether ; alcohol
(c) BBr3 (d) HBF4
(b) it acts as an oxidising reagent Passage for Q.26 to Q.28 :
(c) it is used to identify unsaturation present in a A yellow solid (a) in unaffected by acids and base at
compound room temperature. It is not soluble in water. It dissolves
(d) all slowly in hot concentrated HNO3 to give a brown gas
(b). The solid (a) dissolves slowly in boiling solution of
21. Which of the following statement is correct for
Na2SO3 giving a clear solution (c). Acidification of
salt B ?
solution (c) causes a colourless gas (d). Upon
(a) it solution in water gives yellow precipitate with concentration and cooling, solution (c) deposits as
silver nitrate solution colourless crystals. Pentahydrate solution (c) is a
(b) it solution in water gives yellow precipitate with reducing agents and reacts quantitatively with iodine
lead nitrate solution solution (I2 in KI) and acts as antichlor during bleaching:-
26. Solid A is :-
(c) both (a) and (b)
(a) P4 (b) S8
(d) it is basic compound
(c) Pb3O4 (d) PbO

www.jeebooks.in
6.24 THE p-BLOCK ELEMENTS AND THEIR COMPOUNDS
27. Brown gas (b) & colourless gas (d) are 34. Which of the following options are correct
respectively:- regarding X ?
(a) NH3, PH3 (b) NO2, PO3 (a) X oxidises black PbS into white colour of
(c) NO, PbO2 (d) NO2, SO2 PbSO4
28. Solution C in the form of pentahydrate is :- (b) X oxidises stannous chloride into stannic
chloride
(a) Na2PbO2 . 5H2O (b) Na3PO4 . 5H2O
(c) X is absorbed in turpentine oil
(c) Na2S2O3 . 5H2O (d) NaNH4HPO . 5H2O
(d) All
Passage for Q.29 to Q.31 :
Passage for Q.35 to Q.37 :
A compound of S, O, Cl (vapour density = 67.5) X
reacts with KOH to give substance Y and Z. Y gives a Compound (Z) on reduction with LiAlH4 gives a
white precipitate with AgNO3 solution which is soluble hydride (X) containing 21.72% hydrogen. The one
in NH4OH but insoluble in HNO3. Z give a white mole of hydride (X) reacts with 2 moles of ammonia at
precipitate with BaCl2 which is insoluble in HCl or high temperature gives a compound (Y). (Z) hydrolyses
HNO3. X is amporphous with a compound which is incompletely and forms a compound (W) and H3BO3.
prepared by heating a mixtur of NaCl, K2Cr2O7 and 35. Correct statement regarding X :-
conc. H2SO4 (a) the central atom of X has trigonal planar
29. X is :- geometry
(a) SOCl2 (b) SO2Cl2 (b) all H-atoms in X are in the same plane
(c) S2Cl2 (d) SO3Cl (c) a three centre two-electron bond is observed
in X
30. The product obtained by heating a mixture of NaCl,
K2Cr2O7 and concentrated H2SO4 is (d) all M–H bond lengths are identical in X
(a) CrCl3 (b) CrO2Cl2 36. The hybridisation in central atom of compound W
is
(c) CrOCl2 (d) CrO3Cl
(a) sp2 (b) sp
31. Y & Z are respectively :- 3
(c) sp (d) sp3d
(a) KCl, K2SO3 (b) KCl, K2SO4
37. Correct statement regarding Z :-
(c) ClO, KO2 (d) KCl, K2S
(a) it has trigonal planar geometry
Passage for Q.32 to Q.34 :
(b) the bond lengths between the central atom and
Gas (X) is colourless diamagnetic with fishy smell.
the substituent atoms is shorter than the sum
When X is cooled at –112°C, it condenses to a deep
of the covalent radii.
blue liquid which is dangerously explosive. X is one of
(c) the coordination geometry around central atom
the most powerful oxidising reagent. It is second to F2
in compound Z and in 1 : 1 complex of Z an
in oxidising power.
NH3 is same
32. X is :-
(d) all of these
(a) H2O2 (b) PH3
Passage for Q.38 to Q.40 :
(c) O 3 (d) Cl2
A is a gas with vapour density 8.5. On catalytic
33. When mercury is shaken with X, the metal adheres oxidation in presence of Pt/Rh at high prssure and
to the glass as a mirror. It is because of formation :- temperature it gives a colourless gas (b) which rapidly
(a) HgO (b) Hg2O turns brown in air forming gas C. B and C both are
(c) Hg3P2 (d) HgCl2 paramagnetic in nature :-

www.jeebooks.in
THE p-BLOCK ELEMENTS AND THEIR COMPOUNDS 6.25
38. A, B, C are respectively :- 44. A & B are :-
(a) NH3 , NO, NO2 (b) NH3 , N2O4, N2O3 (a) NH4NO2 , N2 (b) NH4NO3 , N2O
(c) N2H4 , NO, NO2 (d) NH3 , NO2, N2O (c) NH4NO2 ; NO2 (d) NaNO3 , NO2
39. C is dissolved in water to give X which is 45. C and E are respectively :-
concentrated by distillation upto 68% by mass. X is :-
(a) NH3 , HNO2 (b) NH3 , HNO3
(a) HNO2 and the above process to form X is
(c) NH3 , HN3 (d) NH3 , H2N2O2
called ostwald’s process
46. The anionic part of D is :-
(b) NH3 and the above process to form X is called
Haber’s process (a) Linear (b) trigonal planar
(c) HNO2 and the above process to form X is (c) bent (d) tetrahedral
called Birkeland - Eyde process Passage for Q.47 to Q.49 :
(d) HNO3 and the above process to form X is A colourless solid X1 melts at 44°C. When heated with
called Birkeland - Eyde process aqueous KOH/NaOH, it gives off gaseous product
40. Gas C on cooling (< 0°) give :- X2, which is spontaneously inflammable in air at about
(a) N2O3(liquid) (b) N2O4(liquid) 150°C. If is powerful reducing agent.
(c) N2O4(vap.) (d) N2O4(solid) 47. X2 when reacts with CuSO4 then:-
Passage for Q.41 to Q.43 : (a) Red deposit of Cu is formed
An unstable pale blue solution (P) which rapidly (b) Black precipitate of Cu3P2 is formed
decomposes even in cold. The solution oxidises KI in (c) Blue precipitate of Cu3P2 is formed
acidic medium to liberate I2. The solution also oxidises
(d) S gets precipitated
SnCl2 in dilute HCl. P is oxidised by bromine water to
form monobasic acid Q. 48. X2 when reacts with AgNO3 then :-
41. P is :- (a) Brown precipitate of Ag2O is formed
(a) H2CO3 (b) HOCl (b) Yellow soluton of Ag3P is formed
(c) HNO2 (d) HNO3 (c) Black precipitate of metallic Ag is formed
42. P when reacts with hydrazoic acid, it gives :- (d) Black precipitate of Ag2S is formed
(a) a mixture of N2 and NO 49. X1 and X2 are respectively :-
(b) a mixture of N2 and N2O (a) S8 and H2S (b) White P4 and P4O10
(c) a mixture of N2O and NO2 (c) White P4 and PH3 (d) S8 and SO2
(d) a mixture of NO and NO2 Passage for Q.50 to Q.53 :
43. P reacts with which of the following compound to 373K
HCOOH 
conc.H 2SO 4 H2O + (X)
release N2 gas :-
4231273K
(a) Urea (b) CH3CH2NH2 C(s) + H2O   (X) + H2(g)
(c) Sulphamic acid (d) All 50. Select the correct statement about (X) :-
Passage for Q.44 to Q.46 : (a) (X) is a colourless, odourless and almost water
A colourless solid A when heated gives off a colourless insoluble gas
gas B which is supporter of combustion. If heated in (b) (X) is highly poisonous and burns with blue
container the whole of the solid disappears. When A is flame
heated with NaOH, an alkaline gas C is evolvd. When
gas B is heated with sodamide, a colourless solid D is (c) When (X) gas is passed through PdCl2 solution
formed which furthur treated with dilute H2SO4 , a giving rise to black ppt.
colourless liquid E is formed lead and silver salt of E (d) All of these
are explosive.

www.jeebooks.in
6.26 THE p-BLOCK ELEMENTS AND THEIR COMPOUNDS
51. Mixture of (X) gas + H2 is called :- 56. X1 is :-
(a) Water gas or synthesis gas (a) XeF2
(b) Producer gas (b) XeF4
(c) Methane gas (c) XeF6
(d) None of these (d) None
52. In second reaction when air is used instead of 57. X2 is :-
steam a mixed of (X) gas and N2 is produced
which called :- (a) XeO3
(a) Water gas (b) XeO4
(b) Synthesis gas (c) XeO6
(c) Producer gas (d) XeO2
(d) Carbond dioxide gas 58. X3 and X4 are respectively :-
53. Select the correct statement about (X) :- (a) HX eO4– (perxenate ion) , XeO64– (xenate ion)
(a) (X) gas is estimatated by I2O5 (b) HXeO4– (xenate ion) , XeO64– (perxenate ion)
(b) Cu2Cl2 is absorber of (X) gas (c) XeO64– (perxenate ion) , HXeO4– (xenate ion)
(c) (X) gas is the purifying agent for Ni (d) XeO64– (xenate ion) , HXeO4– (perxenate ion)
(d) All of these Passage for Q.59 to Q.60 :
Passage for Q.54 to Q.55 : Consider the following reaction(s)
CH3CH2CN + H2O P + NH3 
A(s) B(s) + C(g) + D(g) +E(v)
CH3CH2NC + H2O CH3CH2NH2 + Q (pale green) (brown)
54. P is :-
Anhydrous CuSO4
(a) CH3CH2CO2H KMnO 4/H
+ F
MnO4– is decolorised (Blue solid)
(b) CH3CH2CH2OH (a)aq. 
NaNO 2  dil.HCl
 unstable brown compound (G)
(c) CH3CH2CH3
µ  15B.M.
(d) CH3CH2CH2NH2
59. Sum of oxidation state of the central atom in C(g)
55. Q is :-
and D(g) is :-
(a) CO2
(a) 6
(b) NH3
(b) 10
(c) CH4
(c) 8
(d) HCO2H
Passage for Q.56 to Q.58 : (d) 4
A Xenon fluoride X1 is formed by heating Xenon with 60. Select incorrect statement about compound F :-
excess of fluorine at very high temperature and pressure. (a) It adsorb orange-red light for d-d transition
X1 has a melting point 47.4°. X1 can acts as fluoride (b) 4 water molecule are directly bonded with
ion donor as well as fluoride ion acceptor. X 1 each Cu+2
undergoes hydrolysis vigorously forming an explosive
(c) It is paramagnetic
compound X2.
X2 + OH– (pH > 10.5) X3 (d) It’s µ = 8B.M.
X3 + OH– X4 + Xe + O2 + H2O

www.jeebooks.in
THE p-BLOCK ELEMENTS AND THEIR COMPOUNDS 6.27
Match the Column Type :
61. Match the column :-
Column-I Column-II
Reactions Products / Use of products
(a) Dry ice + CH3MgX, H+ (P) Urea
(b) CO2 + Gypsum solution + NH3 (Q) Monocarboxylic acid
(c) CO2 + NH3  high P &T
 (R) Ammonium sulphate
OH

(d) + CO2 + NaOH 


high P &T
 (S) Product used as fertiliser

62. Match the column :


Column-I Column-II
(a) I.E. order (P) He > Ne > Ar > Kr > Xe
(b) Boiling point order (Q) He < Ne < Ar < Kr < Xe
(c) Oxidising property order (R) F2 > Cl2 > Br2 > I2
(d) Acid dissociation constant (S) O > S > Se > Te
(T) H2Te > H2Se > H2S > H2O
63. Match the column :
Column-I Column-II
Different types of alkyl chlorosilkane Products on hydrolysis
Me Me Me

(a) Me2SiCl2 (P) O Si O Si O Si O –


Me Me Me
Me Me Me Me
(b) Me3SiCl (Q) Me Si O Si O Si O Si Me

Me Me Me Me
Me Me

O Si O Si O

(c) Me2SiCl2 + Me3SiCl (R) O O


O Si O Si O
Me Me
(d) MeSiCl3 (S) Me3Si – O – SiMe3
64. Match the column :
Column-I Column-II
(a) XeO3 (P) How lone pair of electron
(b) XeF2 (Q) Acts as oxidising reagent
(c) XeF4 (R) Undergoes hydrolysis with H2O
(d) XeF6 (S) Undergoes addition reaction
(T) Give disproportionation reaction with H2O or OH

www.jeebooks.in
6.28 THE p-BLOCK ELEMENTS AND THEIR COMPOUNDS
65. Match the column :
Column-I Column-II
Compound Features of product on treated with NaOH
(a) XeF2 (P) O2 is formed as product
(b) (NH4)2SO4 (Q) Disproportionation reaction
(c) P 4 (R) Product formed is inflammable and used in Holme’s signal
(d) NO 2 (S) Product solution reacts with Zn dust and NH3 gas is liberated
(T) Atlest one product is pyramidal in shape
66. Match the column :
Column-I Column-II
Acids Features
(a) Hypophosphorus acid (P) All hydrogens are ionisable in water
(b) Orthoboric acid (Q) Monobasic acid
(c) Hypophosphoric acid (H4P2O6) (R) sp3 hybridised atoms present
(d) Sulphurous acid (H2SO3) (S) Central atom(s) contain are lone pair of electrons
67. Match the column :
Column-I Column-II
Compounds Features
(a) IF5 (P) show self ionisation
(b) ClF3 (Q) used as potential ionising solvent
(c) ICl (R) slightly conducting
(d) I2 (S) hybridisation of halogen atom is sp3d in ionised state
(T) hybridisation of halogen atom is sp3d2 in ionised state
68. Match the column :
Column-I Column-II
Reaction Nature of reaction/Product
(a) SiCl 4 + H2O  (P) hydrogen halide formation
h
(b) Cl2 + H2O   (Q) O2 gas formation
(c) XeF4 + H2O  (R) redox reaction
(d) H2O2 + ClO2 + OH– (S) nonredox reaction
(T) One of the product has tetrahedral shape
69. Match the column :-
Column-I Column-II
Reaction Product
(a) Cl2O6 + H2O  (P) ClO2
(b) NaClO4 + HCl(conc.)  (Q) HClO3
(c) KClO3 + (CO2H)2  (R) Cl2O
(d) H2O + Cl2 
573K
 (S) HClO4

www.jeebooks.in
THE p-BLOCK ELEMENTS AND THEIR COMPOUNDS 6.29
70. Match the column :-
Column-I Column-II
Compound Shape
(a) ClF3 (P) square pyramidal
(b) ICl3 (Q) orange solid
(c) IF5 (R) T shape
(d) IF7 (S) Colourless gas
(T) Colourless liquid
71. Match the column :-
Column-I Column-II
Reaction Feature
(a) XeF2 + H2O  (P) redox reaction
(b) XeF4 + H2O  (Q) disproportionation reaction
(c) XeF6 + H2O  (R) product formed is used for etching glass
(d) XeF3 + NaOH  (S) Xe formation
72. Match the column :-
Column-I Column-II
(a) XeF6 + SiO2  (P) one of the product has square pyramidal shape
(b) P4(white) + SO2Cl2  (Q) one of the product has sp3 hybridisation
(c) H2SO4 + P4O10  (R) one of the product has p-d overlapping
(d) XeF6 + H2O  (S) one of the product has zero dipole moment
73. Match the column :-
Column-I Column-II
Reactions Products formed
(a) Sn + conc. HNO3  (P) NO
(b) Sn + dil. HNO3  (Q) NO 2
(c) Ag + conc. HNO3  (R) NH4NO3
(d) Ag + dil. HNO3  (S) Sn(NO3)2
(T) AgNO3
74. Match the column :-
Column-I Column-II
Compounds Features
(a) XeF2 (P) Can act as Lewis acid
(b) XeF4 (Q) Can act as Lewis base
(c) XeF6 (R) On reaction with H2O/OH–,disproportionation can take place
(d) XeO3 (S) Has at least one lone pair of electron
(T) At least one lone pair of electron on central atom

www.jeebooks.in
6.30 THE p-BLOCK ELEMENTS AND THEIR COMPOUNDS
75. Match the column :-
Column-I Column-II
Reactions Nature of reaction/product
(a) (NH4)2S2O8 + H2O 
distillation
 (P) Hydrolysis

(b) NaBO2 + H2O + H2O2  OH
 (Q) One of the products has peroxide linkage
fusion
(c) Ag2S2O3 + H2O  temperature
 (R) Disproportionation
(d) 2-Ethyl anthraquinol+air  (S) In one of the products the central atom has sp3 hybridisation
(T) One of the products is used as brightner in washing powder
76. Match the column :-
Column-I Column-II
Compounds Features
(a) H2O2 (P) Non-planar structure
(b) O 3 (Q) Planar structure
(c) H2S (R) Only reducing power
(d) SO2 (S) Only oxidising power
(T) Has oxidising as well as reducing power
77. Match the column :-
Column-I Column-II
Compounds Features
(a) Ca(OCl)Cl (P) Cl has negative oxidation state
(b) Cl2 (Q) Cl has positive oxidation state
(c) NaCl (R) oxidising reagent
(d) Cl2O7 (S) dehydrating reagent
(T) one of the product when Cl2 gas is passed through basic solution
78. Match the column :-
Column-I Column-II
Reaction Features
(a) N2O4 

 (P) One of the product is a mixed anhydride

H
(b) ClO2 + O3   (Q) One of the product is an acidic oxide
(c) K4[Fe(CN)6] + H2SO4 (conc.) + H2O 
 (R) The oxidation state of the central atom of one of
the product is +6
(d) KOH + O3   (S) One of the product is colourless paramagnetic gas
79. Match the column :-
Column-I Column-II
Gas Absorber
(a) CO (P) Turpentine oil
(b) O 2 (Q) Alkaline pyrogallol
(c) O 3 (R) Cu2Cl2+NaOH
(d) CO2 (S) KOH

www.jeebooks.in
THE p-BLOCK ELEMENTS AND THEIR COMPOUNDS 6.31
80. Match the column :-
Column-I Column-II
Allotropes Features
(a) Rhombic sulphur (P) Consisting S8 unit (8 member ring)
(b) Monoclinic sulphur (Q) Most stable allotrope of phosphorous
(c) Black phosphorous (R) Unstable and ordinary temperature
(d) White phosphorous (S) Most stable allotrope of sulphur
(T) Most reactive allotrope of phosphorous
81. Match the column :-
Column-I Column-II
Compounds Products on heating
(a) LiNO3 (P) NH3 gas is evolved
(b) HNO2 (Q) Oxyacid is formed as one of the product
(c) NaNH4HPO4.6H2O (R) Disproportionation reaction takes place
(d) H3PO3 (S) O2 gas is evolved
(T) Oxides of nitrogen is evolved
82. Match the column :-
Column-I Column-II
Ammonium salt Products on heating
(a) NH4IO3 (P) N2 gas
(b) NH4ClO4 (Q) O2 gas
(c) NH4NO2 (R) N2O gas
(d) NH4NO3 (S) Cl2 gas
(T) I2 gas
83. Match the column :-
Column-I Column-II
Compound Feature/Use
(a) NH2CONH2 (P) Chlorinating reagent
(b) NH2NH2 (Q) Dehydrating reagent
(c) SOCl2 (R) Rocket fuel
(d) H2SO4 (S) Nitrogeneous fertiliser
84. Match the column :-
Column-I Column-II
Reaction with ammonia Observation
(a) Cu2+ + NH3(excess)  (P) Deep blue colour solution
(b) AgCl + NH3(excess)  (Q) Brown precipitate
(c) ZnSO4 + NH3(less amount)  (R) Colourless solution
(d) FeCl3 + NH3(less amount)  (S) White precipitate
(T) Complex formation

www.jeebooks.in
6.32 THE p-BLOCK ELEMENTS AND THEIR COMPOUNDS
85. Match the column :-
Column-I Column-II
Group Number Possible oxidation state
(a) Group 13 (P) +3
(b) Group 14 (Q) –3
(c) Group 15 (R) +1
(d) Group 17 (S) –1
(T) +4
86. Match the column :-
S2O32– + Different metal ion Complex
Column-I Column-II
Formula of the complex Metal ion
(a) [M(S2O3)3]3– (P) Cu2+
(b) [M(S2O3)2]2– (Q) Bi3+
(c) [M6(S2O3)5]4– (R) Pb2+
(d) [M(S2O3)2]3– (S) Hg2+
(T) Ag+1
87. Match the column :-
Column-I Column-II
Compound Features
(a) N2O 3 (P) Colourless gas
(b) N2O 4 (Q) Colourless solid / liquid
(c) N2O 5 (R) Blue solid
(d) N 2 O (S) Acidic oxide
(T) Neutral oxide
88. Match the column :-
Column-I Column-II
Compound Features
(a) P4O6 (P) p-d back bonding
(b) P4O10 (Q) p-p back bonding
(c) N2O 4 (R) M–O bond has partial double bond character
(d) N2O (S) Even number of lone pair of electron
(T) Laughing gas
89. Match the column :-
Column-I Column-II
Order Property
(a) SbH3 > NH3 > AsH3 > PH3 (P) Thermal stability
(b) NH3 > PH3 > AsH3 > SbH3 (Q) HEH angle
(c) NH3 > SbH3 > AsH3 > PH3 (R) Boiling point
(d) SbH3 > AsH3 > PH3 > NH3 (S) Melting point
(T) Reducing power

www.jeebooks.in
THE p-BLOCK ELEMENTS AND THEIR COMPOUNDS 6.33
90. Match the column :-
Column-I Column-II
Compound Bond parameters
(a) N2O 5 (P) N–N length is shortest
(b) N2O 4 (Q) N–N length is highest
(c) N2O 3 (R) N–O–N bond length is present and N–O bond length is highest
(d) N 2 O (S) All N–O bond lengths are identical
(T) All N–O bond lengths are not identical

EXERCISE # IV

 Integer Type : 5. How many statement are correct ?


1. How many statements are correct regarding boron (a) H3PO3 is diprotic acid
nitride :- (b) NF3 is stable but NCl3 and NI3 are explosive
(a) It has a graphite like layer structure with (c) PCl5 is covalent in solid stable
hexagonal ring. (d) P–O bond length in POCl3 is less than sum of
(b) B–N bond distance is 1.45Å and comparable the sigle bond covalent radii of P and O atom.
to C–C bond distance in graphite which (e) Metaphosphorus acid (HPO2) is not known.
1.42Å. (f) White phosphorous is more reactive than red
(c) Both B and N in boron nitride are sp 2 phosphorous.
hybridised. 6. What is the value of n in alkyl substituted
(d) It is soft and lubricantes chlorisilane RnSiCl4–n , which is used for the
preparation of cross linked silicone :-
(e) It is a white infusible refractory solid.
7. Number of p-dbonding present in XeO4 .
(f) It is formed by heating boron amide B(NH2)3.
8. Find out the number of hydrogen atoms attached
During this at first boron amide B(NH2)3 to phosphorus atom in hypophosphorus acid is :-
decomposes into boron imide B2(NH)3 and
9. XeF4 + H2O  P(reduced product) + Q (oxidised
then converted into boron nitride. product) + HF + O2
2. In B3O63– find out number of B–O–B linkage. What in the ratio of oxidation states of Xe in
3. A comparative study of following properties reduced and the oxidised products.
between benzene and borazine are taken. For how 10. Find out the number of following oxy acid of
many properties numerical data of benzene is more phosphorus , where P–O–P bond is present :-
than borazene . H3PO4 , H3PO5 , H2PO3, H3P3O9, H4P4O12
Melting point ; Boiling point ; Dipole moment ; 11. No. of S–S linkage in S3O9 = m1
Bond polarity ; Resonance energy ; Molecular No. of P–P linkage in (HPO3)3 = m2
weight No. of S–S linkage in S2O82– = m3
4. How many molecules result symmetrical cleavage No. of –O–O– linkage in S2O62– = m4
of B2H6 molecule ? Find out m1 + m2 + m3 + m4 :-
CO ; N(CH3)3 ; 5(CH3)2 ; O(CH3)2 ; ; 12. How many reagents can be used to dry NH3 gas:-
O Concentrated H2SO4 ; P4O10 ; CaO ; Anhydrous
CH3NH2 ; (CH3)2NH CaCl2

www.jeebooks.in
6.34 THE p-BLOCK ELEMENTS AND THEIR COMPOUNDS
13. How many statements are correct :- (c) K2Cr2O7 + HCl 
(a) Argon is used in arc welding of metals or alloys (d) KMnO4 + concentrated HCl 
to provide an inert atmosphere e Pb3O4 + HCl 
(b) XeF2 , XeF4 , XeF6 arc colourless crystalline (f) NaCl + conc. H3PO4 
solids and sublime readily at 298K. 19. How many statements are correct ?
(c) XeF2 , XeF4 , XeF6 all can undergo hydrolysis. (a) XeF5 is non existent
(d) Se2Cl2 undergoes disproportionation to give (b) For drying H2S, concentrated H2SO4 can be
SeCl4 and Se. used up.
(e) Dioxide like MnO2 , PbO2 do not form H2O2 (c) The inert boble gas which is not found in
with dilute acids but they evolve O2 with atmosphere in reaction.
concentrated H2SO4
(d) Metaphosphoric acid exists in polymeric cyclic
(f) Xenon fluoride reacts with fluoride ion acceptor structure.
to form cationic species of Xe and fluoride ion
(e) SO2 bleaches the colour of article by reduction.
donors to form fluoro anion of Xe.
(f) Cl2 bleaches the colour of article by oxidation.
14. How many statement are incorrect :-
(g) H3PO3 and H3PO4 both are reducing in nature.
(a) Barium azide on heating gives pure nitrogen
gas (h) The first real compound of noble gas was made
by Neil Bartlett.
(b) NH3 + NaOCl (Sodium hypochlorite) N2
(gas) (i) When deep red PtF6 vapour was mixed with
an equal volume of Xe, the gases combined
(c) Oxides of boron and silicon which are B2O3
immediately at room temperature to produce
and SiO2 respectively, are basic in nature
a yellow solid.
(d) Oxides of aluminium (Al2O3) and gallium
20. How many products can be common product
(Ga2O3) are amphoteric in nature
when XeF2 reacts with H2O and XeF4 reacts with
(e) Oxides of indium (In2O3) and thallium (Tl2O3) H2O :-
are acidic in nature
21. Number of p-dbonding present in SO3 .
15. Find out the oxy acids of phosphorus in which
22. Total number of lone pairs of electrons present in
phosphorus has lower oxidation state less than +5
HClO3.
contain either P–P or P–H bonds but not both in
addition to P=O and P–OH bonds :- 23. Number of p-pbonding present in SO2.
H3PO2 ; H3PO3 ; H4I2O5 ; H4P2O6 ; H4P2O7 ; 24. How many statements are correct regarding helium:-
(HPO3)3 (a) It is chemically inert & has very high thermal
16. How many total number of p-block elements conductivity
produce H2 gas with sodium hydroxide :- (b) It has extremely low boiling point & very low
Al, Sn, Pb, P, Zn viscocity
17. Among the following the total number of elements (c) It shows abnormal behaviour on liquification
which undergo disproportionation reaction in (d) It is less soluble in blood than nitrogen under
NaOH :- high pressure
S8, P4, Cl2 , Br2 , N2 (e) It can not form clathrate compound with quinol
18. How many pair of compounds gives Cl2 gas:- (f) It is a inflammable heavy gas therefore it is used
(a) KCl + MnO2  in filling balloons
(b) NaCl + conc.H2SO4  (g) It is used to produce and sustain powerful super
conducting magnets

www.jeebooks.in
THE p-BLOCK ELEMENTS AND THEIR COMPOUNDS 6.35
(h) It is used as a cryogenic agent for carrying out (iii) NO3– +Al + NaOH 
experiments at low temperature (iv) FeSO4 + H2SO4(conc.) + HNO3 
(i) Mixture of helium and oxygen are used as an (v) Au + HCl + HNO3 
aritificial air for divers and other under pressure. 35. How many reactions give N2 gas on heating :-
(j) Because of its light weight and noninflammable (a) (NH4)2Cr2O7 


character it is used in filling balbons.
(b) Ba(N3)2 


25. Amongst the following, the total number of dimeric
halides of group 16th elements which undergoes (c) NaNO2 + NH4Cl  

disproportionation is :- (d) NH4NO3  

S2F2 , S2Cl2 , S2Br2 , Se2Cl2 , Se2Br2 e vapours + Red hot copper 
36. No. of halide ions which change their oxidation
26.
number on hating with MnO2 + conc.H2SO4 ?
27. The number of ionisable hydrogen prsent in
F– ; Cl– ; Br– ; I–
NaH2PO2 :-
37. 1 mole each of H3PO2 , H3PO3 and H3PO4 will
28. In which of the following all bond lengths are
neutrilise X mole of KOH, Y mole of Mg(OH)2
identical :-
and Z mole of Cr(OH)3 (assuming all as strong
CO32–, PCl5, SO52–, NO2, NO3–, BF3, O3, SO2,
electrolytes) respectivley. Find out the value of
HCO3–
x+y+z ?
29. Atomicity of sulphur in rhombic sulphur is X and
38. How many species does not contain peroxide ion?
atomicity of phosphorus in white phosphorus is Y
then find out X – Y :- PbO2 ; BaO2 ; SrO2 ; Na2O2 ; KO2 ; K2O2
30. The oxidation state of S in S8 , S2F2 and H2S are 39. How many reactions involving ozone give O2 in
x, y, z respectively. Find out x + 2y + z. product side ?
31. Alkyl substituted chlorosilane, RnSiCl(4–n) , on (i) PbS(s) + O3 
hydrolysis and then condensation polymerisation (ii) I–(aq.) + O3 
yields silicones. What is the value of n in the alkyl (iii) NO + O3 
substituted chlorosilane used for the preparation (iv) SnCl2 + HCl + O3 
of dimer silicone.
40. How many reagents reacts with H2O2 but not with
32. How many reactions give SO2 gas as one of the ozone ?
product ?
K4[Fe(CN)6] ; K3[Fe(CN)6] + KOH ; PbS ;
(a) S + concentrated H2SO4  SnCl2 + HCl ; KI + H+ ; KMnO4 + H+
(b) Zn + dilute H2SO4  41. How many sulphides are yellow in colour ?
(c) Cu + concentrated H2SO4 
CdS ; Sb2S3 ; As2S3 ; SnS ; SnS2
(d) ZnS + O2 
42. In crystal of polymeric boric acid , ring is formed
(e) Cerussite undergo heating through hydrogen bonding. What is the member
(f) Carbon + conc. H2SO4  of the ring.
33. 1 volume of concentrated HNO3 and x volume of 43. In three dimensional structure of SiO2 , rings are
concentrated HCl mixture is called aqua regia. The formed with alternate silicon and oxygen atoms.
value of x :- What is the member of the ring ?
34. How many reactions give nitric oxide gas one of 44. Buckminster Fullerene (C60) has x1 vertices, x2 six
the gaseous product ? membered ring and x3 five membered ring. Find
(i) KI + HNO3  x1 x1
out  .
(ii) H2S + HNO3  x3 x 2

www.jeebooks.in
6.36 THE p-BLOCK ELEMENTS AND THEIR COMPOUNDS
45. In photosynthesis , green plants convert (c) Cl2 reacts with excess of NH3 to give N2 and
atmospheric CO2 into carbohydrate, such as HCl
glucose. The overall chemical change can be (d) Br2 reacts with hot and strong NaOH solution
expressed as to given NaBr , NaBrO4 and H2O
h
xCO 2  yH 2O 
chlorophyll
 C6 H12O6  zO 2  wH 2O 4. In XeF2 , XeF4 , XeF6 the number of lone pairs on
Find out x+z+w–y. Xe is respectively -[AIEEE - 2002]
46. Xe can exhibit x1 , x2 , x3 , x4 oxidation state in its (a) 2,3,1 (b) 1,2,3
compounds. (c) 4,1,2 (d) 3,2,1
x1 < x2 < x3 < x4 5. Aluminium is industrially prepared by -
Find out x4 – x3 if x4 – x3 = x3 – x2 = x2 – x1 [AIEEE - 2002]
47. A gas G occurs in Earth’s stratosphere. X is (a) Fused cryolite (b) Bauxite ore
diamagnetic. It has bond angle 116.5° and bond (c) Alunite (d) Borax
length 0.128 nm. Dry KOH reacts with G to yeild
6. For making good quality mirrors , plates of float
reddish brown precipitate. What is the molecular
glass are used. These are obtained by floating
weight of G.
molten glass over a liquid metal which does not
48. Total numbers of lone pair is present in H2P2O7. solidify before glass. The metal used can be -
49. Total numbers of P–OH bond present in H4P2O6.
[AIEEE - 2002]
50. Total numbers of P–OH bond present in (HPO3)3
(a) Sodium (b) Magnesium
if x, total numbers of P=O bond present in
(HPO3)3 if y, total numbers of P–O–P bond present (c) Mercury (d) Tin
in (HPO3)3 if z then find out x+y+z ? 7. What may be expected when phosphine gas is
mixed with chlorine gas - [AIEEE - 2003]
EXERCISE # V(A) JEE-MAIN (a) PCl5 and HCl are formed and mixture cools
down
1. In case of nitrogen NCl3 is possible but not NCl5
while in case of phosphorus, PCl3 as well as PCl5 (b) PH3 . Cl2 is formed with warming up
are possible. This is due to - [AIEEE - 2002] (c) The mixture only cools down
(a) A vailability of vacant d-orbitals in P but not in N (d) PCl5 and HCl are formed and mixture warms
(b) Lower electronegativity of P than N up
(c) Lower tendency of H-bond formation in P than N 8. Graphite is a soft solid lubricant extremely difficult
(d) Occurence of P in solid while N in gaseous to melt. The reason for this anomolous behaviour
state at room temperature is that graphite - [AIEEE - 2003]
2. Which products are expected from the (a) has molecules of variable molecular masses like
disproportionation of hypochlorous acid polymers
-[AIEEE - 2002] (b) has carbon atoms arranged in large plated of
(a) HClO3 and Cl2O (b) HClO2 and HClO rings of strongly bonded carbon atoms with
weak interplate bonds
(c) HCl and Cl2O (d) HCl and HClO3
(c) is a non crystalline substance
3. Identify the incorrect statement among the
following - [AIEEE - 2002] (d) is an allotropic from of diamond
(a) ozone reacts with SO2 to give SO3 9. Concentrated hydrochloric acid when kept in open
air sometimes produces a cloud of white fumes.
(b) silicon reacts with NaOH(aq.) in the presence
This is due to - [AIEEE - 2003]
of air to give Na2SiO3 and H2O

www.jeebooks.in
THE p-BLOCK ELEMENTS AND THEIR COMPOUNDS 6.37
(a) strong affinity of HCl gas for moisture in air 14. The correct order of the thermal stability of
results in forming of droplets of liquid solution hydrogen halide (H-X) is - [AIEEE - 2005]
which appears like a cloudy smoke
(a) HI > HBr > HCl > HF
(b) due to strong affinity for water , conc. HCl
(b) HF > HCl > HBr > HI
pulls moisture of air towards self. The moisture
forms droplets of water and hence the cloud (c) HCl > HF > HBr > HI
(c) conc. HCl emits strongly smelling HCl gas all (d) HI > HCl > HF > HBr
the time 15. The structur of diborane contains -[AIEEE - 2005]
(d) oxygen in air reacts with emitted HCl gas to (a) Four 2c-2e bonds and four 3c-2e bonds
form a cloud of Cl2 gas
(b) Two 2c-2e bonds and two 3c-2e bonds
10. Aluminium chloride exists as dimer, Al2Cl6 in solid
state as well as in solution of non-polar solvents (c) Two 2c-2e bonds and four 3c-2e bonds
such as benzene. When dissolved in water , it gives (d) Four 2c-2e bonds and two 3c-2e bonds
-[AIEEE - 2004] 16. Heating an aqueous solution of aluminium chloride
3+
(a) Al + 3Cl – to dryness will give - [AIEEE - 2005]
(b) [Al(H2O)6]3+ + 3Cl– (a) AlCl3 (b) Al2Cl6
(c) [Al(OH)6]3– + 3HCl– (c) Al2O3 (d) Al(OH)Cl2
(d) Al2O3 + 6HCl 17. Which one of the following is the correct statement
11. The soldiers of Napolean army while at Alps during -[AIEEE - 2005]
freezing winter suffered a serious problem as
(a) Boric acid is a protonic acid
regards to the tin buttons of their uniforms. White
metallic tin buttons get converted to grey powder. (b) Beryllium exhibits coordination number of six
This transformation is related to -[AIEEE - 2002] (c) Chlorides of both beryllium and aluminium
(a) An interaction with water vapour contained in have bridged chloride structures in solid phase
humid air (d) B2H6 , 2NH3 is known as “inorganic benzene”
(b) A charge in crystalline structure of tin 18. In silicon dioxide - [AIEEE - 2005]
(c) A change in the partial pressure of O2 in air (a) each silicon atom is surrounded by four oxygen
(d) An interaction with N2 of air at low temperature atoms and each oxygen atom is bonded to two
12. Which one of the following statements regarding silicon atoms
helium is incorrect -[AIEEE - 2004]
(b) each silicon atom is surrounded by two oxygen
(a) It is used to produce and sustain powerful atoms and each oxygen atom is bonded to two
superconducting magnets silicon atoms
(b) It is used as a cryogenic agent for carrying out
(c) silicon atom is bonded to two oxygen atoms
experiments at low temperatures
(d) there are double bonds between silicon and
(c) It is used to fill gas balloons instead of hydrogen
because it is lighter than hydrogen and oxygen atoms
non-inflammable 19. Regular use of which of the following fertilizer
(d) It is used in gas-cooled nuclear reactors increases the acidity of soil -[AIEEE - 2007]
13. The number of hydrogen atoms attached to (a) Potassium nitrate
phosphorus atom in hypophosphorous acid is - (b) Urea
[AIEEE - 2004]
(c) Superphosphate of lime
(a) Zero (b) Two
(d) Ammonium sulphate
(c) One (d) Three

www.jeebooks.in
6.38 THE p-BLOCK ELEMENTS AND THEIR COMPOUNDS
20. The stability of dihalides of Si, Ge, Sn and Pb 26. In view of the signs of rG° for the following
increases steadily in the sequence reactions -
-[AIEEE - 2007] PbO2 + Pb 2PbO, rG° < 0
(a) GeX2 << SiX2 << SnX2 << PbX2 SnO2 + Sn 2SnO, rG° > 0
(b) SiX2 << GeX2 << PbX2 << SnX2 Which oxidation states are more characteristic for
(c) SiX2 << GeX2 << SnX2 << PbX2 lead and tin ? [AIEEE - 2011]
(d) PbX2 << SnX2 << SiX2 << GeX2 (a) For lead + 4 , for tin + 2
21. Among the following substituted silanes the one (b) For lead + 2 , for tin + 2
which will give rise to cross linked silicone polymer (c) For lead + 4 , for tin + 4
on hydrolysis is - [AIEEE - 2008]
(d) For lead + 2 , for tin + 4
(a) R4Si (b) RSiCl3
27. The number of S-S bonds in SO3 , S2O32– , S2O62–
(c) R2SiCl2 (d) R2SiCl and S2O82– respectively are -
22. Which one of the following reactions of Xenon [Jee Main-Online- 2012]
compounds is not feasible -[AIEEE - 2009]
(a) 1,0,1,0 (b) 0,1,1,0
(a) 2XeF2 + 2H2O  2Xe + 4HF + O2
(c) 1,0,0,1 (d) 0,1,0,1
(b) XeF6 + RbF  Rb[XeF7]
28. Which one of the following depletes ozone layer
(c) XeO3 + 6HF  XeF6 + 3H2O
-[Jee Main-Online- 2012]
(d) 3XeF4 + 6H2O  2Xe + XeO3+ 12HF +
1.5O2 (a) NO and freons (b) SO2
23. Which of the following statements is wrong ? (c) CO (d) CO2
[AIEEE - 2011] 29. In which of the following arrangements, the
(a) Single N-N bond is weaker than the single sequence is not strictly according to the property
P-P bond written against it -[Jee Main-Online- 2012]
(b) N2O4 has two resonance structures (a) CO2 < SiO2 > SnO2 > PbO2 : increasing
(c) The stability of hydrides increases from NH3 oxidising power
to BiH3 in group 15 of the periodic table (b) B < C > O > N : increasing first ionisation
(d) Nitrogen cannot form d-p bond enthalpy
24. Which of the following statements regarding (c) NH3 < PH3 > AsH3 > SbH3 : increasing basic
sulphur is incorrect - [AIEEE - 2011] strength
(a) At 600°C the gas mainly consists of S 2 (d) HF < HCl > HBr > HI : increasing acid
molecules strength
(b) The oxidation state of sulphur in nevel less 30. The compound of Xenon with zero dipole moment
than +4 in its compounds is -[Jee Main-Online- 2012]
(c) S2 molecule is paramagnetic (a) XeO3 (b) XeO2
(d) The vapour at 200°C consists mostly of (c) XeF4 (d) XeOF4
S8 rings 31. The formation of molecular complex BF3 – NH3
25. Boron cannot form whcih one of the following results in a change in hybridisation of boron
anions ? [AIEEE - 2011] -[Jee Main-Online- 2012]

(a) B(OH)4 (b) BO2– (a) from sp3 to sp3d (b) from sp2 to dsp2
(3) BF63– (d) BH4– (c) from sp3 to sp2 (d) from sp2 to sp3

www.jeebooks.in
THE p-BLOCK ELEMENTS AND THEIR COMPOUNDS 6.39
32. Trigonal bipyramidal geometry is shown by (a) IF7 : Pentagonal bipyramid
-[Jee Main-Online- 2013] (b) BrF5 : Trigonal bipyramid
(a) XeO3F2 (b) XeOF2 (c) ICl3 : Planar dimeric
2–
(c) [XeF8] (d) FXeOSO2F (d) BrF3 : Planar T-shaped
33. Oxidation state of sulphur in anions SO32–, S2O42– 39. Which of the following series correctly represents
and S2O62– increases in the orders relations between the elements from X to Y ?
-[Jee Main-Online- 2013] XY [Jee Main-Online- 2014]
(a) S2O6 < S2O4 < SO32–
2– 2– (a) 18Ar 54XeNoble character increases
(b) SO32– < S2O42– < S2O62– (b) 3Li 19KIonisation enthalpy increases
(c) S2O42– < SO32– < S2O42– (c) 6C 32GeAtomic radii increases
(d) S2O42– < S2O62– < SO32– (d) 9F 35BrElectron gain enthalpy with negative
sign increases
34. XeO4 molecule is tetrahedral having
40. Which of the following statements about the
-[Jee Main-Online- 2013]
depletion of ozone layer is correct
(a) Two p-d bonds
-[Jee Main-Online- 2014]
(b) Four p-d bonds
(a) the problem of ozone depletion is more serious
(c) One p-d bond at poles because ice crystals in the clouds over
(d) Three p-d bonds poles act as catalyst for photochemical
35. The shape of IF6– is reactions involving the decomposition of ozone
-[Jee Main-Online- 2013] by Cl and ClO radicals
(a) Trigonally distorted octahedron (b) the problem of ozone depletion is less serious
at poles because NO2 solidifies and is not
(b) Pyramidal
available for consuming ClO radicals
(c) Octahedral
(c) Oxides of nitrogen also do not react with ozone
(d) Square antiprism in stratosphere
36. The catenation tendency of C, Si and Ge is in the (d) Freons, chlorofluorocarbons, are inert
order Ge < Si < C. The bond energies (in kJ mol–1) chemically, they do not react with ozone in
of C–C, Si–Si and Ge–Ge bonds are respectively stratosphere
-[Jee Main-Online- 2013] 41. Which of the following xenon-OXO compounds
(a) 348, 260, 297 (b) 348, 297, 260 may not be obtained by hydrolysis of xenon
(c) 297, 348, 260 (d) 260, 297, 348 fluorides -[Jee Main-Online- 2014]
37. The gas evolved on heating CaF2 and SiO2 with (a) XeO2F2 (b) XeO3
concentrated H2SO4 , on hydrolysis gives a white (c) XeO4 (d) XeOF4
gelatinous precipitate. The precipitate is 42. Hydrogen peroxide acts both as an oxidising and
-[Jee Main-Online- 2014] as a reducing agent depending upon the nature of
(a) Silica gel (b) Silicic acid the reacting species. In which of the following cases
(c) hydrofluosilicic acid (d) calciumfluorosilicate H2O2 acts as a reducing agent in acid medium
38. Shape of certain interhalogen compounds are -[Jee Main-Online- 2014]
stated below. Which one of them is not correctly (a) MnO4– (b) SO32–
stated - [Jee Main-Online- 2014] (c) KI (d) Cr2O72–

www.jeebooks.in
6.40 THE p-BLOCK ELEMENTS AND THEIR COMPOUNDS
43. Consider the reaction 49. Which among the following is the most reactive ?
-[Jee Main-Online- 2014] [Jee Main - 2015]
H2SO3(aq) + Sn (aq) + H2O(l) Sn2+(aq) + HSO4–
4+
(a) I2 (b) ICl
(aq)
+ 3H+(aq) (c) Cl2 (d) Br2
Which of the following statement is correct ? 50. Which one has the highest boiling point ?
(a) H2SO3 is the reducing agent because it [Jee Main - 2015]
undergoes oxidation (a) Kr (b) Xe
(b) H2SO3 is the reducing agent because it (c) He (d) Ne
undergoes reduction
51. From the following statements regarding H2O2,
(c) Sn4+ is the reducing agent because it undergoes choose the incorrect statement :
oxidation
-[Jee Main-2015]
(d) Sn4+ is the oxidizing agent because it undergoes
(a) it has to be stored in plastic or wax lined glass
oxidation
bottles in dark
44. The number and type of bonds in C22– ion in CaC2
(b) it has to be kept away from dust
are -[Jee Main-Online- 2014]
(c) it can act only as an oxidizing agent
(a) two  bonds and one  bond
(d) it decomposes on exposure to light
(b) two  bonds and two  bonds
(c) one  bonds and two  bonds EXERCISE # V(B) JEE-ADVANCED
(d) one  bond and one  bond
45. In the following sets of reactants which two sets Fill in the Blanks
best exhibit the amphoteric character of 1. The hydrolysis of alkyl substituted chlorosilanes
Al2O3.xH2O? [Jee Main-Online- 2014] given ............... [IIT-1991]
Set-1 : Al2O3.xH2O(s) and OH– (aq) 2. The hydrolysis of trialkylchlorosiane R3 SiCl, yields
Set-2 : Al2O3.xH2O(s) and H2O () ............... [IIT-1994]
Set-3 : Al2O3.xH2O(s) and H+ (aq) 3. Two types of bonds present in B2H6 are covalent
and ............... [IIT-1994]
Set-4 : Al2O3.xH2O(s) and NH3 (aq)
4. One recently discovered allotrope of carbon (e.g.,
(a) 1 and 2 (b) 2 and 4
C60) is commonly known as ............... [IIT-1994]
(c) 1 and 3 (d) 3 and 4
True/False
46. Example of a three-dimensional silicate is
5. Carbon tetrachloride burns in air when lighted to
-[Jee Main-Online- 2014] give phosgene. [IIT-1983]
(a) Beryls (b) Zeolites 6. Graphite is a better lubricant on the moon than on
(c) Feldspars (d) Ultramarines the earth. [IIT-1987]
47. Which of the following compounds has a P–P 7. All the Al – Cl bonds in Al2Cl6 are equivalent.
bond ? [Jee Main-Online- 2015] [IIT-1989]
(a) H4P2O5 (b) (HPO3)3 8. Diamond is harder than graphite. [IIT-1993]
(c) H4P2O7 (d) H4P2O6 9. The basic nature of the hydroxides of group 13
48. Chlorine water on standing loses its colour and (Gr. IIIB) decreases progressively down the group.
forms -[Jee Main-Online- 2015] [IIT-1993]
(a) HCl and HClO2 (b) HCl only 10. The tendency for catenation is much higher for C
(c) HOCl and HCl2 (d) HCl and HOCl than for Si. [IIT-1993]

www.jeebooks.in
THE p-BLOCK ELEMENTS AND THEIR COMPOUNDS 6.41
11. Complete and balance the following chemical 20. Give reason for the following in one or two
equations - [IIT-1998, 2M] sentences only. [IIT-1999]
(i) P4O10 + PCl5  “BeCl2 can be easily.”
(ii) SnCl4 + C2H5Cl + Na  21. Draw the molecular structures of XeF2 , XeF4 and
12. Work out the following using chemical equations XeO2F2 , indicating the location of lone pair(s) of
electrons. [IIT-2000]
[IIT-1998, 2M]
22. Give reason : [IIT-2000]
“Chlorination of calcium hydroxide produces
bleaching powder” Why elemental nitrogen exists as a diatomic
molecule whereas elemental phosphorus is a tetra
13. Hydrogen peroxide acts both as an oxidizing and
atomic molecule.
a reducing agent in alkaline solution towards certain
23. Givan an example of oxidatioln of one halide by
first row transition metal ion. Illustrate both these
another halogen. Explain the feasibility of the
properties of H2O2 using chemical equations
reaction. [IIT-2000]
-[IIT-1998, 4M]
24. Compound X on reduction with LiAIH4 gives a
14. In the contact process for industrial manufacture hydride Y containing 21.72 % hydrogen alongwith
of sulphuric acid, some amount of sulphuric acid other products. The compound Y reacts with air
is used as a starting material. Explain briefly. What explosively resulting in boron trioxide. Identify X
is the catalyst used in the oxidation of SO2? and Y. Given balanced reactions involved in the
[IIT-1998, 4M] formation of Y and its reaction with air draw the
15. Give reasons in one or two sentences for each of structure of Y. [IIT-2001]
the following : [IIT-1985] 25. Starting from SiCl4 , prepare the following in steps
not exceeding the number given in parenthesis
(i) Graphite is used as a solid lubricant
(reactions only) [IIT-2001]
(ii) Fluorine cannot be prepared from fluorieds by
(i) Silicon (1)
chemical oxidation.
(ii) Linear silicon containing methyl group
16. Write balanced equations for : only (4)
[IIT-1990]
(iii) Na2SiO3 (3)
(i) The preparation of crystalline silicon from SiCl4
26. Write the balanced chemical equation for
(ii) The preparation of phosphine from CaO and developing photographic films. [IIT-2001]
white phosphorus
27. Identify(X) in the following synthetic scheme and
(iii) The preparation of ammonium sulphate from write their structures. [IIT-2001]
gypsum, ammonia and carbon dioxide *

17. Anhydrous AlCl3 is covalent. From the data given  X(gas) (C denotes C14 )
Ba C O3  H 2SO 4 
below, predict whether it would remain covalent 28. Write the balanced equations for the reactions of
or become ionic in aqueous solution. [IIT-1997] the following compounds with water[IIT-2002]
Iinisation energy for Al = 5137 kJ mol–1 (i) Al4C3 (ii) CaNCN
Hhydration for Al3+ = 5137 kJ mol–1 (iii) BF3 (iv) NCl3
Hhydration for Cl– = –381 kJ mol–1 (v) XeF3
18. Aluminium sulphide gives a fourl odour when it 29. Write the balanced equations for the reactions of
becomes damp. Write a balanced chemical the following compounds with water [IIT-2002]
equation for the reaction :- [IIT-1997] (i) Al4C3 (ii) CaNCN
19. Draw the structure of a cyclic silicate, (Si3O9)6 (iii) BF3 (iv) NCl3
with proper labelling- [IIT-1998]
(v) XeF4

www.jeebooks.in
6.42 THE p-BLOCK ELEMENTS AND THEIR COMPOUNDS
30. Identify the following [IIT-2003] 39. Which of the following are hydrolysed :
SO 2
NaCO3  A  Na 2CO3
 B 
  C 
D elemental S I2 [REE-2000]
aq.
(a) NCl3 (b) BCl3
Also mentioln the oxidation state of S in all the
compounds. (c) CCl4 (d) SiCl4
31. Arrange the following oxide in the increasing order 40. The set with correct order of acidity is :
of Bronsted basicity. [IIT-2004] [IIT-2001]
Cl2O7 , BaO7 , SO3 , CO2 , B2O3 (a) HClO < HClO2 < HClO3 < HClO4
32. When zeolite , which is hydrated sodium (b) HClO4 < HClO3 < HClO2 < HClO
aluminium, is treated with hard water, the sodium (c) HClO < HClO4 < HClO3 < HClO2
ions are exchanged with : [IIT-1990] (d) HClO4 < HClO2 < HClO3 < HClO
(a) H+ ions (b) Ca2+ ions 41. The reaction ,
(c) SO42– ions (d) Mg2+ ions 3ClO–(aq) ClO3– (aq) + 2Cl– (aq)
33. Which of the following halides is least stable and is an example of : [IIT-2001]
has doubtful existence : [IIT-1996]
(a) oxidatioln reaction
(a) CCl4 (b) Gel4
(b) reduction reaction
(c) Snl4 (d) Pbl4
(c) disproportionation reaction
34. In compounds of type ECl3 , where E = B, P, As
or Bi, the angles Cl–E–Cl for different E are in the (d) decomposition reaction
order : [IIT-1999] 42. The number of S-S bonds in sulphur trioxide trimer,
(a) B > P = As = Bi (b) B > P > As > Bi (S3O9) is : [IIT-2001]
(c) B < P = As = Bi (d) B < P < As < Bi (a) three (b) two
35. The number of P—O—P bonds in cyclic (c) one (d) zero
tetrametaphosphoric acid is : [IIT-2000] 43. Specify the coordination geometry around and
(a) zero (b) two hybridisation of N and B atoms in a 1:1 complex
(c) three (d) four of BF3 and NH3 : [IIT-2001]
36. The correct order of acidic strength is : (a) N : tetrahedral sp3 ; B: tetrahedral sp3
[IIT-2000] (b) N : pyramidal sp3 ; B: pyramidal sp3
(a) Cl2O7 > SO2 > P4O10 (c) N : pyramidal sp3 ; B: planar sp3
(b) CO2 > N2O5 > SO3 (d) N : pyramidal sp3 ; B: tetrahedral sp3d
(c) Na2O > MgO > Al2O3 44. Statement–1 : Between SiCl4 and CCl4 , only
(d) K2O > CaO > MgO SiCl4 reacts with water. [IIT-2001]
37. Amongest H2O, H2S, H2Se and H2Te , the one Statement–2 : SiCl4 is ionic and CCl4 is covalent.
with the highest boiling point is : [IIT-2000] (a) Statement–1 is True, Statement–2 is True ;
(a) H2O because of hydrogen bonding Statement–2 is a correct explanation for
(b) H2Te because of higher molecular weigtht Statement–1
(c) H2S because of hydrogen bonding (b) Statement–1 is True, Statement–2 is True ;
(d) H2Te because of lower molecular weigtht Statement–2 is NOT a correct explanation for
38. Ammonia can be dried by : [IIT-2000] Statement–1
(a) conc. H2SO4 (b) P 4O10 (c) Statement–1 is True, Staicment–2 is False
(c) CaO (d) Anhydrous CaCl2 (d) Statement–1 is False, Statement–2 is True

www.jeebooks.in
THE p-BLOCK ELEMENTS AND THEIR COMPOUNDS 6.43
45. Polyphosphate are used as water softening agents 52. Which is the most thermodynamically stable
because they : [IIT-2002] allotropic form of phosphorus : [IIT-2004]
(a) form soluble complexes with anionic species (a) red (b) white
(b) precipitate anionic species (c) black (d) yellow
(c) form soluble complexes with cationic species 53. When PbO2 reacts with conc.HNO 3 the gas
evolved is : [IIT-2005]
(d) precipitate cationic species
(a) NO 2 (b) O 2
46. Identify the correct order of solubility of Na2S,
CuS and ZnS in aqueous medium : [IIT-2002] (c) N 2 (d) N 2 O
(a) CuS > ZnS > Na2S 54. Which of the following is not oxidised by O3 :
[IIT-2005]
(b) ZnS > Na2S > CuS
(a) KI (b) FeSO4
(c) Na2S > CuS > ZnS
(c) N 2O 4 (d) N 2O 5
(d) Na2S > ZnS > CuS
55. Which blue liquid is obtained on reacting equimolar
47. Identify, the correct order of acidic strength of amounts of two gases at –30°C? [IIT-2005]
CO2, CuO, CaO, H2O : [IIT-2002]
(a) N 2 O (b) N 2O 3
(a) CaO < CuO < H2O < CO2 (c) N 2O 4 (d) N 2O 5
(b) H2O < CuO < CaO < CO2 56. Name of the structure of silicates in which three
(c) CaO < H2O < CuO < CO2 oxygen atoms of [SiO4]4– are shared is :[IIT-2005]
(d) H2O < CO2 < CaO < CuO (a) Pyrosilicate
48. H3BO3 is : [IIT-2002,3] (b) Sheet silicate
(a) monobasic acid and weak Lewis acid (c) linear chain silicate
(b) monobasic acid and weak Bronsted acid (d) three dimensional silicate
(c) monobasic acid and strong Lewis acid 57. B(OH)3 + NaOH   NaBO + Na[B(OH) ]
2 4
(d) monobasic acid and strong Bronsted acid + H2O how can this reaction is made to proceed
49. When I– is oxidised by MnO4– in alkaline medium, in forward direction ? [IIT-2006]
I– convertes into : [IIT-2003] (a) addition of cis 1, 2 diol

(a) IO 3 (b) I2 (b) addition of borax

(c) IO 4 (d) IO – (c) addition of trans 1, 2 diol
50. Match the column : [IIT-2003] (d) addition of Na2HPO4
Column-I Column-II 58. The percentage of p-character in the orbitals
3+
(a) Bi (BiO) +
(P) Heat forming P-P bonds in P4 is : [IIT-2007]
(a) 25 (b) 33
(b) [AlO2]– Al(OH)3 (Q) Hydrolysis
(c) 50 (d) 75
(c) SiO44– Si2O76– (R) Acidification
2–
59. Among the following , the paramagnetic compound
(d) (B4O7 ) [Bi(OH)3] (S) Dilution by water is : [IIT-2007]
51. (Me)2SiCl2 on hydrolysis will produce : (a) Na2O2 (b) O 3
[IIT-2003]
(c) N 2 O (d) KO 2
(a) (Me)2Si(OH)2 60. Statement–1 : Boron always forms covalent bond.
(b) (Me)2Si = O
[IIT-2007]
(c) [—O—(Me)2Si—O—]n
Statement–2 : The small size of B3+ favours
(d) Me2SiCl(OH) formation of covalent bond.

www.jeebooks.in
6.44 THE p-BLOCK ELEMENTS AND THEIR COMPOUNDS
(a) Statement–1 is True, Statement–2 is True ; 64. XeF4 and XeF6 are expected to be :
Statement–2 is a correct explanation for (a) oxidising (b) reducing
Statement–1 (c) unreactive (d) strongly basic
(b) Statement–1 is True, Statement–2 is True ; Passage for Q.65 to Q.67 :
Statement–2 is NOT a correct explanation for
There are some deposits of nitrates and phosphates in
Statement–1
earth’s crust. Nitrates are more soluble in water.
(c) Statement–1 is True, Staicment–2 is False Nitrates are difficult to reduce under the laboratory
(d) Statement–1 is False, Statement–2 is True conditions but microbes do it easily. Ammonia forms
61. Statement–1 : In water, orthoboric acid behaves large number of complexes with transition metal ions.
as a weak monobasic acid. [IIT-2007] Hybridisation easily explains the case of sigma donation
Statement–2 : In water, orthoboric acid acts as capability of NH3 and PH3. Phosphine is a flammable
a proton donor. gas and is prepared from white phosphorus.
(a) Statement–1 is True, Statement–2 is True ; [IIT-2008]
Statement–2 is a correct explanation for 65. Among the following , the correct statement :
Statement–1 (a) phosphates have no biological significance in
(b) Statement–1 is True, Statement–2 is True ; humans
Statement–2 is NOT a correct explanation for (b) between nitrates and phosphates , phosphates
Statement–1 are less abundant in earth’s crust
(c) Statement–1 is True, Staicment–2 is False (c) between nitrates and phosphates , nitrates are
(d) Statement–1 is False, Statement–2 is True less abundant in earth’s crust
Passage for Q.62 to Q.64 : (d) oxidation of nitrates is possible in soil
The noble gases have closed - shell electronic 66. Among the following , the correct statement :
configuration and are monoatomic gases under normal (a) Between NH3 and PH3, NH3 is a better electron
conditions. The low boiling point of the lighter noble donor because the lone pair of electrons
gases are due to weak dispersion forces between the occupies spherical ‘s’ orbital and is less
atoms and the absence of other interatomic interaction. directional.
The direct reaction of xenon with fluorine leads to a (b) Between NH3 and PH3, NH3 is a better electron
series of compounds with oxidation number +2, +4 donor because the lone pair of electrons
and +6. occupies spherical ‘sp3’ orbital and is more
XeF4 reacts violently with water to give XeO3. The directional.
compounds of xenon exhibit rich stereochemistry and (c) Between NH3 and PH3, NH3 is a better electron
their geometries can be deduced considering the total donor because the lone pair of electrons
number of electron pairs in the valance shell. occupies ‘sp3’ orbital and is more directional.
[IIT-2007]
(d) Between NH3 and PH3, NH3 is a better electron
62. Argon is used in are welding because of its : donor because the lone pair of electrons occupies
(a) low reactivity with metal spherical ‘s’ orbital and is less directional.
(b) ability to lower the melting point of metal 67. The phosphorus on reaction with NaOH gives PH3
(c) flammability as one of the products. This is a :
(d) high calorific value (a) dimerization reaction
63. The structure of XeO3 is : (b) disproportionation reaction
(a) linear (b) planar (c) condensation reaction
(c) pyramidal (d) T-shaped (d) precipitation reaction

www.jeebooks.in
THE p-BLOCK ELEMENTS AND THEIR COMPOUNDS 6.45
68. The reaction of P4 with X leads selectively to P4O6. Passage for Q.76 to Q.77 :
The X is : [IIT-2009] The reaction of Cl2 gas with cold dilute and hot
(a) Dry O2 concentrated NaOH in water give sodium salt of two
(b) A mixture of O2 , N2 (different) oxoacids of chlorine P and Q respectively.
(c) Moist O2 The Cl2 gas reacts with SO2 gas, in presence of charcoal
(d) O2 in the presence of aqueous NaOH to give a product R. R reacts with white phosphorus to
69. The nitrogen oxide(s) the contain(s) N-N bond(s) give a compounds S. On hydrolysis, S gives as oxyacid
is (are) :- [JEE-2009] of phosphorus T. [JEE-2013]
(a) N 2 O (b) N 2O 3 76. R, S and T respectively are :
(c) N2O4 (d) N 2O 5 (a) SO2Cl2, PCl5 and H2PO4
70. In the reaction 2X + B2H6 [BH2(X)2]+ [BH4]– (b) SO2Cl2, PCl3 and H3PO3
the amine(s) X is (are) [JEE-2009]
(c) SO2Cl2, PCl3 and H3PO2
(a) NH3 (b) CH3NH2
(d) SO2Cl2, PCl5 and H3PO4
(c) (CH3)2NH (d) (CH3)3N
77. P and Q , respectively , are the sodium salts of :
71. The reaction of white phosphorus with aqueous
NaOH gives phosphine along with another (a) Hypochlorus and chloric acid
phosphorus containing compound. The reaction (b) Hypochlorus and chlorus acid
type ; the oxidation states of phosphorus in (c) Chloric and perchloric acid
phospine and the other product are respectively : (d) Chloric and hypochlorus acid
- [JEE-2012]
78. The unbalanced chemical reactions given in List-I
(a) redox reaction ; –3 and –5 show missing reagent or condition (?) which are
(b) redox reaction ; +3 and +5 provided in List-II. Match the List-I with List-II
(c) disproportionation reaction ; –3 and +1 and select the correct answer using the code given
(d) disproportionation reaction ; –3 and +3 below the lists : A :- [JEE-2013]
72. Bleaching powder contains a salt of an oxoacid List-I
as one of its components. The anhydride of that (P) PbO2 + H2SO4  ?
 PbSO4 + O2 + other
oxoacid is :- [JEE-2012] product
(a) Graphite is harder than diamond
(Q) Na2S2O3 + H 2O  ?
  NaHSO 4 + other
(b) Graphite has higher electrical conductivity than product
diamond o

(c) Graphite has higher thermal conductivity than (R) N2H4  ?


  N 2 + other product
diamond (S) XeF2  ?
 Xe + other product
(d) Graphite has higher C–C bond than diamond List-II
74. Concentrated nitric acid upon long standing, turns (i) NO
yellow-brown due to the formation of :- (ii) I2
[JEE-2013]
(iii) Warm
(a) NO (b) NO 2
(iv) Cl2
(c) N2O (d) N 2O 4
Codes :
75. The correct statement(s) about O3 is (are) :-
[JEE-2013] P Q R S
(a) O–O bond lengths are equal (a) 1 2 3 1
(b) Thermal decomposition of O3 is endothermic (b) 3 2 1 4
(c) O3 is diamagnetic in nature (c) 1 4 2 3
(d) O3 has bent structure (d) 3 4 2 1

www.jeebooks.in
6.46 THE p-BLOCK ELEMENTS AND THEIR COMPOUNDS
79. Under ambient conditions, the total number of (a) The number of Cl = O bonds in (ii) and (iii)
gases released as products in the final steo of the together is two
reaction scheme shown below is :- (b) The number of lone pairs of electron on Cl in
[JEE-Adv. 2014] (ii) and (iii) together is three
Complete
hydrolysis (c) The hybridisation of Cl in (iv) is sp3
XeF6 P + other product
(d) Amongest (i) to (iv), the strongest acid is (i)
OH– / H2O 83. When O2 is adsorbed on a metallic surface,
electron transfer occurs from the metal to O2. The
Q
TRUE, statements regarding this adsorption is
show disproportionation in OH– / H2O (are) :- [JEE-Adv. 2015]
(a) O2 is physisorbed
product
(b) heat is released
(a) 0 (b) 1
(c) occupancy of *2p of O2 is increased
(c) 2 (d) 3
(d) bond length of O2 is increased
80. The product formed in the reacton of SOCl2 with 84. Under hydrolytic conditions , the compounds used
white phosphorus is :- [JEE-Adv. 2014] for preparation of linear polymer and for chain
(a) It behaves as a weak acid in water due to self termination , respectvley, are :- [JEE-Adv. 2015]
ionization (a) CH3SiCl3 and Si(CH3)4
(b) Acidity of its aqueous solution increases upon (b) (CH3)3SiCl and (CH3)2SiCl2
addition of ethylene glycol (c) (CH3)2SiCl2 and CH3SiCl3
(c) It has a three dimensional structure due to (d) SiCl4 and (CH3)3SiCl
hydrogen bonding
85. Three moles B2H6 are completely reacted with
(d) It is a weak electrolyte in water methanol. The number of moles of boron containing
82. The correct statement(s) regarding (i) HClO, (ii) product formed is :- [JEE-Adv. 2015]
HClO2, (iii) HClO3 and (iv) HClO4 is (are) :- 86. The total number of lone pairs of electron in N2O3
[JEE-Adv. 2015] is :- [JEE-Adv. 2015]


www.jeebooks.in
THE d-BLOCK ELEMENTS AND SOME OF THEIR COMPOUNDS
PROB L EM S B ASED ON GI VEN T OPI CS  Manganate and permanganate
 Variable oxidation state  Preparation
 Stability of the variosu oxidation state  Properties
 Complexes  Silver and its compounds
 Size of atoms and ions  Silver nitrate (AgNO3)
 Density  Zinc compoudns
 Melting and boiling points  Zinc oxide (ZnO)
 Reactivity and boiling points  Zinc chloride (ZnCl2)
 Ionizaition energies
 Zinc sulphate (ZnSO4)
 Colour
 Copper compounds
 Polarization
 Copper oxide (CuO)
 Incompletely filled d or f shell
 Copper chloride(CuCl2)
 Magnetic properties
 Copper sulphate (CuSO4)
 Catalytic properties
 Iron compounds
 Nonstoichiometry
 Iron sulphate (FeSO4 . 7H2O)
 Abundance
 Iron oxide (FeO)
 Chromate and dichromate
 Iron chloride (FeCl2)
 Preparation
 Properties

www.jeebooks.in
CHAPTER
7
The d-Block Elements and
Some of Their Compounds
EXERCISE # I
 Only one correct answer : 7. Which reaction is most spontaneous?
1. Which of the following chromate is brick red colour (a) Mn + 2H+  Mn2+ + H2
precipitate ? (b) Fe + 2H+  Fe2+ + H2
(a) BaCrO4 (b) SrCrO4 (c) Zn + 2H+  Zn2+ + H2
(c) Ag2CrO4 (d) Hg2CrO4 (d) Co + 2H+  Co2+ + H2
2. Which of the following chromate is scarlet red
8. The actual E Zn 2 / Zn is more negative than expected
colour precipitate ?
from the general trend. It is because of :-
(a) BaCrO4 (b) SrCrO4
(a) very low enthalpy of hydration of Zn2+
(c) Ag2CrO4 (d) Hg2CrO4
(b) stability of Zn2+ due to d5 configuration
3. When potassium permanganate in small portion is
(c) stability of Zn2+ due to completely filled d
added to well cooled concentrated sulphuric acid,
subshell
a dark green solution is formed which is used as
explosive. The oxidation state of manganese in (d) None
product is :- 9. Lunar caustic is :-
(a) +2 (b) +4 (a) NaOH (b) KOH
(c) +6 (d) +7 (c) Ba(OH)2 (d) AgNO3
4. The highest oxidation state of Mn is not observed 10. Bordaeux mixture is :-
in which of the following compound of (a) CuCl2 + Cu(OH)2
Mn :- (b) CuSO4 + Ca(OH)2
(a) KMnO4 (b) K2MnO4 (c) Cu(NO3)2 + KOH
(c) Mn2O7 (d) MnO3F (d) Cu2Cl2 + K2CO3
5. Oxalic acid in presence of sulphuric acid produces 11. The correct order of ionic radii of Y3+ , La3+ , Eu3+
CO2 gas when it is allowed to react with KMnO4 and Lu3+ is
solution. The reagent which is used as autocatalyst (a) Y3+ < La3+ < Eu3+ < Lu3+
in this reaction is :- (b) La3+ < Eu3+ < Lu3+ < Y3+
(a) (CO2H)2 (b) KMnO4 (c) Y3+ < Lu3+ < Eu3+ < La3+
(c) dil.H2SO4 (d) MnSO4 (d) Lu3+ < La3+ < Eu3+ < Y3+
6. Which element is not affected by 1(M) H+, among 12. Which of the element in first series of transition
transitional element :- elements has highest heat of atomisation ?
(a) Cu (b) Fe (a) Cr (b) V
(c) Co (d) Ni (c) Co (d) Ni

www.jeebooks.in
7.4 THE d-BLOCK ELEMENTS AND SOME OF THEIR COMPOUNDS
13. E° value is highest +ve for which of the following 21. RCH = O + Fehling solution  RCO2H + Reddish
reaction :- brown precipitate -
(a) MnO4– + e MnO42– Reddish brown precipitate is
(b) MnO4– + 4H+ + 3e– MnO2 + 2H2O (a) CuO (b) Pb3O4
(c) MnO4– + 8H+ + 5e Mn2+ + 4H2O (c) Hg2O (d) Cu2O
(d) Cr2O72– + 14H+ + 6e– 2Cr3+ + 7H2O 22. The stability order :-
15. Metal (M) + dilute HCl  H2 gas (a) CrO42– > MnO42–> FeO42–
Metal (M) (red hot) + steam  H2 gas + Mixed (b) CrO42– > FeO42– >MnO42–
oxide
(c) MnO42–> CrO42– > FeO42–
Metal M can be :-
(d) FeO42– >CrO42– > MnO42–
(a) Copper (b) Zinc
23. + 4 oxidation state of Pt is observed in :-
(c) Iron (d) Cadmium
(a) O2[PtF6] (b) H2[PtCl4]
16. In mild acidic medium manganate ion
(c) H2[PtCl6] (d) K2[Pt(CN)4]
disproportionate in :-
24. The colour of mercurous iodide precipitate is :-
(a) Mn2+, MnO4–
(a) Green (b) Black
(b) MnO2 , MnO4–
(c) Mn2+, Mn2O7 (c) Yellow (d) Scarlet red
(d) MnO2, Mn(OH)2 25. The colour of mercuric iodide precipitate is :-
17. Arrange the following species in the increasing (a) Green (b) Black
order of their magnetic properties :- (c) Yellow (d) Scarlet red
(a) VCl3 > VOSO4 > Na3VO4 > 26. The colour of mercurous oxide precipitate is :-
[V(H2O)6]SO4.H2O (a) Green (b) Black
(b) [V(H2O)6]SO4 > VCl3 > VOSO4 > Na3VO4 (c) Yellow (d) Scarlet red
(c) VCl 3 > VOSO 4 > [V(H 2O) 6 ]SO 4 .H 2O > 27. The colour of mercuric oxide precipitate is :-
Na3VO4
(a) Green (b) Black
(d) All
(c) Yellow (d) Scarlet red
18. Oxide of which d10 ion is amphotetic :-
28. The aqueous solution of CuCrO4 is green because
(a) ZnO (b) CdO it contains :-
(c) HgO (d) OH (a) green Cu2+ ions
19. The aqueous solution of CuCrO4 is green because (b) green CrO42– ions
it consists of :-
(c) blue Cu2+ ions & green CrO42– ions
(a) Green Cu2+ ion
(d) blue Cu2+ ions & yellow CrO42– ions
(b) Blue Cu2+ & Green CrO42– ion
29. The compound of pentavalent gold is ?
(c) Blue CrO42– & Green Cu2+ ion
(a) AuF5 (b) AuCl5
(d) Blue Cu2+ & Yellow CrO42– ion
(c) AuBr5 (d) AuI5
20. The highest oxidation state is exhibited by the
transition metals with configuration - 30. Which cuprous halide is white in colour ?
(a) (n–1)d3ns2 (b) (n–1)d5ns1 (a) CuF (b) CuCl
(c) (n–1)d5ns2 (d) (n–1)d8ns2 (c) CuCl2 (d) CuBr

www.jeebooks.in
THE d-BLOCK ELEMENTS AND SOME OF THEIR COMPOUNDS 7.5
2+ – 2–
31. [Co(H2O)6] + 4Cl  CoCl4 Here :-
Colour of the complex in reactant and product side (a) A is CuS and B is H2S
are respectively (b) A is FeS ; C is FeSO4 ; D is PbS ; E is
(a) Blue ; Pink Fe3[Fe(CN)6]2
(b) Pink ; Blue (c) C is Zn2[Fe(CN)6] and D is PbS
(c) Pink ; Yellow (d) D is PbS and E is Fe4[Fe(CN)6]3
(d) Dark green ; Pink 39. The aqueous solution of P yields a white precipitate
32. When acetylene is passed through cuprous when treated with dilute HNO3 and AgNO3.
chloride:- Another sample of the solution of P when treated
(a) Red colour precipitate of Cu2C2 is formed with NaOH gives a white precipitate which is
dissolved in excess of NaOH. When H2S gas is
(b) White colour precipitate of Cu2C2 is formed
passed through solution, a white precipitate is
(c) Blue colour precipitate of CuC2 is formed obtained. P is :-
(d) Blue colour precipitate of Cu2C2 is formed (a) CoCl2 (b) ZnCl2
33. Microcosmic salt when heated , a transparent bead (c) NiCl2 (d) SnCl2
is fomed. The transparent bead is :-
40. The highly water soluble red violet crystalline
(a) NaBO2 (b) B2O3 compound K3[Mn(C2O4)3].3H2O is octahedral
(c) P 2O 5 (d) NaPO3 complex. The magnetic moment is :-
34. Which of the following black colour precipitate ? (a) 15 B.M. (b) 24 B.M.
(a) Cr2O3 (b) SnS2
(c) 35 B.M. (d) 8 B.M.
(c) HgS (d) AgI
41. Which of the following form an alloy ?
35. Which one of the following ions do not give borax
(a) Zn + Pb
bead test ?
(b) Fe + Hg
(a) Cr3+ (b) Cu2+
(c) Pt + Hg
(c) Mn2+ (d) Zn2+
(d) Fe + C
36. A white powder (A) when strogly heated , it gives
off colourless odourless gas (B) which turns lime 42. KI + Mercuric chloride  Product.
(excess)
water milky (C). Also it gives solid residue D which
is yellow when hot but turns white on cooling. Product along with KOH is very important
laboratory reagent. The reagent and its formula -
(a) PbCO3 , PbO (b) ZnCO3 , ZnO
(a) Nessler’s reagent ; K2[HgI4]
(c) PbCO3 , PbO2 (d) Pb2CO3 , PH3O4
(b) Nessler’s reagent ; K4[HgI6]
37. Coagulation of blood is caused by :-
(c) Nessler’s reagent ; K2[Hg2I4]
(a) Alum
(d) Nessler’s reagent ; K2[Hg(SCN)4]
(b) Microcosmic salt
43. H2S gas is passed through an acidified solution of
(c) Sodium hydrogen phosphate
a mixture containing Cu2+ and Zn2+ ions. What will
(d) Borax happen ?
38. Black coloured (a) no precipitate is formed
(A) + H2SO4  B(g) + C (b) both will get precipitated
B (gas) + Pb(CH3CO2)2  Black ppt. (D) (c) CuS gets precipitated only
C 
K 3 [Fe(CN) 6 ]
 Blue colour(E) (d) ZnS gets precipitated only

www.jeebooks.in
7.6 THE d-BLOCK ELEMENTS AND SOME OF THEIR COMPOUNDS
44. A few Cu (I) salts are coloured. The colour arises Statement -3 : Promithium is a radioactive
from lanthanoide.
(a) d-d transition (a) TTF (b) TFT
(b) Charge transfer spectra (c) FFT (d) FTF
(c) The large wavelengths of the rays absorbed by 50. Purple of cassius is :-
the solutions (a) Colloidal sol of graphite in water
(d) The smaller wavelengths of rays is absorbed (b) Colloidal sol of silver
by the solution (c) Colloidal sol of gold
45. Which of the following does not disproportionate? (d) Colloidal sol of cellulose nitrate in ethanol
(a) Cu+ (b) Au3+ 51. Which of the following has three unpaired
(c) Ga+ (d) Au+ electron:-
46. Acidic KMnO4 is decolourised by (a) Zn2+ (b) Cr3+
(a) The ferric ammonium alum (c) Co2+ (d) Cu2+
(b) Mohr’s salt 52. Which of the following is diamagnetic :-
(c) A neutral ferric chloride solution (a) La3+ (b) Ce4+
(d) All of these (c) Yb2+ (d) All
47. A colourless salt 53. Among the following which has highest oxidising
MnO 4 / H 
power :-
A   decolourises
(a) VO2+ (b) Cr2O72–
A 
Heat
 X + Y + Z ; X and Y are gases (c) MnO4– (d) MnO2
Z + water B 54. The common oxidation state of lanthanoid :-
Y + B milkiness (a) +2 (b) +3
Gas X burns with bue flame. Mark the correct (c) +4 (d) +5
choices. 55. Which of the following is negatively charged sol ?
A X Y Z B (a) CrO3 . xH2O (hydrated metallic oxide)
(a) CaCO3 CaO CO CO 2 H2CO3 (b) Haemoglobin (blood)
(b) CaC2O4 CO CO 2 CaO Ca(OH)2 (c) Oxides TiO2 sol
(c) CaC2O4 CO 2 CO CaO Ca(OH)2 (d) CdS sol
(d) CaSO3 SO2 O 2 CaO Ca(OH)2 56. Correct order for the wavelength of absorption in
48. Statement -1: Mn2Cl10 will be diamagnetic. the visible region for the following :-
Statement -2 : TiCl4 is used for the test of H2O2. (a) [Ni(H2O)6]2+ < [Ni(NH3)6]2+ < [Ni(NO2)6]4–
Statement -3 : In Cr 3O (CH 3COO) 63H 2 O, (b) [Ni(NH3)6]2+ < [Ni(H2O)6]2+ < [Ni(NO2)6]4–
CH3COO– act as bridging ligand. (c) [Ni(H2O)6]2+ < Ni(NO2)6]4– < [Ni(NH3)6]2+
(a) FTT (b) FFT (d) [Ni(NO2)6]4– < [Ni(NH3)6]2+ < [Ni(H2O)6]2+
(c) TTT (d) FFF 57. CuS precipitate is dissolved in :-
49. Statement -1: Actinoide show higher oxidation state (a) NH4OH solution
than lanthanoide. (b) NaOH solution
Statement -2 : All lanthanoide in M+3 state are (c) Potassium cyanide solution
coloured.
(d) Water

www.jeebooks.in
THE d-BLOCK ELEMENTS AND SOME OF THEIR COMPOUNDS 7.7
58. Cr(OH)3 precipitate is dissolved in :- 65. Lanthanoid contraction is caused due to :-
(a) Sodium hydroxide (a) the appreciable shielding on outer electrons by
(b) Sodium peroxide solution 4f electrons from the nuclear charge
(c) 50% HNO3 (b) the appreciable shielding on outer electrons by
5d electrons from the nuclear charge
(d) All
(c) the same effective nuclear charge from Ce to
59. Zn(OH)2 precipitate is dissolved in :- Lu
(a) NH3 (d) the imperfect shielding on outer electron by 4f
(b) Sodium hydroxide solution electrons from the nuclear charge
(c) 50% HNO3 66. Identify the incorect statement among the
(d) All following:-
60. Mischmetal is an alloy which consists of :- (a) d-block elements show irregular and erratic
chemial properties among themselves
(a) 50% of lanthanoid metal and 50% iron & traces
of S, C, Ca and Al (b) La and Lu have partially filled d orbitals and no
other partially filled orbitals
(b) 80% of lanthanoid metal and 20% iron & traces
of S, C, Ca and Al (c) The chemistry of various lanthanoids is very
similar
(c) 95% of lanthanoid metal and 5% iron & traces
of S, C, Ca and Al (d) 4f and 5f orbitals are equally shielded
(d) None 67. The actinoids exhibit more number of oxidation
states in general than the lanthanoids. This is
61. The most stable ion in aqueous solution :-
because :-
(a) V3+ (b) Ti3+
(a) the 5f orbitals are more buried than the 4f
(c) Cr3+ (d) Mn3+ orbitals
62. Cr2O3 + 2Al Al2O3 + 2Cr , What is the H for (b) there is similarity between 4f and 5f orbitals in
the reaction. there angular part of the wave function
Given (Hf)Al O = – 827 kJ-mol–1 , (c) the actinoids are more reactive than the
2 3
(Hf)Cr O = – 540 kJ-mol–1 lanthanoids
2 3
(a) +287 kJ-mol–1 (d) the 5f orbitals extend further from the nucleus
(b) –287 kJ-mol–1 than the 4f orbitals
(c) 1367 kJ-mol–1 68. The lanthanoids contraction is responsible for the
(d) –1367 kJ-mol–1 fact that :-
63. Ag+ does not give white precipitate with :- (a) Zr and Zn have the same oxidation state
(a) potassium thiocyanate solution (b) Zr and Hf have about the same radius
(b) potassium chloride solution (c) Zr and Nb have similar oxidation state
(c) potassium ferrocyanide solution (d) Zr and Y have about the same radius
(d) potassium iodide solution 69. More positive the value of E°Mn+/M, :-
64. Which of the following statement is correct :- (a) greater is the stability of Mn+ ion in aqueous
medium
(a) the +1 oxidation state of Ni is not stable
(b) less is the stability of Mn+ ion in aqueous medium
(b) NiO is an amphoteric oxide
(c) greater is the stability of M ion in aqueous
(c) the most important ore is melachite medium
(d) silver is poor conductor of heat and electricity (d) None of these

www.jeebooks.in
7.8 THE d-BLOCK ELEMENTS AND SOME OF THEIR COMPOUNDS
70. Three separate test tubes contains aq. solution of (c) Potassium manganate disproportionates in
dichloride of a metal : basic medium depositingMnO2 and giving a
(I) test tube -1 
KOH(excess)
 No change purple solution of permanganate.
(II) test tube -2 
dil.H 2SO 4
 No change (d) Potassium manganate disproportionates in
(III) test tube -3 
NH 4 OH(excess)
 No change acidic medium
dichloride of metal is - 7. At least one gaseous product is formed for which
compound when reacts with MnO2 :-
(a) SnCl2 (b) FeCl2
(a) Concentrated HCl
(c) CuCl2 (d) ZnCl2
(b) Concentrated H2SO4
EXERCISE # II (c) Solid NH4Cl
 One or More Than One Correct Answer : (d) Oxalic acid in presence of H2SO4
1. When chromite ore is fused with excess of Na2O2 , 8. In 3d series , which element has positive electron
then:- gain enthalpy ?
(a) brown precipitate is formed (a) Cr (b) Fe
(b) Orange solution is observed (c) Mn (d) Zn
(c) Green colour precipitate is observed 9. Among all d block element :-
(d) Yellow solution is observed (a) Hg has lowest melting point
2. CrO 5 is stabilised in ether solution as the (b) W has highest melting point
coordination complex by :- (c) Fe has highest melting point
(a) Pyridine (b) Trimethyl amine (d) Zn has lowest melting point
(c) Aniline (d) Amyl alcohol 10. Which of the following statements are correct
regarding use of d block element ?
3. Ammonium-dichromate decompose violently on
heating giving - (a) Mo is used in X-ray tube
(b) Co is used for radiotherapy of cancerous
(a) N 2 (b) N 2O 3
tumour
(c) H2O (d) Cr2O3
(c) Pt is a used as catalyst for hydrogenation of
4. Which carboxylic acid can be oxidised by KMnO4 alkene
solution ? (d) Ni is used in nickel plating
(a) HCO2H (b) CH3CO2H 11. Cr2+ is reducing and Mn3+ is oxidising but both are
(c) (CO2H)2 (d) H2CO3 d4 configuration. It is because :-
5. Manganous sulphate is oxidised to permanganic (a) Cr2+ is itself oxidised to Cr3+ which has half filled
acid by treatment with :- t2g orbital
(a) PbO2 is HNO3 (b) Mn3+ is itself reduced to Mn2+ which has half
(b) Sodium bismuthate (NaBiO3) in HNO3 filled d5 configuration
(c) KIO4 is HNO3 (c) Cr2+ is itself oxidised to Cr3+ which has half filled
eg orbital
(d) Ammonium perdisulphate
(d) Mn3+ is itself reduced to Mn4+ which has half
6. What are the correct statement regarding potassium
filled eg orbital
manganate solution ?
12. Example of mixed oxide of transition elements
(a) Potassium mangnate solution is quite stable in
are :-
acidic medium
(a) Pb3O4 (b) Mn3O4
(b) Potassium mangnate solution is quite stable in
basic medium (c) Fe3O4 (d) Co 3O4

www.jeebooks.in
THE d-BLOCK ELEMENTS AND SOME OF THEIR COMPOUNDS 7.9
13. Among the following pair of ions, the lower 20. Which of the following options are correct ?
oxidation state in aqueous solution is more stable (a) CuSO4.5H2O is blue
than the other in :-
(b) Anhydrous CuSO4 is white.
(a) Cu+(aq.) ; Cu2+(aq.)
(c) [Cu(NH3)4]2+ is blue
(b) Mn2+ , Mn3+
(d) CuO is black
(c) Cr2+ ; Cr3+
21. Which reactions are spontaneous ?
(d) Tl+1 ; Tl3+
(a) 2AgNO3 + Cu  2Ag+Cu(NO3)2
14. The aqueous solution of the salt will be coloured.
The salt can be :- (b) CuSO4 + Zn  Cu + ZnSO4
(a) NaNO 3 (c) Fe2(SO4)3 + Cu  Fe + CuSO4
(b) ZnCl2 (d) Fe2(SO4)3 + Cu  2FeSO4 + CuSO4
(c) Cr2(SO4)3 22. Which of the following options are correct ?
(d) Cu(NO3)2 (a) CuO is black colour compound
15. Which of the following options are correct (b) CuO is formed by heating copper nitrate
regarding ionisation potentital of Ni and Pt :- Cu(NO3)2
(a) (IP1 + IP2)Ni < (IP1 + IP2)Pt (c) CuO is insoluble in NH3 solution
(b) (IP1 + IP2)Ni > (IP1 + IP2)Pt (d) CuO is soluble in concentrated NH3 solution
(c) (IP3 + IP4)Ni < (IP3 + IP4)Pt forming deep blue Schwetzer’s reagent.
(d) (IP3 + IP4)Ni > (IP3 + IP4)Pt 23. What can be possible complex formed by
16. Ammoniacal Cu2Cl2 absorbs :- combination between iron and carbon monooxide?
(a) CO 2 (b) CO (a) Fe(CO)5 (b) Fe2(CO)10
(c) C2H4 (d) C2H2 (c) Fe3(CO)12 (d) Fe2(CO)9
17. Mn-an element which forms oxide in +2 to +7 24. KI solution can be used to identify ?
oxidation state. Correct options is - (a) Hg2+ (b) Pb2+
(a) Oxide in +2 oxidation state is ionic (c) Ag+ (d) Cu2+
(b) Oxide in +7 oxidation state is covalent 25. Which of the following reactions give correct
(c) Oxide in +2 oxidation state is covalent product ?
(d) Oxide in +7 oxidation state is ionic (a) Zn + KOH K2ZnO2 + H2
18. Addition of nonmetals like B and C to the interstitial OH
sites of a transition metal results the metal :- (b) + FeSO4 + H2O2 
(a) of more ductability (c) Cu2+ + KICuI2 + K+
(b) of less ductability (d) Cr(OH)3 + NaOH + H2O2  Na2Cr2O7 +
(c) less malleable H2O
(d) more hardness 26. Which of the following pair has/have similar
19. Which of the following options are correct ? chemical properties?
(a) Hydrated Co2+ salt is pink. (a) Zirconium and hafnium
(b) Anhydrous Co2+ salt is blue. (b) Niobium and tantalum
(c) Hybridisation of CoCl2. 6H2O is sp3d2 (c) Cu and Zn
(d) Co2+ has coordination number + 8 (d) Fe and Cd

www.jeebooks.in
7.10 THE d-BLOCK ELEMENTS AND SOME OF THEIR COMPOUNDS
27. The complex of La where it shows a coordination 31. The correct statement about oxide of transition
number of more than 8 :- metal
(a) [La.EDTA.(H2O)4]3H2O (a) Oxide of a metal may be acidic and basic
(b) La2(SO4)3.9H2O (b) Ionic radius of M+2 in MO of 3d series decrease
(c) LaX2 from SC to V and then increase from
(d) Cu2[LaF8] V to Mn
27. Which of the following atomic number are the (c) FeO is a non stoichiometric compound
atomic numbers of the inner transition elements : (d) Mn2O7 is a coloured oxide due to d-d transition
29,59,74,95,102,104? 32. Cu2+ gives black precipitate with :-
(a) 59 (b) 95 (a) saturated solution of H2S
(c) 102 (d) 104 (b) potassium iodide solution
28. The true statements about Re–Re bonding in (c) potassium ferrocyanide solution
[Re2X8]2– are, (d) potassium thiocyanate solution
(a) Re–Re bond length is abnormally short 33. Green precipitate is formed when :-
(b) Re-Re bond is comprised of one , two  and (a) FeSO4 reacts with H2S
one -bond.
(b) Ni(NO3)2 reacts with H2S
(c) If Re-Re bond points along z-axis, then square
(c) FeSO4 reacts with NaOH
plannar ReX4 unit wil use s, px, py and d x 2  y2
(d) Ni(NO3)2 reacts with NaOH
orbitals for the formation of four Re-X  bonds.
34. Black precipitate is formed when :-
(d) dxy orbitals will overlap to form -bond.
(a) FeSO4 reacts with H2S
29. Which of the following is/are correct?
(b) Ni(NO3)2 reacts with H2S
(a) Of the d4 series, Cr+2 is strongly reducing while
(c) FeSO4 reacts with NaOH
Mn3+is strongly oxidising.
(d) Ni(NO3)2 reacts with NaOH
(b) Cobalt (II) is stable in aqueous solution but in
the presence of complexing reagents it easily 35. The oxo metal anions of the first series of the
oxidised. transitional metal exhibits the oxidation state equal
to its group number :-
(c) The d1 configuration is very unstable in ions.
(a) VO33– (b) VO43–
(d) Na 2 Cr 2 O 7 is preferred over K 2 Cr 2 O 7 in
volumetric analysis. (c) CrO42– (d) MnO4–
30. Choose the correct statments :- 36. Which of the following statements are correct :-
(a) First ionisation energy of Cu(29) is higher than (a) Thomas slag is used as fertiliser
thta of K (19) (b) CuO is amphoteric in nature
(b) Seond ionisation of Cu is lower than that of K (c) Cu2O is amphoteric in nature
(19) (d) Thomas slag is used as fuel
(c) The third ionisation energy of Cu is higher than 37. Most common oxidation state of Ce (Cerium)
that K are :-
(d) Third ionisation energy of Cu is lower than that (a) +3 (b) +4
of K
(c) +2 (d) +5

www.jeebooks.in
THE d-BLOCK ELEMENTS AND SOME OF THEIR COMPOUNDS 7.11
38. Which of the following statements are correct :- :- EXERCISE # III
(a) the compounds of lanthanoids are less basic  Linked Comprehension Type :
than those of actinoids
Passage for Q.1 to Q.3
(b) the highest oxidation state by lanthanoids is +7
CrO3 melts at 197°C to a dark red liquid and begins
(c) lanthanoids dissolve in warm water to liberate to decompose at 200°C giving off oxygen :
hydrogen 4CrO3 2Cr2O3 + 3O2
(d) the outer electronic configuration of tungsten is
The decomposition is complete at 420°. A solution of
of the type (n–1)d4ns2
CrO3 in glacial acetic acid is often used as an oxidising
39. Correct statements are :- reagent.
(a) the transition metals from a number of interstitial
1. CrO3 reacts with F2 at normal pressure at 150°C
compounds with hydrogen, carbon, nitrogen,
and 220°C respectively gives :-
boron etc.
(a) CrOF4 , CrO2F2 (b) CrO2F2 , CrOF4
(b) the fluorides of transition metals are ionic nature
(c) CrOF4 , CrF6 (d) CrO2F2 , CrF6
(c) ferric chloride exists as a dimer (Fe2Cl6) at
1023 K and as a monomer at 717 K 2. CrO3/H2SO4 usually taken in acetone is called
(d) the chemical composition is rust is Fe3O4.6H2O Jones reagent. Jones reagents oxidises :-
40. Dental amalgam used for filling teeth consists of :- (a) 1° alcohol to aldehyde RCH = O
(a) Ag (b) Sn (b) 2° alcohol (R2CHOH) to ketone R2C = O
(c) Au (d) Hg (c) 3° alcohol (R3C–OH) to alkene
41. Which of the following reagents are used to convert (d) can not oxidise alcohol
MnO2 to green melt of manganate salt :- 3. CrO3 is similar as SO3 like :-
(a) KOH + KNO3 (b) KOH + KClO3 (a) CrO3 is acidic oxide like SO3
(c) KOH + O2 (d) KOH + H2 (b) CrO3 when reacts with KOH, it gives K2CrO4
42. Which ion can undergo disproportionation :- like SO3 give K2SO4
(a) Hg22+ (b) Zn2+ (c) Crystal structure of CrO3 consists of infinite
(c) Cu+1 (d) Mn2+ chains of linked CrO4 tetrahedra, –SO3 has
43. Which can produce Riemann’s green with cobalt also similar infinite chains of linked SO4
nitrate solution :- tetrahedral.
(a) ZnO (b) ZnSO4 (d) All
(c) 3Zn(OH)3.ZnCO3 (d) Pb(OH)2.PbCO3 Passage for Q.4 to Q.6
44. Identify correct statement with copper sulphate :- When potassium permanganate is heated in test tube,
(a) CuSO4 + KI I2 gas (G1) is evolved and a black residue remains behind.
Upon extracting with a little water and filtering, a green
(b) CuSO4 + KCl Cl2

solution is obtained.
(c) CuSO4   CuO
4. The gas evolved is :-
(d) It’s tartarate complex reacts with NaOH and
(a) O 2 (b) O 3
glucose to give Cu2O
45. The aqueous solution of the salt will be coloured in (c) H2O (d) CO 2
the case of :- 5. Black residue is :-
(a) Zn(NO3)2 (b) LiNO3 (a) Mn2O3 (b) MnO
(c) Co(NO3)2 (d) CrCl3 (c) MnO2 (d) Mn2O7

www.jeebooks.in
7.12 THE d-BLOCK ELEMENTS AND SOME OF THEIR COMPOUNDS
6. Green solution is :- 12. Which element can show +2, +3, +4 oxidation
(a) K2Mn2O3 (b) K2MnO4 state :-
(c) MnO (d) MnSO4 (a) Fe (b) Co
Passage for Q.7 to Q.9 (c) Ni (d) All
KMnO4 when reacts with KI in acidic medium give Passage for Q.13 to Q.15
different products w.r.t. products formed when it reacts The d-block occupies the large middle secton flanked
with KI in basic medium. by s & p-block in the periodic table. The very name
7. In acidic medium KMnO4 oxidises KI to give P1 transition given to the elements of d-block is only
and it is itself converted into P2. P1 and P2 are becuase of their position between s & p-block
respectively :- elements. The d orbitals of the penultimate energy level
(a) I2 , Mn2+ (b) IO3– , Mn2+ in their atoms recieve electrons giving rise to the three
rooms of the transitional metals i.e., 3d, 4d and 5d.
(c) I2 , MnO2 (d) IO3– , MnO2
13. Which of the following elements are occupied in
8. In basic medium KMnO4 oxidises KI to give P1’
d-block but not considered as transition elements.
and it is itself converted into P2’. P1’ and P2’ are
respectively :- (a) Zn (b) Cd

(a) I2 , Mn2+ (b) IO3– , Mn2+ (c) Hg (d) All

(c) I2 , MnO2 (d) IO3– , MnO2 14. Re ; Os ; Ir ; Pt belongs to -

9. When KMnO4 reacts with sodium sulphite or (a) 3d series (b) 4d series
sodium thiosulphate :- (c) 5d series (d) 6d series
(a) MnO2 is precipitated 15. Mo ; Ru ; Rh ; Pd belongs to -
(b) Mn2+ is formed (a) 3d series (b) 4d series
(c) Green MnO42– ion formed (c) 5d series (d) 6d series
(d) None Passage for Q.16 to Q.18
Passage for Q.10 to Q.12 Nearly all the transition elements display typical metallic
One of the notable features of a transition element is properties such as high tensile strength, ductility,
the great variety of oxidation state it may show in its malleability, high thermal and electrical conductivity and
compounds. The element which give the greatest metallic lustre.
number of oxidation state occur in or near the middle The transition elements are very much hard and have
of the series. The maximum oxidation state of resonable low volatility. Their melting and boiling point are high.
stability correspond in value to the sum of the s and d 16. Correct melting point order of 1st series of
electrons upto manganese. transition elements -
10. Lowest oxidation state +1 is exhibited by which of (a) Mn > Cr > V > Ti > Cu
the transition elements :- (b) Cr > Mn > V > Ti > Cu
(a) Mn (b) Fe (c) Cr > V > Ti > Mn > Cu
(c) Co (d) Cu (d) Mn > Fe > Co > Pd > Cu
11. +6 oxidation states are exhibited by which of the 17. The lowest melting point of 2nd series of transition
transition elements :- elements is observed for -
(a) Cr (b) Mn (a) Ru (b) Rh
(c) Fe (d) All (c) Nb (d) Ag

www.jeebooks.in
THE d-BLOCK ELEMENTS AND SOME OF THEIR COMPOUNDS 7.13
18. The structure of Hg :- 25. A is :-
(a) Body centered cubic structure (a) Pb (b) Hg
(b) Hexagonal close packed structure (c) Ag (d) Zn
(c) Cubic close packed structure 26. When B reacts with hypo solution, what will be
(d) A typical metal structure observation :-
Passage for Q.19 to Q.20 (a) white precipitate is formed which on boiling
X (Green colour carbonate salt)  turns reddish brown

  Y (Black
residue) + CO2 + H2O : (b) white precipitate is formed which on boiling
turns yellow
Y (Black residue) 

 Z (Reddish brown residue)
(c) white precipitate is formed which on boiling
+ W gas
turns black
19. X can be :-
(d) none
(a) CuCO3.Cu(OH)2 (b) CuCO3.2Cu(OH)2
27. Cu2+ ion can be reduced to Cu+1 by addition of an
(c) both (a) and (b) (d) FeCO3
aqueous solution of :-
20. Y can be :-
(a) KF (b) KCl
(a) Cu2O (b) FeO
(c) KI (d) KOH
(c) CuO (d) Fe2O3
Passage for Q.28 to Q.29
21. Z can be :-
A brown black solid (A) on fusion with KNO3 is KOH
(a) Cu2O (b) FeO
gives a green compound B and a colourless compound
(c) CuO (d) Fe2O3 C. Green compound B in aquous solution on
Passage for Q.22 to Q.24 electrolytic oxidation gives a violet compound D. KOH
An orange solid X is a very important chemical used in and H2 gas. An aquous solution (D) in cold reacts with
leather industry and an oxidant for preparation of many ethylene to give glycol and its pink solution is discharged
azo compound. When X is heated, it gives yellow due to precipitation of A:-
coloured B and green coloured C. Also during heating 28. A and B are :-
a colourless, odourless gas D is evolved :-
(a) MnO2, K2MnO4 (b) Cr2O3, K2CrO4
22. Compound A is
(c) MnO2, KMnO4 (d) Cr2O3, K2Cr2O7
(a) KMnO4 (b) K2Cr2O7
29. C and D are :-
(c) K2CrO 4 (d) Cr2O3
(a) NO, K2MnO4 (b)KNO2, K2MnO4
23. Compound C is obtained on heating of
(c) NO, KMnO4 (d) NO, K2Cr2O7
(a) (NH4)2Cr2O7 (b) NH4ClO4
(c) NH4NO3 (d) None of these Passage for Q.30 to Q.32
24. Gas D is :- An aqueous solution of a salt upon analysis gives the
following results :
(a) O 2 (b) NO 2
(i) It gives a white precipitate with BaCl2 solution
(c) N 2 (d) H2
insoluble in water.
Passage for Q.25 to Q.27
(ii) Addition of excess KI gives a brown precipitate
A certain metal A is boiled in dilute HNO3 to give
which turns starch-iodide complex blue black.
salt(B) and an oxide of nitrogen(C). Aqueous solution
B with brine solution gives a precipitate D which is (iii) It gives a chocalate brown coloured precipitate
soluble in NH4OH. with potassium ferrocyanide solution.

www.jeebooks.in
7.14 THE d-BLOCK ELEMENTS AND SOME OF THEIR COMPOUNDS
30. What interference can be drawn from (i) above ? 37. Hypo solution is used in photography to dissolve
(a) Presence of SO42– ion (a) Undissociated silver halide to give complex
(b) Presence of Pb2+ ion [Ag(S2O3)2]3–
(c) Presence of Zn2+ ion (b) Undissociated silver halide to give complex
(d) Presence of SO32– [Ag(S2O3)]–1
31. What is the formula of chocolate brown (c) Metallic silver to give complex [Ag(S2O3)2]3–
precipitate:- (d) Metallic silver to give complex [Ag(S2O3)]–1
(a) Cu[Fe(CN)4] (b) Cu2[Fe(CN)6] 38. The developer used in photography is
(c) Cu3[Fe(CN)6]2 (d) Cu2I2 (a) K2[FeII(CO3)2] (b) K2[FeII(C2O4)2]
32. From the above inference, the formula of salt is :- (c) K[FeIII(C2O4)2] (d) K[FeIII(CO3)2]
(a) CuSO4 (b) CuSO3 Matrix Match Type :
(c) CuCl2 (d) Cu(OH)2
39. Match the column :-
Passage for Q.33 to Q.35
Column - I Column - II
HNO3 KI or K2CrO4
[A] [B] + [C] Yellow Ion Colour
Scarlet red Black brown Colorless precipitate
salt residue salt (A) CrO42– (P) Purple/Pink solution
2–
HNO3 (B) Cr2O7 (Q) Green solution
H2S
Mn(NO3)2 (C) MnO42– (R) Yellow
(D) MnO4– (S) Orange
[E] [D]
Pink Black ppt. 40. Match the column :-
solution dissolves in HNO3
Column - I Column - II
33. The compound [A] is
Compound Colour
(a) An oxide (b) A chromate
(A) Cr2O3 (P) Green colour
(c) A maganate (d) Chloride
(B) CrO3 (Q) Red colour
34. The correct formula of compound (E) is
(C) CrO5 (R) Intense blue colour
(a) KMnO4 (b) H2MnO4
(c) HMnO4 (d) K2MnO4 (D) CrO2 (S) Black colour
35. The yellow ppt. formed by reaction of [C] with 41. Match the column :-
K2CrO4 is also given by (other than that in C) Column - I Column - II
(a) Pb2+ (b) Ca2+ Compound Feature
(c) Ba2+ (d) Na+ (A) Mn2O7 (P) Black colour ; most
Passage for Q.36 to Q.38 important ore of Mn
The halides of silver are sensitive to light. This concept (B) MnO2 (Q) Dark green liquid as
is utilised in photography. explosive
36. A photographic plate consist of - (C) MnO (R) Red oxide of manganese
(a) Sensitive emulsion of fine particles of AgF (D) Mn3O4 (S) Olive green powder,
(b) Sensitive emulsion of fine particles of AgCl formed by decomposition
(c) Sensitive emulsion of fine particles of AgBr by MnCO3
(d) Sensitive emulsion of fine particles of AgI (T) Anhydride of HMnO4

www.jeebooks.in
THE d-BLOCK ELEMENTS AND SOME OF THEIR COMPOUNDS 7.15
42. Match the column :- 45. Match the Coloumn
Column - I Column - II Column - I Column - II
Elements Structure type Complex Transitional elements
(A) W (P) Transition elements (A) [M(CN)2]–x (P) x = 1; Ag, Au
(B) Co (Q) Hexagonal close packed (B) [M(CN)4]–x (Q) x = 3; Mn, Fe,Co
(C) Zn (R) Body centered cubic (C) [M(CN)4]–x (R) x = 2; Zn,Cd, Hg
(D) Cd (S) Cubic close packed (D) [M(CN)6] –x
(S) x = 3; Cu only
(T) Typical metal structure 46. Column - I
43. Match the column :- Reaction
Column - I (A) Fe +2HCl 
Conversion (B) Fe +Conc. H2SO4 
2– –
(A) MnO4 MnO4 (C) Fe +HNO3(warm & concentrated) 
2– 2–
(B) CrO4  Cr2O7 (D) Fe +HNO3 (cold & dilute) 
(C) CrO72–  CrO42– Column - II
(D) MnO4– MnO42– Product nature
Column - II
(P) ferrous salt
Feature
(Q) ferric salt
(P) Oxidation state remain unchanged during
(R) colourless, odourless gas
conversion
(S) colourless, paramagnetic gas
(Q) pH < 7, conversion is carried out
(T) Pungent odour gas
(R) pH > 7, conversion is carried out
47. Match the column :-
(S) Oxidation state changes during conversion
Column - I
(T) Electrolytic oxidation in alkaline medium
Reaction
44. Match the column :-
(A) N2H4 + CuO 
Column - I
(B) Quinol + AgBr(s) 
Property

(A) Least Ist ionisation enthalpy (C) FeSO 4 (s)  
(B) Highest Ist ionisation enthalpy (D) CaCl2(s) + K2Cr2O7(s) + H2SO4(conc.)
(C) Highest 2nd ionisation enthalpy
Column - II
(D) Least ionic radii in M2+ ion
Feature
Column - II
(P) Reddish brown vapour is formed
Transitional element
(Q) One of the product is +6 oxidation state
(P) Sc
(R) Redox reaction
(Q) Zn
(S) One of the product is acidic oxide
(R) Cu
(T) Photograpy
(S) Ni

www.jeebooks.in
7.16 THE d-BLOCK ELEMENTS AND SOME OF THEIR COMPOUNDS
48. Match the column :- 51. Match the column :-
Column - I Column - I Column - II
Reaction (A) FeCl3 (P) Oxidising agent and as
mordant in dyeing
(A) Turnbull’s Blue pigment
(B) FeF3 (Q) Exists as Fe3+
(B) Prussian Blue pigment
(C) FeBr3 (R) d5 ion
(C) Brown ring
(D) FeO42– (S) Stable in strongly alkaline
(D) Na4[Fe(CN)5NOS] aqueous medium
Column - II (T) Gives prusian blue colour
Feature with K4[Fe(CN)6]
(P) Oxidation state of iron in coordination sphere 52. Match the column :-
Fe3[Fe(CN)6]2 is +1 Column - I Column - II
(Q) Oxidation state of iron in coordination sphere (A) Fe3+ (P) Precipitate with NH4OH
2+
Fe4[Fe(CN)6]3 is +2 (B) Fe (Q) Precipitate with NaOH
2+
(R) Oxidation state of iron in coordination sphere (C) Cu (R) Complex with KCN (excess)
[Fe(H2O)5NO]SO4 is +3 (D) Ag+ (S) Complex with hypo solution
(S) Iron in anionic part of complex (T) Hydrometallurgical extraction
(T) Iron in cationic part of complex 53. Match the column :-
49. Match the column :- Column - I
(A) Pb2+ gives yellow precipitate with
Column - I Column - II
(B) Bi3+ gives black precipitate with
(A) Cr < W (P) stability of the complex
in +2 oxidation state (C) Hg2Cl2 gives black precipitate with
(D) CuSO4 gives deep blue colour with
(B) Mn < Fe (Q) melting point
Column - II
(C) Zn < Cu (R) magnetic moment
(P) KI
(D) Cd < Hg (S) tendency to form metal
(Q) K2CrO 4
- metal bond
(R) NH4OH
(T) ionisation energy
(S) Na2SnO2 in alkaline medium
50. Match the column :-
54. Match the column :-
Column - I Column - II
Column - I Column - II
(A) Fe2+ > Fe3+ (P) Crystal field splitting
(A) Cu2+ (P) Form amphoteric oxide
energy with strong 2+
(B) Zn (Q) Diamagnetic and colourless
ligand
compounds
(B) Pt2+ > Ni2+ (Q) Complexing tendency 3+
(C) Cr (R) Form complex with NH3
with dipyridyl
(D) Sc3+ (S) Form colourless tetrahedral
(C) Mn2+ > Cr2+ (R) Tendency to form complex with KCN
complex with CO
(T) Form inner orbital
(D) Cu2+ > Cu+ (S) Magnetic moment octahedral complex with
(T) ionic radius NH3

www.jeebooks.in
THE d-BLOCK ELEMENTS AND SOME OF THEIR COMPOUNDS 7.17
55. Match the column 8. Find out the number of ions in solution which are
Column - I colourless :-
(A) Hg2Cl2 
NH3OH
 Product/is
Ti4+ ; Ti3+ ; Cu+ ; Zn2+
(B) Hg2Cl + SnCl2 Product/ is 9. The oxidation state of Mn in the product of oxidative
fusion of pyrolusite ore in basic medium :-
(C) Hg+2 + SnCl2Product/ is
10. The oxidation state of Cr in the product of oxidative
(D) HgCl2 + Cu Product/ is
fusion of chromite ore in basic medium :-
Column - II
11. TiCl4 is example of colourless liquid at room
(P) Hg
temperature.If it reacts with Zn followed by
(Q) Hg(NH2) Cl
addition of water molecule give purple colour
(R) Oxidation compound X. What is the number of unpaired
(S) Metal-metal bond in product electron present in X.
(T) Reduction 12. A metal A which is strongly attracted by a magnet
is attacked slowly by HCl liberating a gas and
EXERCISE # IV producing a blue solution. The addition of water
 Integer Type : of this solution causes it to burn pink. What is the
1. At what pH chromate-dichromate changes :- atomic number of A. A belong to 3d series
2. Find out the n factor of Cr2O72– in acidic medium? element:-
3. MnO4– when oxidises other compounds in acidic 13. Some transitional metals are used as traditional
medium, it is itself converted into Mn2+. The coinage metals in ancient world. Find out number
molecular weight of MnO4– is x times of its of coinage metals.
equivalent weight. The value of x :- 14. When 0.25 mole of hydrated ferric chloride
4. Find out number of electrons present in 4f orbital (FeCl3.6H2O) is made anhydrous by reacting with
in Gadolinium. (At. no. 64) ? 1.5 mole of 2,2-dimethoxy propane :-
5. Find out number of electrons present in 5d orbital 15. What is the number of moles of SnCl2 required for
in Ytterbium (At. no. 70) ? reduction of 1 mole of K2Cr2O7 into Cr3+:-
6. Find out number of electrons present in 6s orbital 16. Peacock ore is CuxFeS4. The value of x ?
in leutetium (At. no. 71) ? 17. The highest oxidation state exhibited among all
7. How many statements are correct ? transitional element :-
(a) Permanganate titration in presence of 18. How many statements are correct :-
hydrochloric acid is unsatisfactory (a) Metre scales are made up of invar alloy
(b) Mn2+ compounds are more stable than Fe2+ (b) Rusting iron is due to formation of Fe2O3 +
toward oxidation to their +3 state Fe(OH)3
(c) Ammoniacal AgNO3 converts glucose to (c) Transitional elements in positive oxidation state
gluconic acid and metallic silver is precipitated can act as Lewis acid.
(d) Titanium and copper both in the first series of (d) Bordaux mixture is used as fungicide. It is a
transition metals exhibits +1 oxidation stae most mixture of CuSO4 + Ca(OH)2
frequently.
(e) Ce4+ (cerric ion) is used as an oxidising reagent
(e) Anhydrous ferric chloride is formed by heating
in volumetric analysis
metallic iron in a stream of dry chlorine gas.

www.jeebooks.in
7.18 THE d-BLOCK ELEMENTS AND SOME OF THEIR COMPOUNDS
19. How many statements are incorrect :- 29. Rust consist of hydrated iron oxide. What is the
(a) Acidified KMnO4 can be decolorised by oxidation state of iron in rust.
Fe2(SO4)3 30. P 1 (imparts violet colour in flame
comp. P
(b) KMnO4 does not act as oxidising reagents in test)  2
conc.H 2SO 4
 P3(reddish brown gas)
alkaline medium NaOH AgNO3
   P4(red ppt.)
(c) Among ZnCl2 , CdCl2 , HgCl2 maximum NH3 solution
P4 (red ppt.)   P5 (soluble)
excess
covalent character is shown byt ZnCl2
HCl solution
(d) Promethium is naturally found element P4 (red ppt.)   P6 (white precipitate)
NaOH solution
20. Number of moles of CrO3 required to oxidise 9 P2  P7 (gives white fumes with HCl)
moles of 2 butanol ? How many products are correctly mentioned -
21. An ornamental gold has 75% gold. It is x carat. P1 = K2Cr2O7 P4 = Ag2CrO4 P7 = NH3
The value of x :- P2 = NH4Cl P5 = [Ag(NH3)2]+1
22. What is the basicity of Perrhenic acid :- P3 = CrO2Cl2 P6 = AgCl
23. A metal ion M2+ gives cherry blood red colour with EXERCISE # V(A) JEE-MAIN
DMG and complex formed is diamagnetic in
1. Which of the following arrangements does not
nature. Which group of qualitative analysis metal
represent the correct order of the property stated
will belong - against it ? [AIEEE 13]
24. CuSO4 + KCN  Complex of Cu. The spin only (1) V2+ < Cr 2+ < Mn 2 < Fe 2+ : paramagnetic
(excess)
behaviour
magnetic moment of the complex.
(2) Ni2+ < Co2+< Fe2+ < Mn2+ : ionic size
25. How many reagents can be used to distinguish
(3) Co3+ < Fe3+< Cr3+ < Sc3+ : stability in aqueous
ferrous salt and ferric salt. :-
solution
NH4OH; NaOH; K4[Fe(CN)6]; K3[Fe(CN)6] (4) Sc < Ti < Cr < Mn : number of oxidation states
26. A metal ion M3+ gives brown red colour with 2. Consider the following reacton : [AIEEE 13]
K3[Fe(CN)6]. With which group it may belong? – 2– +
xMnO4 + yCr2O4 + zH 
27. S2O3–2 + CN– X + SO3–2 
Fe3
 Blood red z
NaF
colour   colourless complex xMn2+ + 2yCO2 + H2O 
excess 2
The coordination number of Fe3+ in colourless The values of x, f and z in the reaction are
complex - respectively :-
28. The general electronic configuration of trasition (a) 5, 2 and 16
element is (n–1)dx–y n sz-w (b) 2, 5 and 8
x = minimum number of electron present in d- (c) 2, 5 and 16
orbital
(d) 5, 2 and 8
y = maximum number of electron present in d-
3. Arrange Ce3+, La3+, Pm3+ and Yb3+ in increasing
orbital
order of their ionic radius :- [AIEEE 02]
z = minimum number of electron present in s- 3+ 3+ 3+ 3+
orbital (a) Yb < Pm < Ce < La
w = maximum number of electron present in s- (b) Ce3+ < Yb3+ < Pm3+ < La3+
orbital (c) Yb3+ < Pm3+ < La3+ < Ce3+
Find out the value of y – z :- (d) Pm3+ < La3+ < Ce3+ < Yb3+

www.jeebooks.in
THE d-BLOCK ELEMENTS AND SOME OF THEIR COMPOUNDS 7.19
3+
4. The radius of La is 1.06 Å, which of the following 10. The lanthanoid contraction is responsible for the
given values will be closest to the radius of Lu3+ fact that :- [AIEEE 05]
(At. no. of Lu = 71, La = 57) [AIEEE 03] (a) Zr and Y have about the same radius
(a) 1.6Å (b) 1.4Å (b) Zr and Nb have similar oxidation state
(c) 1.06Å (d) 0.85Å (c) Zr and Hf have about the same radius
5. The atomic number of vanadium (V), chromium (d) Zr and Zn have similar oxidation state
(Cr), maganese (Mn) and iron (Fe) are 23, 24, 25
11. Lanthanoid contarction is caused due to :-
and 26 respectively. Which one of these may be
expected to have the highest second ionisation [AIEEE 06]
entjhalpy - [AIEEE 03] (a) the same effective nuclear charge from Ce to Lu
(a) V (b) Cr (b) the imperfect shielding on outer electrons by 4f
(c) Mn (d) Fe electrons from the nuclear charge
6. What would happen a solution of potassium (c) the appreciable shielding on outer electrons by
chromate is treated with an excess of dilute nitric 4f electrons from the nuclear charge
acid :- [AIEEE 03] (d) the appreciable shielding on outer electrons by
3+ 2–
(a) Cr and Cr2O7 are formed 5d electrons from the nuclear charge
(b) Cr2O72– and H2O are formed 12. The actinoids exhibits more number of oxidation
(c) Cr2O72– is reduced to +3 state of Cr states in general thatn the lanthanoids.
(d) Cr2O72– is oxidised to +7 state of Cr This becuase :- [AIEEE 07]
7. Among the following series of transition metal ions (a) the 5f-orbitals are more buried than the 4f-orbitals
are ore where all metal ions have 3d2 electronic (b) there is a similarly between 4f -and-5f in the
configuration is :- [AIEEE - 04] their angular part of the wave function
+3 2+ 3+ 4+
(a) Ti , V , Cr , Mn (c) the actinoids are more reactive than the lanthanoids
(b) Ti+, V4+, Cr6+, Mn7+ (d) the 5f-orbitals extend further from the nucleus
(c) Ti4+, V4+, Cr2+, Mn3+ than the 4f-orbitals
(d) Ti2+, V3+, Cr4+, Mn+5 13. Identify the incorrect statement among the
8. Cerium (Z = 58) is an important member of the following :- [AIEEE 07]
lanthanoids. Which of the following statements (a) d-block elements show irregular and erractic
about cersium is incorrec :- [AIEEE 04]
chemical properties among themselves
(a) Cerium (IV) acts as an oxidising agent
(b) La and Lu have partially filled d-orbitals and
(b) The +3 oxidation state of cerium is more stable
no other partially filled orbitals
than the +4 oxidation state
(c) the chemistry of various lanthanoids is very similar
(c) The +4 oxidation state of cerium is not known
in solutions (d) 4f and 5f - orbitals are equally shielded
(d) The common oxidation states of cerium are +3 14. In context with the transition elements, which of the
and +4 following statements is incorrec :- [AIEEE 09]
9. The aqueous solution containing which one of the (a) In the highest oxidation states of the first five
following ions will be colourless ? [AIEEE 05] transition elements (Sc to Mn), all the 4s and
(Atomic number : Sc = 21, Fe = 26, Ti = 22, 3d electrons are used for bonding
Mn = 25) (b) Once the d5 configuration is exceeded, the
(a) Sc3+ (b) Fe2+ tendency to involve all the 3d electrons in
(c) Ti3+ (d) Mn2+ bonding decreases.

www.jeebooks.in
7.20 THE d-BLOCK ELEMENTS AND SOME OF THEIR COMPOUNDS
(c) In addition to the normal oxidation states, the Cl2 ,heat
(a) Fe    FeCl3 
heat ,air

zero oxidation state is also shown by these O 2 , heat CO,600 C
(b) Fe   Fe3O4   FeO
elements in complexes. CO,700 C
  Fe
(d) In the highest oxidation state, the transition H 2SO 4 ,O 2
dil.H 2SO 4
metal show basic character and form cationic (c) Fe   FeSO4   Fe2(SO4)2
Heat
complexes.   Fe
O 2 , heat dil.H 2SO 4
15. Knowing that the chemistry of lanthanoids (Ln) is (d) Fe   FeO   FeSO4
dominated by its +3 oxidation state, which of the Heat
  Fe
following statements is incorrect ? [AIEEE 09] 19. The equation which is balanced and represents the
(a) Ln(III) compounds are generally colourless correct product(s) is :- [Jee-Mains 2014]
(b) Ln(III) hydroxides are mainly basic in character (a) [Mg(H 2 O) 6 ] 2+ + (EDTA) 4– 
excess NaOH

(c) Because of the large size of the Ln(III) ions the [Mg(EDTA)]2+ + 6H2O
bonding in its compounds is predominently (b) CuSO 4 + 4KCN  K 2 [Cu(CN) 4 ] +
ionic in character K2SO 4
(d) The ionic sizes of Ln(III) decrease in general (c) Li2O + 2KCl 2LiCl + K2O
with increasing atomic number
(d) [CoCl(NH3)5]+ + 5H+ Co2+ + 5 NH4+ +
16. The correct order of E°M2+/M values with negative Cl–
sign for the four successive elements
Cr, Mn, Fe and Co is :- [AIEEE 2010] EXERCISE # V(B) JEE-ADVANCED
(a) Cr > Mn > Fe > Co 1. Anhydrous ferric chloride is prepared by :
(b) Mn > Cr > Fe > Co [JEE-2002]
(c) Cr > Fe > Mn > Co (a) heating hydrated ferric chloride at a high tem-
(d) Fe > Mn > Cr > Co perature in a stream of air
17. In content of the lanthanoids, which of the following (b) heating metallic iron in a stream of dry chlorine
stateents is not correct :- [AIEEE 2011] gas
(a) because of similar properties the separation of (c) reaction of ferric oxide with HCl
lanthanoids is not easy (d) reaction of metallic iron with HCl
(b) availability of 4f electrons results in the formation 2. When MnO2 is fused with KOH, a coloured com-
of compounds in +4 state for all the members pound is formed, the product and its colour is :
of the series
[JEE-2003]
(c) there is a gradual decrease in the radii of the
(a) K2MnO4 , green
members with increasing atomic number in the
series (b) KMnO4 , purple
(d) all the members exhibit +3 oxidation state (c) Mn2O3 , brown
18. Which series of reactions correctly represents (d) Mn3O4 , black
chemical relations related to iron and its 
compound ? [Jee-Mains 2014]

www.jeebooks.in
HYDROGEN AND THE HYDRIDE
PROB L EM S B ASED ON GI VEN T OPI CS  Metallic (or interstitial) hadrides
 Electronic structure  Intermediate hydrides
 Position in the periodic table  The hydrogen ion
 Abundance of hydrogen  Acids and Bases
 Preparation of hydrogen  Arrhenius theory
 Properties of molecular hydrogen  Acid and bases in proton solvens
 Isotopes of hydrogen
 Bronsted - Lowry theory
 Ortho and para hydroge
 Lewis theory
 Hydrides
 The lux-flood definition
 Ionic or salf - like hydrides
 Covalent hydrides

www.jeebooks.in
CHAPTER
8
Hydrogen and The Hydride

EXERCISE # I

 Only one correct answer : 7. Heavy water (D2O) is used :-


1. An unreactive diatomic gas A(1 mole) was treated (a) For preparation of polymer
with 3 moles of H2 in presence of catalyst to give (b) For preparation of hydro carbon
another gas (b) which is basic in nature. Gas (b) (c) In atomic reactor to carry out nuclear fission
on further oxidation in moist condition gives a
(d) None
compound (c) which is part of acid rain. Gas (a)
and gas (b) are respectively. 8. An ionic hydride of an alkali metal has significant
covalent charater and is almost unreactive
(a) Cl2 and HCl (b) Br2 and HBr
towareds oxygen and chlorine. It is used to prepare
(c) P2 and PH3 (d) N2 and NH3 the synthesis of other useful hydrides which acts
2. The radioactive isotopes of hydrogen is :- as reducing agent. The hydries is -
(a) Tritium (b) Deuterium (a) LiH (b) BeH2
(c) Protium (d) Hydronium (c) CaH2 (d) KH
3. Electron precise hydrides are formed by 9. H2O + NH2– 
 NH3 + OH– ...(i)
combination of hydrogen with all :-  CH3CO2– + H3O+ ...(ii)
H2O + CH3CO2H 
(a) Elements of group 13 In reation (i) and (ii) water acts as :-
(b) Elements of group 14 (a) acid & acid respectively
(c) Elements of group 15 (b) acid & base respectively
(d) Elements of group 16 (c) base & acid respectively
4. Elements of group 15 form :- (d) base & base respectively
(a) Elements defficient hydrides
10. Electron precise hydrides are :-
(b) Elements rich hydrides
(a) Pyramidal (b) Trigonal planar
(c) Elements precise hydrides
(c) Tetrahedral (d) Bent
(d) None
11. H+ in aqueous solution exists as :-
–1
5. Ionisation energy of hydrogen in kJ mol is :-
(a) OH– (b) H-O-O –
(a) 520 (b) 495
(c) H3O+ (d) H3O2+
(c) 1681 (d) 1312
12. When Na2O2 is treated with dilute H2SO4 , the
6. Hydrolith is :- products obtained are :-
(a) CaH2 (b) BaH2 (a) Na2O2 & H2O2 (b) Na2SO4 & O2
(c) SrH2 (d) BeH2 (c) Na2SO4 & H2 (d) Na2SO4 & H2O2

www.jeebooks.in
8.4 HYDROGEN AND THE HYDRIDE
13. Hydrogen combines with other elements by :- 22. Which of the following can not be oxidised by
(a) Loosing an electron only H2O2 :-
(b) Gaining an electron only (a) PbS (b) KI + HCl
(c) Sharing an electron only (c) Na2SO3 (d) O 3
(d) Loosing, gaining and sharing electron 23. Deionised water is obtained by passing hard water
through :-
14. Hydrogen is :-
(a) Cation exchanger
(a) Colourless and odourless gas
(b) anion exchanger
(b) Colourless and pungent odour gas
(c) Both cation and anion exchanger
(c) Coloured and odourless gas
(d) Permutit process
(d) Coloured and pungent odour gas
24. Elements of which of the following groups in
15. Heavy water is :- periodic table do not form compound with
(a) H2O18 hydrogen :-
(b) D2O (a) Group 7,8,9 (b) Group 13
(c) Water with specific gravity 1 (c) Group 15,16,17 (d) Group 14
(d) Water at 0°C 25. The correct order of ionic character among all
16. Heavy water freezes as :- alkaline earth hydride :-
(a) BeH2 < CaH2 < MgH2 < BaH2 < SrH2
(a) 0°C (b) –3.8°C
(b) BeH2 < CaH2 < MgH2 < SrH2 < BaH2
(c) 3.8°C (d) –5°C
(c) BeH2 < CaH2 < BaH2 < MgH2 < SrH2
17. Electrolysis of molten ionic hydride gives :-
(d) SrH2 < CaH2 < MgH2 < BaH2 < BeH2
(a) Hydrogen at cathode
26. Boiling point of H2O2 is more than H2O. It is due
(b) Hydrogen at anode
to :-
(c) Metal at anode (a) more intermolecular hydrogen bonding in H2O2
(d) No reaction w.r.t. H2O
18. Which of the following represent a pair of covalent (b) more intramolecular hydrogen bonding in H2O2
hydrides :- w.r.t. H2O
(a) TiH1.5–1.8 ; VH0.56 (b) H2S ; HF (c) less polarity in H2O2 w.r.t. H2O
(c) KH ; NaH (d) SiH4 ; RbH (d) higher specific heat in H2O2 w.r.t. H2O
19. Which of the following represent a pair of interstitial 27. Which can not be used to prepare H2O2 , when
hydrides :- BaO2 reacts with:-
(a) TiH1.5–1.8 ; VH0.56 (b) H2S ; HF (a) conc. H2SO4 (b) dilute H2SO4
(c) KH ; NaH (d) SiH4 ; RbH (c) H3PO4 (d) CO2 in H2O
20. Which has least boiling point :- 28. Marck perhydrol is the trade name of which
chemical species ?
(a) Liquid CO2 (b) Liquid O2
(a) H2 (b) H2O
(c) Liquid H2 (d) Liquid N2
(c) H2O2 (d) O 2
21. Hydrogen can not reduced :-
29. Which of the following can be soluble soag ?
(a) Heated CuO (b) Heated Fe2O3
(a) C17H35CO2Na (b) (C17H35CO2)2Mg
(c) Heated SnO2 (d) Heated Al2O3
(c) (C17H35CO2)2Ca (d) C17H35CO2Li

www.jeebooks.in
HYDROGEN AND THE HYDRIDE 8.5
30. Temporary hardness of water is caused by the 35. Which one of the following processes will produce
presence of :- permanent hard water :-
(a) insoluble bicarbonates of calcium , magnesium (a) addition of Na2SO4 to water
and iron in water (b) saturation of water with CaCO3
(b) soluble bicarbonates of calcium , magnesium
(c) saturation of water with MgCO3
and iron in water
(d) saturation of water with CaSO4
(c) insoluble carbonates of calcium , magnesium
and iron in water 36. The reagent commonly used to determine hardness
(d) soluble carbonates of calcium , magnesium and of water titrimetrically is :-
iron in water (a) oxalic acid
31. A clear transparent liquid is taken in a glass. Which (b) disodium salt of EDTA
of the following will provide clue whether it is water (c) sodium citrate
or not
(d) sodium thiosulphate
(a) Adding a litmus paper
37. In context with the industrial preparation of
(b) Adding few drops of the liquid over anhydrous hydrogen form water gas (CO+ H2), which of the
copper sulphate
following is the correct statement :-
(c) Adding few drops of the liquid over anhydrous
(a) CO is oxidised to CO2 with steam in the
Sodium chloride
presence of a catalyst followed by absorption
(d) By testing its smell of CO2 in alkali
32.
(b) CO and H2 are fractionally separated using
differences in their densities
(c) CO is removed by absorption in aqueous
A (parallel spin B (opposed spin
Cu2Cl2 solution
of protons) of protons)
(a) A is called ortho hydrogen and B is called para (d) H2 is removed through occlusion with Pd
hydrogen 38. H2 (Ni) can not reduce :-
(b) A is called para hydrogen and B is called ortho (a) RCN (b) RCHO
hydrogen (c) RCO2H (d) R–CH=CH–R
(c) A is called ortho hydrogen and B is called meta 39. Organic higly branched alkyl groups and higly polar
hydrogen inorganic groups are :-
(d) A is called para hydrogen and B is called meta
(a) hydrophobic and hydrophilic respectively
hydrogen
(b) hydrophilic and hydrophobic respectively
33. When zeolite, which is hydrated sodium aluminium
silicate is treated with hard water, the sodium ion (c) hydrophilic only
are exchanged with :- (d) hydrophobic only
(a) H+ ions (b) Ca2+ ions 40. Roshan heard that instructions were given to
(c) SO42– ions (d) OH– laboratory attendent to store a particular chemical
34. Polyphosphates are used as water softening agnets in dark room. It must be kept away from the dust.
because they :- This chemical is important for use in the pollution
(a) form soluble complexes with anionic species control treatment of domestic and industrial
effluents. The chemical is
(b) percipitate anionic species
(c) form soluble complexes with cationic species (a) O 3 (b) H2O2
(d) percipitate cationic species (c) H2O (d) H2

www.jeebooks.in
8.6 HYDROGEN AND THE HYDRIDE
41. Which of the following statement is incorrect :- EXERCISE # II
(a) Hydrogen peroxide is stored in wax lined
bottles  One or More Than One Correct Answer :
(b) Hard water does not form lather with soap 1. Elements of group 13 form :-
(c) Tritium is a strategic material for thermonuclear (a) Hydrides which are electron defficient
weapons (Hydrogen bomb) (b) Hydrides which are electron rich
(d) None is incorrect (c) Hydrides which have incomplete octet
42. Which of the following reaction is an examples of (d) Hydrides which have complete octet
used of water gas in the synthesis of other 2. What are the characteristic features of ionic
compounds? hydrides :-
1270K
(a) CH4(g) + H2O(g)   CO(g) + H2(g)
( Ni)
(a) ionic hydrides do not conduct electricity in solid
673K
state
(b) CO(g) + H2O(g)   CO2(g) + H2(g)
(Catalyst) (b) ionic hydrides are very good conductors of
1270K
(c) CnH2n + 2 + nH2O(g)   nCO + (2n + 1)H2
( Ni)
electricity in solid state
Cobalt (c) ionic hydrides conduct electricity in aqueous
(d) CO(g) + 2H2(g)   CH3OH(l)
(Catalyst) solution
43. A mixture of two gases AsH3 and SbH3 is passed (d) metallic hydrides are example of ionic hydrides
through a tube heated at the middle portion. It is
3. Hydrogen is released by the action of cold dilute
called Marsh’s Test. What is the observation and
HNO3 on :-
why?
(a) Fe (b) Mn
(a) Black mirror of antimany is deposited far away
from heated spot because of more thermal (c) Cu (d) Mg
stability of SbH3 than AsH3 4. Which of the followings are example of saline
(b) Black mirror of antimany is deposited nearer hydrides :-
from heated spot because of less thermal (a) NH3 (b) NaH
stability of SbH3 than AsH3 (c) SiH4 (d) KH
(c) No black deposit is formed 5. Which elements when combined with hydrogen
(d) Brown colour gas SbH 3 evolved but no give products where hydrogen has negative
evolution of AsH3 takes place. oxidation state :-
44. Which has highest dipole moment ? (a) Li (b) S8
(a) H2 (b) CH4 (c) N 2 (d) Be
(c) H2O (d) H2O2 6. In which of the following compound hydrogen has
45. 2000 gm aqueous solution of CaCO3 contains 20 positive oxidation state :-
gm of CaCO3. Concentration of the solution is (a) HBr (b) NH3
(a) 10 ppm (b) 100 ppm (c) BaH2 (d) HI
(c) 1000 ppm (d) 10000 ppm 7. Which oxidation state are exhibited by hydrogen
46. Excess of KI and dilute H2SO4 were mixed in 50 ml in its compound :-
H2O2. The I2 liberated requires 20 ml of 0.1 (N) (a) 0
Na2S2O3. The strength of H2O2 in gm/lit will be (b) +1
(a) 0.12 (b) 0.24 (c) +2
(c) 0.39 (d) 0.52 (d) –1

www.jeebooks.in
HYDROGEN AND THE HYDRIDE 8.7
8. In what aspect, hydrogen molecule resemble alkali 14. Which reaction indicate reducing property of
metals ? H2O2:-
(a) Hydrogen molecules & alkali metals both are (a) KIO4 + H2O2 KIO3 + H2O + O2
diatomic with almost same ionisation energy (b) 2Fe3+ + 2H+ + H2O2 2Fe2+ + 2H2O + O2
(b) Both elements can loose electron to form mono
(c) HOCl + H2O2 H3O+ + Cl– + O2
positive ion
(d) 2MnO4– + 6H+ + 5H2O2 2Mn2+ + 8H2O +
(c) When HX and MX are electrolysed , hydrogen
5O2
(H2) and M (metal) are generated at cathode
(d) Both are diatomic 15. Which reaction indicate oxidising property of
H2O2:-
9. Which property of hydrogen molecule similar to
4
halogen :- (a) 2  Fe(CN)6   2H   H 2 O 2 

(a) Hydrogen accepts an electron to form noble 3
2  Fe(CN) 6   2H 2 O
gas configuration like halogen
(b) Hydrogen , halogen both are diatomic molecule (b) 2I   2H   H 2O2 
 I 2  2H 2O
(c) Hydrogen , halogen both are polar molecule (c) Mn2+ + H2O2 Mn4+ + 2OH–
(d) Hydrogen , halogen both are nonpolar molecule (d) PbS + 4H2O2 PbSO4 + 4H2O
10. Sodium reacts with which compound to give 16. Correct option regarding H2O.
hydrogen gas :- (a) H2O is a weak field ligand
(a) Cyclopentadine (b) Ethyl alcohol
(b) H2O can act as lewis base
(c) Diethyl ether (d) Ethane
(c) H2O is used as protic solvent
11. Which of the following reaction give correct
product :- (d) H2O is a very important analytical reagent to
identify acidic radical
(a) CaC2 + 2D2O C2D2 + Ca(OD)2
17. F2 when reacts with H2O, the products obtained
(b) CH3MgX + D2O CH3D + (OD)MgX
can be
(c) CD4+ Cl2 CD3Cl + DCl
Li in ND 3
(a) HF (b) O 2
(d) H3C – C  C – CH3  
(c) O 3 (d) F2 O
H3C D
C=C 18. Which of the following process, correct name is
D CH3 given
12. Correct increasing order :- (a) RCH = CHR + H 2  RCH 2 –CH 2 R
(a) BeH2 < TiH2 < CaH2 (electrical conductance) hydrogenolysis
(b) LiH < NaH < CsH (ionic character) (b) RCH = CHR + O3  RCHO + RCHO
(c) F–F < D–D < H–H (bond dissociation energy) ozonolysis
(d) H2O < NH3 < PH3 (reducing property) (c) RCO2R + H2O  RCO2H + ROH hydrolysis
13. Which of the following statements are not true for
hydrogen :- Pd/C/
(d) dehydrogenation
(a) it forms a large number of ionic compound by
loosing an electron 19. H– and He both have 1s2 electronic configuration
(b) it has one electron in outermost shell (a) reactivity order H– = He
(c) it can loose an electron to form a cation which (b) Stability order H– = He
can freely exists
(c) reactivity order H– > He
(d) it can loose an electron to form a cation which
(d) Stability order H– < He
can not freely exists

www.jeebooks.in
8.8 HYDROGEN AND THE HYDRIDE
20. The structure of H2O2 in solid crystal and gas phase Passage for Q.4 to Q.6
is given as follows :- Nascent hydrogen is hydrogen at the moment of its
H H generation. Nascent hydrogen is more reactive than
O

O  O

O 
ordinary hydrogen. It is more powerful reducing agent
H H
than ordinary hydrogen.
4. Correct option regarding Nascent hydrogen.
Solid crystal Gas phase (a) Nascent hydrogen is in ionic state of hydrogen
Correction option (b) Nascent hydrogen is in molecular state of
(a)  >  (b)    hydrogen
(c) > (d) > (c) Nascent hydrogen is in atomic state of
hydrogen
EXERCISE # III (d) Nascent hydrogen is in deactivated state of
 Linked Comprehension Type : hydrogen
Passage for Q.1 to Q.3 5. Which reaction can generate Nascent hydrogen?
Binary compounds of the elements with hydrogen are (a) Devardas alloy + NaOH 
called hydrides. The type of hydride which an element (b) Sodium amalgum + H2O 
forms largely depend on its electronegativity. They
are broadly of three types namely salt like, molecular (c) Zinc + dilute H2SO4 
and interstitial hydride. (d) All
1. Correct option regarding salt like hydride. 6. Nascent hydrogen can reduce.
(a) They are formed by elements of very low (a) Yellow FeCl3 solution
electronegativity, e.g. the alkali and alkaline (b) Pink solution of KMnO4, H+
earth metals of group IA and IIA.
(c) Potassium chlorate
(b) They are colourless solid formed by heating
the metal in H2 at temperature 150°C to 700°C. (d) All
(c) They are ionic hydrides, contain hydride ion Passage for Q.7 to Q.9
(d) All Cold water is rapidly decomposed by alkali metals
2. Correct option regarding molecular hydride. Li, Na, K etc. The vigour of the reaction may be
(a) They are formed by elements of higher moderate by alloying the alkali metal with lead of using
electronegativity, e.g. non-metal of group IIIB an amalgum of the metal.
to VIIB 7. The reactivity of alkali metals towards water-
(b) They are covalent , usually volatile liquid (a) Li > Na > K > Rb > Cs
(c) They are low melting and boiling point and are (b) Cs > Rb > K > Na > Li
nonelectrolyte in the liquid state and in solution
(c) Na > K > Li > Rb > Cs
in nonpolar solvent.
(d) All (d) Cs > Rb > Li > K > Na
3. Correct option regarding interstitial hydride. 8. Which alkaline earth metal does not react with cold
(a) Their formula are non-stoichiometric for e.g., water to give hydrogen gas ?
TiH1.73 ; VH0.6 (a) Be (b) Mg
(b) A number of transition metals when heated to (c) Ca (d) Ba
moderate temperature absorbs large amount 9. Red hot metal when reacts with steam give mixed
of hydrogen form interstitial hydride. oxide. Which metal can not give mixed oxide?
(c) The interstitial hydrides are less than the parent
(a) Zn (b) Mg
metals
(d) All (c) Both Zn and Mg (d) Fe

www.jeebooks.in
HYDROGEN AND THE HYDRIDE 8.9
Passage for Q.10 to Q.12 (c) One by H-bonding & three by normal covalent
Pure hydrogen peroxide is a clear pale blue, viscous bonding
liquid. It is soluble in water in all proportions, also (d) All four by H-bonding
soluble in ether but is insoluble in benzene. It slowly +CaCl2
15. Soap (soluble sodium salt of 
(hard water)
 2NaCl
decomposes at ordinary temperature and readily when
heated into water and oxygen evolving much heat. palmitic acid)
+ P(insoluble calcium palmitate)
10. Which of the following compound / object retards
P is -
the deocmposition of H2O2 -
(a) (C17H35CO2)2Ca (b) (C15H31CO2)2Ca
(a) Metals like gold (b) Dust particles
(c) (C17H32CO2)2Ca (d) None
(c) Phosphoric acid (d) Fe3+
Matrix Match Type :
11. Which of the following statement is not correct
regarding H2O2 ? 1. Match the column :-
(a) H2O2 has dipole moment 1.02 Debye Column - I (Reaction)
(b) H2O2 has boiling point 152°C Debye (a) Steam + Red hot coke 
1000°C

(c) O–O bond distance is 1.49Å (b) Steam + Red hot iron 
600-850°C

(d)  HOO bond angle is 97° in liquid H2O2 (c) 20% Castic soda  electrolysis

12. Which reagents can be used to distinguish H2O2 (d) CH4+ steam 
high temperature

and O3 ? (H2O2 participate in the reaction to change Column - II (Process/Product)
the colour but for O3 , no reaction) (P) Water gas process
(a) K2CrO4 solution + dil. H2SO4 (Q) H2 preparation (industrial)
(b) KMnO4 solution + dil. H2SO4 (R) Steam iron process
(c) TiO2 solution + dil. H2SO4 (S) Oxide formed as biproduct
(d) All (T) Steam hydrocarbon process
Passage for Q.13 to Q.15 2. Match the column :-
Water is a hydride of oxygen. Water is liquid at ordinary Column - I Column - II
temperature. The properties of liquid water are highly
Product formed Type of reaction
anomalous. It’s specific heat, latent heat of fusion and
vaporisation are all abnormally high. It boils at 100°C. (a) D2O (P) electrolysis of 50% H2SO4
13. Individual water molecules are only existent in - (b) H2O2 (Q) prolonged electrolysis of
water (H2O)
(a) Solid state
(c) (H2+CO2) (R) water gas shift reaction
(b) Liquid state
(d) O 3 (S) auto oxidation of 2 ethyl
(c) Vapour state above 100°C
anthraquinol
(d) All
(T) silent discharge of oxygen
14. X ray study of crystalline structure of ice indicates gas
each water molecule is tetrahedrally surrounded
3. Match the column :-
by the other four water molecule. What type of
bonding exists ? Column - I (Behave as)
(a) H2O2 acts as reducing agent
(a) Two by H-bonding & two by normal covalent
bonding (b) H2O2 acts as oxidising agent
(b) Three by H-bonding & one by normal covalent (c) H2 acts as reducing agent
bonding (d) H2 acts as oxidising agent

www.jeebooks.in
8.10 HYDROGEN AND THE HYDRIDE
Column - II (With elements compounds/in 6. Match the column :-
medium) Column - I Column - II
(P) with metals like Na, K, Ca Molecules Features
(Q) in acidic medium (a) H2O (P) Polar solvent
(R) in basic medium (b) H2O2 (Q) Nonlinear molecule
(S) with nonmetals like Cl2 (c) D2O (R) Less acidic than methanol
4. Match the column :- (d) C2H5OH (S) Used in atomic reactor
7. Match the column :-
Column - I (Molecules)
Column - I Column - II
(a) (a) H2 > D2 (P) Boiling point
D (b) D2 > H2 (Q) Freezing point
D D (c) H2O > D2O (R) Vapour pressure
(b) (d) D2O > H2O (S) Latent heat of vaporisation
D D
at boiling point
D
T (T) Latent heat of fusion at
T T melting point
8. Match the column :-
(c) T
T Column - I Column - II
T
(a) Protium (P) one proton
(d) D2O
(b) Deuterium (Q) two neutrons
Column - II (Features)
(c) Tritium (R) one neutron
(P) Polar molecule
(d) Helium (S) zero neutron
(Q) dipole moment is zero
(T) two proton
(R) most reactive towards nitration 9. Match the column :-
(S) most reactive towards sulphonation Column - I Column - II
(T) dielectric constant less than water (a) Hydrolith (P) Hydrogen has oxidation
5. Match the column :- state –1
Column - I (b) NaH (Q) Hydrogen has oxidation
state +1
(a) Use of hexametaphosphate
(c) NH3 (R) Act as lewis base
(b) Use of hydrogen peroxide
(d) HI (S) Act as bronsted acid
(c) Use of sodium borohydride
(T) Covalent molecule
(d) Use of hydrogen (nickel)
10. Match the column :-
Column - II
Column - I
(P) Removal of permanent hardness (a) H2Te > H2Se > H2S
(Q) Reducing agent (b) HF > HI > HBr > HCl
(R) Hydrogenation (c) HF > HCl > HBr > HI
(S) Propellant (d) HF > H2O > NH3 > CH4

www.jeebooks.in
HYDROGEN AND THE HYDRIDE 8.11
Column - II (iv) H2O and H2O2 both are ionic hydride of
(P) Bond polarity order oxygen.
(Q) Acidic strength order (v) H2O2 can be used as fuel as it produces
(R) Boiling point order pollution free atmosphere because its
combustion product is water.
(S) Thermal stability order
7. Commercial 22.4 volume H2O2 solution has a
11. Match the column :-
molarity.
Column - I Column - II
8. 1.5% solution of H2O2 (1.5 gm H2O2 in 100 ml
(a) LiH (P) Non stoichiometric solution) is equivalent to x volume H2O2. What is
(b) (BeH2)n (Q) Bridging bond present the value of x.
OD – / D 2O
(c) CH4 (R) Salt like hydride 9. CH3COCH3   Number of hydrogen
(d) ZrH1.92 (S) Metallic hydride exchange by deuterium in the prodcut.
(T) Vander waals force of 10. Find out number of reactions where phenol can
attraction exists between generate hydrogen
gaseous molecule OH
(i) + Na 
EXERCISE # IV
OH
 Integer Type : (ii) + Ca 
1. Number of isotopes present in hydrogen - OH
2. Number of reactions liberate hydrogen gas - (iii) + Al 
(i) Zn + NaOH  (ii) Zn + dil H2SO4  OH
(iii) Zn + dil HCl  (iv) Ag + HNO3  (iv) + NaOH 
(v) Pb + NaOH  (vi) Sn + NaOH  OH
(vii)Fe + Steam  (v) + NaNH2 
3. During carbocation rearrangement some times shift OH
of Hydride ion takes place. From which carbon (vi) + NaH 
shift of hydride ion takes place. 11. Find out number of hydrides which has more boiling
2 point than H2O.
3 4
1 5 HF ; HCl ; HBr ; CH4 ; NH3 ; B2H6
+
4. Number of element of group 6 forms hydride - 12. Find the number of atoms which are more
electronegative than hydrogen atom.
5. Number of compounds give H2O2 on reaction with
dilute H2SO4 C ; Si ; N ; I ; Br ; Be ; Al; B
PbO2 ; MnO2 ; TiO2 ; BaO2 . 8H2O ; XeO2 13. Find out pH of water at 25°C.
6. Number of statements whch are not true - 14. NiCl2 + 2C6H5MgBr MgBr2 + MgCl2 + P
(i) the electrolysis of molten hydrolith liberates H2 P + 2H2 Q + NiH2
gas at cathaode. Find out the number of  electrons.
(ii) Na2SO3 and KNO2 when reacts with H2O2, 15. Numer of elements which can adsorb large volume
H2O2 acts as reducing agent. of hydrogen
(iii) In the reactions of H2O2 and Na2CO3 , H2O2 Ni ; Pd ; Pt ; Zn ; Hg ; Cd
acts as acid. 

www.jeebooks.in
PERIODIC TABLE
PROB L EM S B ASED ON GI VEN T OPI CS  Problems with ionic radii
 The dural nature of electrons - particls of waves  Trends in inonic radii
 The Heisenberg Uncertainity principle  Ionization energies
 The Schrodinger wave equation  Electron affinity
 Radial and Angular functions  Electronegativity
 Pauli exclusion principle  Pauling
 Build-up of the elements, Hund’s rule  Mulliken
 Arrangement of the elements in groups in the period
 Allred and Rochow
table
 Metallic character
 Size of atoms and ions
 Variable valency and oxidation states
 Size of atoms
 Horizontal , Vertical and Diagonal relationships in
 Size of ions the periodic table.

www.jeebooks.in
CHAPTER
9
Periodic Table

EXERCISE # I
 Only one correct answer : 8. Which of the following order regarding
1. The correct order of first ionisation energies of the electronegativity is correct ?
elements :- (a) Mo(II) > Mo(III) > Mo(IV) > Mo(V) >
(a) Al > Ga (b) P > S Mo(VI)
(c) Br > Cl (d) Mg > Be (b) Cl(VII) > Cl(V) > Cl(I) > Cl(III)
2. The corret order of first ionsiation energy of the (c) sp carbon > sp2 carbon > sp3 carbon
elements :-
(d) F > N > Cl > O
(a) C < O < N (b) C < N < O
9. Which of the following statement is correct ?
(c) O < N < C (d) N < O < C
(a) Be has first ionisation energy lower than that of
3. The electronegativity order of carbon in different
boron
groups :-
(a) (C)CH < (C)CCl < (C)CF (b) Mg has first ionisation energy lower than that
3 3 3 of aluminium
(b) (C)CF < (C)CCl < (C)CH
3 3 3
(c) (C)CH < (C)CF < (C)CCl (c) From Zn to Hg, there is increase in ionisation
3 3 3 energy
(d) None
(d) Among all noble gases, (group 18 elements)
4. The most basic compound out of following
compound Xe has least ionisation energy
Methyl cyanide (a) ; Pyridine (B) ; Methyl amine 10. Which noble gas has highest ionisation energy ?
(C); Aniline (D) (a) He (b) Ne
(a) Methyl cyanide (A) (b) Pyridine (B) (c) Ar (d) Kr
(c) Methyl amine (d) Aniline (D) 11. Correct order of metallic character is :-
5. The most electronegative element and least (a) Rb > K > Na > Li
electronegative element are respectively :-
(b) K > Mg > Al > B
(a) Cl, Na (b) Br, Li
(c) F > O > N > B
(c) I, H (d) F, Cs
th (d) Ba > Be > Mg > B
6. The 5 period of periodic table contains x no. of
elements. The value of x:- 12. The most acidic oxide :-
(a) 2 (b) 8 (a) SO2 (b) SO3
(c) 18 (d) 32 (c) P4O10 (d) BaO
th
7. The 6 period of periodic table contains x no. of 13. The most basic oxide :-
elements. The value of x:- (a) MgO (b) CaO
(a) 2 (b) 8
(c) SrO (d) BaO
(c) 18 (d) 32

www.jeebooks.in
9.4 PERIODIC TABLE
14. The electronic configuration of an element is written 22. C is isomer of PtCl2(NH3)2 = A
as follows 1s22s22p6 3s23p63d74s2, on removal of Trans isomer of PtCl2(NH3)2 = B
one electron, the electronic configuration will
becomes :- If Pt–Cl bond distance is 2.32Å, then Cl–Cl bond
distance in A & B :-
(a) 1s22s22p63s23p53d74s2
(a) 3.28 Å, 3.28 Å (b) 4.64 Å, 3.28 Å
(b) 1s22s22p63s23p63d64s2
(c) 1s22s22p63s23p63d74s1 (c) 4.64 Å, 4.64 Å (d) 3.28 Å, 4.64 Å
(d) 7s12s22p63s23p63d74s2 23. Ionisation energy of AlCl3 is 5137 kJ mol–1
15. The least polar bond :- Heat of hydration of Al3+ is –4665 kJ mol–1
(a) H–F (b) H–Cl Heat of hydration of Cl– is –381 kJ mol–1
(c) H–Br (d) H–I From the data, it is concluded that :-
16. The most polar bond :- (a) AlCl3 in aqueous solution do not conduct
(a) C–F (b) C–O electricity
(c) C–N (d) C–C (b) AlCl3 in gaseous state conduct electricity due
17. The highest melting point for oxide of alkaline earth to presence of free ions.
metal is observed for :- (c) AlCl3 in aqueous solution conduct electricity
(a) MgO (b) CaO due to presence of hydrated ions.
(c) SrO (d) BaO (d) None of these
18. The ionisation potential of two element X1 and X2 24. What is the electronegativity of Cl from the
are 400 and 300 kcal mol–1 respectively. The following data ?
electron affinities of X1 and X2 are 80 and 85 kcal
Bond energy of F2 = 38 Kcal mol–1
mol–1 respectively. Electronegativity orderof X1 and
X2 is :- Bond energy of Cl2 = 58 Kcal mol–1
(a) X1 > X2 (b) X1 = X2 Bond energy of Cl–F = 61 Kcal mol–1
(c) X1 < X2 (d) Can not be predicted Electronegativity of F = 4
19. The first ionisation energy of Al(g) is 577.5 kJ mol–1. (a) 3 (b) 2.8
If second and third ionisation energy of Al are in (c) 3.2 (d) 3.5
the ratio 2 : 3 and if H for Al(g)  Al3+(g) + 3e– is
25. Which of the following does not reflect to
5140 kJ mol–1 then IE2 and IE3 ofAl are respectively :-
periodicity of element ?
(a) 1825 kJ mol–1 , 2737.5 kJ mol–1
(a) Electronegativity
(b) 2000 kJ mol–1 , 3000 kJ mol–1
(c) 1200 kJ mol–1 , 1800 kJ mol–1 (b) Metallic behaviour
(d) 5075 kJ mol–1 , 2525 kJ mol–1 (c) Neutron / Proton ratio
20. Highest size will be of :- (d) Ionisation potential
(a) Cl– (b) Br 26. Two elements P and Q form 3 types of bonds
(c) Br– (d) Br+ P–P; Q–Q ; P–Q. Their bond dissociation energies
are respectively 81 kcal mol–1 ; 64 kcal mol–1 ; 76
21. The As–Cl bond distance in AsCl3 is 2.2 Å. What
is the single bond covalent radius of arsenic. kcal mol–1. If electronegativity of Q is 2.4 then the
(Assuming electronegativity of both to be same and electronegativity of P will be approximately. (P is
radius of chlorine atom is 0.99Å) less electronegativite than Q)
(a) 0.24 Å (b) 0.77Å (a) 2.81 (b) 1.8
(c) 1.21Å (d) 3.19Å (c) 1.99 (d) 1.33

www.jeebooks.in
PERIODIC TABLE 9.5
+ –
27. If interionic distance between Na and F ions is 36. The incorrect statement among the following is :-
2.31 Å then radii of Na+ and F– are :- (a) the first ionisation potential of Al is less that the
(a) 0.95 Å & 1.36 Å (b) 1.155 Å & 1.155 Å first ionisation potential of Mg
(c) 1.36 Å & 0.95 Å (d) None of these (b) the second ionisation potential of Mg is greater
that the second ionisation potential of Na
28. Moving form right to left in a periodic table, the
atomic size is :- (c) the first ionisation potential of Na is less that
the first ionisation potential of Mg
(a) increased (b) decreased
(d) the third ionisation potential of Mg is greater
(c) remains constant (d) None of these that the third ionisation potential of Al
29. The increasing order of electronegativity in the 37. HOCl is example of acid. It can be explained by :-
following elements :-
(a) The electronegativity difference between O and
(a) C,N,Si,P (b) N,Si,C,P Cl is higher than electronegativity difference
(c) Si,P,C,N (d) P,Si,N,C between H and O atom. So Cl – O bond will
30. One element has atomic weight 39. Its electronic rupture.
configuration is 1s2,2s2,2p6,3s2,3p6,4s1. The true (b) The electronegativity difference between O and
statement for that element is :- Cl is lower than electronegativity difference
between H and O atom. So Cl – O bond will
(a) highest value of IE (b) transition element
rupture.
(c) isotone with 18Ar38 (d) None of these
(c) The electronegativity difference between O and
31. The number of paired electrons in oxygen is :- Cl is higher than electronegativity difference
(a) 6 (b) 16 between H and O atom. So O–H bond will
(c) 8 (d) 32 rupture.
32. The decreasing size of K+ , Ca2+ , Cl– & S2– follows (d) The electronegativity difference between O and
the order :- Cl is lower than electronegativity difference
between H and O atom. So O–H bond will
(a) K+ > Ca2+ > S2– > Cl– rupture.
(b) K+ > Ca2+ > Cl– > Sr–2 38. Which of the following species is / are paramagnetic
(c) Ca2+ > K+ > Cl– > S–2 NO2, NO, N2O4, N2O5 :-
(d) S–2 > Cl– > K+ > Ca2+ (a) Only NO2 (b) NO2 , NO
33. Which of the following oxide is neutral ? (c) NO2 , NO , N2O5 (d) All are paramagnetic
(a) CO (b) SnO2 39. Bond order of CO32– and CO+ are respectively :-
(c) ZnO (d) SiO2 (a) 1.33, 2.5 (b) 1.5, 3
34. Which of the following has the maximum number (c) 1.75, 3.5 (d) None of these
of unpaired electrons ? 40. Ionic radii of :-
(a) Mg2+ (b) Ti3+ (a) Ti4+ < Mn7+ (b) 35Cl– < 37Cl–
(c) V3+ (d) Fe2+ (c) K+ < F– (d) P3+ > P5+
35. The following acids have been arranged in the order 41. Identify the correct order of acidic strengths of
of decreasing acid strength. Identify the correct CO2, CuO, CaO, H2O :-
order :- (a) CaO < CuO < H2O < CO2
ClOH(I) BrOH(II) IOH(III) (b) H2O < CuO < CaO < CO2
(a) I > II > III (b) II > I > III (c) CaO < H2O < CuO < CO2
(c) III > II > I (d) I > III > II (d) H2O < CO2 < CaO < CuO

www.jeebooks.in
9.6 PERIODIC TABLE
42. Idenfity the least stable ion amongest the following:- 52. The correct order of basic strength of oxide/
(a) Li– (b) Be– hydroxide
(c) B– (d) C – (a) BaO > SrO > CaO > MgO > BeO
43. The set representing correct order of IP1 is :- (b) Cs2O > Rb2O > K2O > Na2O > Li2O
(a) K > Na > Li (b) Be > Mg > Ca (c) CsOH > Mg(OH)2 > Zn(OH)2 > HOCl
(c) B > C > N (d) Fe > Si > C (d) All
44. The correct order of radii is - 53. 3d series is started and ended with element -
(a) N < Be < B (b) F– < O2– < N3– (a) Scandium and zinc respectively
(c) Na < Li < K (d) Fe3+ < Fe2+ < Fe4+ (b) Titanium and copper respectively
45. Which set is expected to show the smallest (c) Titanium and mercury respectively
difference in Ist ionisation energy ? (d) Scandium and mercury respectively
(a) Fe, Co, Ni (b) N, O, F 54. 4d series is started and ended with element :-
+ 2+
(c) Ca, Ca Ca (d) He, Ar, Xe (a) Yttrium & cadmium respectively
46. The formula of ferrous chloride is FeCl2. The (b) Ytterbium & cadmium respectively
formula of sodium phosphate is Na3PO4. Then the (c) Yttrium and mercury respectively
formula of ferrous rhosphrate
(d) Scandium and mercury respectively
(a) FePO4 (b) Fe3(PO4)2
55. 4f series is started and ended with element :-
(c) Fe2(PO4)3 (d) Fe3(PO4)
(a) Cerium and lawrencium
47. What is the anhydride of lime water ?
(b) Thorium and lawrencium
(a) CaO2 (b) CaO
(c) Cerium and lutetium
(c) Ca (d) Ca(OH)2
(d) Thorium and lutetium respectively
48. What is the oxidation state of Mo in ammonium
phospho molybdate? 56. 5f series is started and ended with element :-
(a) + 4 (b) + 5 (a) Cerium and lawrencium
(c) + 6 (d) + 2 (b) Thorium and lawrencium
49. Which configuration of metal belongs to lowest (c) Cerium and lutetium
melting point? (d) Thorium and lutetium respectively
(a) (n–1)d10 ns2 (b) (n–1)d10 ns1 57. Element tend to lose two electrons :-
(c) (n–1)d8 ns2 (d) (n–1)d6 ns2 (a) Mg (b) Ba
50. Which element forms only one oxoacid ? (c) Ca (d) All
(a) F 58. Element with five electrons in the outer subshell :-
(b) Cl (a) Al (b) Si
(c) I (c) Se (d) As
(d) N 59. For beryllium, Be is more stable than Be3+ from
2+

51. The correct order of acidic strength of oxides that which of the following options are correct ?
(a) Na2O < MgO < ZnO < P4O10 (a) (I2 – I1) of Be is much less than (I3 – I2) of Be.
(b) Al2O3 < SiO2 < P4O10 < SO3 (b) (I2 – I1) of Be is much more than (I3 – I2) of Be.
(c) Cl2O < ClO2 < Cl2O7 (c) (I2 – I1) of Be is equal than (I3 – I2) of Be.
(d) All (d) I3 <<< I2 < I1

www.jeebooks.in
PERIODIC TABLE 9.7
60. Which elements has + charge in compound A, 67. For an element the spin only magnetic moment is
B, C respectively:- 3.83 BM. The number of unpaired electron in the
H–F ; Si–Cl ; C–O valence shell of this element :-
A B C (a) 1 (b) 2
(a) F, Cl, O (b) H, Si, C (c) 3 (d) 4
(c) H, Cl, O (d) H, Si, O 68. Which of the following process is exothermic ?
61. Ist ionisation enthalpy and IInd ionisation enthalpy (a) O(g) O–(g) (b) O–(g) O2–(g)
of Mg are 78 and 348 kcal mol–1 respectively. The (c) O(g) O2–(g) (d) He(g) He–(g)
H for the process M(g) M2+(g) + 2e– is:- 69. Which of the following energy is not associated
(a) +170 kcal mol–1 (b) +426 kcal mol–1 for the conversion M–(g) M4+(g) :-
(c) –426 kcal mol–1 (d) –170 kcal mol–1 (a) Ist ionisation energy of M(g)
62. The largest species and smallest species among (b) IInd ionisation energy of M(g)
Mg, Mg2+, Al, Al3+ are respectively - (c) Ist electron gain enthalpy of M(g)
(a) Mg, Mg2+ (b) Mg, Al3+ (d) IInd electron gain enthalpy of M(g)
(c) Al, Al3+ (d) Al, Mg2+ 70. In the first ninety elements , the number of s-block
elements is :-
63. The first ionisation enthalpy values of the 3rd period
elements Na, Mg and Si are respectively 496, 737, (a) 10 (b) 12
786 kJ mol–1. The first ionisation enthalpy of Al (c) 14 (d) 16
should be :- 71. Which of the following order is correct :-
(a) less than 496 kJ mol–1 (a) F2 > Cl2 > Br2 > I2(Bond dissociation energy)
(b) more than 786 kJ mol–1 (b) F– > Cl– > Br– > I–(Stability)
(c) more than 496 kJ mol –1 but less than (c) F. > Cl. > Br. > I.(Reactivity)
737 kJ mol–1 (d) F > Cl > Br > I(Electron gain enthalpy)
(d) more than 737 kJ mol –1 but less than 72. Select correct order regarding bond dissociation
786 kJ mol–1 energy of hydrogen halide / hydrohalic acid :-
64. The oxidation state of oxygen of two oxygen (a) HF > HCl > HBr > HI
containing compounds OF 2 and Na 2O are (b) HCl > HBr > HF > HI
respectively :-
(c) HI > HBr > HCl > HF
(a) +2, –2 (b) –2, –2 (d) HF > HCl > HI > HBr
(c) +2, +2 (d) +1, –2 73. The Ist , IInd , IIIrd , IVth , Vth ionisation energy of an
65. The correct order of metallic behaviour :- element are 7.1, 14.3, 34.5, 46.8 & 162.2 eV
(a) Na > Mg > Be > Si > P respectively. The element can be :-
(b) Mg > Be > Na > Si > P (a) K (b) Al
(c) Be > Na > Mg > Si > P (c) Cl (d) Si
(d) P > Be > Si > Na > Mg 74. In a period, the elements having least melting point:-
66. If C–C bond distance is 1.54 Å, C=C bond (a) Pnicogens (b) Chalcogens
distance is 1.34 Å, CC bond distance is 1.2 Å. (c) Halogens (d) Noble gas
Then C–C bond distance in benzene is 75. In a period , the elements having highest atomic
approximately equal to :- volume :-
(a) 1.5 Å (b) 1.4 Å (a) Alkali metals (b) Alkaline earth metals
(c) 1.6 Å (d) 1.3 Å (c) Halogens (d) Noble gases

www.jeebooks.in
9.8 PERIODIC TABLE
76. Covalent radius and Vanderwaal’s radius of fluorine 84. The correct order of non metallic character :-
atom are respectively :- (a) B > C > Si > N > F
(a) 71 pm and 71 pm (b) 147 pm and 147 pm (b) Si > C > B > N > F
(c) 71 pm and 147 pm (d) 147 pm and 71 pm (c) F > N > C > B > Si
77. If electronic configuration of A– is [Ar] 3d10 4s2
(d) F > N > C > Si > B
4p6 , then which element has the configuration
identical to the configuration of A2+ :- 85. The correct order of oxidising property :-
(a) Ge (b) Ga (a) F > Cl > O > N (b) F > O > Cl > N
(c) As (d) Se (c) Cl > F > O > N (d) O > F > N > Cl
78. Decrease in size is maximum for which process :- 86. The correct order of ionisation enthalpy :-
(a) O(g) O+(g) (b) O–(g) O(g) (a) B > Al > Ga > In > Tl
(c) O–(g) O+(g) (d) All (b) B > Tl > Ga > Al > In
79. Which of the following represents the correct order (c) Tl > In > Ga > Al > B
of increrasing first ionisation enthalpy for Ca, Ba, (d) Tl > Ga > B > Al > In
S, Se and Ar ?
87. Among the second period element , for whcih
(a) Ba < Ca < Se < S < Ar
property the following order is observed :-
(b) Ca < Ba < S < Se < Ar
Li < B < Be < C < O < N < F < Ne
(c) Ca < S < Ba < Se < Ar
(a) electronegativity (b) covalent radius
(d) S < Se < Ca < Ba < Ar
(c) ionisation energy (d) electron gain enthalpy
80. The first ionisation potential of Na is 5.1 eV. The
value of electron gain enthalpy of Na+ will be :- 88. The correct order of metallic property :-
(a) –10.2 eV (b) +2.55 eV (a) B > Al > Mg > K (b) Al > Mg > B > K
(c) –2.55 eV (d) –5.1 eV (c) Mg > Al > K > B (d) K > Mg > Al > B
81. Among the following oxoacids, the correct 89. Electronegativity of nitrogen element is highest for
decreasing order of acid strength is :- which of its compound :-
(a) HClO4 > HClO3 > HClO2 > HOCl (a) NO (b) NH3
(b) HClO2 > HClO4 > HClO3 > HOCl (c) NO 2 (d) N2O 5
(c) HOCl > HClO2 > HClO3 > HOCl4 90. Formula of oxides of grou 14 element will be :-
(d) HClO4 > HOCl > HClO2 > HClO3 (a) M2O5 (b) MO3
82. Fluorine is the most reactive among all the halogens, (c) MO2 (d) M2O3
because of its :- 91. What is the oxidation state and covalency of Mg
(a) high electron affinity in Grignard reagent (alkyl magnesium halide) :-
(b) low dissociation energy of F–F bond (a) +1, +1
(c) large size (b) +2, +2
(d) high dissociation energy of F–F bond
(c) +2, +4
83. In the long form of the periodic table, the valence
(d) +2, +3
shell electronic configuration of 5s25p4 corresponds
to the element present in :- 92. Uuu and Uub is symbol of :-
(a) group 16 and period 5 (a) Unununnium, Ununbium
(b) group 17 and period 6 (b) Unununium, Unununbium
(c) group 17 and period 5 (c) Ununium, Unbium
(d) group 16 and period 6 (d) Unununnim, Unununbium

www.jeebooks.in
PERIODIC TABLE 9.9
2+ 2–
93. The internuclear distance of Mg and O is 212 pm. 101. The bromine atom possesses 35 electrons. It
The inter ionic distance of Mg2+ and S2– is 256 pm. contains 6 electrons in 2p orbital. 6 electrons in
If radius of O2– is 140 pm, what is the radius of S2– ? 3p orbital and 5 electrons in 4p orbital. Which of
(a) 72 pm (b) 184 pm these electron experience the lowest effective
(c) 82 pm (d) 116 pm nuclear charge ?
94. (Ionic radius)O2– = 126 pm (Ionic radius)Na+ = 116 pm. (a) 4s (b) 4p
Ionic radius of F– should be (c) 4d (d) 4f
(a) 114 pm (b) 128 pm 102. Which element has odd number of unpaired
(c) 119 pm (d) 110 pm electron :-
95. (Ionic radius)K+ = 152 pm (Ionic radius)Cl– = 167 pm. (a) P (b) Si
Ionic radius of S2– should be (c) Cr (d) Fe
(a) 148 pm (b) 158 pm 103. Increasing order of energy :-
(c) 164 pm (d) 170 pm (a) 5s < 5d < 4f < 6p < 6d
96. After the filling of np orbitals, next orbital filled will (b) 5s < 4f < 5d < 6p < 6d
be (c) 4f < 5s < 5d < 6p < 6d
(a) ns (b) (n–1)d (d) 4f < 5s < 5d < 6d < 6p
(c) (n+1)s (d) nd 104. The quantum number of four electrons are given
97. The element having atomic number 117 has not below.
yet been discovered. It should be placed in :- (I) n = 4,  = 2, m = –2, ms = –1/2
(a) Group 14 (b) Group 15 (II) n = 3,  = 2, m = 1, ms = +1/2
(c) Group 16 (d) Group 17 (III) n = 3,  = 1, m = 1, ms = +1/2
98. The element having atomic number 120 has not (IV) n = 4,  = 1, m = 0, ms = +1/2
yet been discovered. It should be placed in :- Correct order of increasing energy is -
(a) Group 1 (b) Group 2 (a) I < IV < III < II (b) III < IV < II < I
(c) Group 3 (d) Group 4 (c) III < II < IV < I (d) II < IV < III < I
99. Anything that influences the valene electrons will 105. Which of the following has least pH in aqueous
affect the chemistry of the element. Which one of solution ?
the follwing factors does not affect the valence (a) HOCl (b) HClO2
shell? (c) HClO3 (d) HClO4
(a) Valence principal quantum number (n) 106. Which arrangement of electrons is wrongly written :-
(b) Nuclear charge (z) 1s 2s 3p 3s 3p
(c) Nuclear mass (a) P :
(d) Number of core electrons 1s 2s 2p 3s 3p 4s 3d
(b) Cr :
100. The size of isoelctronic species – F–. Ne and Na+
1s 2s 2p 3s 3p 4s 3d
is affected by (c) Cu :
(a) Nuclear charge (Z) 1s 2s 2p
(b) Valence principal quantum number (n) (d) N :
(c) Electron-electron intraction in the outer orbitals 107. Ist ionisation energy is least with which atom :-
(d) None of the factors because their size is the (a) Lead (b) Silicon
same (c) Carbon (d) Tin

www.jeebooks.in
9.10 PERIODIC TABLE
108. Ist ionisation energy is least with which atom :- 117. Which is called superhalogen ?
(a) Boron (b) Indium (a) F (b) Cl
(c) Galium (d) Thallium (c) Br (d) I
109. The highest oxidising power is shown by :- 118. The preference of filling the shells runs :-
(a) I2 (b) B2 (a) K > L > M > N (b) L > M > N > K
(c) Cl2 (d) F2 (c) N > M > L > K (d) N > L > M > K
110. Hydration energy is highest for :- 119. Reaction Energy Involved
(a) Li+ (b) Na+ CO(g)  CO+(g) + e IE1
(c) K+ (d) Cs+ +
N2(g)  N2 (g) + e IE2
+
111. Hydration energy is highest for :- O2(g)  O2 (g) + e IE3
(a) Li+ (b) Be2+ Correct option :
(c) Mg2+ (d) Al3+ (a) IE1 > IE2 > IE3 (b) IE2 > IE3 > IE1
112. Correct order of stability of following anions :- (c) IE1 > IE3 > IE2 (d) IE2 > IE1 > IE3
CH3CH 2 ; CH 2  CH; HC  C; NH 2 ; OH 120. Which of the following represent d9 ion :-
A B C D E (a) Cu+ (b) Cu2+
(a) E > D > C > B > A (b) E > A > B > C > D (c) Zn2+ (d) Ni2+
(c) E > C > D > B > A (d) D > E > C > B > A 121. Among alkaline earth metal which one has lowest
113. The ionisation energy of Zn is :- ionisation energy value :-
(a) More than Cu and Ga (a) Mg (b) Ca
(b) Less than Cu and Ga (c) Sr (d) Ba
(c) More than Ga and less than Cu 122. The formula of magnesium nitride and sodium azide
(d) More than Cu and less than Ga are :-
114. Which of the following property undergoes gradual (a) Mg3N2 , Na3N respectively
steady shrinkage along the series :- (b) Mg(N3)2, NaN3 respectively
(a) Metallic radii of lanthanide elements (c) Mg3N2 , NaN3 respectively
(b) Covalent radii of lanthanide elements (d) Mg(N3)2 , Na3N respectively
(c) Ionic radii of lanthanide elements 123. Formula of phosphide ion and sulphate ion are
(d) All respectively P 3– and SO42–. The formula of
magnesium phosphide and aluminium sulphate are
115. For a particular principal quantum number, the
respectively :-
penetrating power of the orbitals.
(a) MgP, AlSO4 (b) Mg3P2, AlSO4
(a) nf > nd > np > ns (b) nd > nf > ns > np
(c) Mg3P2, Al2(SO4)3 (d) MgP, Al2(SO4)3
(c) ns > np > nd > nf (d) np > ns > nf > nd
124. Which of the following is largest cation ?
116. Reaction Energy involved
+ (a) Na+ (b) Mg2+
P(g)  P (g) + e E1
+ (c) Ca2+ (d) Al3+
S(g)  S (g) + e E2
125. Which of the following is smallest cation ?
P+(g)  P2+(g) + e E3
+ 2+ (a) Li+
S (g)  S (g) + e E4
(b) Be2+
Correct option :-
(c) Mg2+
(a) E1 > E2 > E3 > E4 (b) E4 > E3 > E1 > E2
(d) Al3+
(c) E4 > E3 > E2 > E1 (d) E3 > E4 > E1 > E2

www.jeebooks.in
PERIODIC TABLE 9.11
126. What will be the period number, group number, 137. The ratio of unpaired electrons present in the orbital
block of the element having atomic number 20 ? of Mn2+ and Cr3+ :-
(a) 4, 3, p (b) 3, 2, s (a) 4 : 1 (b) 2 : 1
(c) 4, 2, s (d) 3, 3, p (c) 5 : 2 (d) 5 : 3
127. Which of the following is largest ? 138. The five successive ionisation energies of an element
(a) Mg (b) Mg+1 are 800, 2427, 3658, 25024 and 32824 kJ mol–1
(c) Mg2+ (d) Al3+ respectively. The number of valence electrons is :-
128. The most electropositive element :- (a) 3 (b) 5
(a) S (b) O (c) 1 (d) 2
(c) Rb (d) K 139. Which of the following oxide is examples of
129. Which of the following pair of atomic numbers suboxide :-
represents s-block elements ? (a) NO 2 (b) KO 2
(a) 3, 9 (b) 11, 38 (c) C3O2 (d) N 2 O
(c) 55, 5 (d) 4, 33 140. NaOH is example of base it can be explained by :-
130. Alkali metals and alkaline earth metals form the (a) The electronegativity difference between O and
ion of the type :- H is higher than electronegativity difference
(a) M+1 ; M+2 (b) M+2 ; M+1 between Na and O atom. So Na – O bond
will rupture.
(c) M–1 ; M–2 (d) M–2 ; M–1
131. What is the electronic configuraiton for Co3+(g) :- (b) The electronegativity difference between O and
H is lower than electronegativity difference
(a) [Ar] 3d5 (b) [Ar] 3d6
between Na and O atom. So Na – O bond
(c) [Ar] 3d7 (d) [Ar] 4s13d5 will rupture.
132. Which of the following element is example of (c) The electronegativity difference between O and
metalloid ? H is higher than electronegativity difference
(a) N (b) As between Na and O atom. So O–H bond will
(c) Al (d) Te rupture.
133. The outermost electron resides in K, Cr, Cl, (d) The electronegativity difference between O and
respectively :- H is lower than electronegativity difference
(a) 4s, 4p, 3d orbital (b) 3s, 3d, 3p orbital between Na and O atom. So O–H bond will
(c) 3s, 4d, 4p orbital (d) 4s, 3d, 3p orbital rupture.
134. If an element X forms the highest oxide of the 141. The energy required to convert all atoms of Mg to
formula X2O7 , then it belongs to group :- Mg2+ ion present in 24×10–3 g of Mg vapour is :-
(a) 14 (b) 15 (a) 2.67 kJ (b) 2.19 kJ
(c) 16 (d) 17 (c) 3.56 kJ (d) 4.38 kJ
135. In Mn, the maximum number of electrons having 142. Which of the following pair of elements belongs to
ms = –½ is :- the same period :-
(a) 15 (b) 14 (a) Ca & Cl (b) Na & Ca
(c) 16 (d) 20 (c) Mg & Sb (d) Ca & Zn
136. In Mn, the minimum number of electrons having 143. Which of the following has highest second
ms = +½ is :- ionisation energy ?
(a) 10 (b) 12 (a) Sc (b) Ti
(c) 14 (d) 15 (c) V (d) Cr

www.jeebooks.in
9.12 PERIODIC TABLE
144. Which of the following has highest third ionisation EXERCISE # II
energy ?
 One or More Than One Correct Answer :
(a) Ca (b) Tl
1. Which of the ion pair is isoelectronic :-
(c) Mn (d) V
(a) F–, K+ (b) Cl–, Na+
145. Correct ionisation energy order :-
(c) S2–, Ca2+ (d) O2–, Mg2+
(a) Cu > Ag > Au (b) Au > Cu > Ag
2. Which process requires energy to take place ?
(c) Au > Ag > Cu (d) Cu > Au > Ag
(a) Ne Ne–1 (b) F F–1
146. Bond polarity order :-
(c) O–1 O2– (d) Mg Mg2+
(a) F–H > O–H > Cl–H > S–H
3. Which of the following reactions should , are
(b) F–H > Cl–H > O–H > S–H
nonspontaneous in gas phase :-
(c) S–H > Cl–H > O–H > F–H
(a) Xe + He+ Xe+ + He
(d) Cl–H > S–H > F–H > O–H
(b) Si + Cl+ Si+ + Cl
147. Which orbital is filled after filling of np orbitals :-
(c) F– + I I– + F
(a) ns (b) nd
(d) I– + F F– + I
(c) (n–1)d (d) (n+1)s
4. Bond dissociation energy order :-
148. If x, y, z are electronegativity on Mulliken scale,
ionisation potential (in V) and electron affinity (a) C–C > C–H > H–H
(+ in eV) respectively, then the electron affinity in (b) F–F > Cl–Cl > Br–Br
the terms of electronegativity and ionisation (c) CC > C=C > C–C
potential will be :- (d) O=O > S=S > Se=Se
xy xy
(a) z  (b) y  5. Which of these ions are paramagnetic ?
2 2
zy (a) Sr2+ (b) Fe3+
(c) x  (d) z = 2x – y
2 (c) Co2+ (d) S2–
149. Fluorine has the highest electronegativity among
6. Which of the following electronic configuration
the ns2 np5 group on the Pauling scale, but the
electron affinity of fluorine is less than that of belongs to d block element ?
chlorine because :- (a) 1s22s22p63s23p63d54s1
(a) the atomic number of fluoride is less than that (b) 1s22s22p63s23p63d104s1
of chlorine (c) 1s22s22p63s23p6
(b) fluorine being the first member of the family (d) 1s22s22p63s23p63d104s24p1
behaves in an unusual manner 7. Which of the following statements are correct ?
(c) chlorine can accomodate an electron better than (a) MgO is more basic than BaO
fluorine by utilising its cacant 3d-orbital
(b) Na2(g) molecule exhibit metallic properties
(d) small size, high electron density and an
(c) Ca2+ has smaller ionic radius than K+
increased electron repulsion makes addition of
an electron to fluorine less favourable than that (d) Atomic size order Cs > Na > Mg > Si > Cl
in the case of chlorine in isolated stage. 8. Which of the following statements are correct ?
150. First and second ionisation enthalpies of Mg are (a) Lithium is better reducing agent than Ceasium
720 kJ/mol and 1440 kJ/mol respectively. The % (b) Ionisation energy and sublimation energy of Li
of Mg+ ions, if one gram of Mg(g) absorbs 50 kJ is less than Ceasium
of energy. (Given : Atomic mass of Mg = 24 amu):- (c) 2s orbital is lower in energy than 2p orbital
(a) 33.33 (b) 66.67 (d) The first ionisation energy of Berylium is more
(c) 55.55 (d) 99.99 than that of boron

www.jeebooks.in
PERIODIC TABLE 9.13

9. Which of the following statements are correct ? (c) N2 and CN are isoelectronic species.
(a) The electronic configuration of Cr is [Ar] 3d5 (d) Third ionisation energy of phosphorous is more
4s1. (atomic number of Cr = 24) than sulphur.
(b) The magnetic quantum number may have a 17. Which of the following statements are incorrect ?
negative value (a) Formation of Se2– and Ar–, both require
(c) In silver atom, 23 electrons have a spin of one absorption of energy
type and 24 of the opposite type. (atomic (b) Metallic and covalent radii of potassium are
number of Ag = 47) 203 and 230 pm respectively.
(d) The oxidation state of nitrogen in HN3 is –3
(c) If the same element is forming oxide in different
10. In which case, Ist bond is more polar than 2nd oxidation state, then the oxide will be highest
bond? acidic in nature in which element will be in its
(a) P-Cl, P-Br (b) S-Cl, S-O highest oxidation state.
(c) N-O, N-F (d) B-F, B-Cl (d) The electron gain enthalpy of oxygen is more
11. Diagonal relationship is observed in negative than that for sulphure.
(a) F, Ne (b) C, P 18. Elements tend to gain two electron :-
(c) B, Si (d) Li, Mg (a) S (b) O
12. Electronic configuration ns2(n–1) d0–1 (n–2)f1–14 (c) Te (d) Cl
represents 19. Glenn Seaborg took part in the discovery of ten of
(a) Representative elements the periodic table’s chemical elements. Which
(b) Lanthanides element it can be :-
(c) Radioactive elements (a) Nobelium (atomic number 102)
(d) Actinides (b) Californium (atomic number 98)
13. Which of the following pair is isoelectronic? (c) Berkelium (atomic number 97)
(a) Te2–, Xe (b) Mn2+, Cr3+ (d) Curium (atomic number 96)
(c) Ni, Zn2+ (d) Br–, Kr 20. Aqueous solution of which of the following oxide
14. Which of the following properties are correctly are acidic in nature :-
given? (a) Cl2O7 (b) CO2
(a) N > P > As > Sb(Third ionisation energy) (c) BaO (d) BeO
(b) Na+ < F– < O2– N3–(Ionic size) 21. Which of the following options are incorrect :-
(c) O > F > N > C(Second ionisation energy) (a) Second period element oxygen has more
(d) Mg > Al > Si > P(Covalent radius) negative electron gain enthalpy than third period
15. Which of the following atoms has positive electron element sulphur.
gain enthalpy ? (b) Second period element lithium has more
(a) N (b) Be negative electron gain enthalpy than third period
(c) Mg (d) Mn element sodium.
16. Which of the following statements are correct ? (c) Second period element neon has more negative
(a) The 5th period of periodic table contains 32 electron gain enthalpy than third period element
elements argon.
(b) The 4f and 5f inner transition series of elements (d) Second period element fluorine has more
are placed separately at the bottom of the negative electron gain enthalpy than third period
periodic table element chlorine.

www.jeebooks.in
9.14 PERIODIC TABLE
22. In which case all M–O bond lengths are identical :- 30. Formula of oxide of group 15 elements can be :-
(a) PO43– (b) CO32– (a) M2O3 (b) M2O5
(c) MnO4– (d) CH3CO2H (c) M4O10 (d) MO2
23. Which of the following options are correct :- 31. Which formulas are correctly given :-
(a) Across the period effective nuclear charge (a) Compound formed by silicon and bromine :
decreases SiBr4 (Silicon tetrabromide)
(b) Atomic gaseous of inert gases is highest in the (b) Compound formed by aluminium and sulphur :
period Al2S3 (Aluminium sulphide)
(c) Ionisation energy of Ne is greater as compared (c) Compound formed by mercury and iodine :
to Ne+ HgI2 (Mercuric iodid)
(d) Ionisation energy of Zn is higher as compared (d) Compound formed by lithium and oxygen : Li2O
to Ga (Lithium oxide)
24. Out of lithium and neon :- 32. Which of the following elements are called
(a) IE2 of Li is more than IE2 of Ne semimetals or metalloid :-
(b) IE2 of Li is less than IE2 of Ne (a) Si (b) Ge
(c) I (d) As
(c) IE1 of Li is more than IE1 of Ne
34. Select correct option regarding Uuh(atomic
(d) IE1 of Li is less than IE1 of Ne
number 116)
25. Which of the following options are incorrect :- :-
(a) It belongs to 7th period element
(a) HF bond length is smaller then sum of radius of
(b) It belongs to group 16 element
H atom and F atom
(c) It is p-block element
(b) Effective nuclear charge of inert gases is minium
(d) It is an inert gas
(c) C(g)C2+(g) the energy involved in this
process is called 2nd ionisation energy 35. Cl35 and Cl– has
(d) Electron affinity of oxygen atom is higher than (a) Same number of electrons
that of sulphur atom (b) Same number of neutrons
26. 2px orbital and 3px orbital of an atom have same :- (c) Same number of protons
(a) size (b) orientation (d) Same stability
(c) shape (d) nodal plane 36. CH2–CH2
| | CH2 = CH2 ;
27. Which of the following is associated with the I I –I2

removal of electron from d-subshell :- (A) (vicinal dihalide)


(a) IE1 for Mn (b) IE2 for Mn OH
(c) IE3 for Mn (d) IE4 for Mn CH2 .
OH H 2C = O
28. The correct order of reactivity :- –H2O
(a) Cs > Rb > K > Na > Li (B) Gem diol
(b) I > Br > Cl > F Correct option
(c) Li > Na > K > Rb > Cs (a) Vicinal diiodide does not exist because C–I has
(d) F > Cl > Br > I less bond dissociation energy
29. Chemical elements discovered at Lawerance (b) Gem diol does not exist because C= O has
Berkeley National Laboratory are :- high bond dissociation energy
(a) Technetium (b) Astatine (c) Vicinial diol does exist
(c) Plutonium (d) Curium (d) Gem diol exists

www.jeebooks.in
PERIODIC TABLE 9.15
37. Which of the following statements are correct :- 45. Which of the following statements is/are true for
(a) the size of hydride ion is greater than that of F– the long form of the periodic table :-
(b) (n+l) values for 5s and 4p orbitals are identical (a) it reflects the sequence of filling the electrons in
(c) H– is example of stronger reducing agent as the order of sub-energy levels s,p,d and f.
compared to hydrogen atom (b) it helps to predict the stable valency states of
(d) The lanthanoid contraction is less as compared the element
to actinide contraction (c) it reflects trends in physical and chemical
38. H–CCH can form salt with :- properties of the elements
(a) Ca metal (b) Na metal (d) it helps to predict the relative ionicity of the
bond between any two elements
(c) Li metal (d) K metal
46. Which of the following statements are true ?
39. Which of the elements with given atomic number
belong to p block element ? (a) the first ionisation potential of Al is less than the
first ionisation potential of Mg
(a) Atomic number 83 (b) Atomic number 34
(b) the second ionisation potential of Mg is greater
(c) Atomic number 54 (d) Atomic number 64 than the second ionisation potential of Na
40. Which of the elements with given atomic number (c) the first ionisation potential of Na is less than
belong to inner transition series ? the first ionisation potential of Mg
(a) Atomic number 64 (b) Atomic number 42 (d) the third ionisation potential of Mg is greater
(c) Atomic number 79 (d) Atomic number 91 than the third ionisation potential of Al
41. The order of ionic mobility :- 47. Which one of the following statements are correct
(a) K+(aq.) > Na+ (aq.) (b) Ca2+(aq.) > Sr2+(aq.) in relation to ionization enthalpy?
(c) Rb+(aq.) > Na+ (aq.) (d) Na+(aq.) > K+ (aq.) (a) Ionization enthalpy increases for each
42. CH3I + OH– CH3OH + I– successive electron.
Correct option regarding this reaction :- (b) The greatest increase in ionization enthalpy is
(a) Here OH– attacks + charge of carbon experienced on removal of electron from core
noble gas configuration.
(b) Here OH– acts as nucleophile
(c) End of valence electrons is marked by a big
(c) It is example of nucleophilic substitution jump in ionization enthalpy.
(d) (Electronegativity of carbon) CH group > (d) Removal of electron from orbitals bearing lower
3
(Electronegativity of iodine) n value is easier than from orbital having higher
43. Which of the following option are correct :- n value.
(a) Pauling scale electronegativity is based on bond 48. Which of the following statements are correct for
energy value the preiodic classification of elements ?
(b) Alred rochow scale electronegativity is based (a) The properties of elements are the periodic
on Zeff and covalent radius functions of their atomic number
(c) Electronegativity in Muliken’s scale is based (b) Non-metallic elements are lesser in number
on ionisation energy and electron affinity than metallic character
(d) Electronegativity in Sanderson’s scale is based (c) The first ionisation energy of the element along
on compactness of electron cloud around the a period do not vary in a regular manner with
nucleus increase in atomic number
44. Ionic radii of :- (d) For transition elements, the d-subshells are filled
(a) Ti4+ < Mn7+ (b) 35Cl– < 37Cl– with electrons monotonically with increase in
(c) K+ < Cl– (d) P3+ > P5+ atomic number

www.jeebooks.in
9.16 PERIODIC TABLE
49. The correct order of radii is :- 58. Which of the following elements have non metallic
(a) N < Be < B (b) F– < O2– < N3– properties :-
(c) Na < Li < K (d) Fe3+ < Fe2+ < Fe4+ (a) Cs (b) Cl
50. Which of the following statements are correct ? (c) P (d) S
(a) Among Sn, Pb, Fe, Ag ; +2 oxidation state is 59. Reducing property is shown by which element :-
most stable for Pb (a) Li (b) K
(b) Among Mg2+, Ti3+, V3+, Fe2+ ; maximum number (c) Ba (d) Ca
of unpaired electron is present in Fe2+ 60. Which of the following are example of interhalogen
(c) Among O, F, N, C ; the highest second compound ?
ionisation potential is observed for oxygen (a) ClF3 (b) HBr
(d) Among the following configuration [Ne] 3s23p1, (c) Cl2O7 (d) IF7
[Ne] 3s23p3, [Ne] 3s23p2, [Ne] 3d104s24p3;
61. Which of the following are example of
[Ne] 3s23p3 has highest ionisation energy
pseudohalide ?
51. Which of the following sets contain only
(a) CN– (b) ICl2–
isoelectronic ions ?
(c) SCN– (d) SeCN–
(a) Zn2+, Ca2+, Ga3+, Al3+
62. Correct order of stability of following ion / radical :-
(b) K+, Ca2+, Sc3+, Cl–
(c) P3–, S2–, Cl–, K+ (a) F– < Cl– < Br– < I– (b) F+ < Cl+ < Br+ < I+
. . . .
(d) Ti4+, Ar, Cr3+, V5+ (c) F < Cl < Br < I (d) F– > Cl– > Br– > I–
52. Which of the following have no unit ? 63. Which of the following ion are example of
(a) Electronegativity (b) Electron gain enthalpy diamagnetic ?
(c) Ionisation enthalpy (d) Metallic character (a) Co2+ (b) Zn2+
53. 0 :- (c) Ti4+ (d) Sc3+
(a) 0 (b) 0 64. Which of the following ion represent d6 ion ?
(c) 0 (d) 0 (a) Co2+ (b) Fe3+
54. Which of the following pair of atomic numbers (c) Fe2+ (d) Co3+
represents p-block elements ? 65. The correct order of ionic mobility :-
(a) 19, 33 (b) 14, 53 (a) Li+(g) > Na+(g) (b) Li+(aq.) > Na+(aq.)
(c) 6, 35 (d) 30, 31 (c) Be2+(g) > Mg2+(g) (d) Be2+(aq.) > Mg2+(aq.)
55. Which of the following pair of atomic numbers 66. Which of the following orbitals are represented by
represents d-block elements :- n = 3,  = 1 ?
(a) 23, 49 (b) 24, 46 (a) 3py (b) 4px
(c) 56, 28 (d) 80, 21 (c) 3dxy (d) 3px
56. Correct electron affinity order :- 67. Which of the following are example of isoelectronic
(a) Ir > Rh > Co (b) Pt > Pd > Ni species ?
(c) Au > Ag > Cu (d) I > Cl > F (a) Cr6+ , Sc3+ (b) Te2– , I–
57. Which of the polyatomic anion are isoelectronic :- (c) N2O, CS2 (d) NO2+ , O3
(a) NO3–, CO32–, BO33– 68. In which pair, the first species has more size as
(b) SO42– , PO43–, SiO44– compared to second species :-
(c) SeO32– , SO32– , TeO33– (a) Ar, Ne (b) I– , Cl–
(d) N3– , CN22– (c) Na+(aq), K+(aq) (d) Na, Na+

www.jeebooks.in
PERIODIC TABLE 9.17
69. Correct options are :- 77. The first element of a group differs in many ways
(a) F– is larger than H– from the other heavier members of the group. It is
(b) Cl is more electronegative than Br due to :-
(c) Cs is more electropositive than Na (a) small size
(d) Inert gas can form cation and anion easily (b) high electronegativity and high ionisation
potential
70. Correct order of acidic strength :-
(c) odd atomic number
(a) HNO2 > HNO3
(d) magic numbers of atomic weight
(b) H2SO4 > H2SO3
78. The first three ionisation energy of an element are
(c) HClO3 > HBrO3 > HIO3
9.3, 18.2 and 553.8 eV. What inforamations are
(d) H3PO2 > H3PO4 > H3PO3 reflected by following data ?
71. 1 eV atom–1 is equivalent to:- (a) The element belongs to 15th group of Modern
(a) 3.83×10–20 Cals atom–1 periodic table
(b) 1.6×10–19 J atom–1 (b) The element has three electrons in the valence
(c) 7.68×10–19 Cals atom–1 shell
(d) 2.26×10–19 Cals atom–1 (c) The element belongs to 2nd group of Modern
72. Which of the following are examples of mixed periodic table
oxide :- (d) The element has two electrons in the valence
(a) Fe3O4 (b) Mn3O4 shell
(c) N2O 5 (d) KO 2 79. Correct molecular formula :-
73. How many compound has more lattice energy than (a) Hg2(N3)2 : Mercurous Azide
NaCl :- (b) HN3 : Hydra acid which contains nitrogen
(a) RbCl (b) MgO (c) Cl2O7 : Anhydride of HClO4
(c) ScN (d) TiC (d) Fe[Fe(CN)6] : Ferri-Ferri cyanide
74. Which oxides are more basic than Li2O :- 80. Which of the following statements are correct :-
(a) Na2O (a) The halogen that shows same oxidation state
(b) Al2O3 in all its compounds with other elements is
(c) BeO chlorine.
(d) BaO (b) Most stable oxidation state of chromium is +3
75. Which of the following ions have pseudo noble (c) Among Na, Mg, Fe, Al ; Fe only exhibits more
gas configuration ? than one oxidation state (other than zero)
(a) Cu+1 (d) The number of electrons present in its outermost
(b) Cd2+ shell of an element (whose most common
oxidation state is –2) is six
(c) Au+
81. What is the change in oxidation state during
(d) Tl3+
following conversion :-
76. Correct option regarding As (atomic number 33)
(a) PbS PbSO4 : +2 to +6 for S
(a) It is one of the typical element
(b) Br2 BrO3– : 0 to +5 for Br
(b) It belongs to 16th group
(c) H2O2 H2O : +1 to –2 for O
(c) It is a 4th period element
(d) NH3 NO : –3 to +2 for N
(d) It is a p-block element

www.jeebooks.in
9.18 PERIODIC TABLE
82. 3py and 4py orbital of an atom have different :- 89. Ionisation energy of atoms A and B are 350 and
(a) Number of nodal plane 250 kcalmol–1 respectively. The electron affinities
(b) Quantum number of these atoms are 70 and 90 kcalmol–1 respectively.
Then:-
(c) Size
(a) electron cloud is more attrached by A
(d) Shape
(b) electron cloud is more attrached by B
83. 1 2 3 4 5
For the compound, CH3–CH=N–CN (c) on Mulliken scale, electronegativity of A is more
than B
Th correct order of electronegativity :-
(d) on Mulliken scale, electronegativity of A is less
(a) N5 > N3 (b) N5 > C4
than B
(c) C4 > C2 (d) C2 > C1
90. Which of the following has/have no unit ?
84. Which of the following options are correct
(a) Electronegativity
regarding
(b) Electron gain enthalpy
ns2(n–1)d0–1(n–2)f1–14 configuration ?
(c) Ionisation enthalpy
(a) For lanthanides n = 4
(b) For lanthanides n = 5 (d) Metallic character
(c) For actinides n = 5 91. A , B and C are oxides of element X, Y and
Z respectively. X, Y and Z are in the same period
(d) For actinides n = 6
of the Modern periodic table. A gives an
85. Which of the following sequences contain atomic aqueous solution which turns blue litmus red.
numbers of only representative elements? B reacts with both strong acids and strong alkalies.
(a) 3,33,53,87 (b) 2,10,22,36 C gives an aqueous solution which is strongly
(c) 7,17,25,37,48 (d) 9,35,51,88 alkaline. Which of the following statements is / are
86. Ionic radii vary in :- true ?
(a) inverse proportion to the effective nuclear charge (a) All the three elements are metals
(b) inverse proportion to the square of effective (b) The Pauling electronegativities decrease from
nuclear charge X to Y to Z
(c) direct proportion to the screening effect (c) The atomic radius increases in the order
(d) direct proportion to the square of screening X<Y<Z
effect (d) X, Y and Z could be phosphorus, aluminium
87. Those elements impart colour to the flame on and sodium respectively
heating in it, the atoms of which require low energy N0
92. If atoms of X(g) are conveted into X+(g) by
for the ioisation (i.e., absorbs energy in the visible 2
region of spectrum). The elements of which of the N
energy E1 and 0 atoms of X(g) are conveted into
following groups in Modern periodic table will 2
impart colour to the flame ? X–(g) by energy E2, then :-
(a) 2 (b) 13 2E1
(a) Ionisation potential of X would be N
(c) 1 (d) 17 0

88. Which of the following elements will gain one (b) Ionisation potential of X would be 2E1
electron more readily in comparison to other 2E 2
elements of their group :- (c) Electron affinity of X would be N
0
(a) S(g) (b) Na(g) (d) Electron affinity of X would be 2E2
(c) O(g) (d) C(g)

www.jeebooks.in
PERIODIC TABLE 9.19
93. Electronic configuration Ist ionisation energy EXERCISE # III
ns2np1 (IE)
 Linked Comprehension Type :
2 3
ns np IE’ Passage for Q.1 to Q.3
ns2np4 IE’’ A(g) A (aq.) + 2e–H1 = 700 kJ mol–1
2+
2 5
ns np IE’’’ A–(g) A2+(g) + 3e–H2 = 1400 kJ mol–1
then - Electron gain enthalpy for A+(g) = –350 kJ mol–1
(a) IE > IE’ (b) IE’ > IE’’ IE1 IE2 for A(g) = 950 kJ mol–1
(c) IE’’ > IE’’’ (d) IE’’’ > IE 1. What are IE1 andIE2 of A ?
94. Which of the following compounds exists :- (a) 600, 350 kJ mol–1 (b) 400, 550 kJ mol–1
(a) BiF5 (b) PbO2 (c) 350, 600 kJ mol–1 (d) 500, 450 kJ mol–1
(c) PbI4 (d) As2O3 2. Hydration energy of A2+(g) is
95. In the long form of periodic table the valence shell (a) – 200 kJ mol–1 (b) – 150 kJ mol–1
electronic configuration of an element is 5s25p4.
(c) – 100 kJ mol–1 (d) – 250 kJ mol–1
The element resides in :-
3. Electron gain enthalpy of A is -
(a) Group 17 (b) Group 16
(a) – 300 kJ mol–1 (b) – 350 kJ mol–1
(c) Period 5 (d) Period 6
(c) – 400 kJ mol–1 (d) – 450 kJ mol–1
96. Which of the following oxyacid are examples of –
ic acid :- Passage for Q.4 to Q.6
(a) H2SO4 (b) HNO2 In periodic table, some of the elements are given with
their atomic number.
(c) H2CO3 (d) H2SO3
Element Atomic number
97. Which of the following oxyacid are examples of –
ous acid :- Cl 17
(a) H2SO4 (b) HNO2 F 9
(c) H2CO3 (d) H2SO3 Cs 55
98. Prefix pyro is attached to the names :- Al 13
(a) S2O72– (b) H4As2O7 C 6
(c) H2SO5 (d) H4B2O7 Xe 54
99. Identify the meta acids :- 4. The element with highest electronegativity and
lowest ionisation potential are respectively.
(a) HPO3 (b) H2SnO3
(a) F, Xe (b) Cl, Cs
(c) HMnO4 (d) HBO2
(c) C, Al (d) Xe, C
100. Which of the following options are correct :-
5. The element which has smallest radius and the
(a) The magnitude of Ist ionisation energy of Cl(g)
element whose oxide is amphoteric are
is same as electron gain enthalpy of Cl+(g)
respectively.
(b) The corresponding thioether of ethyl methyl
(a) F, Al (b) F, C
ether is CH3–S–C2H5
(c) Cl, C (d) Cl, Al
(c) The last element of the p block in 6th peirod is
represented by the outermost electronic 6. The element which has 8 electrons in outer most
configuration 4f145d106s26p6 shell.
(d) The oxidation state and covalency of sulphur (a) C (b) F
in sulphur molecule (S8) are respectively 0 and 2 (c) Cs (d) Xe

www.jeebooks.in
9.20 PERIODIC TABLE
Passage for Q.7 to Q.9 Passage for Q.13 to Q.15
The IE1 and the IE2 in kJ/mol of few elements are given According to Aufbau principle , the atomic orbitals
below fill in the order of increasing (n + l) value when n
Element IE1 IE2 is principle quantum number and l is azimuthal
P 2372 5270 quantum number. Higher the (n + l) value for an
Q 500 7300 orbital, higher will be the energy. If the two orbitals
R 900 1700 have the same value (n + l) the one which has lower
S 1680 3400 value of n will be of lower energy and would be filled
Based on the above information answer the following first.
questions :
13. The correct order of energy level for different
7. Which of the above elements is likely to be a orbitals -
reactive metal ?
(a) 6d < 8s < 5f < 7p
(a) P (b) Q
(b) 8s < 5f < 6d < 7p
(c) R (d) S
(c) 5f < 6d < 7p < 8s
8. Which of the above elements is likely to be a
reactive non metal ? (d) 7p < 6d < 8s < 5f
(a) P (b) Q 14. For which orbitals (n+l) value becomes 7
(c) R (d) S (a) 4f (b) 5d
9. Which represents a noble gas ? (c) 6p (d) All
(a) P (b) Q 15. The maximum number of electron if is denoted by
(c) R (d) S Nmax and if n is th principle quantum number then
Passage for Q.10 to Q.12 (a) Nmax = 2n2 (b) Nmax = n2
When an electron is added to a neutral gaseous atom (c) Nmax = 3n2 (d) Nmax = 4n2
energy is released. This is called electron affinity which Passage for Q.16 to Q.18
is defined as the amount of energy released when an A solid melts when the force holding its constituent
electron is added to an isolated gaseous atom is called
units in position is overcomed by thermal energy as
electron affinity. This process is represent as
a consequence of increase in temperature. The melting
atom(g) + electron anion(g) + energy temperature is determined by a number of different
The magnitude of electron affinity measures the ability factors : nature of packing the atoms, ions or molecules
of an atom to hold an additional electron. the lattice energy , association through hydgoren
10. Which of the following process is endothermic - bond etc.
(a) P + e–  P– (b) O + e–  O– 16. Which element of 2nd period has highest melting
(c) Li + e–  Li– (d) N + e–  N– point?
11. Choose the correct pair regarding electron affinity (a) B (b) C
(magnitude) - (c) N (d) O
(a) N > P (b) Li > Na 17. Which element of 3rd period has highest melting
(c) Be > Li (d) N > C point?
12. According to Millikan’s, if an atom has high (a) Al (b) Si
electronegativity than other atom then it must have -
(c) Cl (d) Ar
(a) high ionisation energy only
18. Correct melting point order
(b) high electron affinity only
(a) P > N (b) S > O
(c) less s character in hybrid orbital
(c) Cl > F (d) all
(d) high (ionisation energy + electron affinity)

www.jeebooks.in
PERIODIC TABLE 9.21
Passage for Q.19 to Q.21 24. The most reactive non metal.
In periodic table elements are devided with metals and (a) VI (b) III
non-metals. Metals comprise more than 78% of all (c) IV (d) V
known elements and appear on the left side on the 25. The least reactive non metal.
periodic table. Non-metals which are located at the
(a) I (b) III
top high hard of the periodic table. Metallic character
increases with increasing atomic number in a group (c) IV (d) V
whereas decreases from left to right in a period. 26. The metal which can form a stable binary halide of
19. Non-metals are usually solids or gases at room the formula MX2 (X = halogen).
temperature with low melting and boiling point. The (a) I (b) III
exception is - (c) IV (d) V
(a) F (b) B 27. The metal which can form a predominantly stable
(c) C (d) both Boron & carbon covalent halide of the formula MX (X = halogen).
20. Which properties are characteristic feature of (a) I (b) III
metals. (c) IV (d) V
(a) Metals are good conductor of heat of electricity Passage for Q.28 to Q.30
(b) Metals are malleable and ductile When Schrodinger equation is solved for hydrogen atom,
(c) Metals are usually solid at room temperature the solution gives the possible energy levels the electron
(d) All can occupy and the corresponding wave functions ()
of the electron associated with each energy level. The
21. Which of the following is not a metalloid/semi
quantised energy states and the corresponding wave
metal ?
functions which are characterised by a set of quantum
(a) Bi (b) As numbers (principal quantum number n, azimuthal quantum
(c) Ge (d) Sb number l, magnetic quantum number ml).
Passage for Q.22 to Q.27 28. Principal quantum number n -
The first (iH1) and the second (iH2) ionization (a) Can be positive or can be negative
enthalpies (in kJ mol–1) and the (egH) electron gain (b) Is always positive integer
enthalpy (in kJ mol–1) of a few elements are given below:
(c) Can be zero
Element H1 H2 egH
(d) Can be fraction
I 520 7300 –60
29. Principal quantum number determines -
II 419 3051 –48
(a) shape of the orbital
III 1681 3374 –328
(b) the size and to large extent the energy of the
IV 1008 1846 –295 orbital
V 2372 5251 +48 (c) the spatial orientation of the orbital with respect
VI 738 1451 –40 to standard sets of coordination axis.
22. The least reactive element. (d) the spin of electrons
(a) II (b) III 30. The value of n is highest for
(c) IV (d) V (a) 4s orbital
23. The most reactive element. (b) 6s orbital
(a) II (b) III (c) 4f orbital
(c) IV (d) V (d) 3d orbital

www.jeebooks.in
9.22 PERIODIC TABLE
Passage for Q.31 to Q.33 36. Similarities in the pair B and Si is illustrated by
Azimuthal quantum number ‘’ defines the three which of the following fact
dimensional shape of the orbital. For a given value of n, (a) Oxides of boron and silicon both are reduced
 can have n number of values ranging 0 to n–1. For in electric furnance to give elemental B and Si
example, when n = 1, value of  is only zero. For n = 2, respectively.
the possible value of  can be 0 and 1. For n = 3 the (b) Both B and Si forms oxyacid H3BO3 and
possible values of  are 0,1,2. H4SiO4 which are monobasic acid
31. For  = 4, number of orbitals will be (c) Both halides of boron and silicon are reduced
(a) 5 (b) 7 by LiAlH4 to produce their corresponding
(c) 9 (d) 11 hydrides
32. Which orbitals are described by n = 5,  = 3 and (d) Maximum valency of boron and silicon are 6
n = 4,  = 0. Passage for Q.37 to Q.39
(a) 5f, 4p (b) 5g, 4d Phosphorous is an element with atomic number 15. It
(c) 5d, 4s (d) 5f, 4s is considered as nonmetal. It forms some oxyacid like
H3PO2 , H3PO3 , H3PO4. It has different allotropes like
33. Value of (n + ) = 5 is applicable for which orbital
white phosphorous , red phosphorous, black
(a) 4p (b) 3d phosphorous.
(c) 4d (d) both (a) and (b) 37. Total number of orbitals which have (n+) = 3 for
Passage for Q.34 to Q.36 phosphorous atom
The diagonal relationship signifies the similarities in (a) 3 (b) 4
properties of the elements of the 2nd period with the (c) 5 (d) 6
respective diagonally opposite 3rd period elements
38. Total number of electrons which can have ml = 0
between the succesive groups in the periodic table. Such
for phosphorous atom
diagonally opposite pairs are Li and Mg ; Be and Al ; B
and Si. (a) 3 (b) 6
34. Li and Mg resembles in which of the following way. (c) 9 (d) 12
(a) Both Li and Mg reacts with water to give O2 39. Total number of maximum electrons which can have
gas s = +1/2 for phosphorous atom
(b) Both Li and Mg-nitride reacts with water to (a) 6 (b) 9
give NH3 gas (c) 12 (d) 15
(c) Both Li and Mg-nitrate undergo heating to give Passage for Q.40 to Q.42
N2 gas Atomic radii increase down a group, within the s and p
(d) Both LiOH and Mg(OH)2 are very much water block, decrease from left to right across a period. The
soluble lanthanoide contraction results in a decrease in atomic
35. Be and Al properties are similar,. It is supported radius for elements following the f -block. All
by which fact monoatomic anions are lareger than their parent atoms
and all monoatomic cations are smaller.
(a) Both Be and Al reacts with caustic soda to give
berylate and aluminate with molecular formula 40. The correct order of increasing atomic radius of
NaBeO2 and NaAlO2 the following elements is -
(b) Both Be-carbide and aluminium carbide react (a) S < O < Se < C
with water to give methane gas. (b) O < C < Se < S
(c) Maximum covalency of both Be and Al are 6 (c) O < C < S < Se
(d) Maximum covalency of both Be and Al are 4 (d) C < O < S < Se

www.jeebooks.in
PERIODIC TABLE 9.23
41. The order of increasing ionic radius of the following 46. What is the electronegativity of Arsenic atom
is (having atomic number 33). Given covalent radius
(a) K+ < Li+ < Mg2+ < Al3+ of Arsenic = 120 pm
(b) Mg2+ < Al3+ < K+ < Li+ (a) 3.2 (b) 2.84
(c) 2.61 (d) 2.32
(c) Al3+ < Li+ < Mg2+ < K+
47. According to this equation -
(d) Al3+ < Mg2+ < Li+ < K+
(a) Down the group, electronegativity decreases
42. The order of increasing ionic radius of the following
is (b) Across a period, from left to right
electronegativity increases
(a) S2– < F– < Br– < Te2–
(c) Both (a) and (b) are correct
(b) F– < Br– < Se2– < Te2–
(d) Electronegativity does not depend on covelent
(c) O2– < Cl– < I– < Se2– radius
(d) O2– < Se2– < Cl– < Te2– Passage for Q.48 to Q.50
Passage for Q.43 to Q.45 Hund’s rule suggest that the ground state of an atom
The polarisability of an atom is its ability to be distorted should contain maximum number of unpaired electrons
by an electric field. An atom or ion (mostly anion) is (within the same subshell). Also electrons in different
highly polarisable if its electron distribution can be orbitals of the same energy will have their spins parallel.
distorted readily. A polarisable atom or ion is one with 48. Spin multiplicity for np3 electronic configuration
orbitals that lie close in energy, large, heavy atoms and (a) 1 (b) 2
tend to be highly polarisable. On the other hand, species (c) 3 (d) 4
that effectively distorted the electron distribution of an 5
49. Spin multiplicity for nd electronic configuration
neighbouring atom or cation are described as having
(a) 4 (b) 5
polarising ability.
(c) 6 (d) 7
43. The anion which is most polarisable among halide
50. For d7 ion number of unpaired electron with parallel
ion is
spin will be
(a) F– (b) Cl–
(a) 1 (b) 2
(c) Br– (d) I–
(c) 3 (d) 4
44. Correct option - Passage for Q.51 to Q.53
(a) Na+ has less polarising power than Cs+ The half filled or full filled orbitals are more stable
(b) Te2– has less polarisability than O2– compared to the nearest half filled or full filled orbitals
(c) Ag+ has more polarising power than K+ respectively. This enhanced stability for half filled and
full filled orbitals will be explained with the help of
(d) Hg2+ has less polarising power than Ca2+
exchange energy. d10 configuration has additional
45. Which of th following has highest polarisability - exchange energy which overcomes the disfavour created
(a) F– (b) O2– due to additional pairing.
(c) S2– (d) N3 – 51. The actual electronic configuration of Pd (atomic
Passage for Q.46 to Q.47 number = 46) is -
Allred-Rochow equation regarding electronegativity of (a) [Kr] 4d10 5s0 (b) [Kr] 4d9 5s1
element is given as follows (c) [Kr] 4d8 5s2 (d) [Kr] 4d10 5s1
0.36Z* 52. The actual electronic configuration of Ag (atomic
en   0.744
d number = 47) is -
d = covalent radius of atom in Å (a) [Kr] 4d10 5s0 (b) [Kr] 4d9 5s1
Z = effective nuclear charge of atom (c) [Kr] 4d8 5s2 (d) [Kr] 4d10 5s1

www.jeebooks.in
9.24 PERIODIC TABLE
53. The actual electronic configuration of Gd (atomic Passage for Q.57 to Q.58
number = 64) is - The principal quantum number of the outermost shell
(a) [Xe] 4f8 5d0 6s2 (b) [Xe] 4f7 5d1 6s2 of M1,M2 and M3 is n = 3. M1 forms the only oxide
(c) [Xe] 4f7 5d0 6s2 (d) [Xe] 4f8 5d1 6s1 (M1)2O with oxygen. The oxide on being dissolved in
Passage for Q.54 to Q.56 water produces strong alkali. The covalent hydride of
M2 is (M2)H3 and the formula of highest oxides of M2 is
Born-Haber cycle is the thermodynamic cycle which
(M2)2O5. The hydride of M3 is strong acid and on being
determines lattice energy of a compound. The most
ionised M3 produces M3– ion.
stable crystal structure of the compound is commonly
the structure with the greatest lattice energy under 57. M2 & M3 are respectively -
prevailing condition (a) N, F
1 (b) P, Cl
Na(s) + Cl (g) NaCl(s) ............(i) (c) N, Cl
2 2
1 (d) P, F
Mg(s) + O (g) MgO(s) ............(ii)
2 2 58. Incorrect option regarding M1 -
3 (a) M1 is example of strong reducing element
Al(s) + F (g) AlF3(s) ............(iii)
2 2 (b) M1 is example of strong oxidising element
Given : Sublimation energy of sodium = 109 kJ mol–1
(c) M1 is hightly reactive metal
Ionisation energy of sodium = 494 kJ mol–1
(d) Dianion of M1 does not exist
Enthalpy of formation of NaCl(s)
Passage for Q.59 to Q.60
= –414 kJ mol–1
If we have Born Haber cycle with this equation, then
Electrone gain enthalpy of Cl = –347 kJ mol–1 following energies are associated :
Bond dissociation energy of Cl2 = 242 kJ mol–1 Na(s)
+
Na (aq) + e
54. Lattice energy of NaCl(s) - H1
H2 H4
(a) 2560 kJ mol–1 (b) 791 kJ mol–1
+ –
(c) 1582 kJ mol–1 (d) 320 kJ mol–1 Na(g) Na (g) + e
H3
55. The correct expression of heat of formation of MgO 59. H2 and H3, H4 refers :-
with other thermodynamical data is
(a) Fusion energy of sodium, ionisation energy of
(a) Hf = S+IE1 + IE2 + D – EA1 – U sodium, hydration energy of sodium
(b) Hf = S+IE1 + IE2 + D – EA1 – EA2 – U (b) Vaporisation energy of sodium, electron affinity
(c) Hf = S+2IE1 + D/2 – 2(EA1) – U of sodium, hydration energy of sodium
( Hf = S+IE1 + IE2 + D/2 – (EA1) – (EA2) – U (c) Sublimation energy of sodium, ionisation energy
56. To prepare AlF3 form Al(s) and F2(g), at first Al3+(g) of sodium, hydration energy of sodium
and F–(g) has to be generated. Correct option (d) Bond dissociation energy of sodium, hydration
(a) For Al(g) Al3+(g) energy required is 3IE1 energy of sodium
(b) For Al(g) Al3+(g) energy required is IE1 + 60. The expression of H1 in terms of H2,H3,H4 (with
IE2 + IE3 sign) is -
3 (a) –H1=–H2–H3+H4
(c) For F (g)  3F–(g) energy required is
2 2 (b) –H1=H2+H3–H4
3D (c) –H1=–H2+H3–H4
– 3EA1
2
(d) –H1=H2–H3+H4
(d) Both (b) and (c) are correct

www.jeebooks.in
PERIODIC TABLE 9.25
Matrix Match Type : 65. Match the column :-
61. Match the column :- Column - I Column - II
Atomic number Feature
Column - I Column - II
(a) 38 (P) s-block element
Property Ordered
(b) 51 (Q) p-block element
(a) Electronegativity order (P) N>B (c) 29 (R) d-block element
(b) Electron affinity order (Q) Se>Br (d) 64 (S) 5th period element
(c) Ionisation energy order (R) C>B (T) Rare earth element
(d) No. of valance electron (S) O>S 66. Match the column :-
62. Match the column :- Column - I Column - II
Element Feature
Column - I Column - II
(a) Hg (P) Solid at room temperature
Electronic configuration Value of born
(b) Mn (Q) Liquid at room temperature
exponent (c) Zn (R) d-block element
(a) 1s2 (P) 10 (d) W (S) Transition element
(b) 2s22p6 (Q) 9 (T) (n–1)d 10 ns 2 or (n–1)d 5 ns 2
(c) 3s23p6 or 3s23p63d10 (R) 7 configuration
67. Match the column :-
(d) 4s 4p or 4s 4p 4d
2 6 2 6 10
(S) 5
Column - I Column - II
63. Match the column :- Element Feature
Column - I Column - II (a) Osmium (Os) (P) Inner transition element
Compound Melting point (b) Promethium (Pm) (Q) Transition element
(a) BeCl2 (P) 872°C (c) Magnesium (Mg) (R) Highest positive
oxidation state
(b) MgCl2 (Q) 772°C
(d) Germanium (Ge) (S) Typical element
(c) CaCl2 (R) 712°C
(T) Used as semiconductor
(d) SrCl2 (S) 405°C 68. Match the column :-
64. Match the column :- Column - I Column - II
Column - I (Order) Pair of atoms Feature
(a) Na   F   O 2   N 3 (a) S,Se (P) Chalcogen family
(b) Cl, Br (Q) Halogen family
(b) Li   Na   K   Rb   Cs  (c) N, P (R) Pnicogen family
(c) O > S > F > Cl (d) Be,Al (S) Diagonal relationship
(T) Non metals
(d) Cl  K   Ca 2   Sc3
69. Match the column :-
Column - II (Property) Column - I Column - II
(P) Electronegativity Order Property
(Q)Mobility of hydrated ions (a) As > Se (P) Atomic number
(R)Ionisation energy (b) Se < Br (Q) Atomic radius
(S) Electron affinity (c) Mg < Al (R) Electronegativity
(d) K < Mg (S) Number of valance electrons
(T) Ionic size
(T) Ist ionisation energy

www.jeebooks.in
9.26 PERIODIC TABLE
70. Match the column :- 75. Match the column :-
Column - I Column - II Column - I Column - II
Formula of hydride Elements Metal ion Ionic radii (symmetric)
(a) MH4 (P) Groups 16 elements (a) Mg2+ (P) 202 pm
(b) MH3 (Q) Groups 15 elements (b) Li +
(Q) 179 pm
(c) MH2 (R) Groups 14 elements (c) I– (R) 102 pm
(d) MH (S) Groups 13 elements (d) Br –
(S) 92 pm
(T) Groups 17 elements 76. Match the column :-
71. Notation of IUPAC nomenclature of elements Consider chlorine atom for given property
Column - I Column - II Column - I Column - II
Name Digit/Abbreviation (a) Electronegativity in Pauling (P) 1.732
(a) nil (P) 0/n Scale
(b) enn (Q) 9/e (b) Spin magnetic moment in BM (Q) 0
(c) sept (R) 7/s (c) Z/e ratio (R) 3
(d) un (S) 1/u (d) Number of vacant orbital in (S) 1
72. Match the column :- outermost shell
Column - I Column - II 77. Match the column :-
C–F bond distance Molecule Column - I Column - II
(a) 139.1 pm (P) CH3F Element Number of valence
(b) 135.8 pm (Q) CH2F2 electrons
(c) 133.2 pm (R) CHF3 (a) Group 13 elements (P) 3
(d) 132 pm (S) CF4 (b) Group 14 elements (Q) 4
73. Match the column :- (c) Group 18 elements (R) 8
Column - I Column - II (d) Group 2 elements (S) 2
Molecular formula Atomic number 78. Match the column :-
(a) AB2 (P) Atomic number ofA and B Column - I Column - II
are respectively 13 and 53
Order Property
(b) A3B2 (Q) Atomic number ofA and B
(a) Cl < K < Ca < Sc
– + 2+ 3+
(P) Electronegativity
are respectively 12 and 07
(b) O < S< F <Cl (Q) Nuclear charge
(c) AB3 (R) Atomic number ofA and B
are respectively 56 and 09 (c) Li < Na >K < Rb < Cs (R) Size
+ + + + +

(d) AB5 (S) Atomic number ofA and B (d) Na+ < F– < O2– < N3– (S) Electron affinity
are respectively 15 and 17 (T) Ionisation energy
74. Match the column :- 79. Match the column :-
Column - I Column - II Column - I Column - II
Metal Metallic Radii (pm) (a) Fe Fe
3+ 2+
(P) Exothermic in nature
(a) Rb (P) 137 pm (b) Br Br– (Q) Endothermic in nature
(b) Ca (Q) 157 pm (c) H H –
(R) Becomes diamagnetic
(c) Mn (R) 197 pm (d) N2 N (S) µ undergoes a change
(d) Li (S) 250 pm (T) Becomes paramagnetic

www.jeebooks.in
PERIODIC TABLE 9.27
80. Match the column :- 85. Match the column :-
Column - I Column - II Column - I Column - II
(a) MnO (P) Aqueous solution is basic in Orbital Number of nodal plane
nature (a) py orbital (P) Two nodal plane present in xy
(b) Mn2O7 (Q) Aqueous solution is acidic in and yz plane
nature (b) dzx orbital (Q) One nodal plane present in xz
(c) Al2O3 (R) Amphoteric in nature plane
(d) ZnO (S) Highest oxidation state (c) dz2 orbital (R) Zero nodal plane present but
(T) Transition metal oxide two nodal cones are present
81. Match the column :- (d) s orbital (S) Zero nodal plane is present.
Column - I Column - II 86. Match the column :-
Element Atomic radius (pm) Column - I Column - II
(a) Be (P) 88 Overlap of orbitals Bond formation
(b) C (Q) 80 (a) px + dxz (P) bond is formed
(c) O (R) 77 (z axis is internuclear axis)
(d) B (S) 66 (b) px + dx2 – y2 (Q) bond is formed
82. Match the column :- (x axis is internuclear axis)
Column - I Column - II (c) dz2 + dz2 (R) No bond is formed
Electronic Electron gain (z axis is internuclear axis)
configuration enthalpy / kJ mol–1 (d) px + dxy (S) Head on overlap
(a) 1s2 2s2 sp6 (P) –53 (z axis is internuclear axis)
(b) 1s 2s 2p 3s
2 2 6 1
(Q) –328 (T) Side way overlap
(c) 1s 2s 2p
2 2 5
(R) –141 87. Match the column :-
(d) 1s2 2s2 2p4 (S) +48 Column - I Column - II
83. Match the column :- Oxidation state Compound
of sulphur
Column - I Column - II
(a) Zero (P) Dimethyl sulphoxide
Element H 1 H 2 egH
(b) +6 (Q) Sulphuryl chloride
(a) Most reactive non (P) 419 3051 –48
metal (c) +4 (R) Sulphonyl chloride
(d) –2 (S) Sodium sulphide
(b) Most reactive metal (Q) 1681 3374 –328
(T) Sulphuric acid
(c) Least reactive element (R) 738 1451 –328
88.Match the column :-
(d) Metal forming binary (S) 2372 5251 +48
halide Column - I Column - II
84.Match the column :- Shell Number of electrons in
subshell or shell
Column - I Column - II
(a) K (P) 2 electrons in s subshell
Electrons Z effective
(b) L (Q) 6 electrons in p subshell
(a) 4d electron in Pd (P) 6.85
(c) M (R) 10 electrons in d subshell
(b) 3d electron in Ni (Q) 3.9
(d) N (S) 14 electrons in f subshell
(c) 3p electron in Ca (R) 8.75
(T) Even number of electron in
(d) 2s electron in N (S) 7.55
each shell

www.jeebooks.in
9.28 PERIODIC TABLE
89. Match the column :- 93. Match the column :-
Column - I Column - II Column - I Column - II
Type of elements Elements Oxide Nature of oxide
(a) Typical elements (P) Aluminium (a) Cs2O (P) Acidic oxide
(b) Eka-boron (Q) Gallium (b) Cl2O7 (Q) Basic oxide
(c) Eka-silicon (R) Scandium (c) BeO (R) Amphoteric oxide
(d) Eka-aluminium (S) Germenium (d) CO (S) Neutral oxide
(T) Chlorine 94. Match the column :-
90. Match the column :- Column - I Column - II
Column - I Column - II Type of elements Example/Features
Period Number and types orbitals (a) Inert gases (P) He, Ne, Ar, Kr, Xe
involved / No. of elements (b) Representative (Q) II-A to VII-A elements
(a) 3rd (P) s,p,d,f all orbitals involved elements
(b) 4 th
(Q) 9, 18 (c) Transition (R) Incomplete d-shell either
(c) 6th (R) 4, 8 elements in ground state or in most
(d) 7 th
(S) 16, 32 stable valency state
91. Match the column :- (d) Inner transition (S) The last electron goes to
Column - I Column - II elements the f-orbital of antepenul-
timate shell either in
Elements Features
ground state or in any
(a) Chromium (P) half filled d-orbitals common oxidation state
(At. No. 24)
95. Match the column :-
(b) Rhodium (Q) full filled d-orbitals
Column - I Column - II
(At. No. 45)
Elements Feature
(c) Platinum (R) one electron present in ns
(At. No. 78) orbitals (a) Sc (P) Not considered as transition
elements
(d) Molybdenum (S) eight electrons present in
(At. No. 42) (n–1) d orbitals (b) Ti (Q) Highest covalent radii
(T) nine electrons present in (c) Ni (R) Least covalent radii
(n–1) d orbitals (d) Zn (S) Odd number of unpaired d
92.Match the column :- electrons
Column - I (Symbol Name) (T) Even number of unpaired d
(a) Unniltrium electrons (excluding zero)
(b) Ununqnadium 96. Match the column :-
(c) F > O > N Column - I Column - II
(d) F2 > O2 > N2 Elements Feature
Column - II (Atomic number) (a) Na (P) Transition element
(P) 117 (b) Ag (Q) Member of the 4th peirod
(Q) 108 (c) Co (R) Behaves as metal
(R) 103 (d) Br (S) Behaves as non metal
(S) 114 (T) One unpaired electron

www.jeebooks.in
PERIODIC TABLE 9.29
97. Match the column :- 100. Match the column :-
Column - I (Pair of elements) Column - I Column - II
(a) Mg, Cl Reactions Nature of reactions
(b) Se, Te (a) O(g) O (g)
2–
(P) Endothermic process
(c) V, Cd (b) O–(g) O2–(g) (Q) Exothermic process
(d) B, Si (c) O(g) O–1(g) (R) I st electron gain
Column - II (Feature) enthalpy of oxygen
(P) Same group (d) O+1(g) O2+(g) (S) IInd ionisation energy
(Q)Diagonal relationship of oxygen
(R)Same period (T) II nd electron gain
(S) Same block elements enthalpy ofoxygen
(T) Both element in pair have atomic number less than EXERCISE # IV
50
98. Match the column :-  Integer Type :
Column - I Column - II 1. Number of elements present in group-17.
Compounds Feature 2. Number of elements present in group-18.
(a) Magnesium (P) ns np6 configuration for both 3. An element (M) has atomic number, A M2– has
oxide cation and anion electronic configuration resembles with inert gas
(b) Potassium (Q) Isoelectronic cation and configuration. That element belongs to 2nd period.
chloride anion What is the atomic number of A.
(c) Sodium (R) Cation and anion have +1 and 4. An element has electronic configuration
fluoride –1 oxiation state respectively [Kr]4d75s2. On the basis of this electronic
(d) Barium (S) Cation and anion have 2 and configuration, find out the group number of elements
sulphide –2 oxidation state respectively according to modern periodic table :-
(T) Largest internuclear distance 5. Number of unpaired electron present in 4p orbital
99. Match the column :- of arsenic atom.
Column - I 6. Ionisation potential of Bex+ is found to be 217.6
Element electron volt. What is the value of x -
(a) Gadolinium 7. How many hydroxides are basic in nature ?
(b) Potasssium CsOH ; Se(OH)2 ; IOH ; ClOH ; Sr(OH)2 ;
P(OH)3
(c) Chromium
(d) Gallium 8. On the Pauling electronegativity scale, what is
the atomic number of the element next to fluorine
Column - II
atom.
Fullfilled orbital Half filled orbital
9. Find out electronegativity of chlorine atom on
(P) 1s, 2s, 2p, 3s, 3p, 3d, 4s 4p
Pauling scale if ionisation energy of Cl– is 4 eV and
(Q) 1s, 2s, 2p, 3s, 3p, 3d, 4s electron affinity of Cl+ is 13 eV.
(R) 1s, 2s, 2p, 3s, 3p 4s 10. In the periodic table, if there are 10 periods, then
(S) 1s, 2s, 2p, 3s, 3p, 3d, 4s, 4f what is the number of maximum elements present
4p, 4d, 5s, 5p, 6s in 10th period.

www.jeebooks.in
9.30 PERIODIC TABLE
11. During formation of NaCl, from Na(s) and Cl2(g), Hf + –

how many steps are endothermic 19. ½F2(g)+BF3(g)+O2(g) O2 BF4 (s)

(a) Na(s)  Na(g) ½ H1


1
(b) Cl (g)  Cl(g) F(g)+BF3(g)+O2(g)
2 2
(c) Na(g) – e–  Na+(g)
H2 H5
(d) Cl(g) + e–  Cl–(g)
(d) Na+(g) + Cl–(g)  NaCl(s) –
F (g)+BF3(g)+O2(g)
12. Total number of gaseous molecules
H3
He ; N2 ; O2 ; Cl2; F2 ; H2; Xe ; Kr ; Ar
13. How many columns are present in p block –
BF4 (g)+O2(g) –
BF4 (g)+O2 (g)
+

elements ? H4
14. If columns are present in d block elements is p, How many statements are correct :
then find out p–1 (a) H1 corresponds to bond dissociation energy
15. Find the number of elements which has higher EA2 of F2.
than EA1 (b) H2 corresponds to electron affinity of F.
P; N ; C ; Se ; Br ; Na ; Be ; Mg ; O ; B (c) H3 corresponds to electron affinity of BF3.
16. Count the number of properties which have higher (d H4 corresponds to ionisation energy of O2.
values for nitrogen atom as compared to (e) H5 corresponds to lattice energy of O2+ BF4–.
phosphorus atom (magnitude) :- H6 step-VI
20. NH4Cl(s) NH4+(g) + Cl–(g)
(i) electronegativity
(ii) electron affinity H1 step-I H5 step-V
(iii) ionisation potential NH3(g)+HCl(g)
+
NH3(g)+H (g)+Cl (g)

(iv)covalent radius
H2 step-II H4 step-IV
(v) proton electron ratio
step-III
17. Oxidation state of nitrogen is positive integer for NH3(g)+H(g)+Cl(g) NH3(g)+H+(g)+Cl(g)
H3
how many of the following compounds : How many statements are correct :
(a) Hydrazoic acid (a) H1 corresponds to heat of formation of
(b) Magnesium nitride NH4Cl.
(c) Ammonium chloride (b) H2 corresponds to bond dissociation energy
(d) Hydrazine of HCl.
(e) Laughing gas (c) H3 corresponds to ionisation energy of H(g)
(f) Nitrosyl chloride (d H4 corresponds to electron gain enthalpy of
18. Find out lattice energy of KI Cl(g)
Given : Sublimation energy of I2 = 14 kcals mol–1 (e) H5 corresponds to proton affinity of NH3
Sublimation energy of K = 20 kcals mol–1 (f) H6 corresponds to lattice energy of NH4Cl.
IE1 of K = 92 kcals mol–1 (g) Step II and Step III are endothermic process.
Dissociation energy of I2 = 28 kcals mol–1 (h) Step IV and Step V are exothermic process.
Electron affinity of I2 = –70 kcals mol–1 (i) Step I is exothermic process
(j) Step VI is exothermic process.

www.jeebooks.in
PERIODIC TABLE 9.31
21. Find the total number of cations for which I.P. of (f) Mercury and Bromine are liquids at room
carbon is lower than the corresponding atom. temperature.
(a) Na+ (b) Mg+ (g) Screening effect is not observed in He+1, Li+2,
(c) Sn4+ (d) Fe2+ Be3+ ion.
(e) Cl+ (f) S+ (h) Oxidation state of S in S2Cl2 is +1.
22. Find out the total number of acidic compound. 28. Find out number of electrons lost by Cr atom to
(a) ClO2(OH) produce an isoelectronic ion of S2– ion.
(b) BrO3(OH) 29. Number of electrons in Na (atomic number = 11)
having m = 0.
(c) Sr(OH)2
30. Number of electrons in S (atomic number = 16)
(d) NO2(OH)
having (n+) = 3.
(e) Mg(OH)2 (f) PO(OH)3
31. Number of lanthanoids in Modern periodic table.
(g) SO2(OH)2
32. The maximum number of electrons in phosphorous
23. How many electrons satisfy n = 3, l = 2, s = +½ atom for which n +  + m = 3.
24. Calculate Zeff. of valence electron of Gd (atomic 33. Find out maximum number of electrons in Cr atom
number 64) for which  + m = 0.
25. Ionisation enthalpies of elements of second period 34. Electrons of one subshell do not participate in
are given below. (in Kcals mole–1) [not in sequence] bonding due to inert pair effect. Find out (n+) for
2080 ; 1086 ; 899 ; 1314 ; 801 ; 520 ; 1402 ; that subshell.
1681 35. For one element shielding effect is not possible.
If the atomic number of element with ionisation What is the atomic number of that element ?
energy 1314 Kcal mole–1 is x. 36. According to Long form of modern periodic table,
If the atomic number of element with ionisation chalcogens are elements of group number x. Find
enthalpy 1080 Kcal moles is y. the value of x.
Then find out x - y : 37. The elements which exhibit both vertical and
O O horizontal similarities are called transition elements.
What is the atomic number of the Ist element of
26. HO – S – S – S* – S – S –OH
transition elements.
O O 38. An element has spin magnetic moment equal to
What is the oxidation state of S* : 1.73 B.M.. It has atomic number in between 22
27. How many statements are correct ? to 30. Find out atomic number of element which is
(a) Na+ has higher value of ionisation enthalpy than just after it along a period in the modern periodic
Ne change both have same electronic table.
configuration. 39. How many compouds having higher melting point
(b) Ist ionisation energy of nitrogen is higher than TiC.
than oxygen and opposite for 2nd ionisation NaF, BeO, MgO, CaO, BaO, CsCl
energy. 40. How many elements have 3d 7 electronic
(c) Electrons gain enthalpy of oxygen less negative configuration in M+3 state. (Ti to Ni)
than selenium. Co, Fe, Ni, Mn, Cu, Zn
(d) As and Sb are example of metalloid. 41. How many ions are diamagnetic.
(e) d block elements generally exhibit variable Sc3+, Ti3+, Ti2+, V2+, Cr3+, Mn2+, Fe2+, Cu2+, Co2+,
valency. Zn2+

www.jeebooks.in
9.32 PERIODIC TABLE
42. Number of 3d block elements which can have +4 49. How many of the following statements related to
oxidation state. the modern periodic table is incorrect ?
43. Internuclear axis (i) The p-block has 6 columns, because a
x y z maximum of 6 electrons can occupy all the
orbitals in a p-shell.
Px+Px (ii) The d-block has 8 columns, because a
Pz+Pz maximum of 8 electrons can occupy all the
orbitals in a d-subshell.
Pz+Pz
(iii) Each block contains a number of columns equal
Number of overlap give  bond = x. to the number of electrons that can occupy that
Number of overlap giving  bond = y. subshell.
Find out x + y. (iv)The block indicates value of azimuthal quantum
44. Total numbe of ions present in solution when TlI3 number (l) for the last subshell that received
dissociated with ions completely. electrons in building up the electronic
45. How many overlaps give  bond. (If inter nuclear configuration.
axis is x axis). (v) An element with mass number 37 possesses
(i) s + s (v) px + py one unit of negative charge. If the ion contains
(ii) s + px (vi) px + px 11.1% more neutrons than the electrons, then
(iii) s + py (vii) px + pz the symbol of ion is 3717Cl–1
(iv)s + pz (viii) py + py (vi)An element with mass number 81 contains
31.7% more neutrons as compared to proton
46. Internuclear axis the atomic symbol of the element is 8135Br.
x y z 50. If the number of electrons in the species H2+, H2
z
dxy+dxy and O2+ are x,y,z respectively then find out x  y .
dxz+dxz
EXERCISE # V
dx2– y2 +dx2– y2
(JEE- MAIN & ADVANCED)
dz2 + dz2
1. The correct order of second ionisation potentital
Find out total how many overlaps give  bond. of carbon, nitrogen, oxygen and fluorine is :
47. Internuclear axis [IIT-JEE, 1981]
x y z (a) C > N > O > F (b) O > N > F > C
(c) O > F > N > C (d) F > O > N > C
dxy+dxy
2. The element with the highest first ionization potential
dxz+dxz is : [IIT-JEE, 1982]
dx2– y2 +dx2– y2 (a) boron (b) carbon
(c) nitrogen (d) oxygen
dz2 + dz2 3. The first ionization potentials (in electron volts) of
Find out number of overlaps which give  bond. nitrogen and oxygen atoms are, respectively, given
48. If number of pentagon and number of hexagon in by : [IIT-JEE, 1987]
fullerene C60 is x and y then find out y – x (a) 14.5, 13.6 (b) 13.6, 14.6
(c) 13.6, 13.6 (d) 14.6, 14.6

www.jeebooks.in
PERIODIC TABLE 9.33
4. Atomic radii of fluorine and neon (in Angstron units) 11. Which of the following has the maximum number
are, respectively given by : [IIT-JEE, 1987] of unpaired electrons : [IIT-JEE, 1996]
(a) 0.72, 1.60 (b) 1.60, 1.60 (a) Mg2+ (b) Ti3+
(c) 0.72, 0.72 (d) None of these (c) V3+ (d) Fe2+
5. The electronegativity of the following elements 12. Which of the following statements is wrong ?
increases in the order : [IIT-JEE, 1987] [IIT-JEE, 1997]
(a) C, N, Si, P (b) N, Si, C, P (a) The first ionisation potential of Al is less than
(c) Si, P, C, N (d) P, Si, N, C the first ionization potential of Mg.
6. The first ionization potentials of Na, Mg, Al and Si (b) The second ionisation potential of Mg is greater
are in the order : [IIT-JEE, 1988] than the second ionization potential of Na.
(a) Na < Mg < Al < Si (b) Na > Mg > Al > Si (c) The first ionisation potential of Na is less than
(c) Na < Mg < Al > Si (d) Na > Mg > Al < Si the first ionization potential of Mg.
7. Which of the following is smallest is size : (d) The third ionisation potential of Mg is greater
than the third ionization potential of Mg.
[IIT-JEE, 1988]
13. The correct order of acidic strength is :
(a) N3– (b) O2–
(a) Cl2O7 > SO2 > P4O10 [IIT-JEE, 2000]
(c) F– (d) Na+
(b) CO2 > N2O5 > SO3
8. Among the following elements (whose electronic
configurations are given below) , the one having (c) Na2O > MgO > Al2O3
the highest ionization energy : [IIT-JEE, 1990] (d) K2O > CaO > MgO
(a) [Ne]3s23p1 (b) [Ne]3s23p3 14. The correct order of radii is : [IIT-JEE, 200]
(c) [Ne]3s23p2 (d) [Ne]3d104s24p1 (a) N < Be < B
9. Which of the following statements is not correct (b) F– < O2– < N3–
for the periodic classification of elements ? (c) Na < Li < K
[IIT-JEE, 1992] (d) Fe3+ < Fe2+ < Fe4+
(a) The properties of elements are the periodic 15. The incorrect statements among the following is :
functions of their atomic numbers. [JEE, 1997]
(b) Non-metallic elements are lesser in number (a) The first ionisation energy of Al is less than first
than metallic elements ionization energy of Mg
(c) The first ionisation energies of elements along (b) The second ionisation energy of Mg is greater
a period do not vary in a regular manner with than second ionization energy of Na.
increase in atomic number. (c) The first ionisation energy of Na is less than
(d) For transition elements, the d-subshells are filled first ionization energy of Mg.
with electrons monotonically with increase in (d) The third ionisation energy of Mg is greater than
atomic number. third ionization energy of Al.
10. Which of the following has the most stable +2 16. Property of the alkaline earth metals that increases
oxidation state ? [IIT-JEE, 1995] with their atomic number is : [JEE, 2000]
(a) Sn (a) ionisation energy
(b) Pb (b) solubility of their hydroxides
(c) Fe (c) solubility of their sulphates
(d) Ag (d) electronegativity

www.jeebooks.in
9.34 PERIODIC TABLE
17. Arrange the following ions in order of their 24. The set with correct order of acidity is :
increasing size : Li+, Mg2+, K+, Al3+:[JEE, 1997] [JEE, 2001]
18. Compounds that formally contain Pb4+ are easily (a) HClO < HClO2 < HClO3 < HClO4
reduced to Pb2+. The stability of the lower oxidation (b) HClO4 < HClO3 < HClO2 < HClO
state is due to .............. [JEE, 1997] (c) HClO < HClO4 < HClO3 < HClO2
19. Asseration : F atom has a less negative electron (d) HClO4 < HClO2 < HClO3 < HClO
affinity than Cl atom. [JEE, 1998]
25. Identify the correct order of acidity strengths of
Reason : Additional electron are repelled more CO2, CuO, CaO, H2O : [JEE, 2002]
effectively by 3p electrons in Cl atom than by 2p
(a) CaO < CuO < H2O < CO2
electrons in F atom.
(b) H2O < CuO < CaO < CO2
(a) Both Asseration and Reason are true, and
Reason is the correct explanation of Asseration. (c) CaO < H2O < CuO < CO2
(b) Both Asseration and Reason are true, and (d) H2O < CO2 < CaO < CuO
Reason is not correct explanation of Asseration. 26. Identify the least stable ion amongst the following :
(c) Asseration is true but Reason is false. (a) Li– [JEE, 2002]
(d) Asseration is fasle but Reason is true. (b) Be–
20. Ionic radii of : [JEE, 1999] (c) B–
(a) Ti4+ < Mn7+ (d) C–
(b) 35Cl– < 37Cl– 27. Arrange the following oxides in the increasing order
(c) K+ > Cl– of Bronsted basicity : [JEE, 2004]
(d) P3+ > P5+ Cl2O7 , BaO , SO3 , CO2 , B2O3
21. Give reason for the following in one or two 28. Asseration : Pb 4+ compounds are stronger
sentences only : [JEE 1999] oxidizing agents than Sn4+ compounds. [JEE,
2008]
CrO3 is an acid anhydride.
Reason : The higher oxidation states for the group
22. Asseration : The first ionization energy of Be is
14 elements are more stable for the heavier
greater than that of B. [JEE, 2000]
members of the group due to inert part effect.
Reason : 2p orbital is lower in energy than 2s.
(a) Both Asseration and Reason are true, and
(a) Both Asseration and Reason are true, and Reason is the correct explanation of Asseration.
Reason is the correct explanation of Asseration.
(b) Both Asseration and Reason are true, and
(b) Both Asseration and Reason are true, and Reason is not correct explanation of Asseration.
Reason is not correct explanation of Asseration.
(c) Asseration is true but Reason is false.
(c) Asseration is true but Reason is false.
(d) Asseration is fasle but Reason is true.
(d) Asseration is fasle but Reason is true.
29. Which one of the following ions has the highest
23. The set representing the correct order of fist value of ionic radius - [AIEEE, 2004]
ionization potential is [JEE, 2001]
(a) Li+
(a) K > Na > Li
(b) B3+
(b) Be > Mg > Ca
(c) O2–
(c) B > C > N
(d) F–
(d) Ge > Si > C

www.jeebooks.in
PERIODIC TABLE 9.35
2–
30. The formation of theoxide ion O (g) requires first 35. The lanthanide contraction is responsible for the
an exothermic and then an endothermic step as fact that : [AIEEE 2005]
shown below : [AIEEE, 2004] (a) Zr and Y have about the same radius
O(g) + e = O (g) ; H° = – 142 kJmol–1
– –
(b) Zr and Nb have similar oxidation state
O–(g) + e– = O2–(g) ; H° = 844 kJmol–1 (c) Zr and Hf have about the same radius
This is because - (d) Zr and Zn have similar oxidation state
(a) oxygen is more electronegative 36. The increasing order of the first ionization enthalpies
(b) oxygen has high electron affinity of the elements B, P, S and F (lowest first) is :
(c) O– ion will tend to resist the addition of another [AIEEE 2006]
electrons (a) F < S < P < B (b) P < S < B < F
(d) O– ion has comparatively larger size than (c) B < P < S < F (d) B < S < P < F
oxygen atom 37. Which of the following statements is true ?
31. Among Al2O3, SiO2, P2O3 and SO2 the correct [AIEEE 2006]
order of acid strength is: [AIEEE, 2004] (a) H3PO4 is a stronger acid than H2SO3
(a) SO2 < P2O3 < SIO2 < Al2O3 (b) In aqueous medium, HF is stronger acid than
(b) SiO2 < SO2 < Al2O3 < P2O3 HCl
(c) Al2O3 < SiO2 < SO2 < P2O3 (c) HClO4 is a weaker acid than HClO3
(d) P2O3 < SiO2 < Al2O3 < SO2 (d) HNO3 is a stronger acid than HNO2
32. Which of the following oxides is amphoteric in 38. Lanthanoid contraction is caused due to :
nature :: [AIEEE, 2005] [AIEEE, 2007]
(a) CaO (b) CO2 (a) the appreciable shielding on outher electrons
(c) SiO2 (d) SnO2 by 4f electrons from the nuclear charge
33. In which of the following arrangements the order (b) the appreciable shielding on outher electrons
is NOT according to the property indicated against by 5f electrons from the nuclear charge
it ? [AIEEE, 2005] (c) the same effecitive nuclear charge from Ce
(a) Al3+ < Mg2+ < Na+ < F– - increasing ionic size to Lu
(b) B < C < N < O - increasing first ionisation (d) the imperfect shielding on outer electrons by 4f
enthalpy electrons from the nuclear charge
(c) I < Br < F < Cl - increasing electron gain 39. The stability of dihalides of Si, Ge, Sn and Pb increases
enthalpy (with negative size) steadily in the sequence : [AIEEE, 2007]
(d) Li < Na < K < Rb - increasing metallic radius (a) SiX2 << GeX2 << SnX2 << PbX2
34. Which of the following factors may be (b) PbX2 << SnX2 << GeX2 << SiX2
regarded as the main cause of lanthanide (c) GeX2 << SiX2 << SnX2 << PbX2
contraction : [AIEEE, 2005] (d) SiX2 << GeX2 << PbX2 << SnX2
(a) greater shielding of 5d-electrons by 4f- 40. The set representing the correct order of ionic
electrons radius is: [AIEEE, 2009]
(b) poorer shielding of 5d-electrons by 4f-electrons (a) Na+ > Li+ > Mg2+ > Be2+
(c) effective shielding of one of 4f-electrons by (b) Li+ > Na+ > Mg2+ > Be2+
another in the sub-shell (c) Mg2+ > Be2+ > Li+ > Na+
(d) poor shielding of one of 4f-electrons by another (d) Li+ > Be2+ > Na+ > Mg2+
in the sub-shell

www.jeebooks.in
9.36 PERIODIC TABLE
41. In which of the following arrangements , the 46. The increasing order of the ionic radii of the given
sequence is not strictly according to the property isoelectronic species is - [AIEEE, 2012]
written against it ? [AIEEE, 2009] (a) Cl–, Ca2+, K+, S2–
(a) HF < HCl < HBr < HI : increasing acid strength (b) S2–, Cl–, Ca2+, K+
(b) NH3 < PH3 < AsH3 < SbH3 : increasing basic (c) Ca2+, K+, Cl–, S2–
strength (d) K+, S2–, Ca2+, Cl–
(c) B < C < O < N : increasing first ionization 47. Which of the following represent the correct order
enthalpy of increasing first ionization enthalpy for Ca, Ba,
(d) CO2 < SiO2 < SnO2 < PbO2 : increasing S, Se and Ar ? [AIEEE, 2013]
oxidising power (a) Ca < S < Ba < Se < Ar
42. The correct sequence which shows decreasing (b) S < Se < Ca < Ba < Ar
order of the ionic radii of the elements is :
(c) Ba < Ca < Se < S < Ar
[AIEEE, 2010]
(d) Ca < Ba < S < Se < Ar
(a) Al3+ > Mg2+ > Na+ > F– > O2–
48. The first ionization potential of Na is 5.1 eV. The
(b) Na+ > Mg2+ > Al3+ > O2– > F–
value of electron gain enthalpy of Na + will
(c) Na+ > F– > Mg2+ > O2– > Al3+ be: [AIEEE, 2014]
(d) O2– > F– > Na+ > Mg2+ > Al3+ (a) –2.55 eV
43. The outer electron configuration of Gd (atomic (b) –5.1 eV
number 64) is: [AIEEE, 2011]
(c) –10.2 eV
(a) 4f35d56s2
(d) +2.55 eV
(b) 4f85d106s2
49. Among the followng oxoacids, the correct
(c) 4f45d56s2 decreasing order of acid stength is:
(d) 4f735d16s2 [AIEEE, 2014]
44. Which of the following orders present the correct (a) HClO4 > HClO3 > HClO2 > HOCl
sequence of the increasing basic nature of the given
(b) HClO2 > HClO4 > HClO3 > HOCl
oxides ? [AIEEE 2011]
(c) HOCl > HClO2 > HClO3 > HClO4
(a) Al2O3 < MgO < Na2O < K2O
(d) HClO4 > HOCl > HClO2 > HClO3
(b) MgO < K2O < Al2O3 < Na2O
50. In the long form of the periodic table, the valence
(c) Na2O < K2O < MgO < Al2O3
shell electronic configuration of 5s25p4 corresponds
(d) K2O < Na2O < Al2O3 < MgO to the element present in : [AIEEE, 2015]
45. The correct order of electron gain enthalpy (a) Group 16 and period 5
with negative sign of F, Cl, Br and I having
(b) Group 17 and period 6
atomic number 9, 17, 35 and 53 respectively
as : [AIEEE, 2012] (c) Group 17 and period 5
(a) F > Cl > Br > I (d) Group 16 and period 6
(b) Cl > F > Br > I 51. In an atom, the total number of electrons
having quantum numbers n = 4, |ml| = 1 and
(c) Br > Cl > I > F
ms = –½ is : [AIEEE, 2014]
(d) I > Br > Cl > F


www.jeebooks.in
ANSWERS
CH -1 CH EM I CAL B ON D I N G
EXERCISE # I
1. (d) 2. (d) 3. (c) 4. (c) 5. (d) 6. (b) 7. (c) 8. (b) 9. (d) 10. (c)
11. (d) 12. (d) 13. (c) 14. (c) 15. (c) 16. (b) 17. (a) 18. (d) 19. (b) 20. (d)
21. (d) 22. (a) 23. (b) 24. (a) 25. (c) 26. (b) 27. (c) 28. (a) 29. (a) 30. (c)
31. (d) 32. (c) 33. (c) 34. (c) 35. (c) 36. (d) 37. (d) 38. (a) 39. (c) 40. (d)
41. (b) 42. (d) 43. (d) 44. (b) 45. (a) 46. (c) 47. (a) 48. (a) 49. (a) 50. (c)
51. (b) 52. (b) 53. (b) 54. (b) 55. (c) 56. (c) 57. (d) 58. (d) 59. (b) 60. (a)
61. (c) 62. (d) 63. (c) 64. (a) 65. (a) 66. (a) 67. (b) 68. (d) 69. (a) 70. (d)
71. (d) 72. (a) 73. (c) 74. (d) 75. (d) 76. (d) 77. (a) 78. (c) 79. (a) 80. (d)
81. (d) 82. (c) 83. (b) 84. (c) 85. (c) 86. (c) 87. (a) 88. (c) 89. (c) 90. (b)
91. (d) 92. (a) 93. (d) 94. (c) 95. (b) 96. (c) 97. (c) 98. (b) 99. (d) 100. (c)
101. (d) 102. (a) 103. (c) 104. (c) 105. (a) 106. (d) 107. (b) 108. (c) 109. (c) 110. (c)
111. (d) 112. (b) 113. (c) 114. (b) 115. (c) 116. (c) 117. (a) 118. (d) 119. (d) 120. (d)
121. (b) 122. (d) 123. (c) 124. (d) 125. (b) 126. (b) 127. (b) 128. (d) 129. (d) 130. (c)
131. (a) 132. (d) 133. (d) 134. (c) 135. (a) 136. (c) 137. (a) 138. (c) 139. (d) 140. (d)
141. (c) 142. (a) 143. (c) 144. (c) 145. (c) 146. (a) 147. (b) 148. (d) 149. (b) 150. (a)
151. (d) 152. (b) 153. (d) 154. (d) 155. (c) 156. (a) 157. (a) 158. (d) 159. (d) 160. (a)
161. (d) 162. (b) 163. (c) 164. (c) 165. (a) 166. (b) 167. (d) 168. (a) 169. (c) 170. (b)
171. (b) 172. (c) 173. (a) 174. (b) 175. (b) 176. (c) 177. (d) 178. (c) 179. (c) 180. (c)
181. (d) 182. (a) 183. (c) 184. (d) 185. (a) 186. (c) 187. (a) 188. (b) 189. (d) 190. (d)
191. (d) 192. (d) 193. (b) 194. (a) 195. (b) 196. (d) 197. (a) 198. (d) 199. (c) 200. (b)
EXERCISE-II
1. (a,b,c,d) 2. (a,c,d) 3. (a,b,d) 4. (a,c,d) 5. (b,c,d) 6. (a,b,c,d)
7. (a,b) 8. (b,c) 9. (a,d) 10. (a,b,c) 11. (b,d) 12. (a,b,d)
13. (a,d) 14. (b,c,d) 15. (a,c,d) 16. (b,c) 17. (a,b,d) 18. (a,b,d)
19. (c,d) 20. (a,d) 21. (c,d) 22. (a,b) 23. (b,d) 24. (a,c,d)
25. (b,c) 26. (a,c,d) 27. (a,c,d) 28. (a,d) 29. (a,b) 30. (b,c)
31. (a,c,d) 32. (b,c) 33. (a,b,c) 34. (a,b,c,d) 35. (a,b,c,d) 36. (b,d)
37. (a,b,c,d) 38. (a,b) 39. (c,d) 40. (a,b) 41. (a,d) 42. (b,c,d)
43. (a,c,d) 44. (a,b) 45. (b,d) 46. (b,d) 47. (a,c) 48. (a,b)
49. (b,d) 50. (b,c) 51. (a,b,c,d) 52. (a,d) 53. (c,d) 54. (a,b)
55. (a,d) 56. (c,d,f) 57. (a,c) 58. (a,b,d) 59. (a,c,d) 60. (b,c)
61. (a,b,c) 62. (b,c) 63. (a,c) 64. (c,d) 65. (a,c) 66. (a,b,c)
67. (a,b,d) 68. (a,c,d) 69. (a,b) 70. (a,b,c) 71. (b,c,d) 72. (a,d)
73. (a,b,c,d) 74. (a,b) 75. (a,b,c,d) 76. (a,b,c) 77. (b,c) 78. (c,d)
79. (a,c) 80. (a,d) 81. (a,b,d) 82. (a,d) 83. (b,c) 84. (a,b,c)
85. (a,b) 86. (a,b,c) 87. (a,b,c,d) 88. (a,b) 89. (a,c) 90. (a,c,d)

www.jeebooks.in
10.2 ANSWERS
91. (a,c) 92. (a,b,d) 93. (a,b,c,d) 94. (c,d) 95. (a,d) 96. (b,d)
97. (a,b,c) 98. (a,b,d) 99. (c,d) 100. (a,b) 101. (c,d) 102. (a,d)
103. (a,b,c) 104. (a,c) 105. (a,b,c) 106. (a,d) 107. (a,b,d) 108. (a,b,c,d)
109. (a,b) 110. (a,d) 111. (a,b,c,d) 112. (a,b,c,d) 113. (a,c,d) 114. (a,b,c)
115. (a,d) 116. (c,d) 117. (a,d) 118. (b,c) 119. (a,b) 120. (a,b)
121. (a,b,c) 122. (a,d) 123. (a,b,c) 124. (c,d) 125. (a,b,d) 126. (a,b)
127. (a,b,c,d) 128. (b,c,d) 129. (a,d) 130. (a,c,d) 131. (b,c) 132. (a,b,c)
133. (a,b,c,d) 134. (b,c) 135. (a,d) 136. (a,b,c,d) 137. (a,c,d) 138. (b,c)
139. (b,c) 140. (a,b,c) 141. (a,b,c) 142. (b,c,d) 143. (a,b,c,d) 144. (a,b,c)
145. (a,c,d) 146. (a,b,c,d) 147. (a,b,d) 148. (a,b,c) 149. (c,d) 150. (c,d)
EXERCISE-III : Paragraph Type & Matrix Match Type
1. (c) 2. (c) 3. (b) 4. (a) 5. (c) 6. (d) 7. (b) 8. (c) 9. (d) 10. (d)
11. (b,d) 12. (c) 13. (b) 14. (d) 15. (b) 16. (a) 17. (c) 18. (b) 19. (b) 20. (c)
21. (a) 22. (c) 23. (c) 24. (d) 25. (d) 26. (d) 27. (d) 28. (a) 29. (b) 30. (c)
31. (b) 32. (b) 33. (d) 34. (c) 35. (d) 36. (c) 37. (d) 38. (d) 39. (a) 40. (d)
41. (d) 42. (c) 43. (a) 44. (b) 45. (a) 46. (a) 47. (d) 48. (c) 49. (b) 50. (a)
51. (b) 52. (d) 53. (d) 54. (d) 55. (d) 56. (d) 57. (d) 58. (d) 59. (d) 60. (b)
61. (d) 62. (a) 63. (c) 64. (b) 65. (d)
66. (A)-P ; (B)-Q ; (C)-S ; (D)-R 67. (A)-P ; (B)-Q ; (C)-R ; (D)-S
68. (A)-Q ; (B)-P ; (C)-R ; (D)-S 69. (A)-P, Q, R,S ; (B)-P,Q,R,S ; (C)-P,R,S ; (D)-P,R,S
70. (A)-Q, R, S ; (B)-Q,R ; (C)-P,R ; (D)-Q,R,S 71. (A)-S ; (B)-R ; (C)-Q; (D)-P
72. (A)-P ; (B)-Q,S ; (C)-P,R ; (D)-Q,R 73. (A)-P ; (B)-Q,S ; (C)-P,R ; (D)-Q,S
74. (A)-Q,R,T ; (B)-P,Q,R,S ; (C)-P,R,S ; (D)-Q,R,T 75. (A)-P,R,S ; (B)-R ; (C)-Q,R ; (D)-Q,R
76. (A)-P,S ; (B)-S ; (C)-Q,R ; (D)-P,Q,R 77. (A)-R, T ; (B)-Q,S,T ; (C)-P,R,T; (D)-P,S,T
78. (A)-P,S ; (B)-Q,S ; (C)-R,S; (D)-S 79. (A)-R ; (B)-S ; (C)-Q ; (D)-P
80. (A)-P ; (B)-P,Q ; (C)-R,S ; (D)-S,T 81. (A)-P,S,T ; (B)-P,Q,T ; (C)-P,R,T ; (D)-P,T
82. (A)-P,Q,R,S,T ; (B)-P,Q,R,S,T ; (C)-Q,R,T ; (D)-Q,R,T
83. (A)-P,R ; (B)-P,Q ; (C)-P,R ; (D)-P,S 84. (A)-S ; (B)-R ; (C)-Q ; (D)-P
85. (A)-R ; (B)-P,Q,S,T ; (C)-P,R,S ; (D) -T 86. (A)-P, Q ; (B)-P ; (C)-R ; (D)-S
87. (A)-P, S, T ; (B)-P,S ; (C)-P,Q ; (D)-R 88. (A)-P ; (B)-Q ; (C)-R,T ; (D)-S
89. (A)-Q, S, T ; (B)-R,S ; (C)-S ; (D)-P,S 90. (A)-P,Q ; (B)-R ; (C)-S,T ; (D)-P,S
91. (A)-R ; (B)-Q ; (C)-P ; (D)-S 92. (A)-R,T ; (B)-P,T ; (C)-Q,T ; (D)-P,S
93. (A)-P ; (B)-Q ; (C)-R ; (D)-S
94. (i) -X, Z (ii) -P,Q,S (iii) -P,R,S (iv) -T,X,Y
(v) -T,X (vi) -T,V,X,Z (vii) -U,Y (viii) -W,X,Z
95. (i) -Q, W (ii) -Q,W,Y (iii) -Q,U,W,Y (iv) -Q,R,U,W,Y
(v) -P,Q,U,W,Y (vi) -Q,R,U,W,Y (vii) -Q,S,U,W,Y (viii) -Q,S,W,X,Y
(ix) -Q,T,V,W,Y (x) -Q,T,V,W,Y

www.jeebooks.in
ANSWERS 10.3
96. (A)-P,Q,R,S ; (B)- P,Q,R,S; (C)-Q,R,S ; (D)-Q,R 97. (A)-R ; (B)-R,S ; (C)-P,Q,R ; (D)-P,R
98. (A)-S ; (B)-P ; (C)-R ; (D)-Q 99. (A)-P, Q, R ; (B)-Q,R ; (C)-P,Q,R,S ; (D)-P,Q,R
100. (A)-P ; (B)-Q ; (C)-R ; (D)-S
EXERCISE-IV : Integer Type
1. (5) 2. (5) 3. (6) 4. (1) 5. (3) 6. (5) 7. (4) 8. (7) 9. (7) 10. (6)
11. (2) 12. (3) 13. (9) 14. (1) 15. (6) 16. (5) 17. (2) 18. 12 or 3 19. (1) 20. (3)
21. (5) 22. (5) 23. (3) 24. (5) 25. (3) 26. (2) 27. (9) 28. (2) 29. (1) 30. (5)
31. (4) 32. (3) 33. (0) 34. (0) 35. (2) 36. (3) 37. (1) 38. (3) 39. (0) 40. (7)
41. (2) 42. (4) 43. (8) 44. (4) 45. (7) 46. (4) 47. (8) 48. (5) 49. (5) 50. (3)
51. (6) 52. (1) 53. (2) 54. (6) 55. (3) 56. (2) 57. (5) 58. 7, 25 59. (3) 60. (9)
61. (5) 62. (4) 63. (2) 64. (6) 65. (6) 66. (4) 67. (1) 68. (2) 69. (4) 70. (3)
71. (9) 72. (4) 73. (5) 74. (4) 75. (4) 76. (0) 77. 24 or 6 78. (6) 79. (3) 80. (6)
EXERCISE-V(A) : (JEE-MAIN)
1. (3) 2. (2) 3. (2) 4. (1) 5. (1) 6. (2) 7. (3) 8. (1) 9. (4) 10. (3)
11. (2) 12. (4) 13. (4) 14. (3) 15. (2) 16. (3) 17. (4) 18. (1) 19. (3) 20. (4)
21. (1) 22. (1) 23. (4) 24. (1) 25. (1) 26. (3) 27. (4) 28. (2) 29. (1) 30. (4)
31. (1) 32. (4) 33. (4) 34. (2) 35. (2) 36. (Bonus) 37. (2) 38. (4) 39. (3)4 0 .
(3)
41. (3) 42. (4) 43. (2) 44. (1) 45. (3) 46. (2) 47. (2) 48. (1) 49. (1) 50. (3)
51. (3) 52. (2) 53. (1) 54. (2) 55. (4) 56. (1) 57. (3) 58. (2) 59. (3) 60. (3)
61. (3) 62. (2) 63. (1) 64. (3) 65. (2) 66. (1) 67. (2) 68. (2) 69. (1) 70. (1)
71. (3) 72. (1) 73. (4) 74. (2,3,4) 75. (4) 76. (2,4) 77. (2) 78. (4) 79. (2) 80. (2)
81. (2) 82. (2) 83. (2) 84. (4)
EXERCISE-V(B) : (JEE-ADVANCED)
1. (b) 2. (b) 3. (a) 4. (a) 5. (a) 6. (a) 7. (c) 8. (a) 9. (b) 10. (d)
11. (d) 12. (d) 13. (a) 14. (c) 15. (c) 16. (c) 17. (c)
18. (A)-P,Q,R,T ; (B)-Q,R,S,T ;(C)-P,Q,R ;(D)-P,Q,R,S
19. (a,b,c) 20. (b,c) 21. (d) 22. (a) 23. (b) 24. (b) 25. (d) 26. (4) 27. (0) 28. (3)
29. (6) : H2SO4 ; H3PO4 ;H2CO3 ;H2S2O7 ;H2CrO4 ;H2SO3
..
NH2
.. ..
N N
30. (2) ; F, Na 31. (5) 32. (6) ; 33. (c) 34. (c)
.. ..
NH2 N
.. NH2
35. (D) ; BeCl2 , N3– , N2O , NO2+
..
O
..
..

..
..

36. (8) ; N – N .. N–O–N 37. (b,c)38. (b,c,d)


O O
..
..

..O O
..

..

.. .. ..

www.jeebooks.in
10.4 ANSWERS
CH -2 COORDI N AT I ON COM POU ND S
EXERCISE # I
1. (b) 2. (c) 3. (c) 4. (c) 5. (d) 6. (b) 7. (d) 8. (c) 9. (d) 10. (a)
11. (c) 12. (d) 13. (c) 14. (b) 15. (b) 16. (c) 17. (d) 18. (d) 19. (c) 20. (a)
21. (d) 22. (c) 23. (c) 24. (c) 25. (a) 26. (c) 27. (b) 28. (d) 29. (d) 30. (c)
31. (d) 32. (a) 33. (c) 34. (c) 35. (c) 36. (c) 37. (d) 38. (b) 39. (d) 40. (a)
41. (a) 42. (b) 43. (a) 44. (a) 45. (d) 46. (c) 47. (d) 48. (c) 49. (d) 50. (b)
51. (d) 52. (c) 53. (a) 54. (a) 55. (a) 56. (d) 57. (c) 58. (b) 59. (a) 60. (c)
61. (a) 62. (c) 63. (c) 64. (d) 65. (d) 66. (a) 67. (b) 68. (c) 69. (b) 70. (c)
71. (d) 72. (b) 73. (a) 74. (a) 75. (d) 76. (c) 77. (d) 78. (d) 79. (c) 80. (a)
81. (b) 82. (a) 83. (a) 84. (a) 85. (d) 86. (b) 87. (c) 88. (b) 89. (a) 90. (b)
91. (d) 92. (a) 93. (d) 94. (c) 95. (c) 96. (c) 97. (a) 98. (c) 99. (d) 100. (c)
101. (d) 102. (d) 103. (d) 104. (b) 105. (c) 106. (b) 107. (b) 108. (b) 109. (a) 110. (d)
111. (d) 112. (d) 113. (a) 114. (d) 115. (a) 116. (c) 117. (a) 118. (d) 119. (c) 120. (d)
121. (d) 122. (d) 123. (c) 124. (c) 125. (d) 126. (d) 127. (d) 128. (c) 129. (a) 130. (c)
131. (a) 132. (c) 133. (b) 134. (b) 135. (a) 136. (d) 137. (b) 138. (c) 139. (c) 140. (c)
141. (c) 142. (a) 143. (d) 144. (c) 145. (d) 146. (b) 147. (b) 148. (d) 149. (d) 150. (a)
EXERCISE # II
1. (a,b,c) 2. (a,b,c) 3. (b,d) 4. (b,c,d) 5. (a,b,d) 6. (a,b)
7. (a,d) 8. (a,d) 9. (b,c,d) 10. (b,c) 11. (a,d) 12. (a,b)
13. (a,b) 14. (a,b,c) 15. (a,b,c,d) 16. (a,b,c,d) 17. (a,b,c,d) 18. (b,c,d)
19. (a,b,c) 20. (a,b,c,d) 21. (b,d) 22. (a,b,c,d) 23. (a,b,c) 24. (b,c)
25. (a,d) 26. (a,b,d) 27. (a,b,c,d) 28. (b,c) 29. (a,b,c) 30. (b,c)
31. (c,d) 32. (a,b,c) 33. (b,c,d) 34. (a,c) 35. (a,b,c,d) 36. (a,b,c)
37. (a,b,c) 38. (b,c,d) 39. (b,c) 40. (a,b,c) 41. (a,b,c) 42. (a,c)
43. (a,d) 44. (a,b) 45. (a,d) 46. (a,c) 47. (a,c) 48. (a,b,c,d)
49. (b,c) 50. (a,b,c) 51. (b,d) 52. (a,c) 53. (b,c) 54. (b,c,d)
55. (a,b,d) 56. (c,d) 57. (a,b,d) 58. (a,b) 59. (b,c) 60. (a,b,c)
61. (b,c,d) 62. (a,b) 63. (b,d) 64. (a,c,d) 65. (b,c,d) 66. (a,d)
67. (a,b,d) 68. (a,b,c,d) 69. (a,b) 70. (c,d) 71. (c,d) 72. (a,b,d)
73. (a,c) 74. (b,c) 75. (c,d) 76. (a,b,c) 77. (b,c,d) 78. (a,b,d)
79. (b,c,d) 80. (a,b,c,d) 81. (a,b) 82. (a,c,d) 83. (a,b,c,d) 84. (a,b,c,d)
85. (a,b,c,d) 86. (a,b) 87. (a,b,d) 88. (b,c,d) 89. (b,c) 90. (a,c,d)
91. (b,d) 92. (a,b,d) 93. (b,c) 94. (b,c,d) 95. (c,d) 96. (a,b,c,d)
97. (a,b,c) 98. (a,b) 99. (a,b) 100. (a,b,c)

www.jeebooks.in
ANSWERS 10.5
EXERCISE # III
Paragraph Type :
1. (c) 2. (c) 3. (a) 4. (d) 5. (a) 6. (b) 7. (d) 8. (a) 9. (a) 10. (c)
11. (b) 12. (b) 13. (a) 14. (c) 15. (b) 16. (c) 17. (d) 18. (b) 19. (c) 20. (c)
21. (b) 22. (c) 23. (d) 24. (b) 25. (d) 26. (d) 27. (d) 28. (c) 29. (a) 30. (c)
31. (c) 32. (d) 33. (b) 34. (c) 35. (d) 36. (b) 37. (d) 38. (c) 39. (a) 40. (b)
41. (a) 42. (c) 43. (c) 44. (a) 45. (b) 46. (d) 47. (b) 48. (a,b,c,d)
49. (a,b,c,d) 50. (a,b,c,d) 51. (a,b,c,d) 52. (a,c) 53. (a,c) 54. (b) 55. (a)
56. (a,b,c,d) 57. (b) 58. (c) 59. (b) 60. (d) 61. (b) 62. (b) 63. (c) 64. (c)
65. (c) 66. (a) 67. (c) 68. (a) 69. (b) 70. (c)
Matrix Match Type :
(a) (b) (c) (d) (a) (b) (c) (d)
71. S; -P,T ; Q,R,T; R,T 72. -P,T; -R,S,T; -Q,S,T; -P
73. -P, R, S; -P,Q,R,S,T; -P,Q,R; -P,Q,R 74. -P, R, T; -Q,S,T; -P,R,T; -Q,S,T
75. -P; -Q; -R,T; -S 76. -P; -Q; -R; -S
77. -S,T; -R,T; -S,T; -Q,T 78. -R; -Q; -S; -P
79. -Q; -P; -S; -R 80. -P,R; -T; -P,S; -Q
81. -P; -Q; -R; -S 82. P; -Q; -R; -S
83. -P, Q, S; -P,Q,S,T; -P,Q,R; -P,Q,R 84. P,R,S; -R, S; -R, S; -W,R,S
85. -P,S; -P,Q; -R,T; -P,Q 86. -Q,R,S; -P,R; -Q,R; -R,S
87. -P,Q,R,S; -Q,R; -Q,R; -P,R,S 88. Q,R; -Q,S; -Q,S; -P,R,S
89. P; -P,Q; -S,T; -P,Q,R 90. -P,Q; -Q,R,S; -P; -P,Q
91. -R; -T; -P; -P 92. -R,S; -P,Q; -P; -P,Q
93. -Q,R; -P,T; -Q,R; -S,T 94. -P; -Q; -R; -S
95. -R; -S; -P; -Q 96. -P; -Q; -R; -S
97. - P; - Q; - R; -S 98. - S; - R; - Q; -P
99. - S; - R; - Q; -P 100. - P,S; - R; - P,Q,S; - P,S,T
101. - P,R,S; - P,T; - P,Q,T; - P,R,S
EXERCISE # IV
1. (4) 2. (4) 3. (4) 4. (6) 5. (7) 6. (7) 7. (2) 8. (3) 9. (3) 10. (5)
11. (4), a,b,f,g 12. (3) 13. (6) 14. (6) 15. (2) 16. (4) 17. (6) 18. (7) 19. (6)
20. (4) 21. (6) 22. (5) 23. (3) 24. (8) 25. (7) 26. (0) 27. (9) 28. 32/5 29. (5)
30. (4) 31. (5) 32. (2) 33. (2) 34. (6) 35. 5 36. 2 37. 2 38. 3 39. 8
40. 11/2 41. 2 42. 3 43. 6 44. 0 45. 9 46. 1 47. 3 48. 4 49. 9/18
50. 3

www.jeebooks.in
10.6 ANSWERS
EXERCISE # V(A) JEE-MAIN
1. (b) 2. (c) 3. (d) 4. (a) 5. (c) 6. (b) 7. (a) 8. (c) 9. (d) 10. (c)
11. (c) 12. (a) 13. (b) 14. (d) 15. (d) 16. (a) 17. (c) 18. (d) 19. (d) 20. (a)
21. (d) 22. (b) 23. (c) 24. (b) 25. (a) 26. (a) 27. (b) 28. (c) 29. (c) 30. (c)
31. (a) 32. (c) 33. (c) 34. (d) 35. (c) 36. (d) 37. (d) 38. (d) 39. (a)40. (c)
41. (c) 42. (c) 43. (b) 44. (d) 45. (d) 46. (a) 47. (c) 48. (c) 49. (d)50. (c)
51. (c) 52. (a) 53. (b)
EXERCISE # V(B) JEE-ADVANCED
True/False
1. (F) 2. (b) 3. (F) 4. (a) 5. (a) 6. (b) 7. (b) 8. (b) 9. Paramagnetism
10. Hexaamminecobalt(III)chloride 11. (c,d) 12. (d) 13. (d) 14. (c) 15. (c) 16. (a)
17. (b) 18. (d) 19. (b) 20. (b) 21. (b) 22. (a) 23. (a) 24. (c) 25. (a) 26. (a)
27. (d) 28. (a)-P,Q,S ; (b)-P,R,S ; (c)-Q,S ; (d)-Q,S 29. (c) 30. (c) 31. (b) 32. (b)
2+
H H -
O
O O O
H H H O
33. (a) 34. (a) 35. (c,d) 36. Cu O S 37. (b) 38. (3)
H H H O
O O O
H H -
O

39. (c) 40. (b) 41. (c) 42. (6) 43. (d) 44. (a) 45. (c) 46. (b) 47. (8) 48. (b,d)
– –
49. (b) 50. (4) 51. I is reducing agent than cl 52. (i) Pentaamminenitritocobalt(III)
chloride (ii) Potassiumhexacyanochromate(III) 53. Pentaamminecarnonatochromium(III)chloride
2+
54. (i) [co(NH3)5cl] (ii) Li[alH4] 55. (a) [cr(H2O)6]cl3 ; (b) [cr(H2O)5cl]cl2.H2O ; (c) 
[cr(H2O)4cl2]cl.H2O 56. d2sp3, dsp2 and sp3 57. (a) [cr(NH3)4clbr]cl ;
(b) [cr(NH3)4cl2]br In both cr is d2 sp3 hybridised and magnetic moment is 15BM .
58. [Nicl4]2–  sp3, 8BM [Ni(cN)4]2–  dsp2, 0 59. Potassium amminetetracyanonitrosochro


O–H.............O
N N
mate(I) d2sp3 , octahedral 60. H3C – C C–CH3 61. (4) 62. (3)
Ni
H3C – C
N N=C–CH3

O..........H — O
2 2+
dsp , Ni , diamagnetic
63. (6) 64. (b) 65. (5) 66. (a)

www.jeebooks.in
ANSWERS 10.7

CH -3 M ETAL L U RGY
EXERCISE # I
1. (d) 2. (d) 3. (b) 4. (d) 5. (d) 6. (a) 7. (b) 8. (b) 9. (a) 10. (c)
11. (a) 12. (d) 13. (d) 14. (c) 15. (c) 16. (c) 17. (b) 18. (d) 19. (b) 20. (a)
21. (b) 22. (c) 23. ( ) 24. (a) 25. (c) 26. (c) 27. (c) 28. (d) 29. (d) 30. (a)
31. (b) 32. (a) 33. (c) 34. (c) 35. (b) 36. (c) 37. (c) 38. (c) 39. (d) 40. (d)
41. (a) 42. (a) 43. (a) 44. (c) 45. (a) 46. (a) 47. (b) 48. (c) 49. (c) 50. (a)
51. (b) 52. (a) 53. (b) 54. (c) 55. (b) 56. (b) 57. (b) 58. (a) 59. (c) 60. (c)
61. (b) 62. (b) 63. (c) 64. (a) 65. (b) 66. (c) 67. (c) 68. (d) 69. (c) 70. (d)
71. (d) 72. (c) 73. (b) 74. (a) 75. (d) 76. (a) 77. (b) 78. (a) 79. (b) 80. (c)
81. (a) 82. (d) 83. (d) 84. (c) 85. (d) 86. (c) 87. (d) 88. (c) 89. (b) 90. (a)
91. (c) 92. (d) 93. (b) 94. (d) 95. (d) 96. (c) 97. (d) 98. (c) 99. (d) 100. (c)
EXERCISE # II
1. (b,c,d) 2. (a,b) 3. (a,v,c) 4. (a,b,c) 5. (a,c)
6. (b,c) 7. (a,c) 8. (a,b,c) 9. (a,b,c,d) 10. (a,b,c,d)
11. (a,d) 12. (a,b,c,d) 13. (a,b) 14. (b,c,d) 15. (a,b,d)
16. (b,c) 17. (a,b,d) 18. (b,c,d) 19. (b,c,d) 20. (b,c)
21. (a) 22. (c,d) 23. (a,b,c,d) 24. (a,b,c,d) 25. (a,b,d)
26. (a,b) 27. (a,c,d) 28. (a,c,d) 29. (a,b,c) 30. (a,b,c)
31. (a,b) 32. (a,b,c) 33. (a,b,c) 34. (a,c,d) 35. (a,b)
36. (a,b) 37. (a,c) 38. (b,d) 39. (a,b,d) 40. (a,b,c,d)
41. (b) 42. (a,b,c,d) 43. (a,b,c,d) 44. (a,b,c,d) 44. (a,c)
45. (a,b,d) 46. (a,d) 47. (a,b,c) 48. (b,c) 49. (a,d)
50. (c,d) 51. (a,b,c) 52. (a,b,c) 53. (a,c) 54. (c,d)
55. (c,d) 56. (a,b) 57. (a,b,c) 58. (b,c) 59. (a,b,c)
60. (a,c,d)
EXERCISE # III
1. (a,d) 2. (a) 3. (c) 4. (c) 5. (d) 6. (c) 7. (d) 8. (b) 9. (d) 10. (c)
11. (d) 12. (c) 13. (d) 14. (d) 15. (c) 16. (b) 17. (b) 18. (c) 19. (a) 20.(a)
Matrix Match Type :
(a) (b) (c) (d) (a) (b) (c) (d)
21. S P Q R 22. P R Q S
23. S,T R,T Q,R P,S 24. P, Q, S R R Q,T
25. R S Q P 26. P,R Q S,T Q
27. Q,S,T P,R Q,R,T R,T 28. P,T R Q,R,S Q,S,T
29. P S R Q 30. S P Q R
31. Q,S P P,R Q,S 32. P Q P,R P,R,S
33. S R Q P 34. R S Q P
35. S P R Q

www.jeebooks.in
10.8 ANSWERS
EXERCISE # IV
1. (3) 2. (5) 3. (5) 4. (4) 5. (3) 6. (4) 7. (3) 8. (9) 9. (3) 10. (6)
11. (5) 12. (4) 13. (6) 14. (2) 15. (4) 16. (4) 17. (4) 18. (4) 19. (6) 20. (4)
21. (3) 22. (3) 22. (2) 23. (3) 24. (0) 25. (2) 26. (6) 27. (6) 28. (2) 29. (5)
30. (8) 31. (7) 32. (0) 33. (3) 34. (3) 35. (2) 36. (2) 37. (1) 38. (2) 39. (2)
40. (1)
EXERCISE # V(a) JEE-MAIN
1. (3) 2. (1) 3. (1) 4. (3) 5. (1) 6. (2) 7. (4) 8. (4) 9.(3) 10.(4)
11. (4) 12. (2)
EXERCISE # V(b) JEE-ADVANCED
1.Smelting, Sintering
2. (b) 3. (b) 4. (a,c) 5. (c) 6. (c) 7. (c) 8. (b) 9. (a) 10. (a)
11. (a) 12. (c) 13. (b) 14. (b) 15. (a) 16. (d) 17. (c)
18. (a) (P,R) ; (b) (P) ; (c) (Q) ; (d) (S) 19. (a) (P) ; (b) (Q) ; (c) (P,R) ; (d) (S)
20.Sintering, Smelting 21. (a,c,d) 22. (d) 23. (b) 24. (a) 25. (c,d)
26. (a,c,d) 27. ( ) 28. (a) (P,Q,S) ; (b) (T) ; (c) (Q,R) ; (d) (R)

CH -4 QU AL I TAT I VE SAL T AN ALYSI S


EXERCISE # I
1. (c) 2. (a) 3. (a) 4. (c) 5. (b) 6. (b) 7. (c) 8. (a) 9. (d) 10. (c)
11. (c) 12. (c) 13. (b) 14. (b) 15. (b) 16. (c) 17. (a) 18. (a) 19. (c) 20. (c)
21. (b) 22. (a) 23. (c) 24. (c) 25. (b) 26. (b) 27. (a) 28. (d) 29. (a) 30. (a)
31. (a) 32. (b) 33. (b) 34. (c) 35. (b) 36. (c) 37. (a) 38. (c) 39. (c) 40. (c)
41. (b) 42. (c) 43. (b) 44. (a) 45. (c) 46. (b) 47. (a) 48. (d) 49. (b) 50. (c)
51. (b) 52. (b) 53. (d) 54. (b) 55. (b) 56. (a) 57. (c) 58. (d) 59. (a) 60. (b)
61. (b) 62. (d) 63. (c) 64. (d) 65. (d) 66. (c) 67. (c) 68. (d) 69. (b) 70. (b)
71. (c) 72. (b) 73. (a) 74. (c) 75. (b) 76. (c) 77. (c) 78. (d) 79. (a) 80. (c)
81. (c) 82. (c) 83. (d) 84. (d) 85. (d) 86. (b) 87. (a) 88. (d) 89. (d) 90. (c)
91. (c) 92. (d) 93. (c) 94. (d) 95. (a) 96. (d) 97. (b) 98. (c) 99. (c) 100. (c)
101. (a) 102. (d) 103. (c) 104. (d) 105. (c) 106. (d) 107. (b) 108. (b) 109. (b) 110. (d)
111. (c) 112. (b) 113. (a) 114. (d) 115. (c) 116. (d) 117. (d) 118. (c) 119. (a) 120. (a)
EXERCISE # II
1. (a,b,c,d) 2. (a,c) 3. (a,b) 4. (a,b,c,d) 5. (a,b,c)
6. (b,c) 7. (a,c,d) 8. (a,b,c,d) 9. (b,d) 10. (a,d)
11. (b,c,d) 12. (a,c) 13. (a,c) 14. (a,b) 15. (b,c)
16. (a,b,c,d) 17. (a,b) 18. (a,b) 19. (a,b,c,d) 20. (a,b,c)

www.jeebooks.in
ANSWERS 10.9
21. (a,b,c) 22. (a,b,c,d) 23. (a,b,c,d) 24. (a,b) 25. (a,b,c,d)
26. (a,b,d) 27. (b,c) 28. (c,d) 29. (a,c,d) 30. (a,b,c)
31. (a,b) 32. (b,c,d) 33. (a,b,c) 34. (a,b,c,d) 35. (a,b,c)
36. (a,b,c) 37. (a,b,c,d) 38. (a,b,c) 39. (a,b,c) 40. (a,c,d)
41. (a,b,c) 42. ( a,b,c,d) 43. (b,d) 44. (a,b,c,d) 45. (a,b)
46. (a,b,c) 47. (b,c) 48. (a,c) 49. (b,c) 50. (a,b,d)
51. (a,b,c,d) 52. (a,b,c) 53. (a,b,c) 54. (b,d) 55. (a,c,d)
56. (a,b,c) 57. (a,d) 58. (a,b,c) 59. (a,b,d) 60. (a,b,c,d)
61. (a,c,d) 62. (a,d) 63. (a,b,d) 64. (a,d) 65. (a,b,c,d)
EXERCISE # III
1. (b) 2. (c) 3. (b) 4. (a) 5. (b,c) 6. (b) 7. (a) 8. (c) 9. (c) 10. (b)
11. (c) 12. (d) 13. (b) 14. (d) 15. (a) 16. (b) 17. (a,c,d) 18. (b) 19. (d) 20. (a,c)
21. (a,b,c,d) 22. (a,b,c) 23. (a,b,c,d) 24. (a) 25. (a) 26. (b) 27. (a,b,c,d)
28. (b) 29. (d) 30. (d) 31. (a) 32. (b) 33. (c) 34. (c) 35. (b) 36. (a) 37. (b)
38. (b) 39. (b) 40. (d) 41. (d) 42. (a) 43. (b) 44. (b) 45. (d) 46. (d) 47. (b)
48. (b) 49. (c) 50. (c) 51. (a) 52. (b) 53. (d) 54. (c) 55. (b) 56. (c) 57. (c)
58. (d) 59. (a) 60. (c) 61. (a) 62. (c) 63. (a) 64. (d) 65. (b) 66. (c) 67. (c)
68. (b) 69. (a) 70. (c) 71. (d) 72. (a) 73. (b) 74. (d) 75. (c) 76. (a) 77. (c)
78. (d) 79. (b) 80. (a) 81. (b) 82. (c) 83. (a) 84. (c) 85. (c) 86. (c) 87. (c)
88. (d) 89. (d) 90. (d) 91. (c) 92. (b) 93. (b) 94. (b) 95. (c) 96. (b) 97. (b)
98. (a) 99. (a) 100. (a)
Matrix Match Type :
(a) (b) (c) (d) (a) (b) (c) (d)
101. P,T ; P,S,T ; P,Q,T ; Q,R,T 102. P,R,T ; P,Q,T ; P,S,T ; P,S,T
103. P,Q,R ; P; R; P 104. P,T; P,T; Q; R,S
105. S,T; P,Q,R,S,T; P,S,T; R,S 106. Q,R,S; P,R,S; P,Q,R,S; R,S
107. S; P,Q; R; T 108. R,T; R,T; Q,S; P
109. S; P,Q,R,S; P,R,T; P,S,T 110. R,T; P,Q; Q,S; Q,S
111. S; R; Q; P 112. Q,T; P; R; S
113. P; Q; R; S 114. P,T; Q,R,S; P; P
115. Q,R,T; P,T; P,Q,R,S; P,R,S 116. P; Q; R; S
117. P; P,Q; R,T; Q,R,S 118. R; Q; P; S
119. P.Q; R; P,Q,S; P,Q,S 120. S; P,Q,R,S; P,S; P
121. S; P,Q,T; Q,R,S,T; Q,R,T 122. Q; S; P; R
123. P.Q; R; S; T 124. S; R; Q; P
125. P; Q; R; S

www.jeebooks.in
10.10 ANSWERS
EXERCISE # IV
1. (3) 2. (0) 3. (0) 4. (7) 5. (3) 6. (5) 7. (2) 8. (5) 9. (7) 10. (3)
11. (2) 12. (2) 13. (2) 14. (4) 15. (6) 16. (2) 17. (4) 18. (1) 19. (2) 20. (5)
21. (4) 22. (5) 23. (0) 24. (2) 25. (3) 26. (3) 27. (3) 28. (2) 29. (5) 30. (3)
31. (5) 32. (3) 33. (5) 34. (5) 35. (4) 36. (0) 37. (4) 38. (2) 39. (4) 40. (3)
41. (6) 42. (3) 43. (3) 44. (3) 45. (4) 46. (4) 47. (7) 48. (4) 49. (3) 50. (5)
EXERCISE # V(A) JEE-MAIN
1. (d) 2. (d) 3. (d) 4. (d) 5. (b) 6. (b) 7. (b) 8. (a) 9. (b) 10. (b)
11. (c) 12. (d) 13. (d) 14. (d) 15. (b) 16. (c) 17. (b) 18. (b) 19. (c) 20. (c)
21. (d) 22. (b) 23. (c) 24. (a) 25. (d) 26. (d) 27. (a) 28. (b) 29. (c) 30. (d)
31. (d) 32. (d) 33. (a) 34. (a) 35. (a) 36. (d) 37. (d)
EXERCISE # V(B) JEE-ADVANCED
1. (a,b,c) 2. (a,b,d) 3. (d) 4. (c) 5. (a) 6. (b) 7. (a) 8. (b) 9. (a) 10. (d)
11. (d) 12. (a) 13. (a) 14. AP,S ; BR ; CP,Q ; DP 15. (b) 16. (a,b) 17. (d)
18. (c) 19. (b) 20. AP,S; BQ,S; CR,T; DQ,T 21. (a) 22. (a,c,d) 23. (b,c,d)
24. (b) 25. (a) 26. (c) 27. (a,c,d) 28. (c) 29. (a,c,d) 30. (d) 31. (d)
32. (a) 33. (7) 34. (6) 35. (b) 36. (d) 37. (a,b) 38. (c,d)

CH -5 T H E s-B L OCK EL EM EN T S AN D T H EI R COM POU N D S


EXERCISE # I
1. (b) 2. (d) 3. (b) 4. (d) 5. (c) 6. (b) 7. (d) 8. (d) 9. (a) 10. (b)
11. (c) 12. (c) 13. (b) 14. (c) 15. (c) 16. (b) 17. (a) 18. (a) 19. (d) 20. (d)
21. (b) 22. (a) 23. (a) 24. (a) 25. (c) 26. (a) 27. (d) 28. (c) 29. (d) 30. (c)
31. (d) 32. (a) 33. (b) 34. (c) 35. (c) 36. (a) 37. (c) 38. (c) 39. (a) 40. (b)
41. (c) 42. (d) 43. (b) 44. (b) 45. (c) 46. (a) 47. (d) 48. (d) 49. (b) 50. (c)
51. (b) 52. (c) 53. (c) 54. (a) 55. (d) 56. (b) 57. (a) 58. (d) 59. (c) 60. (b)
61. (c) 62. (a) 63. (a) 64. (a) 65. (b) 66. (c) 67. (d) 68. (c) 69. (b) 70. (a)
71. (a) 72. (d) 73. (c) 74. (c) 75. (b)
EXERCISE # II
1. (a,b,d) 2. (a,b,c,d) 3. (a,b,c) 4. (a,b,c,d) 5. (a,b,d) 6. (a,b,c,d)
7. (a,c) 8. (b,d) 9. (a,c,d) 10. (a,d) 11. (a,c,d) 12. (a,b,c)
13. (a,b) 14. (a,b,c,d) 15. (b,d) 16. (a,b) 17. (a,b,c) 18. (a,b,c,d,)
19. (c,d) 20. (a,d) 21. (a,b) 22. (a,b,c) 23. (a,d) 24. (a,b,c,d)
25. (a,c,d) 26. (b,c) 27. (b,c) 28. (a,b,d) 29. (b,c,d) 30. (b,c)
31. (a,b,c) 32. (b,c) 33. (a,d) 34. (a,d) 35. (a,b,c,d) 36. (a,b)
37. (b,c,d) 38. (a,b,c,d) 39. (a,b,d) 40. (a,b,c) 41. (a,b,c,d) 42. (a,b,c,d)

www.jeebooks.in
ANSWERS 10.11
43. (a,b,c,d) 44. (a,b,c,d) 45. (a,b,c,d) 46. (b,c) 47. (c,d) 48. (a,b,c)
49. (a,b,c,d) 50. (a,b,c,d) 51. (a,b,d) 52. (a,c) 53. (a,c) 54. (c,d)
55. (a,c) 56. (a,b,c) 57. (a,d) 58. (a,b,c) 59. (a,b) 60. (a,b,d)
61. (a,b,c) 62. (a,c) 63. (a,b,c,d) 64. (a,b,c) 65. (a,c,d)
EXERCISE # III : Paragraph Type & Matrix Match Type
1. (b) 2. (a) 3. (b) 4. (d) 5. (b,d) 6. (b) 7. (d) 8. (b) 9. (c) 10. (b)
11. (d) 12. (a) 13. (a) 14. (d) 15. (d) 16. (a) 17. (c) 18. (d) 19. (b) 20. (a)
21. (a) 22. (b) 23. (d) 24. (d) 25. (c) 26. (b) 27. (c) 28. (a)
29. (A)P ; (B)S ; (C)Q ; (D)R 30. (A)P; (B)Q ; (C)R ; (D)S
31. (A)P,S ; (B)R ; (C)Q ; (D)P,S 32. (A)P ; (B)Q ; (C)R ; (D)R,S
33. (A)R,T ; (B)P,Q ; (C)R ; (D)P,S
34. (A)Q,R ; (B)P,Q,R,S ; (C)P,Q,R ; (D)P,Q,R
35. (A)Q ; (B)R ; (C)P ; (D)S 36. (A)P ; (B)Q ; (C)R ; (D)S
37. (A)–P, R ; (B)–P,S ; (C)–P,Q ; (D)–P 38. (A)–Q ; (B)–R ; (C)–P,Q,R,S ; (D)–S
39. (A)–S ; (B)–Q ; (C)–R ; (D)–P 40. (A)P,Q (B) Q (C)  R(D)  S,T
41. (A)  R (B) S (C) P (D)  Q,T 42. (A) P ; (B)  Q ; (C)  R ; (D)  S
43. (A)  P,Q,R,S,T (B)  P,Q,R,S,T (C) P,R,S,T (D) S,T
44. (A)  P,Q,T (B)  P,Q,T (C)  P,S (D)  R,S 45. (A)  S (B)  Q,R,T (C)  P,R (D)  P,R
EXERCISE # IV : Integer Type
1. (7) 2. (6) 3. (3) 4. (8/17) 5. (4) 6. (5) 7. (4) 8. (4) 9. (6) 10. (9)
11. (4) 12. (5) 13. (4) 14. (3) 15. (2) 16. (1) 17. (4) 18. (3) 19. (6) 20. (6)
EXERCISE # V(A) : (JEE-MAIN)
1. (4) 2. (1) 3. (3) 4. (4) 5. (4) 6. (2) 7. (1) 8. (1) 9. (2) 10. (3)
11. (4) 12. (4) 13. (4) 14. (3) 15. (3) 16. (4) 17. (3) 18. (4) 19. (3) 20. (3)
21. (4)
EXERCISE # V(B) : (JEE-ADVANCED)
1. (a) 2. (a) 3. (b) 4. (b) 5. (a,b) 6. (b) 7. (b) 8. (a,b)

CH -6 T H E p -B L OCK EL EM EN T S AN D T H EI R COM POU N D S


EXERCISE # I
1. (d) 2. (b) 3. (a) 4. (c) 5. (c) 6. (d) 7. (a) 8. (c) 9. (d) 10. (d)
11. (c) 12. (a) 13. (d) 14. (b) 15. (a) 16. (c) 17. (d) 18. (c) 19. (b) 20. (a)
21. (c) 22. (c) 23. (c) 24. (a) 25. (a) 26. (b) 27. (a) 28. (b) 29. (d) 30. (a)
31. (c) 32. (c) 33. (a) 34. (d) 35. (c) 36. (d) 37. (b) 38. (b) 39. (c) 40. (c)
41. (b) 42. (c) 43. (a) 44. (d) 45. (a) 46. (b) 47. (b) 48. (b) 49. (c) 50. (d)
51. (c) 52. (a) 53. (d) 54. (d) 55. (c) 56. (d) 57. (b) 58. (a) 59. (c) 60. (b)

www.jeebooks.in
10.12 ANSWERS
61. (d) 62. (d) 63. (c) 64. (c) 65. (b) 66. (d) 67. (d) 68. (a) 69. (b) 70. (d)
71. (c) 72. (d) 73. (c) 74. (c) 75. (b) 76. (b) 77. (a) 78. (a) 79. (d) 80. (a)
81. (b) 82. (a) 83. (b) 84. (b) 85. (b) 86. (d) 87. (c) 88. (d) 89. (c) 90. (d)
91. (c) 92. (a) 93. (b) 94. (b) 95. (b) 96. (c) 97. (c) 98. (d) 99. (b) 100. (b)
101. (c) 102. (b) 103. (b) 104. (a) 105. (d) 106. (b) 107. (a) 108. (a) 109. (a) 110. (d)
111. (c) 112. (c) 113. (a) 114. (b) 115. (a) 116. (c) 117. (c) 118. (a) 119. (c) 120. (d)
121. (b) 122. (d) 123. (d) 124. (a) 125. (a) 126. (b) 127. (c) 128. (d) 129. (d) 130. (c)
131. (c) 132. (d) 133. (b) 134. (d) 135. (c) 136. (a) 137. (c) 138. (c) 139. (b) 140. (d)
141. (c) 142. (c) 143. (a) 144. (d) 145. (b) 146. (a) 147. (b) 148. (a) 149. (d) 150. (d)
EXERCISE # II
1. (a,b,d) 2. (a,d) 3. (a,b,c,d) 4. (a,b) 5. (b,c,d) 6. (a,b,c,d) 7. (b,c) 8. (c,d)
9. (a,c,d) 10. (a,b) 11. (c,d) 12. (c,d) 13. (c,d) 14. (a,b,c,d) 15. (a,b,c) 16. (a,b,c,d)
17. (a,b,c,d) 18. (a,b) 19. (a,b) 20. (b,c,d) 21. (a,b,c) 22. (c,d) 23. (b,c,d) 24. (a,c,d)
25. (b,d) 26. (b,d) 27. (a,b,d) 28. (a,b,c,d) 29. (a,b,d) 30. (a,b,c) 31. (a,b,c,d) 32. (c,d)
33. (b,c) 34. (b,c,d) 35. (b,c) 36. (a,b,c) 37. (a,b,d) 38. (a,b,d) 39. (a,b,c,d) 40. (a,c,d)
41. (a,b,c,d) 42. (c,d) 43. (a,c) 44. (a,b,d) 45. (a,b,c) 46. (a,c,d) 47. (a,b,c,d) 48. (a,b,c)
49. (b,d) 50. (a,b,d) 51. (a,c) 52. (b,d) 53. (b,c,d) 54. (c,d) 55. (c,d) 56. (b,c,d)
57. (a,b,c) 58. (b,c) 59. (a,b,c,d) 60. (a,c,d) 61. (a,b,c,d) 62. (a,b) 63. (a,c) 64. (a,c)
65. (b,d) 66. (a,d) 67. (a,b,c) 68. (b,c,d) 69. (a,b,c,d) 70. (a,c,d) 71. (a,b,c,d) 72. (a,d)
73. (a,b,c,d) 74. (b,d) 75. (a,b,c) 76. (a,b,d) 77. (c,d) 78. (a,b,c) 79. (c,d) 80. (c,d)
EXERCISE # III
Paragraph Type :
1. (d) 2. (b) 3. (d) 4. (a) 5. (c) 6. (b) 7. (b) 8. (a) 9. (b) 10. (a)
11. (c) 12. (d) 13. (a) 14. (a) 15. (d) 16. (b) 17. (c) 18. (d) 19. (d) 20. (d)
21. (b) 22. (c) 23. (d) 24. (c) 25. (d) 26. (b) 27. (d) 28. (c) 29. (b) 30. (b)
31. (b) 32. (c) 33. (b) 34. (d) 35. (c) 36. (c) 37. (d) 38. (a) 39. (b) 40. (d)
41. (c) 42. (d) 43. (c) 44. (b) 45. (c) 46. (a) 47. (b) 48. (c) 49. (c) 50. (d)
51. (a) 52. (c) 53. (d) 54. (a) 55. (d) 56. (c) 57. (a) 58. (b) 59. (b) 60. (d)
Matrix Match Type :
(a) (b) (c) (d) (a) (b) (c) (d)
61. Q; R,S; P, S ; Q 62. P,S; Q; R; T
63. P; S; Q; R 64. P,R,S; P,Q,R,T; P,Q,R,T; P,Q,R,S
65. P; T; Q,R,T; Q,S 66. Q,R; Q; P,R; P,R,S
67. P,Q,R,S, T;P,Q,R,S,T; P,Q,R,S P,R,S 68. P,S; P,Q,R; P,Q,R; Q,R
69. Q,S; S; P R 70. R,S; Q,R; P,T; S
71. P,Q,R,S; P,Q,R,S; R; P,Q,S 72. P,Q,R,S,T; S; P,Q,R,T; P,Q,R
73. Q; R,S; Q,T; P,T 74. P; R,S; Q,T; P,T

www.jeebooks.in
ANSWERS 10.13
(a) (b) (c) (d) (a) (b) (c) (d)
75. P,R,S; P,R,S,T; Q,R; P,R,S 76. P,T; Q,S; Q,R; Q,T
77. P,Q,R,T; R; P,T; R,S 78. P,Q; P,Q,R,S; R; S
79. R; Q; P; S 80. P,S; P,R; Q; R,T
81. R,S,T; Q,R,T; P; Q,R 82. P,Q,T; P,Q,S; P; R
83. S; R; P; Q 84. P,T; R,T; S; Q
85. P,R; T; P,Q,R; P,R,S 86. Q; R,S; P; T
87. R,S; Q,S; Q,S; P,T 88. P,R,S; P,R,S; Q,R,S; Q,R,S,T
89. R; P,Q; S; T 90. R,T; S; Q,T; P
EXERCISE # IV
1. (6) 2. (3) 3. (3) 4. (5) 5. (5) 6. (1) 7. (4) 8. (2) 9. (0) 10. (2)
11. (0) 12. (1) 13. (6) 14. (2) 15. (4) 16. (3) 17. (4) 18. (4) 19. (7) 20. (2)
21. (2) 22. (7) 23. (1) 24. (9) 25. (5) 26. (0) 27. (0) 28. (6) 29. (4) 30. (0)
31. (3) 32. (4) 33. (3) 34. (4) 35. (4) a,b,c,d,e 36. (3) 37. (3) 38. (2) 39. (3)
40. (2) 41. (3) 42. (8) 43. (8) 44. (2) 45. (6) 46. (2) 47. 3/48 48. 5/14 49. (4)
50. (9)
EXERCISE # V(A) JEE-MAIN
1. (a) 2. (d) 3. (d) 4. (d) 5. (b) 6. (c) 7. (d) 8. (b) 9. (b) 10. (b)
11. (b) 12. (c) 13. (b) 14. (b) 15. (d) 16. (c) 17. (c) 18. (a) 19. (d) 20. (c)

21. (b) 22. (c) 23. (c) 24. (b) 25. (c) 26. (d) 27. (b) 28. (a) 29. (c) 30. (c)
31. (d) 32. (a) 33. (c) 34. (b) 35. (a) 36. (b) 37. (b) 38. (b) 39. (c) 40. (a)
41. (c) 42. (a) 43. (a) 44. (c) 45. (c) 46. (b,c,d) 47. (d) 48. (d) 49. (b) 50. (b)
51. (c)
EXCERCISE # V(B) JEE-ADVANCED
Fill in the blanks :
1. Silicones 2. R3Si(OH) 3. Three centre two electron bond or banana bond
4. buckminstre fullerene
True/False :
5. (F) 6. (T) 7. (F) 8. (T) 9. (F) 10. (T)
Objective :
32. (b,d) 33. (d) 34. (b) 35. (c) 36. (a) 37. (a) 38. (c) 39. (b,d) 40. (a) 41. (c)
42. (d) 43. (a) 44. (c) 45. (c) 46. (d) 47. (a) 18. (a) 49. (a) 51. (c) 52. (c)
53. (a,b) 54. (c) 55. (b) 56. (b) 57. (a) 58. (d) 59. (d) 60. (a) 61. (c) 62. (a)
63. (c) 64. (a) 65. (c) 66. (c) 67. (b) 68. (b) 69. (a,b,c) 70. (b,c) 71. (c) 72. (a)
73. (b,d) 74. (b) 75. (a,c,d )76. (a) 77. (a) 78. (d) 79. (c) 80. (a) 81. (b,d) 82. (b,c)
83. (b,c,d) 84. (b) 85. (6) 86. (8)

www.jeebooks.in
10.14 ANSWERS
CH -7 T H E d -B L OCK EL EM EN T S AN D SOM E OF T H EI R COM POU N D S
EXERCISE # I
1. (c) 2. (d) 3. (d) 4. (b) 5. (d) 6. (a) 7. (b) 8. (c) 9. (d) 10. (b)
11. (b) 12. (b) 13. (b) 15. (c) 16. (b) 17. (b) 18. (a) 19. (d) 20. (c)
21. (d) 22. (a) 23. (c) 24. (a) 25. (d) 26. (b) 27. (c) 28. (d) 29. (a) 30. (b)
31. (b) 32. (a) 33. (d) 34. (c) 35. (d) 36. (b) 37. (a) 38. (b) 39. (b) 40. (d)
41. (d) 42. (a) 43. (c) 44. (b) 45. (a) 46. (b) 47. (b) 48. (a) 49. (b) 50. (c)
51. (b) 52. (d) 53. (c) 54. (b) 55. (d) 56. (a) 57. (c) 58. (d) 59. (d) 60. (c)
61. (c) 62. (b) 63. (d) 64. (a) 65. (d) 66. (d) 67. (d) 68. (b) 69. (a) 70. (d)
EXERCISE # II
1. (a,d) 2. (a,b,c,d) 3. (a,c,d) 4. (a,c) 5. (a,b,c,d) 6. (b,d)
7. (a,b,c,d) 8. (c,d) 9. (a,b) 10. a,b,c,d) 11. (a,b) 12. (b,c)
13. (b,d) 14. (c,d) 15. (a,d) 16. (b,d) 17. (a,b) 18. (b,c)
19. (a,b,c) 20. (a,b,c,d) 21. (a,b,d) 22. (a,b,d) 23. (a,c,d) 24. (a,b,c,d)
25. (a,b) 26. (a,b) 27. (a,b) 27. (a,b,c) 28. (a,b,d) 29. (a,b,c)
30. (a,b,d) 31. (a,b,c) 32. (a,d) 33. (c,d) 34. (a,b) 35. (a,c,d)
36. (a,b) 37. (a,b) 38. (c,d) 39. (a,b) 40. (a,b,d) 41. (a,b,c)
42. (a,c) 43. (a,b,c) 44. (a,c,d) 45. (c,d)
EXERCISE # III : Paragraph Type & Matrix Match Type
1. (b) 2. (b) 3. (d) 4. (a) 5. (c) 6. (b) 7. (a) 8. (d) 9. (a) 10. (d)
11. (d) 12. (d) 13. (d) 14. (c) 15. (b) 16. (c) 17. (d) 18. (d) 19. (c) 20. (c)
21. (a) 22. (b) 23. (a) 24. (c) 25. (c) 26. (c) 27. (c) 28. (a) 29. (a) 30. (b)
31. (b) 32. (a) 33. (a) 34. (c) 35. (a) 36. (c) 37. (a) 38. (b)
39. A  R ; B  S ; C  Q ; D  P 40. A  P ; B  Q ; C  R ; D  S
41. A  Q,T ; B  P ; C  S ; D  R 42. A  P,R ; B  P,S ; C  Q,T ; D  Q,T
43. A  S,T ; B  P,Q ; C  P,R ; D  R,S,T 44. A  P ; B  Q ; C  R ; D  S
45. A  P ; B  R ; C  S ; D  Q 46. A  P,R ; B  Q,T ; C  Q,R,S ; D  P
47. A  Q,R ; B  R,T ; C  P,Q,T ; D  P,Q 48. A  ; B  ; C  ; D 
49. A  P ; B  Q ; C  R ; D  S 50. A  P ; B  Q ; C  R ; D  S
51. A  P ; B  Q ; C  R ; D  S 52. A  P ; B  Q ; C  R ; D  S
53. A  P,Q ; B  P,S ; C  R ; D  R 54. A  P ; B  Q ; C  R ; D  S
55. A  P,Q,R,T ; B  P,R,T ; C  P,R,S,T ; D  P,R,T
EXERCISE # IV : Integer Type
1. (4) 2. (6) 3. (5) 4. (7) 5. (0) 6. (2) 7. (4) 8. (3) 9. (6) 10. (6)
11. (1) 12. (9) 13. (3) 14. (3) 15. (3) 16. (5) 17. (8) 18. (5) 19. (4) 20. (6)
21. (9/18) 22. (1) 23. (4) 24. (0) 25. (4) 26. (3) 27. (6) 28. (9) 29. (3) 30. (7)

www.jeebooks.in
ANSWERS 10.15
EXERCISE # V(A) : (JEE-MAIN)
1. (1) 2. (1) 3. (1) 4. (4) 5. (2) 6. (2) 7. (4) 8. (3) 9. (1) 10. (3)
11. (2) 12. (4) 13. (4) 14. (4) 15. (1) 16. (2) 17. (2) 18. (2) 19. (1)
EXERCISE # V(B) : (JEE-ADVANCED)
1. (b) 2. (a)

CH -8 H YD ROGEN AN D T H E H YD RI D E
EXERCISE # I
1. (c) 2. (a) 3. (b) 4. (c) 5. (d) 6. (a) 7. (c) 8. (a) 9. (b) 10. (c)
11. (c) 12. (d) 13. (d) 14. (a) 15. (b) 16. (c) 17. (b) 18. (b) 19. (a) 20. (c)
21. (c) 22. (d) 23. (c) 24. (a) 25. (b) 26. (a) 27. (a) 28. (c) 29. (a) 30. (b)
31. (b) 32. (a) 33. (b) 34. (c) 35. (d) 36. (b) 37. (d) 38. (c) 39. (a) 40. (b)
41. (d) 42. (d) 43. (b) 44. (d) 45. (c) 46. (a)
EXERCISE # II
1. (a,c) 2. (c,d) 3. (b,d) 4. (b,d) 5. (a,d) 6. (a,b,d)
7. (b,d) 8. (b,c) 9. (a,b,d) 10. (a,b) 11. (a,b,c,d) 12. (a,b,d)
13. (a,b,d) 14. (a,b,c,d) 15. (a,b,c,d) 16. (a,b,c) 17. (a,b,c) 18. (a,b,c,d)
19. (c,d) 20. (b,c)
EXERCISE # III
Paragraph Type :
1. (d) 2. (d) 3. (d) 4. (c) 5. (d) 6. (d) 7. (b) 8. (a) 9. (b) 10. (c)
11. (a) 12. (d) 13. (c) 14. (a) 15. (b)
Matrix Match Type :
(a) (b) (c) (d) (a) (b) (c) (d)
1. P,Q,S; Q,R,S; Q; Q,S,T 2. Q; P,S; R; T
3. R,S; Q; S; P 4. Q,R,S,T; Q,R,T; Q,R,T; P,T
5. P; S; Q; R 6. P,Q,R; P,Q; P,Q,R,S; P,Q,R
7. R; P,Q,S,T; R,S,T; P,Q,R 8. P,S; P,R; P,Q; T
9. P,R; P,R; Q,R,T; Q,S,T 10. Q,R; R; P,Q; P,S
11. R; Q; T; P,S
EXERCISE # IV
1. 3 2. 6 3. 4 4. 1 5. 1 6. 4 7. 2 8. 5 9. 6 10. 6
11. 0 12. 4 13. 7 14. 6 15. 3

www.jeebooks.in
10.16 ANSWERS

CH -9 PERI OD I C T AB L E
EXERCISE # I
1. (b) 2. (a) 3. (a) 4. (c) 5. (d) 6. (c) 7. (d) 8. (c) 9. (c) 10. (a)
11. (a) 12. (d) 13. (d) 14. (c) 15. (d) 16. (a) 17. (a) 18. (a) 19. (a) 20. (c)
21. (c) 22. (d) 23. (c) 24. (c) 25. (c) 26. (d) 27. (a) 28. (a) 29. (c) 30. (c)
31. (a) 32. (d) 33. (a) 34. (d) 35. (a) 36. (b) 37. (d) 38. (b) 39. (d) 40. (d)
41. (a) 42. (a) 43. (b) 44. (b) 45. (a) 46. (b) 47. (b) 48. (c) 49. (a) 50. (a)
51. (d) 52. (d) 53. (a) 54. (a) 55. (c) 56. (b) 57. (d) 58. (a) 59. (b) 60. (b)
61. (b) 62. (b) 63. (c) 64. (a) 65. (a) 66. (b) 67. (c) 68. (a) 69. (d) 70. (b)
71. (c) 72. (a) 73. (d) 74. (d) 75. (a) 76. (c) 77. (c) 78. (c) 79. (a) 80. (d)
81. (a) 82. (b) 83. (a) 84. (c) 85. (b) 86. (b) 87. (c) 88. (d) 89. (d) 90. (c)
91. (b) 92. (a) 93. (b) 94. (c) 95. (d) 96. (c) 97. (d) 98. (b) 99. (a) 100. (a)
101. (b) 102. (a) 103. (b) 104. (c) 105. (d) 106. (a) 107. (d) 108. (b) 109. (d) 110. (a)
111. (d) 112. (b) 113. (a) 114. (c) 115. (c) 116. (b) 117. (a) 118. (a) 119. (d) 120. (b)
121. (d) 122. (c) 123. (c) 124. (c) 125. (b) 126. (c) 127. (a) 128. (c) 129. (b) 130. (a)
131. (b) 132. (b) 133. (d) 134. (d) 135. (a) 136. (a) 137. (d) 138. (a) 139. (a,b,c) 140. (b)
141. (a) 142. (d) 143. (d) 144. (a) 145. (c) 146. (a) 147. (d) 148. (d) 149. (d) 150. (b)
EXERCISE # II
1. (c,d) 2. (a,d) 3. (a,b,c) 4. (c,d) 5. (b,c) 6. (a,b)
7. (c,d) 8. (a,c,d) 9. (a,b,c) 10. (a,d) 11. (b,c,d) 12. (b,d)
13. (a,c,d) 14. (a,b,c,d) 15. (a,b,c,d) 16. (b,c,d) 17. (b,d) 18. (a,b,c)
19. (a,b,c,d) 20. (a,b) 21. (a,d) 22. (a,b,c) 23. (b,d) 24. (a,d)
25. (b,c,d) 26. (b,c,d) 27. (c,d) 28. (a,d) 29. (a,b,c,d) 30. (b,c,d)
31. (a,b,c,d) 32. (a,b,d) 34. (a,b,c) 35. (b,c) 36. (a,b) 37. (a,b,c,d)
38. (a,b,c,d) 39. (a,b,c) 40. (a,d) 41. (a,c) 42. (a,b,c) 43. (a,b,c,d)
44. (c,d) 45. (a,c,d) 46. (a,c,d) 47. (a,b,c) 48. (a,b,c) 49. (b,c)
50. (a,b,c,d) 51. (b,c) 52. (a,d) 53. (c,d) 54. (b,c) 55. (b,d)
56. (a,b,c) 57. (a,b,d) 58. (b,c,d) 59. (a,b,c,d) 60. (a,d) 61. (a,c,d)
62. (a,b,d) 63. (b,c,d) 64. (c,d) 65. (d) 66. (a,d) 67. (a,b)
68. (a,b,c,d) 69. (b,c) 70. (b,c) 71. (a,b) 72. (a,b) 73. (b,c,d)
74. (a,d) 75. (a,b,c,d) 76. (c,d) 77. (a,b) 78. (c,d) 79. ( a,b,c,d)
80. (b,c,d) 81. (b,d) 82. (b,d) 83. (b,c,d) 84. (a,c) 85. (a,d)
86. (a,c) 87. (a,c) 88. (a,c) 89. (a,c) 90. (a,d) 91. (b,c,d)
92. (a,c) 93. (b,d) 94. (a,b,d) 95. (b,c) 96. (a,c) 97. (b,d)
98. (a,b,d) 99. (a,b,d) 100. (a,b,c,d)

www.jeebooks.in
ANSWERS 10.17
EXERCISE # III
Paragraph Type :
1. (c) 2. (d) 3. (d) 4. (b) 5. (a) 6. (d) 7. (b) 8. (d) 9. (a) 10. (d)
11. (b) 12. (d) 13. (c) 14. (d) 15. (a) 16. (b) 17. (b) 18. (d) 19. (d) 20. (d)
21. (a) 22. (d) 23. (a) 24. (b) 25. (c) 26. (d) 27. (a) 28. (b) 29. (b) 30. (b)
31. (c) 32. (d) 33. (d) 34. (b) 35. (b) 36. (c) 37. (b) 38. (c) 39. (b) 40. (c)
41. (c) 42. (b) 43. (d) 44. (c) 45. (c) 46. (d) 47. (c) 48. (d) 49. (c) 50. (c)
51. (a) 52. (d) 53. (b) 54. (b) 55. (d) 56. (d) 57. (b) 58. (b) 59. (c) 60. (b)
Matrix Match Type :
(a) (b) (c) (d) (a) (b) (c) (d)
61. P; Q; R; S 62. S; R; Q; P
63. S; R; Q; P 64. P; Q; R; S
65. P,S; Q ,S; R; T 66. Q,R,T; P,R,S,T; P,R,S,T; P,R,S
67. Q,R; P; S; T 68. P,T; Q,T; R,S; S
69. Q,T; P,R,S,T; R,S,T; P,R,S,T 70. P; Q; R; S
71. P; Q; R; S 72. P; Q; R; S
73. P; Q; R; S 74. S; R; P; Q
75. R; S; P; Q 76. R; P; S; Q
77. P; Q; R; S 78. P; Q; R; S
79. Q,R,T; P,R,S; P,R,S; Q,S,T 80. P,T; Q,S,T; R,S; R,S
81. P; R; S; Q 82. S; P; Q; R
83. Q; R; P; S; 84. P; Q; R; S
85. Q; P; R; S 86. Q,T; P,S; P,S; R
87. P; Q,T; R; S 88. P,T; P,Q,T; P,Q,R,T; P,Q,R,S,T
89. P,T; R; S; Q 90. R; Q; P,S; P,S
91. P,R; R,S; Q,R; P,R 92. P; Q; R; S
93. P; Q; R; S 94. P; Q; R; S
95. Q,S; T; R,T; P 96. R,T; P,R,T; P,Q,R; Q,S,T
97. R,T; P,S; S,T; Q,S,T 98. P,Q,S; P,Q,R; P,Q,R; P,S,T
99. S; R; Q; P 100. P; P,T; Q,S; P,S
EXERCISE # IV
1. (5) 2. (6) 3. (8) 4. (9) 5. (3) 6. (3) 7. (2) 8. (8) 9. (3) 10. (9)72
11. (3) 12. (9) 13. (6) 14. (9) 15. (0) 16. (2) 17. (2) 18. (141/6) 19. (8) 20. (9)
21. (6) 22. (5) 23. (5) 24. (3) 25. (2) 26. (0) 27. (8) 28. (6) 29. (7) 30. (8)
31. (14/5) 32. (4) 33. (4) 34. (6) 35. (1) 36. (16/7) 37. (21/3) 38. (3/30) 39. (0) 40. (1)
41. (2) 42. (7) 43. (9) 44. (2) 45. (3) 46. (4) 47. (3) 48. (8) 49. (1) 50. (5)

www.jeebooks.in
10.18 ANSWERS
EXERCISE # V(JEE-MAIN & ADVANCED)
1. (c) 2. (c) 3. (a) 4.(a) 5. (c) 6. (a) 7. (d) 8. (b) 9. (d) 10. (b)
11. (d) 12. (b) 13. (a) 14. (b) 15. (b) 16. (b) 17. Al3+ < Mg2+ < Li+ < K+
18. Inert pair effect 19. (c) 20. (d)
21. CrO3 on reaction with water produces chromic acid (H2CrO4). So CrO3 is an acid anhydride.
22. (c) 23. (b) 24. (a) 25. (a) 26. (b) 27. Cl2O7 < SO3 < CO2 < B2O3 < BaO 28. (c)
29. (c) 30. (c) 31. (d) 32. (d) 33. (b) 34. (d) 35. (c) 36. (d) 37. (d) 38. (d)
39. (a) 40. (a) 41. (b) 42. (d) 43. (d) 44. (a) 45. (b) 46. (c) 47. (c) 48. (b)
49. (a) 50. (a) 51. (6)



www.jeebooks.in
www.jeebooks.in

You might also like